Mental Health FINAL

Ace your homework & exams now with Quizwiz!

Which criterion is NOT essential when the nurse plans nursing interventions designed to meet a specific goal? Economic

Economical Correct Although expense should be considered, interventions are chosen based on the other options and not on their economic value.

A nursing diagnosis for a client with a psychiatric disorder serves the purpose of....

providing a framework for selecting appropriate interventions

A newly diagnosed patient is prescribed lithium. Which information from the patient's history indicates that monitoring of serum concentrations of the drug will be challenging and critical?

Heart failure

A patient demonstrating behaviors associated with acute mania has exhausted the staff by noon. Staff members are feeling defensive and fatigued. Which action will the staff take initially?

Hold a staff meeting to discuss consistency and limit-setting approaches.

The premise underlying behavioral therapy is a. Behavior is learned and can be modified. b. Behavior is a product of unconscious drives. c. Motives must change before behavior changes. d. Behavior is determined by cognitions; change in cognitions produces new behavior.

A Behavior is learned and can be modified.

A patient diagnosed with bipolar disorder becomes hyperactive after discontinuing lithium. The patient threatens to hit another patient. Which comment by the nurse is appropriate?

"Do not hit anyone. If you are unable to control yourself, we will help you."

A 4-year-old cries for 5 minutes when the parents leave the child at preschool. The parents ask the nurse, "What should we do?" Select the nurse's best response. a. "Ask the teacher to let the child call you at play time." b. "Withdraw the child from preschool until maturity increases." c. "Remain with your child for the first hour of preschool time." d. "Give your child a kiss before you leave the preschool program."

"Give your child a kiss before you leave the preschool program."

Current information suggests that the most disabling mental disorders are the result of: A) biological influences. B) psychological trauma. C) learned ways of behaving. D) faulty patterns of early nurturance.

A biological influences

A nurse interacts with patients diagnosed with various mental illnesses. Which statements reflect use of therapeutic communication? (Select all that apply.) a. "Tell me more about that situation." b. "Let's talk about something else." c. "I notice you are pacing a lot." d. "I'll stay with you a while." e. "Why did you do that?"

A, C, D The correct responses demonstrate use of the therapeutic techniques making an observation and showing empathy. The incorrect responses demonstrate changing the subject and probing, which are nontherapeutic techniques

Which comments by an adult best indicate self-actualization? (Select all that apply.) a. "I am content with a good book." b. "I often wonder if I chose the right career." c. "Sometimes I think about how my parents would have handled problems." d. "It's important for our country to provide basic health care services for everyone." e. "When I was lost at sea for 2 days, I gained an understanding of what is important."

A, D, E Self-actualized persons enjoy privacy, have a sense of democracy, and show positive outcomes associated with peak experiences. Self-doubt, defensiveness, and blaming are not consistent with self-actualization.

4. Which documentation of family assessment indicates a healthy and functional family? a. Members provide mutual support. b. Power is distributed equally among all members. c. Members believe there are specific causes for events. d. Under stress, members turn inward and become enmeshed.

ANS: A Healthy families nurture and support their members, buffer against stress, and provide stability and cohesion. The distracters are unrelated or incorrect.

11. It has been 5 days since a suicidal patient was hospitalized and prescribed an antidepressant medication. The patient is now more talkative and shows increased energy. Select the highest priority nursing intervention. a. Supervise the patient 24 hours a day. b. Begin discharge planning for the patient. c. Refer the patient to art and music therapists. d. Consider discontinuation of suicide precautions.

ANS: A The patient now has more energy and may have decided on suicide, especially given the prior suicide attempt history. The patient must be supervised 24 hours per day. The patient is still a suicide risk.

4. During group therapy, one patient says to another, "When I first started in this group, you were unable to make a decision, but now you can. You've made so much progress that I am beginning to think maybe I can conquer my fears too." Which therapeutic factor is evident by this statement? a. Hope b. Altruism c. Catharsis d. Cohesiveness

ANS: A The patient's profession that he may be able to learn to cope more effectively reflects hope. Groups can instill hope in individuals who are demoralized or pessimistic. Altruism refers to doing good for others, which can result in positive feelings about oneself. Catharsis refers to venting of strong emotions. Cohesion refers to coming together and developing a connection with other group members.

18. During morning care, a nurse asks a patient diagnosed with dementia, "How was your night?" The patient replies, "It was lovely. I went out to dinner and a movie with my friend." Which term applies to the patient's response? a. Sundown syndrome b. Confabulation c. Perseveration d. Delirium

ANS: B Confabulation refers to making up of stories or answers to questions by a person who does not remember. It is a defensive tactic to protect self-esteem and prevent others from noticing memory loss. The patient's response was not sundown syndrome. Perseveration refers to repeating a word or phrase over and over. Delirium is not present in this scenario.

3. After the sudden death of his wife, a man says, "I can't live without her ... she was my whole life." Select the nurse's most therapeutic reply. a. "Each day will get a little better." b. "Her death is a terrible loss for you." c. "It's important to recognize that she is no longer suffering." d. "Your friends will help you cope with this change in your life."

ANS: B Adjustment disorders may be associated with grief. A statement that validates a bereaved person's loss is more helpful than false reassurances and clichés. It signifies understanding.

11. A parent is admitted to a unit for treatment of addictions. The patient's spouse and adolescent children attend a family session. Which initial assessment question should the nurse ask of family members? a. "What changes are most important to you?" b. "How are feelings expressed in your family?" c. "What types of family education would benefit your family?" d. "Can you identify a long-term goal for improved functioning?"

ANS: B It is important to understand family characteristics, particularly in a family under stress. Expression of feelings is an important aspect of assessment of the family's function (or dysfunction). The distracters relate more to outcome identification and planning interventions, both of which should be delayed until the assessment is complete.

16. A nurse wants to enhance growth of a patient by showing positive regard. The nurse's action most likely to achieve this goal is a. making rounds daily. b. staying with a tearful patient. c. administering medication as prescribed. d. examining personal feelings about a patient.

ANS: B Staying with a crying patient offers support and shows positive regard. Administering daily medication and making rounds are tasks that could be part of an assignment and do not necessarily reflect positive regard. Examining feelings regarding a patient addresses the nurse's ability to be therapeutic.

Sullivan viewed anxiety as: A) emotional experience felt after the age of 5 years. B) a sign of guilt in adults. C) any painful feeling or emotion arising from social insecurity. D) adults trying to go beyond experiences of guilt and pain.

C Any painful feeling or emotion arising from social insecurity

Which branch of epidemiology is the nurse involved in when seeking outcomes for patients whose depression was treated with electroconvulsive therapy (ECT)? A. experimental B. descriptive C. clinical D. analytic

C Clinical

1. A married couple has two biologic children who live with them as well as a child from the wife's first marriage. What type of family is evident? a. Homogeneous b. Extended c. Blended d. Nuclear

ANS: C A blended family is made up of members from two or more unrelated families. It is not a nuclear family because a stepchild is present. It is not an extended family, because there are only two generations present. Homogeneous is not a family type.

8. During which phase of the nurse-patient relationship can the nurse anticipate that identified patient issues will be explored and resolved? a. Preorientation b. Orientation c. Working d. Termination

ANS: C During the working phase, the nurse strives to assist the patient in making connections among dysfunctional behaviors, thinking, and emotions and offers support while alternative coping behaviors are tried.

18. Which agency provides coordination in the event of a terrorist attack? a. Food and Drug Administration (FDA) b. Environmental Protection Agency (EPA) c. National Incident Management System (NIMS) d. Federal Emergency Management Agency (FEMA)

ANS: C The NIMS provides a systematic approach to guide departments and agencies at all levels of government, nongovernmental organizations, and the private sector during disaster situations.

13. A nurse cares for a rape victim who was given a drink that contained flunitrazepam by an assailant. Which intervention has priority? Monitoring for a. coma. b. seizures. c. hypotonia. d. respiratory depression.

ANS: D Monitoring for respiratory depression takes priority over hypotonia, seizures, or coma.

2. Four teenagers died in an automobile accident. One week later, which behavior by the parents of these teenagers most clearly demonstrates resilience? The parents who a. visit their teenager's grave daily. b. return immediately to employment. c. discuss the accident within the family only. d. create a scholarship fund at their child's high school.

ANS: D Resilience refers to positive adaptation or the ability to maintain or regain mental health despite adversity. Loss of a child is among the highest risk situations for maladaptive grieving. The parents who create a scholarship fund are openly expressing their feelings and memorializing their child. The other parents in this question are isolating themselves and/or denying their feelings. Visiting the grave daily shows active continued mourning but is not as strongly indicative of resilience as the correct response.

19. A nurse counsels a patient with recent suicidal ideation. Which is the nurse's most therapeutic comment? a. "Let's make a list of all your problems and think of solutions for each one." b. "I'm happy you're taking control of your problems and trying to find solutions." c. "When you have bad feelings, try to focus on positive experiences from your life." d. "Let's consider which problems are very important and which are less important."

ANS: D The nurse helps the patient develop effective coping skills. Assist the patient to reduce the overwhelming effects of problems by prioritizing them. The incorrect options continue to present overwhelming approaches to problem solving.

A patient diagnosed with bipolar disorder will be discharged tomorrow. The patient is taking a mood stabilizing medication. What is the priority nursing intervention for the patient as well as the patient's family during this phase of treatment?

Attending psychoeducation sessions

A patient fearfully runs from chair to chair crying, "They're coming! They're coming!" The patient does not follow the staff's directions or respond to verbal interventions. Which nursing diagnosis has the highest priority? a.Fear b.Risk for injury c.Self-care deficit d.Disturbed thought processes

B A patient experiencing panic-level anxiety is at high risk for injury related to increased non-goal-directed motor activity, distorted perceptions, and disordered thoughts. Data are not present to support a nursing diagnosis of self-care deficit or disturbed thought processes. The patient may have fear, but the risk for injury has a higher priority.

A patient experiences a sudden episode of severe anxiety. Of these medications in the patient's medical record, which is most appropriate to give as a prn anxiolytic? a.buspirone (BuSpar) c.amitriptyline (Elavil) b.lorazepam (Ativan) d.desipramine (Norpramin)

B Lorazepam is a benzodiazepine used to treat anxiety. It may be given as a prn medication. Buspirone is long acting and is not useful as a prn drug. Amitriptyline and desipramine are tricyclic antidepressants and considered second- or third-line agents.

A patient with a long history of hypertension and diabetes now develops confusion. The health care provider wants to make a differential diagnosis between Alzheimer's disease and multiple infarcts. Which diagnostic procedure should the nurse expect to prepare the patient for first? a. Skull x-rays b. CT scan c. PET d. Single photon emission computed tomography (SPECT)

B A CT scan shows the presence or absence of structural changes, including cortical atrophy, ventricular enlargement, and areas of infarct, information that would be helpful to the health care provider. PET and SPECT show brain activity rather than structure and may occur later. See relationship to audience response question.

A patient says to the nurse, "My life doesn't have any happiness in it anymore. I once enjoyed holidays, but now they're just another day." The nurse documents this report as an example of: a. dysthymia. b. anhedonia. c. euphoria. d. anergia.

B Anhedonia is a common finding in many types of depression. It refers to feelings of a loss of pleasure in formerly pleasurable activities. Dysthymia is a diagnosis. Euphoria refers to an elated mood. Anergia means "without energy."

A nurse is part of a multidisciplinary team working with groups of depressed patients. Half the patients receive supportive interventions and antidepressant medication. The other half receives only medication. The team measures outcomes for each group. Which type of study is evident? a. Prevalence b. Clinical epidemiology c. Descriptive epidemiology d. Experimental epidemiology

B Clinical epidemiology is a broad field that addresses what happens to people with illnesses seen by providers of clinical care. This study is concerned with the effectiveness of various interventions. Prevalence refers to numbers of new cases. Descriptive epidemiology provides estimates of the rates of disorders in a general population and its subgroups. Experimental epidemiology tests presumed assumptions between a risk factor and a disorder.

A black patient says to a white nurse, "There's no sense talking about how I feel. You wouldn't understand because you live in a white world." The nurse's best action would be to a. explain, "Yes, I do understand. Everyone goes through the same experiences." b. say, "Please give an example of something you think I wouldn't understand." c. reassure the patient that nurses interact with people from all cultures. d. change the subject to one that is less emotionally disturbing.

B Having the patient speak in specifics rather than globally will help the nurse understand the patient's perspective. This approach will help the nurse engage the patient. Reassurance and changing the subject are not therapeutic techniques.

A medical-surgical nurse works with a patient diagnosed with a somatic symptom disorder. Care planning is facilitated by understanding that the patient will probably: a. readily seek psychiatric counseling. b. be resistant to accepting psychiatric help. c. attend psychotherapy sessions without encouragement. d. be eager to discover the true reasons for physical symptoms.

B Patients with somatic symptom disorders go from one health care provider to another trying to establish a physical cause for their symptoms. When a psychological basis is suggested and a referral for counseling offered, these patients reject both.

A patient has blindness related to conversion (functional neurological) disorder but is unconcerned about this problem. Which understanding should guide the nurse's planning for this patient? a. The patient is suppressing accurate feelings regarding the problem. b. The patient's anxiety is relieved through the physical symptom. c. The patient's optic nerve transmission has been impaired. d. The patient will not disclose genuine fears.

B Psychoanalytical theory suggests conversion reduces anxiety through production of a physical symptom symbolically linked to an underlying conflict. Conversion, not suppression, is the operative defense mechanism in this disorder. While some MRI studies suggest that patients with conversion disorder have an abnormal pattern of cerebral activation, there is no actual alternation of nerve transmission. The other distracters oversimplify the dynamics, suggesting that only dependency needs are of concern, or suggest conscious motivation (conversion operates unconsciously). See relationship to audience response question.

Which action by a nurse constitutes a breach of a patient's right to privacy? a. Documenting the patient's daily behavior during hospitalization b. Releasing information to the patient's employer without consent c. Discussing the patient's history with other staff during care planning d. Asking family to share information about a patient's pre-hospitalization behavior

B Release of information without patient authorization violates the patient's right to privacy. The other options are acceptable nursing practices. See relationship to audience response question.

Two staff nurses applied for a charge nurse position. After the promotion was announced, the nurse who was not promoted said, "The nurse manager had a headache the day I was interviewed." Which defense mechanism is evident? a.Introjection b.Conversion c.Projection d.Splitting

C Projection is the hallmark of blaming, scapegoating, prejudicial thinking, and stigmatizing others. Conversion involves the unconscious transformation of anxiety into a physical symptom. Introjection involves intense, unconscious identification with another person. Splitting is the inability to integrate the positive and negative qualities of oneself or others into a cohesive image.

A patient diagnosed with a somatic symptom disorder has been in treatment for 4 weeks. The patient says, "Although I'm still having pain, I notice it less and am able to perform more activities." The nurse should evaluate the treatment plan as: a. marginally successful. b. minimally successful. c. partially successful. d. totally achieved.

C Decreased preoccupation with symptoms and increased ability to perform activities of daily living suggest partial success of the treatment plan. Total success is rare because of patient resistance.

A Haitian patient diagnosed with major depressive disorder tells the nurse, "There's nothing you can do. This is a punishment. The only thing I can do is see a healer." The culturally aware nurse assesses that the patient a. has delusions of persecution. b. has likely been misdiagnosed with depression. c. may believe the distress is the result of a curse or spell. d. feels hopeless and helpless related to an unidentified cause.

C Individuals of African American or Caribbean cultures who have a fatalistic attitude about illness may believe they are being punished for wrongdoing or are victims of witchcraft or voodoo. They may be reticent to share information about curses with therapists. No data are present in the scenario to support delusions. Misdiagnosis more often labels a patient with depression as having schizophrenia.

Which communication strategy would be most effective for a nurse to use during an assessment interview with an adult Native American patient? a. Open and friendly, ask direct questions, touch the patient's arm or hand occasionally for reassurance. b. Frequent nonverbal behaviors, such as gestures and smiles, make an unemotional face to express negatives. c. Soft voice, break eye contact occasionally, general leads and reflective techniques. d. Stern voice, unbroken eye contact, minimal gestures, direct questions.

C Native American culture stresses living in harmony with nature. Cooperative, sharing styles rather than competitive or intrusive approaches are preferred.

Which treatment modality should a nurse recommend to help a patient diagnosed with a somatic symptom disorder to cope more effectively? a. Flooding c. Relaxation techniques b. Response prevention d. Systematic desensitization

C Somatic symptom disorders are commonly associated with complicated reactions to stress. These reactions are accompanied by muscle tension and pain. Relaxation can diminish the patient's perceptions of pain and reduce muscle tension. The distracters are modalities useful in treating selected anxiety disorders.

19. A patient reports, "The medicine prescribed to help me get to sleep worked well for about a month, but I don't have any more of those pills. Now my insomnia is worse than ever. I had nightmares the last 2 nights." Which type of medication did the health care provider most likely prescribe? a. Benzodiazepine b. Tricyclic antidepressant c. Conventional antipsychotic d. Central nervous system (CNS) stimulant

ANS: A Benzodiazepines, members of the hypnotic's family of medications, can worsen existing sleep disturbances when they are discontinued. This class of medications produces tolerance. Once the drug is discontinued, the individual may have rebound insomnia and nightmares. CNS stimulants worsen insomnia while they are in use. Tricyclic antidepressants and atypical antipsychotics may help insomnia but would not be used for initial therapy.

11. Which season would be most associated with increased periods of wakefulness in the general population? a. Summer b. Winter c. Spring d. Fall

ANS: A Circadian drive is associated with physiology. Light is the main exogenous factor that drives wakefulness. Days are longest in summer.

For a patient experiencing panic, which nursing intervention should be implemented first? a.Teach relaxation techniques. b.Administer an anxiolytic medication. c.Prepare to implement physical controls. d.Provide calm, brief, directive communication.

D Calm, brief, directive verbal interaction can help the patient gain control of overwhelming feelings and impulses related to anxiety. Patients experiencing panic-level anxiety are unable to focus on reality; thus, learning relaxation techniques is virtually impossible. Administering anxiolytic medication should be considered if providing calm, brief, directive communication is ineffective. Although the patient is disorganized, violence may not be imminent, ruling out the intervention of preparing for physical control until other less-restrictive measures are proven ineffective.

A nurse assesses a patient with a tentative diagnosis of generalized anxiety disorder. Which question would be most appropriate for the nurse to ask? a."Have you been a victim of a crime or seen someone badly injured or killed?" b."Do you feel especially uncomfortable in social situations involving people?" c."Do you repeatedly do certain things over and over again?" d."Do you find it difficult to control your worrying?"

D Patients with generalized anxiety disorder frequently engage in excessive worrying. They are less likely to engage in ritualistic behavior, fear social situations, or have been involved in a highly traumatic event.

A patient has acute anxiety related to an automobile accident 2 hours ago. The nurse should teach the patient about medication from which group? a. Tricyclic antidepressants b. Antipsychotic drugs c. Mood stabilizers d. Benzodiazepines

D Benzodiazepines provide anxiety relief. Tricyclic antidepressants are used to treat symptoms of depression. Mood stabilizers are used to treat bipolar disorder. Antipsychotic drugs are used to treat psychosis.

A nurse begins work in an agency that provides care to members of a minority ethnic population. The nurse will be better able to demonstrate cultural competence after a. identifying culture-bound issues. b. implementing scientifically proven interventions. c. correcting inferior health practices of the population. d. exploring commonly held beliefs and values of the population.

D Cultural competence is dependent on understanding the beliefs and values of members of a different culture. A nurse who works with an individual or group of a culture different from his or her own must be open to learning about the culture. The other options have little to do with cultural competence or represent only a portion of the answer.

Which Western cultural feature may result in establishing unrealistic outcomes for patients of other cultural groups? a. Interdependence b. Present orientation c. Flexible perception of time d. Direct confrontation to solve problems

D Directly confronting problems is a highly valued approach in the American culture but not part of many other cultures in which harmony and restraint are valued

A disheveled patient with severe depression and psychomotor retardation has not showered for several days. The nurse will: a. bring up the issue at the community meeting. b. calmly tell the patient, "You must bathe daily." c. avoid forcing the issue in order to minimize stress. d. firmly and neutrally assist the patient with showering.

D When patients are unable to perform self-care activities, staff must assist them rather than ignore the issue. Better grooming increases self-esteem. Calmly telling the patient to bathe daily and bringing up the issue at a community meeting are punitive.

A person was directing traffic on a busy street, rapidly shouting, "To work, you jerk, for perks" and making obscene gestures at cars. The person has not slept or eaten for 3 days. Which assessment findings will have priority concern for this patient's plan of care?

Hyperactivity; not eating and sleeping

Which statement made by a patient diagnosed with borderline personality disorder indicates the treatment plan is effective? a."I think you are the best nurse on the unit." b."I'm never going to get high on drugs again." c."I felt empty and wanted to hurt myself, so I called you." d."I hate my mother. I called her today, and she wasn't home."

I felt empty and wanted to hurt myself, so i called you

A client with a history of three failed engagements is concerned about being "too possessive." This concern supports a need for which type of therapy?

Interpersonal

Which statement best describes the DSM-5?

It is a medical psychiatric assessment system.

A patient says, "I get in trouble sometimes because I make quick decisions and act on them." Select the nurse's most therapeutic response. a."Let's consider the advantages of being able to stop and think before acting." b."It sounds as though you've developed some insight into your situation." c."I bet you have some interesting stories to share about overreacting." d."It's good that you're showing readiness for behavioral change."

Lets consider the advantages of being able to stop and think before acting

A nurse will prepare teaching materials for the parents of a child newly diagnosed with attention deficit hyperactivity disorder (ADHD). Which medication will the information focus on?* a. Paroxetine (Paxil) b. Imipramine (Tofranil) c. Methyphenidate (Ritalin) d. Carbamazepine (Tegretol)

Methyphenidate (Ritalin)

At a unit meeting, the staff discusses decor for a special room for patients with acute mania. Which suggestion is appropriate?

Neutral walls with pale, simple accessories

A tool the novice nurse might refer to when writing treatment results criteria is th Nursing Outcomes Classification (NOC).

Nursing Outcomes Classification (NOC). Correct The Nursing Outcomes Classification is a publication used as a resource across the United States.

For which patients diagnosed with personality disorders would a family history of similar problems be most likely? (Select all that apply.) a.Obsessive-compulsive b.Antisocial c.Borderline d. Schizotypal e.Narcissistic

OCD, antisocial, Borderline, schizotypical

What three structural components comprise a nursing diagnosis? Problem, etiology, supporting data

Problem, etiology, supporting data Correct

A desired outcome for a 12-year-old diagnosed with attention deficit hyperactivity disorder (ADHD) is to improve relationships with other children. Which treatment modality should the nurse suggest for the plan of care? a. Reality therapy b. Simple restitution c. Social skills group d. Insight-oriented group therapy

Social skills group

A nurse sits with a patient diagnosed with schizophrenia. The patient starts to laugh uncontrollably, although the nurse has not said anything funny. Select the nurse's most therapeutic response.

a. "Why are you laughing?" b. "Please share the joke with me." c. "I don't think I said anything funny." d. "You're laughing. Tell me what's happening." ANSWER: D

A patient diagnosed with schizophrenia exhibits little spontaneous movement and demonstrates waxy flexibility. Which patient needs are of priority importance?

a. Self-esteem b. Psychosocial c. Physiological d. Self-actualization ANSWER: C

The child prescribed an antipsychotic medication to manage violent behavior is one most likely diagnosed with: a. attention deficit hyperactivity disorder. b. posttraumatic stress disorder. c. communication disorder. d. an anxiety disorder.

attention deficit hyperactivity disorder.

The parent of a 6-year-old says, "My child is in constant motion and talks all the time. My child isn't interested in toys but is out of bed every morning before me." The child's behavior is most consistent with diagnostic criteria for: a. communication disorder. b. stereotypic movement disorder. c. intellectual development disorder. d. attention deficit hyperactivity disorder.

attention deficit hyperactivity disorder.

Outcome identification for the treatment plan of a patient experiencing grandiose thinking associated with acute mania will focus on:

distorted thought self-control.

Interviewer anxiety during an assessment interview is most likely to be a result of the client's perception of the interviewer's ability to help.

the client's perception of the interviewer's ability to help. Correct Whenever a client is in doubt about the helpfulness of the interviewer, anxiety is generated. The interviewer can "tune in" to the client's anxiety by empathy. REF: Page 116-117

The most likely factor to interfere with data collection in an initial assessment interview of an older adult is whether the client has any physical deficiencies.

whether the client has any physical deficiencies. Correct While all the options can interfere, the most prevalent one affecting the data collected is any physical and/or cognitive deficiencies that client may possess. REF: Page 122-123

The spouse of a patient diagnosed with bipolar disorder asks what evidence supports the possibility of genetic transmission of bipolar disorders. Which response should the nurse provide?

"A higher rate of relatives with bipolar disorder is found among patients with bipolar disorder."

A nurse assesses a patient who takes lithium. Which findings demonstrate evidence of complications?

Diaphoresis, weakness, and nausea

Four new patients were admitted to the behavioral health unit in the past 12 hours. The nurse directs a psychiatric technician to monitor these patients for safety. Which patient will need the most watchful supervision? A patient diagnosed with:

bipolar I disorder.

What is the priority nursing diagnosis for a patient diagnosed with antisocial personality disorder who has made threats against staff, ripped art off the walls, and thrown objects? a.Risk for other-directed violence b.Risk for self-directed violence c.Impaired social interaction d.Ineffective denial

risk for other directed violence

When a hyperactive patient diagnosed with acute mania is hospitalized, what is the initial nursing intervention?

Set limits on patient behavior as necessary.

1. A patient in good health and without any major health needs says, "I want to try some techniques to improve my mental and physical well-being but I'm overwhelmed by all the suggestions on the Internet." Which techniques would be appropriate for the nurse to suggest? (Select all that apply.) a. Yoga b. Exercise c. Meditation d. Aromatherapy e. Acupuncture f. Spinal manipulation

ANS: A, B, C, D Yoga, exercise, meditation, and aromatherapy are self-help techniques that may have a positive effect on the patient's physical and mental well-being. These techniques are unlikely to cause harm. The patient is in good health; therefore, acupuncture and spinal manipulation are not indicated.

8. Consider these health problems: Lewy body disease, frontal-temporal lobar degeneration, and Huntington's disease. Which term unifies these problems? a. Cyclothymia b. Dementia c. Delirium d. Amnesia

ANS: B The listed health problems are all forms of dementia.

A nurse cares for a first-generation American whose family emigrated from Germany. Which worldview about the source of knowledge would this patient likely have? a. Knowledge is acquired through use of affective or feeling senses. b. Science is the foundation of knowledge and proves something exists. c. Knowledge develops by striving for transcendence of the mind and body. d. Knowledge evolves from an individual's relationship with a supreme being.

B The European-American perspective of acquiring knowledge evolves from science. The distracters describe the beliefs of other cultural groups.

Which dinner menu is best suited for a patient with acute mania?

Broiled chicken breast on a roll, an ear of corn, and an apple

A patient asks the nurse, "I read an article online about psychosocial factors that influence depression. What are psychosocial factors?" Examples a nurse could cite to support the premise that a patient's depression may be influenced by psychosocial factors include: (Select all that apply) a. having a hostile and over involved family. b. having two first-degree relatives with bipolar disorder. c. feeling strong guilt over having an abortion when one's religion forbids it. d. experiencing the death of a parent a month before the onset of depression. e. experiencing symptom remission when treated with antidepressant medication.

A, C, & D Family influence is a psychosocial factor affecting a patient's mental health. A hostile, overinvolved family is critical of the patient and contributes to low self-esteem. Genetic factors influence an individual's risk for mental disorder but are not psychosocial factors. Religious influences are psychosocial in nature. Life experiences, especially crises and losses, are psychosocial influences on mental health. Treatment with a biological agent, such as antidepressant medication, is an example of a biological influence.

A nurse plans health teaching for a patient with generalized anxiety disorder who begins a new prescription for lorazepam (Ativan). What information should be included? Select all that apply. a.Caution in use of machinery b.Foods allowed on a tyramine-free diet c.The importance of caffeine restriction d.Avoidance of alcohol and other sedatives e.Take the medication on an empty stomach

A, C, D Caffeine is a central nervous system stimulant that acts as an antagonist to the benzodiazepine lorazepam. Daily caffeine intake should be reduced to the amount contained in one cup of coffee. Benzodiazepines are sedatives, thus the importance of exercising caution when driving or using machinery and the importance of not using other central nervous system depressants such as alcohol or sedatives to avoid potentiation. Benzodiazepines do not require a special diet. Food will reduce gastric irritation from the medication.

A nurse's neighbor says, "I saw a news story about a man without any known illness who died suddenly after his ex-wife committed suicide. Was that a coincidence, or can emotional shock be fatal?" The nurse should respond by noting that some serious medical conditions may be complicated by emotional stress, including: (select all that apply) a. cancer. b. hip fractures. c. hypertension. d. immune disorders. e. cardiovascular disease.

A, C, D, E A number of diseases can be worsened or brought to awareness by intense emotional stress. Immune disorders can be complicated associated with detrimental effects of stress on the immune system. Others can be brought about indirectly, such as cardiovascular disease due to acute or chronic hypertension. Hip fractures are not in this group.

A student nurse caring for a patient diagnosed with depression reads in the patient's medical record, "This patient shows vegetative signs of depression." Which nursing diagnoses most clearly relate to the vegetative signs? Select all that apply. a. Imbalanced nutrition: less than body requirements b. Chronic low self-esteem c. Sexual dysfunction d. Self-care deficit e. Powerlessness f. Insomnia

A, C, D, F Vegetative signs of depression are alterations in body processes necessary to support life and growth, such as eating, sleeping, elimination, and sexual activity. These diagnoses are more closely related to vegetative signs than diagnoses associated with feelings about self. See relationship to audience response question.

The nurse assesses an adult who is socially withdrawn and hoards. Which nursing diagnoses most likely apply to this individual? Select all that apply. a.Ineffective home maintenance b.Situational low self-esteem c.Chronic low self-esteem d.Disturbed body image e.Risk for injury

A, C, E Shame regarding the appearance of one's home is associated with hoarding. The behavior is usually associated with chronic low self-esteem. Hoarding results in problems of home maintenance, which may precipitate injury. The self-concept may be affected, but not body image.

A nurse prepares to administer a second-generation antipsychotic medication to a patient diagnosed with schizophrenia. Additional monitoring for adverse effects will be most important if the patient has which co-morbid health problems? (Select all that apply.) a. Parkinson's disease b. Grave's disease c. Hyperlipidemia d. Osteoarthritis e. Diabetes

A, C, E Antipsychotic medications may produce weight gain, which would complicate care of a patient with diabetes, and increase serum triglycerides, which would complicate care of a patient with hyperlipidemia. Parkinson's disease involves changes in transmission of dopamine and acetylcholine, so these drugs would also complicate care of this patient. Osteoarthritis and Grave's disease should have no synergistic effect with this medication.

The health care team at an inpatient psychiatric facility drafts these criteria for admission. Which criteria should be included in the final version of the admission policy? (Select all that apply) a. Clear risk of danger to self or others b. Adjustment needed for doses of psychotropic medication c. Detoxification from long-term heavy alcohol consumption needed d. Respite for caregivers of persons with serious and persistent mental illness e. Failure of community-based treatment, demonstrating need for intensive treatment

A, C, E Medication doses can be adjusted on an outpatient basis. The goal of caregiver respite can be accomplished without hospitalizing the patient. The other options are acceptable, evidence-based criteria for admission of a patient to an inpatient service.

Which comments by an elderly person best indicate successful completion of the individual's psychosocial developmental task? (Select all that apply.) a. "I am proud of my children's successes in life." b. "I should have given to community charities more often." c. "My relationship with my father made life more difficult for me." d. "My experiences in the war helped me appreciate the meaning of life." e. "I often wonder what would have happened if I had chosen a different career."

A, D The developmental crisis for an elderly person relates to integrity versus despair. Pride in one's offspring indicates a sense of fulfillment. Recognition of the wisdom gained from difficult experiences (such as being in a war) indicates a sense of integrity. Blaming and regret indicate despair and unsuccessful resolution of the crisis.

Which findings are signs of a person who is mentally healthy? (Select all that apply.) a. Says, "I have some weaknesses, but I feel I'm important to my family and friends." b. Adheres strictly to religious beliefs of parents and family of origin. c. Spends all holidays alone watching old movies on television. d. Considers past experiences when deciding about the future. e. Experiences feelings of conflict related to changing jobs.

A, D & E Mental health is a state of well being in which each individual is able to realize his or her own potential, cope with the normal stress of life, work productively and make a contribution to the community.

A patient diagnosed with a somatic symptom disorder says, "Why has God chosen me to be sick all the time and unable to provide for my family? The burden on my family is worse than the pain I bear." Which nursing diagnoses apply to this patient? Select all that apply. a. Spiritual distress b. Decisional conflict c. Adult failure to thrive d. Impaired social interaction e. Ineffective role performance

A, E The patient's verbalization is consistent with spiritual distress. The patient's description of being unable to provide for and burdening the family indicates ineffective role performance. No data support diagnoses of adult failure to thrive, impaired social interaction, or decisional conflict.

1. An older adult patient takes multiple medications daily. Over 2 days, the patient developed confusion, slurred speech, an unsteady gait, and fluctuating levels of orientation. These findings are most characteristic of: a. delirium. b. dementia c. amnestic syndrome. d. Alzheimer's disease.

ANS: A Delirium is characterized by an abrupt onset of fluctuating levels of awareness, clouded consciousness, perceptual disturbances, and disturbed memory and orientation. The onset of dementia or Alzheimer's disease, a type of dementia, is more insidious. Amnestic syndrome involves memory impairment without other cognitive problems.

28. An elderly person presents with symptoms of delirium. The family reports, "Everything was fine until yesterday." What is the most important assessment information for the nurse to gather? a. A list of all medications the person currently takes b. Whether the person has experienced any recent losses c. Whether the person has ingested aged or fermented foods d. The person's recent personality characteristics and changes

ANS: A Delirium is often the result of medication interactions or toxicity. The distracters relate to MAOI therapy and depression.

25. An older adult is prescribed digoxin (Lanoxin) and hydrochlorothiazide daily as well as lorazepam (Ativan) as needed for anxiety. Over 2 days, the patient developed confusion, slurred speech, an unsteady gait, and fluctuating levels of orientation. What is the most likely reason for the patient's change in mental status? a. Drug actions and interactions b. Benzodiazepine withdrawal c. Hypotensive episodes d. Renal failure

ANS: A Drug actions and interactions are common among elderly persons and predispose this population to delirium. Delirium is characterized by an abrupt onset of fluctuating levels of awareness, clouded consciousness, perceptual disturbances, and disturbed memory and orientation. The patient takes lorazepam on a PRN basis, so withdrawal is unlikely. Hypotensive episodes or problems with renal function may occur associated with the patient's drug regime, but interactions are more likely the problem.

14. A patient has progressive memory deficits associated with dementia. Which nursing intervention would best help the individual function in the environment? a. Assist the patient to perform simple tasks by giving step-by-step directions. b. Reduce frustration by performing activities of daily living for the patient. c. Stimulate intellectual function by discussing new topics with the patient. d. Read one story from the newspaper to the patient every day.

ANS: A Patients with cognitive impairment should perform all tasks of which they are capable. When simple directions are given in a systematic fashion, the patient is better able to process information and perform simple tasks. Stimulating intellectual functioning by discussing new topics is likely to prove frustrating for the patient. Patients with cognitive deficits may enjoy the attention of someone reading to them, but this activity does not promote their function in the environment.

26. A hospitalized patient diagnosed with delirium misinterprets reality, while a patient diagnosed with dementia wanders about the home. Which outcome is the priority in both scenarios? The patients will: a. remain safe in the environment. b. participate actively in self-care. c. communicate verbally. d. acknowledge reality.

ANS: A Risk for injury is the nurse's priority concern. Safety maintenance is the desired outcome. The other outcomes are lower priorities and may not be realistic.

21. An older adult with moderately severe dementia forgets where the bathroom is and has episodes of incontinence. Which intervention should the nurse suggest to the patient's family? a. Label the bathroom door. b. Take the older adult to the bathroom hourly. c. Place the older adult in disposable adult briefs. d. Limit the intake of oral fluids to 1000 ml per day.

ANS: A The patient with moderately severe dementia has memory loss that begins to interfere with activities. This patient may be able to use environmental cues such as labels on doors to compensate for memory loss. Regular toileting may be helpful, but a 2-hour schedule is often more reasonable. Placing the patient in disposable briefs is more appropriate at a later stage. Severely limiting oral fluid intake would predispose the patient to a urinary tract infection

4. What is the priority nursing diagnosis for a patient with fluctuating levels of consciousness, disturbed orientation, and visual and tactile hallucinations? a. Risk for injury related to altered cerebral function, fluctuating levels of consciousness, disturbed orientation, and misperception of the environment b. Bathing/hygiene self-care deficit related to cerebral dysfunction, as evidenced by confusion and inability to perform personal hygiene tasks c. Disturbed thought processes related to medication intoxication, as evidenced by confusion, disorientation, and hallucinations d. Fear related to sensory perceptual alterations as evidenced by visual and tactile hallucinations

ANS: A The physical safety of the patient is of highest priority among the diagnoses given. Many opportunities for injury exist when a patient misperceives the environment as distorted, threatening, or harmful or when the patient exercises poor judgment or when the patient's sensorium is clouded. The other diagnoses may be concerns, but are lower priorities.

20. A patient in a support group says, "I'm tired of being sick. Everyone always helps me, but I will be glad when I can help someone else." This statement reflects a. altruism. b. universality. c. cohesiveness. d. corrective recapitulation.

ANS: A Altruism refers to the experience of being helpful or useful to others, a condition that the patient anticipates will happen. The other options are also therapeutic factors identified by Yalom.

9. The nurse who is counseling a patient with dissociative identity disorder should understand that the assessment of highest priority is a. risk for self-harm. b. cognitive function. c. memory impairment. d. condition of self-esteem.

ANS: A Assessments that relate to patient safety take priority. Patients with dissociative disorders may be at risk for suicide or self-mutilation, so the nurse must be alert for indicators of risk for self-injury. The other options are important assessments but rank below safety. Treatment motivation, while an important consideration, is not necessarily a part of the nursing assessment.

2. A patient who has been diagnosed with depression is scheduled for cognitive therapy in addition to receiving prescribed antidepressant medication. The nurse understands that the goal of cognitive therapy will be met when what is reported by the patient? a. "I will tell myself that I am a good person when things don't go well at work." b. "My medications will make my problems go away." c. "My family will help take care of my children while I am in the hospital." d. "This therapy will improve my response to neurotransmitter impulses."

ANS: A Cognitive therapy helps patients restructure their patterns of thinking to various events or thoughts in a more healthy way. Medication alters neurotransmitters but does not make problems go away. Family support is important but is not the goal of cognitive therapy. Neurotransmitters are affected by medication and brain stimulation therapy, not by cognitive therapy.

25. Relaxation techniques help patients who have experienced major traumas because they a. engage the parasympathetic nervous system. b. increase sympathetic stimulation. c. increase the metabolic rate. d. release hormones.

ANS: A In response to trauma, the sympathetic arousal symptoms of rapid heart rate and rapid respiration prepare the person for flight or fight responses. Afterward, the dorsal vagal response damps down the sympathetic nervous system. This is a parasympathetic response with the heart rate and respiration slowing down and decreasing the blood pressure. Relaxation techniques promote activity of the parasympathetic nervous system.

21. The feeling experienced by a patient that should be assessed by the nurse as most predictive of elevated suicide risk is a. hopelessness. b. sadness. c. elation. d. anger.

ANS: A Of the feelings listed, hopelessness is most closely associated with increased suicide risk. Depression, aggression, impulsivity, and shame are other feelings noted as risk factors for suicide.

13. A nurse explains to the family of a mentally ill patient how a nurse-patient relationship differs from social relationships. Which is the best explanation? a. "The focus is on the patient. Problems are discussed by the nurse and patient, but solutions are implemented by the patient." b. "The focus shifts from nurse to patient as the relationship develops. Advice is given by both, and solutions are implemented." c. "The focus of the relationship is socialization. Mutual needs are met, and feelings are shared openly." d. "The focus is creation of a partnership in which each member is concerned with growth and satisfaction of the other."

ANS: A Only the correct response describes elements of a therapeutic relationship. The remaining responses describe events that occur in social or intimate relationships

22. A nurse leads a psychoeducational group for patients in the community diagnosed with schizophrenia. A realistic outcome for group members is that they will a. discuss ways to manage their illness. b. develop a high level of trust and cohesiveness. c. understand unconscious motivation for behavior. d. demonstrate insight about development of their illness.

ANS: A Patients with schizophrenia almost universally have problems associated with everyday living in the community, so discussing ways to manage the illness would be an important aspect of psychoeducation. Discussing concerns about daily life would be a goal to which each could relate. Developing trust and cohesion is desirable but is not the priority outcome of a psychoeducational group. Understanding unconscious motivation would not be addressed. Insight would be difficult for a patient with residual schizophrenia because of the tendency toward concrete thinking.

20. A soldier in a combat zone tells the nurse, "I saw a child get blown up over a year ago, and I still keep seeing bits of flesh everywhere. I see something red, and the visions race back to my mind." Which phenomenon associated with PTSD is the soldier describing? a. Reexperiencing b. Hyperarousal c. Avoidance d. Psychosis

ANS: A Spontaneous or cued recurrent, involuntary, and intrusive distressing memories of the traumatic events are often associated with PTSD. The soldier has described intrusive thoughts and visions associated with reexperiencing the traumatic event. This description does not indicate psychosis, hypervigilance, or avoidance.

22. As a nurse discharges a patient, the patient gives the nurse a card of appreciation made in an arts and crafts group. What is the nurse's best action? a. Recognize the effectiveness of the relationship and patient's thoughtfulness. Accept the card. b. Inform the patient that accepting gifts violates policies of the facility. Decline the card. c. Acknowledge the patient's transition through the termination phase but decline the card. d. Accept the card and invite the patient to return to participate in other arts and crafts groups.

ANS: A The nurse must consider the meaning, timing, and value of the gift. In this instance, the nurse should accept the patient's expression of gratitude. See relationship to audience response question.

9. A college student who attempted suicide by overdose was hospitalized. When the parents were contacted, they responded, "We should have seen this coming. We did not do enough." The parents' reaction reflects a. guilt. b. denial. c. shame. d. rescue feelings.

ANS: A The parents' statements indicate guilt. Guilt is evident from the parents' self-chastisement. The feelings suggested in the distracters are not clearly described in the scenario.

9. At what point in the nurse-patient relationship should a nurse plan to first address termination? a. During the orientation phase b. At the end of the working phase c. Near the beginning of the termination phase d. When the patient initially brings up the topic

ANS: A The patient has a right to know the conditions of the nurse-patient relationship. If the relationship is to be time-limited, the patient should be informed of the number of sessions. If it is open-ended, the termination date will not be known at the outset, and the patient should know that the issue will be negotiated at a later date. The nurse is responsible for bringing up the topic of termination early in the relationship, usually during the orientation phase.

8. Which scenario demonstrates a dissociative fugue? a. After being caught in an extramarital affair, a man disappeared but then reappeared months later with no memory of what occurred while he was missing. b. A man is extremely anxious about his problems and sometimes experiences dazed periods of several minutes passing without conscious awareness of them. c. A woman finds unfamiliar clothes in her closet, is recognized when she goes to new restaurants, and complains of "blackouts" despite not drinking. d. A woman reports that when she feels tired or stressed, it seems like her body is not real and is somehow growing smaller.

ANS: A The patient in a dissociative fugue state relocates and lacks recall of his life before the fugue began. Often fugue states follow traumatic experiences and sometimes involve assuming a new identity. Such persons at some point find themselves in their new surroundings, unable to recall who they are or how they got there. A feeling of detachment from one's body or from the external reality is an indication of depersonalization disorder. Losing track of several minutes when highly anxious is not an indication of a dissociative disorder and is common in states of elevated anxiety. Finding evidence of having bought clothes or gone to restaurants without any explanation for these is suggestive of dissociative identity disorder, particularly when periods are "lost" to the patient (blackouts).

23. Which assessment finding best supports dissociative fugue? The patient states a. "I cannot recall why I'm living in this town." b. "I feel as if I'm living in a fuzzy dream state." c. "I feel like different parts of my body are at war." d. "I feel very anxious and worried about my problems."

ANS: A The patient in a fugue state frequently relocates and assumes a new identity while not recalling previous identity or places previously inhabited. The distracters are more consistent with depersonalization disorder, generalized anxiety disorder, or dissociative identity disorder.

24. After major reconstructive surgery, a patient's wounds dehisced. Extensive wound care was required for 6 months, causing the patient to miss work and social activities. Which physiological response would be expected for this patient? a. Vital signs return to normal. b. Release of endogenous opioids would cease. c. Pulse and blood pressure readings are elevated. d. Psychomotor abilities of the right brain become limited.

ANS: A The scenario presents chronic and potentially debilitating stress. The helpless and out of control feelings produce pathophysiological changes. Unmyelinated ventral vagus responses initially result in rapid heart rate and respiration. After many hours, days, or months the body cannot sustain this state, so the dorsal vagal response dampens the sympathetic nervous system. This parasympathetic response results in the heart rate and respiration slowing down and a decrease in blood pressure. Individuals with dissociative disorders have altered communication between higher and lower brain structures due to the massive release of endogenous opioids at the time of severe threat.

26. A nurse says, "I am the only one who truly understands this patient. Other staff members are too critical." The nurse's statement indicates a. boundary blurring. b. sexual harassment. c. positive regard. d. advocacy.

ANS: A When the role of the nurse and the role of the patient shift, boundary blurring may arise. In this situation the nurse is becoming overinvolved with the patient as a probable result of unrecognized countertransference. When boundary issues occur, the need for supervision exists. The situation does not describe sexual harassment. Data are not present to suggest positive regard or advocacy.

24. A patient approaches the nurse in the clinic waiting room and says, "I want to talk to you about a sexual matter." The nurse can best facilitate the discussion by a. saying, "Let's go my office." b. responding, "I want to help. Go ahead; I'm listening." c. telling the patient, "Let's schedule another appointment." d. offering to sit in a corner of the waiting room with the patient.

ANS: A A discussion of sexual concerns requires privacy. Suggesting use of office space is preferable to using the waiting room, where others cannot help but overhear sensitive material. The distracters block communication.

1. Which behavior best demonstrates aggression? a. Stomping away from the nurses' station, going to the hallway, and grabbing a tray from the meal cart. b. Bursting into tears, leaving the community meeting, and sitting on a bed hugging a pillow and sobbing. c. Telling the primary nurse, "I felt angry when you said I could not have a second helping at lunch." d. Telling the medication nurse, "I am not going to take that, or any other, medication you try to give me."

ANS: A Aggression is harsh physical or verbal action that reflects rage, hostility, and potential for physical or verbal destructiveness. Aggressive behavior violates the rights of others. Refusing medication is a patient's right and may be appropriate. The other incorrect options do not feature violation of another's rights.

7. A rape victim says to the nurse, "I always try to be so careful. I know I should not have walked to my car alone. Was this attack my fault?" Which communication by the nurse is most therapeutic? a. Support the victim to separate issues of vulnerability from blame. b. Emphasize the importance of using a buddy system in public places. c. Reassure the victim that the outcome of the situation will be positive. d. Pose questions about the rape and help the patient explore why it happened.

ANS: A Although the victim may have made choices that made her vulnerable, she is not to blame for the rape. Correcting this distortion in thinking allows the victim to begin to restore a sense of control. This is a positive response to victimization. The distracters do not permit the victim to begin to restore a sense of control or offer use of nontherapeutic communication techniques. In this interaction, the victim needs to talk about feelings rather than prevention.

26. Emergency response workers arrive in a community after a large-scale natural disaster. What is the workers' first action? a. Report to the incident command system (ICS) center. b. Determine whether the community is safe. c. Establish teams of workers with varied skills. d. Evaluate actions completed by local law enforcement.

ANS: A An ICS provides a common organizational structure facilitating an immediate response. It establishes a clear chain of command that supports the coordination of personnel and equipment at an event site. The incorrect responses describe actions that may or may not be taken by the ICS.

11. A patient tells the nurse, "My husband lost his job. He's abusive only when he drinks too much. His family was like that when he was growing up. He always apologizes and regrets hurting me." What risk factor was most predictive for the husband to become abusive? a. History of family violence b. Loss of employment c. Abuse of alcohol d. Poverty

ANS: A An abuse-prone individual is an individual who has experienced family violence and was often abused as a child. This phenomenon is part of the cycle of violence. The other options may be present but are not as predictive.

23. A 10-year-old boy is diagnosed with gender dysphoria. Which assessment finding would the nurse expect? a. Having tea parties with dolls b. A compromised sexual response cycle c. Identifying with boys who are athletic d. Intense urges to watch his parents have sex

ANS: A An individual with gender dysphoria feels at odds with the roles associated with that gender. A child with this diagnosis is likely to engage in play associated with the opposite gender. The other options are not age appropriate or characteristically seen in children with gender dysphoria.

11. Which is an effective nursing intervention to assist an angry patient learn to manage anger without violence? a. Help a patient identify a thought that produces anger, evaluate the validity of the belief, and substitute reality-based thinking. b. Provide negative reinforcement such as restraint or seclusion in response to angry outbursts, whether or not violence is present. c. Use aversive conditioning, such as popping a rubber band on the wrist, to help extinguish angry feelings. d. Administer an antipsychotic or antianxiety medication.

ANS: A Anger has a strong cognitive component, so using cognition techniques to manage anger is logical. The incorrect options do nothing to help the patient learn anger management.

21. The principle most useful to a nurse planning crisis intervention for any patient is that the patient a. is experiencing a state of disequilibrium. b. is experiencing a type of mental illness. c. poses a threat of violence to others. d. has high potential for self-injury.

ANS: A Disequilibrium is the only answer universally true for all patients in crisis. A crisis represents a struggle for equilibrium when problems seem unsolvable. Crisis does not reflect mental illness. Potential for self-violence or other-directed violence may or may not be a factor in crisis.

13. A nurse is anxious about assessing the sexual history of a patient who is considerably older than the nurse is. Which statement would be most appropriate for obtaining information about the patient's sexual practices? a. "Some people are not sexually active, others have a partner, and some have several partners. What has been your pattern?" b. "Sexual health can reflect a number of medical problems, so I'd like to ask if you have any sexual problems you think we should know about." c. "It's your own business, of course, but it might be helpful for us to have some information about your sexual history. Could you tell me about that, please?" d. "I would appreciate it if you could share your sexual history with me so I can share it with your healthcare provider. It might be helpful in planning your treatment."

ANS: A Explaining that sexual practices vary helps reduce patient anxiety about the topic by normalizing the full range of sexual practices so that whatever his situation, the patient can feel comfortable sharing it. "It's your business of course ..." implies the nurse does not have a valid reason to seek the information and in effect suggests that the patient perhaps should not answer the question. "It might be helpful ..." makes the information seem less valid or important for the nurse to pursue and, again, could discourage the patient from responding fully. Asking if the patient has any sexual problems that staff should know about is not unprofessional, but it is a very broad question that may increase a patient's uncertainty about what the nurse wants to hear, thus increasing his anxiety. Defining or giving an example of "sexual problem" would make this inquiry more effective.

17. A nurse plans health education for a patient who will be receiving warfarin for several weeks after knee-replacement surgery. Which substance should the nurse caution the patient to avoid? a. Fish oil b. Black cohosh c. Lavender d. Mandarin

ANS: A Fish oil may increase bleeding time and therefore has a potentially hazardous interaction with the anticoagulant warfarin. Black cohosh is an herbal treatment for hot flashes. Mandarin and lavender may have calming effects, which may be helpful, but would not cause increased risk of bleeding.

14. Which nursing intervention has the highest priority for a patient diagnosed with bulimia nervosa? a. Assist the patient to identify triggers to binge eating. b. Provide corrective consequences for weight loss. c. Assess for signs of impulsive eating. d. Explore needs for health teaching.

ANS: A For most patients with bulimia nervosa, certain situations trigger the urge to binge; purging then follows. Often the triggers are anxiety-producing situations. Identification of triggers makes it possible to break the binge-purge cycle. Because binge eating and purging directly affect physical status, the need to promote physical safety assumes highest priority.

12. A nurse provides care for an adolescent patient diagnosed with an eating disorder. Which behavior by this nurse indicates that additional clinical supervision is needed? a. The nurse interacts with the patient in a protective fashion. b. The nurse's comments to the patient are compassionate and nonjudgmental. c. The nurse teaches the patient to recognize signs of increasing anxiety and ways to intervene. d. The nurse refers the patient to a self-help group for individuals with eating disorders.

ANS: A In the effort to motivate the patient and take advantage of the decision to seek help and be healthier, the nurse must take care not to cross the line toward authoritarianism and assumption of a parental role. Protective behaviors are part of the parent's role. The helpful nurse uses a problem-solving approach and focuses on the patient's feelings of shame and low self-esteem. Referring a patient to a self-help group is an appropriate intervention.

9. The staff development coordinator plans to teach use of physical management techniques for use when patients become assaultive. Which topic should the coordinator emphasize? a. Practice and teamwork b. Spontaneity and surprise c. Caution and superior size d. Diversion and physical outlets

ANS: A Intervention techniques are learned behaviors and must be practiced to be used in a smooth, organized fashion. Every member of the intervention team should be assigned a specific task to carry out before beginning the intervention. The other options are useless if the staff does not know how to use physical techniques and how to apply them in an organized fashion.

26. The treatment team discusses adding a new prescription for lisdexamfetamine dimesylate to the plan of care for a patient diagnosed with binge eating disorder. Which finding from the nursing assessment is most important for the nurse to share with the team? a. The patient's history of poly-substance abuse b. The patient's preference for homeopathic remedies c. The patient's family history of autoimmune disorders d. The patient's comorbid diagnosis of a learning disability

ANS: A Lisdexamfetamine dimesylate is designed to suppress the appetite and presents a risk for abuse. The patient with a history of substance abuse is at risk to abuse this medication as well. The patient's preference for homeopathic remedies is a consideration, but the history of substance abuse has a higher priority. Lisdexamfetamine dimesylate is commonly used to treat attention deficit hyperactivity disorder rather than learning disabilities. A history of autoimmune disorders in the family is irrelevant.

6. A patient who was responding to auditory hallucinations earlier in the morning now approaches the nurse shaking a fist and shouts, "Back off!" and then goes to the dayroom. While following the patient into the dayroom, the nurse should a. make sure there is adequate physical space between the nurse and patient. b. move into a position that places the patient close to the door. c. maintain one arm's length distance from the patient. d. begin talking to the patient about appropriate behavior.

ANS: A Making sure space is present between the nurse and the patient avoids invading the patient's personal space. Personal space needs increase when a patient feels anxious and threatened. Allowing the patient to block the nurse's exit from the room may result in injury to the nurse. Closeness may be threatening to the patient and provoke aggression. Sitting is inadvisable until further assessment suggests the patient's aggression is abating. One arm's length is inadequate space.

8. Which person would be most likely to experience sleep fragmentation? a. An obese adult b. A toddler who attends day care c. A person diagnosed with mild osteoarthritis d. An adolescent diagnosed with anorexia nervosa

ANS: A Obese adults experience more disruption of sleep stages, resulting in fragmentation. Obesity is the leading factor for obstructive sleep apnea, which causes sleep fragmentation. These changes are also associated with illness and some medications. The changes are evident on a hypnogram. An adolescent with anorexia nervosa would have a low body weight and therefore decreased risk for sleep fragmentation. Persons with arthritis have pain that may sometimes interrupt sleep, but it would not have as high risk as would obesity. Toddlers do not generally experience sleep fragmentation.

18. An adult seeks treatment for urges involving sexual contact with children. The adult has not acted on these urges but feels shame. Which finding best indicates that this adult is making progress in treatment? The adult a. consistently avoids schools and shops at malls only during school hours. b. indicates sexual drive and enjoyment from sex have decreased. c. reports an active and satisfying sex life with an adult partner. d. volunteers to become a scout troop leader.

ANS: A One strategy for avoiding acting on inappropriate urges is to avoid environments and circumstances that evoke those urges; for a pedophile this would include avoiding all situations that would likely result in contact with children. Pedophilic disorder is persistent; elimination of fantasies about children would be unrealistic. A person who volunteers to lead a scout troop is placing himself/herself around children. A diminished sex drive or a healthy sex life with an appropriate partner does not necessarily reduce the desire for sexual contact with children.

3. An adult experienced a myocardial infarction six months ago. At a follow-up visit, this adult says, "I haven't had much interest in sex since my heart attack. I finished my rehabilitation program, but having sex strains my heart. I don't know if my heart is strong enough." Which nursing diagnosis applies? a. Deficient knowledge related to faulty perception of health status b. Disturbed self-concept related to required lifestyle changes c. Disturbed body image related to treatment side effects d. Sexual dysfunction related to self-esteem disturbance

ANS: A Patients who have had a myocardial infarction often believe sexual intercourse will cause another heart attack. The patient has completed the rehabilitation, but education is needed regarding sexual activity. These patients should receive information about when sexual activity may begin, positions that conserve energy, and so forth. The scenario does not suggest self-concept or body image disturbance.

15. One bed is available on the inpatient eating-disorder unit. Which patient should be admitted to this bed? The patient whose weight decreased from a. 150 to 100 pounds over a 4-month period. Vital signs are temperature, 35.9 C; pulse, 38 beats/min; blood pressure 60/40 mm Hg b. 120 to 90 pounds over a 3-month period. Vital signs are temperature, 36 C; pulse, 50 beats/min; blood pressure 70/50 mm Hg c. 110 to 70 pounds over a 4-month period. Vital signs are temperature 36.5 C; pulse, 60 beats/min; blood pressure 80/66 mm Hg d. 90 to 78 pounds over a 5-month period. Vital signs are temperature, 36.7 C; pulse, 62 beats/min; blood pressure 74/48 mm Hg

ANS: A Physical criteria for hospitalization include weight loss of more than 30% of body weight NURSINGTB.COM within 6 months, temperature below 36 C (hypothermia), heart rate less than 40 beats/min, and systolic blood pressure less than 70 mm Hg.

27. A 7-year-old child was diagnosed with pica. Which assessment finding would the nurse expect associated with this diagnosis? a. The child frequently eats newspapers and magazines. b. The child refuses to eat peanut butter and jelly sandwiches. c. The child often rechews and reswallows foods at mealtimes. d. The parents feed the child clay because of concerns about anemia.

ANS: A Pica refers to eating nonfood items after maturing past toddlerhood. Some cultures practice eating nonfood items; however, this factor is a cultural preference rather than a disorder. Refusing to eat peanut butter and jelly sandwiches is an example of a simple food preference in a child. Rumination refers to regurgitation with rechewing, reswallowing, or spitting.

20. Physical assessment of a patient diagnosed with bulimia often reveals a. prominent parotid glands. b. peripheral edema. c. thin, brittle hair. d. 25% underweight.

ANS: A Prominent parotid glands are associated with repeated vomiting. The other options are signs of anorexia nervosa and not usually seen in bulimia.

20. A patient experiencing primary insomnia asks the nurse, "I take a nap during the day. Doesn't that make up for a lost night's sleep?" Select the nurse's best reply. a. "Circadian drives give daytime naps a structure different from nighttime sleep." b. "The body clock operates on a 24-hour cycle, making nap effectiveness unpredictable." c. "It is a matter of habit and expectation. We expect to be more refreshed from a night's sleep." d. "Sleep restores homeostasis but works more efficiently when aided by melatonin secreted at night."

ANS: A Regular sleep cycles occur with nighttime sleep, with progression through two distinct physiological states: four stages of NREM and a period of REM sleep. Naps often contain different amounts of REM sleep, thus changing the physiology of sleep as well as the psychological and behavioral effects of sleep.

13. After treatment for a detached retina, a survivor of intimate partner abuse says, "My partner only abuses me when I make mistakes. I've considered leaving, but I was brought up to believe you stay together, no matter what happens." Which diagnosis should be the focus of the nurse's initial actions? a. Risk for injury related to physical abuse from partner b. Social isolation related to lack of a community support system c. Ineffective coping related to uneven distribution of power within a relationship d. Deficient knowledge related to resources for escape from an abusive relationship

ANS: A Risk for injury is the priority diagnosis because the partner has already inflicted physical injury during violent episodes. The other diagnoses are applicable, but the nurse must first address the patient's safety.

3. The nurse provides health education for an adult experiencing sleep deprivation. Which instruction has the highest priority? a. "It's important to limit your driving to short periods. Sleep deprivation increases your risks for serious accidents." b. "Sleep deprivation is usually self-limiting. See your health care provider if it lasts more than a year." c. "Turn the radio on with a soft volume as you prepare for bed each evening. It will help you relax." d. "Three glasses of wine each evening helps many patients who suffer from sleep deprivation."

ANS: A Safety is the highest priority for this patient. Sleep deprivation causes psychomotor deficits. Driver drowsiness and fatigue lead to many automobile injuries and fatalities. Alcohol compounds problems associated with sleep deprivation. Sleep deprivation should be evaluated and treated; a 1-year delay is too long.

5. The parents of a 15-year-old seek to have this teen declared a delinquent because of excessive drinking, habitually running away, and prostitution. The nurse interviewing the patient should recognize these behaviors often occur in adolescents who a. have been abused. b. are attention seeking. c. have eating disorders. d. are developmentally delayed.

ANS: A Self-mutilation, alcohol and drug abuse, bulimia, and unstable and unsatisfactory relationships are frequently seen in teens who are abused. These behaviors are not as closely aligned with any of the other options.

5. While conducting the initial interview with a patient in crisis, the nurse should a. speak in short, concise sentences. b. convey a sense of urgency to the patient. c. be forthright about time limits of the interview. d. let the patient know the nurse controls the interview.

ANS: A Severe anxiety narrows perceptions and concentration. By speaking in short concise sentences, the nurse enables the patient to grasp what is being said. Conveying urgency will increase the patient's anxiety. Letting the patient know who controls the interview or stating that time is limited is nontherapeutic.

2. A nurse is performing an assessment for a 59-year-old man with a long history of hypertension. What is the rationale for including questions about prescribed medications and their effects on sexual function in the assessment? a. Sexual dysfunction may result from use of prescription medications for management of hypertension. b. Such questions are an indirect way of learning about the patient's medication adherence. c. These questions ease the transition to questions about sexual practices in general. d. Sexual dysfunction can cause stress and contribute to increased blood pressure.

ANS: A Some of the drugs used to treat hypertension can interfere with normal sexual functioning and lead to sexual disorders. Hypertension itself can lead to acquired erectile dysfunction. It would not be appropriate or necessary to use such inquiries as a lead-in to other sexual health topics. Sexual dysfunction, while stressful, does not cause hypertension.

15. A victim of intimate partner violence comes to the crisis center seeking help. Crisis intervention strategies the nurse applies will focus on a. supporting emotional security and reestablishing equilibrium. b. long-term resolution of issues precipitating the crisis. c. promoting growth of the individual. d. providing legal assistance.

ANS: A Strategies of crisis intervention address the immediate cause of the crisis and restoration of emotional security and equilibrium. The goal is to return the individual to the precrisis level of function. Crisis intervention is, by definition, short term. The correct response is the most global answer. Promoting growth is a focus of long-term therapy. Providing legal assistance might or might not be applicable.

20. An adult has cared for a debilitated parent for 10 years. The health care provider recently recommended transfer of the parent to a skilled nursing facility. The adult says, "I've always been able to care for my parents. Nursing home placement goes against everything I believe." Successful resolution of this adult's crisis will most closely relate to a. resolving the feelings associated with the threat to the person's self-concept. b. ability of the person to identify situational supports in the community. c. reliance on assistance from role models within the person's culture. d. mobilization of automatic relief behaviors by the person.

ANS: A The adult's crisis clearly relates to a loss of (or threatened change in) self-concept. Her capacity to care for her parents, regardless of the parent's condition, has been challenged. Crisis resolution will involve coming to terms with the feelings associated with this loss. Identifying situational supports is relevant, but less so than coming to terms with the threat to self-concept. Reliance on lessons from role models can be helpful but not the primary factor associated with resolution in this case. Automatic relief behaviors include withdrawal or flight and will not be helpful. Automatic relief behaviors are part of the third phase of crisis.

8. An 11-year-old says, "My parents don't like me. They call me stupid and say they wish I were never born. It doesn't matter what they think because I already know I'm dumb." Which nursing diagnosis applies to this child? a. Chronic low self-esteem related to negative feedback from parents b. Deficient knowledge related to interpersonal skills with parents c. Disturbed personal identity related to negative self-evaluation d. Complicated grieving related to poor academic performance

ANS: A The child has indicated a belief in being too dumb to learn. The child receives negative and demeaning feedback from the parents. The child has internalized these messages, resulting in a low self-esteem. Deficient knowledge refers to knowledge of health care measures. Disturbed personal identity refers to an alteration in the ability to distinguish between self and nonself. Grieving may apply, but a specific loss is not evident in the scenario. Low self-esteem is more relevant to the child's statements.

19. During the initial interview at the crisis center, a patient says, "I've been served with divorce papers. I'm so upset and anxious that I can't think clearly." Which comment should the nurse use to assess personal coping skills? a. "In the past, how have you handled difficult or stressful situations?" b. "What would you like us to do to help you feel more relaxed?" c. "Tell me more about how it feels to be anxious and upset." d. "Can you describe your role in the marital relationship?"

ANS: A The correct answer is the only option that assesses coping skills. The incorrect options are concerned with self-esteem, ask the patient to decide on treatment at a time when he or she "cannot think clearly," and seek to explore issues tangential to the crisis.

1. Which comment by the nurse would best support relationship building with a survivor of intimate partner abuse? a. "You are feeling violated because you thought you could trust your partner." b. "I'm here for you. I want you to tell me about the bad things that happened to you." c. "I was very worried about you. I knew you were living in a potentially violent situation." d. "Abusers often target people who are passive. I will refer you to an assertiveness class."

ANS: A The correct option uses the therapeutic technique of reflection. It shows empathy, an important nursing attribute for establishing rapport and building a relationship. None of the other options would help the patient feel accepted.

12. Normally, most people sleep at night. What is the physiological rationale? a. The master biological clock responds to darkness with sleep. b. Darkness stimulates histamine release, which promotes sleep. c. Cooler environmental temperatures stimulate retinal messages. d. Stimulation of the sympathetic nervous system promotes sleep.

ANS: A The master biological clock in the suprachiasmatic nucleus (SCN) of the hypothalamus regulates sleep as well as other physiological processes. Darkness cues the clock for sleep. Light cues it for wakefulness. Light stimulates retinal messages. Histamine release is associated with wakefulness. Stimulation of the sympathetic nervous system promotes alertness.

7. A patient reports good results from taking an herb to manage migraine headache pain. The nurse confirms there are no hazardous interactions between the herb and the patient's current prescription drugs. Select the nurse's best comment to the patient. a. "Thanks for telling me. I'll make a note in your medical record that you take it." b. "You are experiencing a placebo effect. When we believe something will help, it usually does." c. "Self-management of health problems can be dangerous. You should have notified me sooner." d. "Research studies show that herbals actually increase migraine pain by inflaming nerve cells in the brain."

ANS: A The nurse should reinforce the patient for reporting use of the herb. Many patients keep secrets about use of alternative therapies. If it poses no danger, the nurse can document the use. The patient may also get placebo effect from the herb, but it is not necessary for the nurse to point out that information. The distracters are judgmental and may discourage the patient from openly sharing in the future.

15. A parent who is very concerned about a 3-year-old son says, "He likes to play with girls' toys. Do you think he is homosexual or mentally ill?" Which response by the nurse most professionally describes the current understanding of gender identity? a. "A child's interest in the activities of the opposite gender is not unusual or related to sexuality. Most children do not carry cross-gender interests into adulthood." b. "It's difficult to say for sure because the research is incomplete so far, but chances are that he will grow up to be a normal adult." c. "The research is incomplete, but many boys play with girls' toys and turn out normal as adults." d. "I am sure that whatever happens, he will be a loving son, and you will be a proud parent."

ANS: A The parent's inquiry is representing two questions: (1) whether the child's behavior suggests an increased risk of developing mental illness and (2) what the child's future sexual preference will be. The psychiatric disorder that most directly addresses gender preferences and cross-gender activities is gender identity disorder. Pointing out that cross-gender activities are not necessarily related to gender identity and not likely to be carried into adulthood is supported by current research. Saying the child will grow up to be "normal" implies that to be homosexual is to be abnormal, which reflects a cultural perspective that most professionals would believe to be inappropriate to share in a professional setting. Research provides information about the relationship between cross-gender interests in childhood and adulthood, so a comment that "research is incomplete" is not entirely accurate. Stating that the child is a wonderful boy the father will be proud of, whatever happens, evades the parent's question and suggests that parental bonds should not be affected by gender issues. The nurse has a professional obligation to maintain an objective, therapeutic relationship.

17. An older adult with Lewy body dementia lives with family. After observing multiple bruises, the home health nurse talked with the daughter, who became defensive and said, "My mother often wanders at night. Last night she fell down the stairs." Which nursing diagnosis has priority? a. Risk for injury related to poor judgment, cognitive impairments, and inadequate supervision b. Wandering related to confusion and disorientation as evidenced by sleepwalking and falls c. Chronic confusion related to degenerative changes in brain tissue as evidenced by nighttime wandering d. Insomnia related to sleep disruptions associated with cognitive impairment as evidenced by wandering at night

ANS: A The patient is at high risk for injury because of her confusion. The risk increases when caregivers are unable to give constant supervision. Insomnia, chronic confusion, and wandering apply to this patient; however, the risk for injury is a higher priority.

19. An adult has a history of physical violence against family when frustrated, followed by periods of remorse after each outburst. Which finding indicates a successful plan of care? The adult a. expresses frustration verbally instead of physically. b. explains the rationale for behaviors to the victim. c. identifies three personal strengths. d. agrees to seek counseling.

ANS: A The patient will have developed a healthier way of coping with frustration if it is expressed verbally instead of physically. The incorrect options do not confirm achievement of outcomes.

18. A victim of a violent rape was treated in the emergency department. As discharge preparation begins, the victim says softly, "I will never be the same again. I can't face my friends. There is no reason to go on." Select the nurse's most appropriate response. a. "Are you thinking of harming yourself?" b. "It will take time, but you will feel the same as before the attack." c. "Your friends will understand when you explain it was not your fault." d. "You will be able to find meaning from this experience as time goes on."

ANS: A The patient's words suggest hopelessness. Whenever hopelessness is present, so is suicide risk. The nurse should directly address the possibility of suicidal ideation with the patient. The other options attempt to offer reassurance before making an assessment.

12. A nurse works at rape telephone hotline. Communication with potential victims should focus on a. explaining immediate steps victims should take. b. providing callers with a sympathetic listener. c. obtaining information for law enforcement. d. arranging counseling.

ANS: A The telephone counselor establishes where the victim is and what has happened and provides the necessary information to enable the victim to decide what steps to take immediately. Counseling is not the focus until immediate problems are resolved. The victim remains anonymous. The other distracters are inappropriate or incorrect because counselors are trained to be empathetic rather than sympathetic.

15. A victim of a sexual assault who sits in the emergency department is rocking back and forth and repeatedly saying, "I can't believe I've been raped." This behavior is characteristic of which stage of rape-trauma syndrome? a. The acute phase reaction b. The long-term phase c. A delayed reaction d. The angry stage

ANS: A The victim's response is typical of the acute phase and shows cognitive, affective, and behavioral disruptions. This response is immediate and does not include a display of behaviors suggestive of the long-term (reorganization) phase, anger, or a delayed reaction.

4. A patient diagnosed with anorexia nervosa virtually stopped eating 5 months ago and lost 25% of body weight. A nurse asks, "Describe what you think about your present weight and how you look." Which response by the patient is most consistent with the diagnosis? a. "I am fat and ugly." b. "What I think about myself is my business." c. "I'm grossly underweight, but that's what I want." d. "I'm a few pounds' overweight, but I can live with it."

ANS: A Untreated patients with anorexia nervosa do not recognize their thinness. They perceive themselves to be overweight and unattractive. The patient with anorexia will usually tell people perceptions of self. The patient with anorexia does not recognize his or her thinness and will persist in trying to lose more weight.

21. A man who regularly experiences premature ejaculation tells the nurse, "I feel like such a failure. It's so awful for both me and my partner." Select the nurse's most therapeutic response. a. "I sense you are feeling frustrated and upset." b. "Tell me more about feeling like a failure." c. "You are too hard on yourself." d. "What do you mean by awful?"

ANS: A Using reflection and empathy promotes trust and conveys concern to the patient. The distracters do not offer empathy, probe, and offer premature reassurance.

25. An outpatient diagnosed with anorexia nervosa has begun refeeding. Between the first and second appointments, the patient gained 8 pounds. The nurse should a. assess lung sounds and extremities. b. suggest use of an aerobic exercise program. c. positively reinforce the patient for the weight gain. d. establish a higher goal for weight gain the next week.

ANS: A Weight gain of more than 2 to 5 pounds weekly may overwhelm the heart's capacity to pump, leading to cardiac failure. The nurse must assess for signs of pulmonary edema and congestive heart failure. The incorrect options are undesirable because they increase the risk for cardiac complications.

1. A nurse ends a relationship with a patient. Which actions by the nurse should be included in the termination phase? (Select all that apply.) a. Focus dialogues with the patient on problems that may occur in the future. b. Help the patient express feelings about the relationship with the nurse. c. Help the patient prioritize and modify socially unacceptable behaviors. d. Reinforce expectations regarding the parameters of the relationship. e. Help the patient to identify strengths, limitations, and problems.

ANS: A, B The correct actions are part of the termination phase. The other actions would be used in the working and orientation phases.

2. Which assessment findings would the nurse expect in a patient experiencing delirium? Select all that apply. a. Impaired level of consciousness b. Disorientation to place, time c. Wandering attention d. Apathy e. Agnosia

ANS: A, B, C Disorientation to place and time is an expected finding. Orientation to person (self) usually remains intact. Attention span is short, and difficulty focusing or shifting attention as directed is often noted. Patients with delirium commonly experience illusions and hallucinations. Fluctuating levels of consciousness are expected. Agnosia occurs with dementia. Apathy is associated with depression.

3. A community health nurse visits a family with four children. The father behaves angrily, finds fault with the oldest child, and asks twice, "Why are you such a stupid kid?" The wife says, "I have difficulty disciplining the children. It's so frustrating." Which comments by the nurse will facilitate an interview with these parents? (Select all that apply.) a. "Tell me how you discipline your children." b. "How do you stop your baby from crying?" c. "Caring for four small children must be difficult." d. "Do you or your husband ever spank your children?" e. "Calling children 'stupid' injures their self-esteem."

ANS: A, B, C An interview with possible abusing individuals should be built on concern and carried out in a nonthreatening, nonjudgmental way. Empathetic remarks are helpful in creating rapport. Questions requiring a descriptive response are less threatening and elicit more relevant information than questions that can be answered by yes or no.

1. A 10-year-old cares for siblings while the parents work because the family cannot afford a babysitter. This child says, "My father doesn't like me. He calls me stupid all the time." The mother says the father is easily frustrated and has trouble disciplining the children. The community health nurse should consider which resources as priorities to stabilize the home situation? (Select all that apply.) a. Parental sessions to teach childrearing practices b. Anger management counseling for the father c. Continuing home visits to give support d. A safety plan for the wife and children e. Placing the children in foster care

ANS: A, B, C Anger management counseling for the father is appropriate. Support for this family will be an important component of treatment. By the wife's admission, the family has deficient knowledge of parenting practices. Whenever possible, the goal of intervention should be to keep the family together; thus, removing the children from the home should be considered a last resort. Physical abuse is not suspected, so a safety plan would not be a priority at this time.

3. An emergency department nurse prepares to assist with examination of a sexual assault victim. What equipment will be needed to collect and document forensic evidence? (Select all that apply.) a. Camera b. Body map c. DNA swabs d. Pulse oximeter e. Sphygmomanometer

ANS: A, B, C Body maps, DNA swabs, and photographs are used to collect and preserve body fluids and other forensic evidence.

2. A patient was abducted and raped at gunpoint by an unknown assailant. Which nursing interventions are appropriate while caring for the patient in the emergency department? (Select all that apply.) a. Allow the patient to talk at a comfortable pace. b. Place the patient in a private room with a caregiver. c. Pose questions in nonjudgmental, empathetic ways. d. Invite the patient's family members to the examination room. e. Put an arm around the patient to demonstrate support and compassion.

ANS: A, B, C Neutral, nonjudgmental care and emotional support are critical to crisis management for the rape victim. The rape victim should have privacy but not be left alone. The rape victim's anxiety may escalate when touched by a stranger, even when the stranger is a nurse. Some rape victims prefer not to have family involved. The patient's privacy may be compromised by family presence.

2. A parent was recently hospitalized with severe depression. Family members say, "We're falling apart. Nobody knows what to expect, who should make decisions, or how to keep the family together." Which interventions should the nurse use when working with this family? (Select all that apply.) a. Help the family set realistic expectations. b. Provide empathy, acceptance, and support. c. Empower the family by teaching problem solving. d. Negotiate role flexibility among family members. e. Focus planning on the family rather than on the patient.

ANS: A, B, C, D The correct answers address expressed needs of the family. The distracter is inappropriate.

2. A 10-year-old child was placed in a foster home after being removed from parental contact because of abuse. The child has apprehension, tremulousness, and impaired concentration. The foster parent also reports the child has an upset stomach, urinates frequently, and does not understand what has happened. What helpful measures should the nurse suggest to the foster parents? The nurse should recommend (Select all that apply) a. conveying empathy and acknowledging the child's distress. b. explaining and reinforcing reality to avoid distortions. c. using a calm manner and low, comforting voice. d. avoiding repetition in what is said to the child. e. staying with the child until the anxiety decreases. f. minimizing opportunities for exercise and play.

ANS: A, B, C, E The child's symptoms and behavior suggest that he is exhibiting PTSD. Interventions appropriate for this level of anxiety include using a calm, reassuring tone, acknowledging the child's distress, repeating content as needed when there is impaired cognitive processing and memory, providing opportunities for comforting and normalizing play and physical activities, correcting any distortion of reality, and staying with the child to increase his sense of security.

3. The nurse interviewing a patient with suspected PTSD should be alert to findings indicating the patient (Select all that apply) a. avoids people and places that arouse painful memories. b. experiences flashbacks or re-experiences the trauma. c. experiences symptoms suggestive of a heart attack. d. feels compelled to repeat selected ritualistic behaviors. e. demonstrates hypervigilance or distrusts others. f. feels detached, estranged, or empty inside.

ANS: A, B, C, E, F These assessment findings are consistent with the symptoms of PTSD. Ritualistic behaviors are expected in obsessive-compulsive disorder.

1. A young adult says, "I was sexually abused by my older brother. During those assaults, I went somewhere else in my mind. I don't remember the details. Now, I often feel numb or unreal in romantic relationships, so I just avoid them." Which disorders should the nurse suspect based on this history? (Select all that apply.) a. Acute stress disorder b. Depersonalization disorder c. Generalized anxiety disorder d. PTSD e. Reactive attachment disorder f. Disinhibited social engagement disorder

ANS: A, B, D Acute stress disorder, depersonalization disorder, and PTSD can involve dissociative elements, such as numbing, feeling unreal, and being amnesic for traumatic events. All three disorders are also responses to acute stress or trauma, which has occurred here. The distracters are disorders not evident in this patient's presentation. Generalized anxiety disorder involves extensive worrying that is disproportionate to the stressors or foci of the worrying. Reactive attachment disorder and disinhibited social engagement disorder are problems of childhood.

1. A patient diagnosed with moderately severe Alzheimer's disease has a self-care deficit of dressing and grooming. Designate appropriate interventions to include in the patient's plan of care. Select all that apply. a. Provide clothing with elastic and hook-and-loop closures. b. Label clothing with the patient's name and name of the item. c. Administer anti-anxiety medication before bathing and dressing. d. Provide necessary items and direct the patient to proceed independently. e. If the patient resists dressing, use distraction and try again after a short interval.

ANS: A, B, E Providing clothing with elastic and hook-and-loop closures facilitates patient independence. Labeling clothing with the patient's name and the name of the item maintains patient identity and dignity (provides information if the patient has agnosia). When a patient resists, it is appropriate to use distraction and try again after a short interval because patient moods are often labile. The patient may be willing to cooperate given a later opportunity. Providing the necessary items for grooming and directing the patient to proceed independently are inappropriate. Be prepared to coach by giving step-by-step directions for each task as it occurs. Administering anxiolytic medication before bathing and dressing is inappropriate. This measure would result in unnecessary overmedication.

2. A night shift worker reports, "I'm having trouble getting to sleep after a night's work. I have a hearty breakfast with coffee, read the paper, do my exercises, and then go to bed. However, I just lie awake until it is nearly time to get up to be with my family for dinner." What changes should the nurse suggest? (Select all that apply.) a. Drink juice with breakfast rather than coffee. b. Exercise after awakening rather than before. c. Turn on the television when going to bed. d. Do not read the paper. e. Eat a light breakfast.

ANS: A, B, E Sleep can be disrupted by caffeine, a CNS stimulant, exercise performed just before trying to sleep, and eating a heavy meal before retiring. Reading the newspaper is not likely to be so stimulating that it disrupts the patient's ability to sleep. Television will be disruptive to sleep.

1. A nurse assesses a patient diagnosed with a paraphilic disorder. Which findings are most likely? (Select all that apply.) a. Childhood history of attention deficit hyperactivity disorder (ADHD) b. A poorly managed endocrine disorder c. History of brain injury d. Cognitive distortions e. Grandiosity

ANS: A, C, D ADHD in childhood, substance abuse, phobic disorders, and major depressive disorder/dysthymia are strongly associated with paraphilic disorders. Errors in thought make it seem acceptable for deviant and destructive sexual behaviors to occur. Patients who have experienced head trauma with damage to the frontal lobe of the brain may display symptoms of promiscuity, poor judgment, inability to recognize triggers that set off sexual desires, and poor impulse control. Endocrine problems are not associated with pedophilic disorder. Self-confidence is lacking; therefore, grandiosity would not be expected.

6. Which scenarios describe completed rape? (Select all that apply.) a. A husband forces vaginal sex when he comes home intoxicated from a party. The wife objects. b. A woman's lover pleads with her to have oral sex. She gives in but later regrets the decision. c. A person is beaten, robbed, and forcibly subjected to anal penetration by an assailant. d. A dentist gives anesthesia for a procedure and then has intercourse with the unconscious patient. e. A perpetrator grabs a potential victim, tears off most of her clothing, and fondles her breasts before she escapes.

ANS: A, C, D The correct responses depict scenarios of completed rape. The incorrect responses represent consensual sexual contact and sexual assault. Consensual sex is not considered rape if the participants are of legal age.

1. When an emergency department nurse teaches a victim of rape-trauma syndrome about reactions that may occur during the long-term phase, which symptoms should be included? (Select all that apply.) a. Development of fears and phobias b. Decreased motor activity c. Feelings of numbness d. Flashbacks, dreams e. Syncopal episodes

ANS: A, C, D These reactions are common to the long-term phase. Victims of rape frequently have a period of increased motor activity rather than decreased motor activity during the long-term reorganization phase. Syncopal episodes would not be expected.

3. A college student is extremely upset after failing two examinations. The student said, "No one understands how this will hurt my chances of getting into medical school." The student then suspends access to his social networking website and turns off his cell phone. Which suicide risk factors are evident? (Select all that apply.) a. Shame b. Panic attack c. Humiliation d. Self-imposed isolation e. Recent stressful life event

ANS: A, C, D, E Failing examinations in the academic major constitutes a recent stressful life event. Shame and humiliation related to the failure can be hypothesized. The statement, "No one can understand," can be seen as recent lack of social support. Terminating access to one's social networking site and turning off the cell phone represents self-imposed isolation. The scenario does not provide evidence of panic attack.

2. A nurse directs the intervention team who places an aggressive patient in seclusion. Before approaching the patient, which actions will the nurse direct team members to take? (Select all that apply.) a. Appoint a person to clear a path and open, close, or lock doors. b. Quickly approach the patient and take the closest extremity. c. Select the person who will communicate with the patient. d. Move behind the patient when the patient is not looking. e. Remove jewelry, glasses, and harmful items.

ANS: A, C, E Injury to staff and the patient should be prevented. Only one person should explain what will happen and direct the patient. This may be the nurse or a staff member with a good relationship with the patient. A clear pathway is essential because those restraining a limb cannot use keys, move furniture, or open doors. The nurse is usually responsible for administering medication once the patient is restrained. Each staff member should have an assigned limb rather than just grabbing the closest. This system could leave one or two limbs unrestrained. Approaching in full view of the patient reduces suspicion.

2. A nurse assists a victim of intimate partner abuse to create a plan for escape if it becomes necessary. Which components should the plan include? (Select all that apply.) a. Keep a cell phone fully charged. b. Hide money with which to buy new clothes. c. Have the phone number for the nearest shelter. d. Take enough toys to amuse the children for 2 days. e. Secure a supply of current medications for self and children. f. Assemble birth certificates, Social Security cards, and licenses. g. Determine a code word to signal children when it is time to leave.

ANS: A, C, E, F, G The victim must prepare for a quick exit and so should assemble necessary items. Keeping a cell phone fully charged will help with access to support persons or agencies. Taking a large supply of toys would be cumbersome and might compromise the plan. People are advised to take one favorite small toy or security object for each child, but most shelters have toys to further engage the children. Accumulating enough money to purchase clothing may be difficult.

1. A nurse driving home after work comes upon a serious automobile accident. The driver gets out of the car with no apparent physical injuries. Which assessment findings would the nurse expect from the driver immediately after this event? (Select all that apply.) a. Difficulty using a cell phone b. Long-term memory losses c. Fecal incontinence d. Rapid speech e. Trembling

ANS: A, D, E Immediate responses to crisis commonly include shock, numbness, denial, confusion, disorganization, difficulty with decision making, and physical symptoms such as nausea, vomiting, tremors, profuse sweating, and dizziness associated with anxiety. Incontinence and long-term memory losses would not be expected.

1. A patient with a history of command hallucinations approaches the nurse yelling obscenities. Which nursing actions are most likely to be effective in deescalation for this scenario? (Select all that apply.) a. Stating the expectation that the patient will stay in control. b. Asking the patient, "Do you want to go into seclusion?" c. Telling the patient, "You are behaving inappropriately." d. Offering to provide the patient with medication to help. e. Speaking in a firm but calm voice.

ANS: A, D, E Stating the expectation that the patient will maintain control of behavior reinforces positive, healthy behavior and avoids challenging the patient. Offering as-needed medication provides support for the patient trying to maintain control. A firm but calm voice will likely comfort and calm the patient. Belittling remarks may lead to aggression. Criticism will probably prompt the patient to begin shouting.

3. Which central nervous system structures are most associated with anger and aggression? (Select all that apply.) a. Amygdala b. Cerebellum c. Basal ganglia d. Temporal lobe e. Prefrontal cortex

ANS: A, D, E The amygdala and prefrontal cortex mediate anger experiences and help a person judge an event as either rewarding or aversive. The temporal lobe, which is part of the limbic system, also plays a role in aggressive behavior. The basal ganglia are involved in movement. The cerebellum manages equilibrium, muscle tone, and movement.

4. Which aspects of assessment have priority when a nurse interviews a rape victim in an acute setting? (Select all that apply.) a. Coping mechanisms, the patient is using b. The patient's previous sexual experiences c. The patient's history of sexually transmitted diseases d. Signs and symptoms of emotional and physical trauma e. Adequacy and availability of the patient's support system

ANS: A, D, E The nurse assesses the victim's level of anxiety, coping mechanisms, available support systems, signs and symptoms of emotional trauma, and signs and symptoms of physical trauma. The history of STDs or previous sexual experiences has little relevance.

2. A novice psychiatric nurse has a parent diagnosed with bipolar disorder. This nurse angrily recalls feelings of embarrassment about the parent's behavior in the community. Select the best ways for this nurse to cope with these feelings. (Select all that apply.) a. Seek ways to use the understanding gained from childhood to help patients cope with their own illnesses. b. Recognize that these feelings are unhealthy. The nurse should try to suppress them when working with patients. c. Recognize that psychiatric nursing is not an appropriate career choice. Explore other nursing specialties. d. The nurse should begin new patient relationships by saying, "My own parent had mental illness, so I accept it without stigma." e. Recognize that the feelings may add sensitivity to the nurse's practice, but supervision is important.

ANS: A, E The nurse needs support to explore these feelings. An experienced psychiatric nurse is a resource that may be helpful. The knowledge and experience gained from the nurse's relationship with a mentally ill parent may contribute sensitivity to compassionate practice. Self-disclosure and suppression are not adaptive coping strategies. The nurse should not give up on this area of practice without first seeking ways to cope with the memories.

7. Which assessment finding would be likely for a patient experiencing a hallucination? The patient: a. looks at shadows on a wall and says, "I see scary faces." b. states, "I feel bugs crawling on my legs and biting me." c. reports telepathic messages from the television. d. speaks in rhymes.

ANS: B A hallucination is a false sensory perception occurring without a corresponding sensory stimulus. Feeling bugs on the body when none are present is a tactile hallucination. Misinterpreting shadows as faces is an illusion. An illusion is a misinterpreted sensory perception. The other incorrect options apply to thought insertion and clang associations.

12. Consider these diagnostic findings: apolipoprotein E (apoE) malfunction, neurofibrillary tangles, neuronal degeneration in the hippocampus, and brain atrophy. Which health problem corresponds to these diagnostic findings? a. Huntington's disease b. Alzheimer's Disease c. Parkinson's disease d. Vascular dementia

ANS: B All of the options relate to dementias; however, the pathophysiological phenomena described apply to Alzheimer's disease. Parkinson's disease is associated with dopamine dysregulation. Huntington's disease is genetic. Vascular dementia is the consequence of circulatory changes.

19. A nurse counsels the family of a patient diagnosed with Alzheimer's disease who lives at home and wanders at night. Which action is most important for the nurse to recommend to enhance safety? a. Apply a medical alert bracelet to the patient. b. Place locks at the tops of doors. c. Discourage daytime napping. d. Obtain a bed with side rails.

ANS: B Placing door locks at the top of the door makes it more difficult for the patient with dementia to unlock the door because the ability to look up and reach upward is diminished. The patient will try to climb over side rails, increasing the risk for injury and falls. Avoiding daytime naps may improve the patient's sleep pattern but does not assure safety. A medical alert bracelet will be helpful if the patient leaves the home, but it does not prevent wandering or assure the patient's safety.

23. A patient with severe dementia no longer recognizes family members and becomes anxious and agitated when they attempt reorientation. Which alternative could the nurse suggest to the family members? a. Wear large name tags. b. Focus interaction on familiar topics. c. Frequently repeat the reorientation strategies. d. Place large clocks and calendars strategically.

ANS: B Reorientation may seem like arguing to a patient with cognitive deficit and increases the patient's anxiety. Validating, talking with the patient about familiar, meaningful things, and reminiscing give meaning to existence both for the patient and family members. The option that suggests using validating techniques when communicating is the only option that addresses an interactional strategy. Wearing large name tags and placing large clocks and calendars strategically are reorientation strategies. Frequently repeating the reorientation strategies is inadvisable because patients with dementia sometimes become more agitated with reorientation.

19. A group has two more sessions before it ends. One member was previously vocal and has shown much progress but has now grown silent. What explanation most likely underlies this behavior? The silent member a. has participated in the group and now has nothing more to offer. b. is having trouble dealing with feelings about termination of this group. c. wants to give quieter members a chance to talk in the remaining sessions. d. is engaging in attention-seeking behavior aimed at continuation of the group.

ANS: B A chief task during the termination phase of a group is to take what has been learned in group and transition to life without the group. The end of a group can be a significant loss for members, who may experience loss and grief and respond with sadness or anger. It is unlikely he would have nothing to say; at the very least, he could be responding to the comments of others even if not focusing on his own issues. He may wish to give quieter members a chance to talk, but again, this would not require or explain his complete silence. Some members, faced with only two remaining sessions, may be becoming more dominant under this pressure of time, but here too this is unlikely to lead a previously active participant to fall completely silent. The member is not attention-seeking.

10. A nurse introduces the matter of a contract during the first session with a new patient because contracts a. specify what the nurse will do for the patient. b. spell out the participation and responsibilities of each party. c. indicate the feeling tone established between the participants. d. are binding and prevent either party from prematurely ending the relationship.

ANS: B A contract emphasizes that the nurse works with the patient rather than doing something for the patient. "Working with" is a process that suggests each party is expected to participate and share responsibility for outcomes. Contracts do not, however, stipulate roles or feeling tone, and premature termination is forbidden.

21. During a support group, a patient diagnosed with schizophrenia says, "Sometimes I feel sad that I will never have a good job like my brother. Then I dwell on it and maybe I should not." Select the nurse leader's best comment to facilitate discussion of this issue. a. "It is often better to focus on our successes rather than our failures." b. "How have others in the group handled painful feelings like these?" c. "Grieving for what is lost is a normal part of having a mental disorder." d. "I wonder if you might also experience feelings of anger and helplessness."

ANS: B Asking others to share their experiences will facilitate discussion of an issue. Giving information may serve to close discussion of the issue because it sounds final. Suggesting a focus on the positives implies a discussion of the issue is not appropriate. Suggesting other possible feelings is inappropriate at this point, considering the patient has identified feelings of sadness and seems to have a desire to explore this feeling. Focusing on other feelings will derail discussion of the patient's grief for his perceived lost potential.

8. An adult diagnosed with schizophrenia lives with elderly parents. The patient was recently hospitalized with acute psychosis. One parent is very anxious, and the other is ill because of the stress. Which nursing diagnosis is most applicable to this scenario? a. Ineffective family coping related to parental role conflict b. Caregiver role strain related to the stress of chronic illness c. Impaired parenting related to patient's repeated hospitalizations d. Interrupted family processes related to relapse of acute psychosis

ANS: B Caregiver role strain refers to a caregiver's felt or exhibited difficulty in performing a family caregiver role. In this case, one parent exhibits stress-related illness and the other exhibits increased anxiety. The other nursing diagnoses are not substantiated by the information given and are incorrectly formatted (one nursing diagnosis should not be the etiology for another).

8. An older adult has experienced severe depression for many years and is unable to tolerate most antidepressant medications due to adverse effects of the medications. He is scheduled for electroconvulsive therapy (ECT) as a treatment for his depression. What teaching should the nurse give the patient regarding this treatment? a. There are no special preparations needed before this treatment. b. Common side effects include headache and short-term memory loss. c. One treatment will be needed to cure the depression. d. This treatment will leave you unconscious for several hours.

ANS: B Common side effects of ECT include headache, sleepiness, short-term memory loss, nausea, and muscle aches. Preparations before and after the procedure are the same as any operative procedure involving the patient receiving anesthesia. Treatment is typically three sessions a week for 4 weeks, not once. Patients are not unconscious after the procedure due to the use of precisely placed electrodes and the use of anesthesia.

1. A nurse assesses a confused older adult. The nurse experiences sadness and reflects, "This patient is like one of my grandparents ... so helpless." Which response is the nurse demonstrating? a. Transference b. Countertransference c. Catastrophic reaction d. Defensive coping reaction

ANS: B Countertransference is the nurse's transference or response to a patient that is based on the nurse's unconscious needs, conflicts, problems, or view of the world. See relationship to audience response question.

20. An advanced practice nurse observes a novice nurse expressing irritability regarding a patient with a long history of alcoholism and suspects the new nurse is experiencing countertransference. Which comment by the new nurse confirms this suspicion? a. "This patient continues to deny problems resulting from drinking." b. "My parents were alcoholics and often neglected our family." c. "The patient cannot identify any goals for improvement." d. "The patient said I have many traits like her mother."

ANS: B Countertransference occurs when the nurse unconsciously and inappropriately displaces onto the patient feelings and behaviors related to significant figures in the nurse's past. In this instance, the new nurse's irritability stems from relationships with parents. The distracters indicate transference or accurate analysis of the patient's behavior.

9. An adult, recently diagnosed with AIDS, is hospitalized with pneumonia. The patient and family are very anxious. Select the best outcome to add to the plan of care for this family. a. Describe the stages of the anticipatory grieving process. b. Identify and describe effective methods for coping with anxiety. c. Recognize ways dysfunctional communication is expressed in the family. d. Examine previously unexpressed feelings related to the patient's sexuality.

ANS: B Desired outcomes might be set for the family as a whole or for individuals within the family. The outcome most closely associated with the anxiety that each member is experiencing is to focus on identifying and describing ways of coping with the anxiety. The other options are not appropriate at this time.

11. The unlicensed assistive personnel (UAP) says to the nurse, "That patient with amnesia looks fine, but when I talk to her, she seems vague. What should I be doing for her?" Select the nurse's best reply. a. "Spend as much time with her as you can and ask questions about her life." b. "Use short, simple sentences and keep the environment calm and protective." c. "Provide more information about her past to reduce the mysteries that are causing anxiety." d. "Structure her time with activities to keep her busy, stimulated, and regaining concentration."

ANS: B Disruptions in ability to perform activities of daily living, confusion, and anxiety are often apparent in patients with amnesia. Offering simple directions to promote activities of daily living and reduce confusion helps increase feelings of safety and security. A calm, secure, predictable, protective environment is also helpful when a person is dealing with a great deal of uncertainty. Recollection of memories should proceed at its own pace, and the patient should only gradually be given information about her past. Asking questions that require recall that the patient does not possess will only add frustration. Quiet, undemanding activities should be provided as the patient tolerates them and should be balanced with rest periods; the patient's time should not be loaded with demanding or stimulating activities.

24. A new patient acts out so aggressively that seclusion is required before the admission assessment is completed or orders written. Immediately after safely secluding the patient, which action is the nurse's priority? a. Complete the physical assessment. b. Notify the health care provider to obtain a seclusion order. c. Document the incident objectively in the patient's medical record. d. Explain to the patient that seclusion will be discontinued when self-control is regained.

ANS: B Emergency seclusion can be effected by a credentialed nurse but must be followed by securing a medical order within a period of time specified by the state and the agency. The incorrect options are not immediately necessary from a legal standpoint. See related audience response question.

17. A soldier returned home from active duty in a combat zone and was diagnosed with PTSD. The soldier says, "If there's a loud noise at night, I get under my bed because I think we're getting bombed." What type of experience has the soldier described? a. Illusion b. Flashback c. Nightmare d. Auditory hallucination

ANS: B Flashbacks are dissociative reactions in which an individual feels or acts as if the traumatic event were recurring. Illusions are misinterpretations of stimuli, and although the experience is similar, it is better termed a flashback because of the diagnosis of PTSD. Auditory hallucinations have no external stimuli. Nightmares commonly accompany PTSD, but this experience was stimulated by an actual environmental sound.

14. A nurse wants to demonstrate genuineness with a patient diagnosed with schizophrenia. The nurse should a. restate what the patient says. b. use congruent communication strategies. c. use self-revelation in patient interactions. d. consistently interpret the patient's behaviors.

ANS: B Genuineness is a desirable characteristic involving awareness of one's own feelings as they arise and the ability to communicate them when appropriate. The incorrect options are undesirable in a therapeutic relationship.

12. A patient diagnosed with depersonalization disorder tells the nurse, "It's starting again. I feel as though I'm going to float away." Which intervention would be most appropriate at this point? a. Notify the health care provider of this change in the patient's behavior. b. Engage the patient in a physical activity such as exercise. c. Isolate the patient until the sensation has diminished. d. Administer a prn dose of antianxiety medication.

ANS: B Helping the patient apply a grounding technique, such as exercise, assists the patient to interrupt the dissociative process. Medication can help reduce anxiety but does not directly interrupt the dissociative process. Isolation would allow the sensation to overpower the patient. It is not necessary to notify the health care provider.

1. A nurse works with a patient diagnosed with posttraumatic stress disorder (PTSD) who has frequent flashbacks as well as persistent symptoms of arousal. Which intervention should be included in the plan of care? a. Trigger flashbacks intentionally in order to help the patient learn to cope with them. b. Explain that the physical symptoms are related to the psychological state. c. Encourage repression of memories associated with the traumatic event. d. Support "numbing" as a temporary way to manage intolerable feelings.

ANS: B Persons with PTSD often experience somatic symptoms or sympathetic nervous system arousal that can be confusing and distressing. Explaining that these are the body's responses to psychological trauma helps the patient understand how such symptoms are part of the illness and something that will respond to treatment. This decreases powerlessness over the symptoms and helps instill a sense of hope. It also helps the patient to understand how relaxation, breathing exercises, and imagery can be helpful in symptom reduction. The goal of treatment for PTSD is to come to terms with the event so treatment efforts would not include repression of memories or numbing. Triggering flashbacks would increase patient distress.

3. When a nurse assesses a family, which family task has the highest priority for healthy family functioning? a. Allocation of family resources b. Physical maintenance and safety c. Maintenance of order and authority d. Reproduction of new family members

ANS: B Physical and safety needs have greater importance in Maslow's hierarchy than other needs.

19. Which scenario best illustrates scapegoating within a family? a. The identified patient sends messages of aggression to selected family members. b. Family members project problems of the family onto one particular family member. c. The identified patient threatens separation from the family to induce feelings of isolation and despair. d. Family members give the identified patient nonverbal messages that conflict with verbal messages.

ANS: B Scapegoating projects blame for family problems onto a member who is less powerful. The purpose of this projection is to distract from issues or dysfunctional behaviors in the members of the family.

19. Which comment by the parents of young children best demonstrates support of development of resilience and effective stress management? a. "Our children will be stronger if they make their own decisions." b. "We spend daily family time talking about experiences and feelings." c. "We use three different babysitters. All of them have college degrees." d. "Our parenting strategies are different from those our own parents used."

ANS: B The correct response demonstrates consistent nurturing, which is a vital component of building resilience in children. The incorrect responses are not necessarily unhealthy parenting behaviors, but they do not clearly demonstrate parental nurturing.

16. Two weeks ago, a soldier returned to the United States from active duty in a combat zone. The soldier was diagnosed with PTSD. Which comment by the soldier requires the nurse's immediate attention? a. "It's good to be home. I missed my home, family, and friends." b. "I saw my best friend get killed by a roadside bomb. I don't understand why it wasn't me." c. "Sometimes I think I hear bombs exploding, but it's just the noise of traffic in my hometown." d. "I want to continue my education, but I'm not sure how I will fit in with other college students."

ANS: B The correct response indicates the soldier is thinking about death and feeling survivor's guilt. These emotions may accompany suicidal ideation, which warrants the nurse's follow-up assessment. Suicide is a high risk among military personnel diagnosed with PTSD. One distracter indicates flashbacks, common with persons with PTSD, but not solely indicative that further problems exist. The other distracters are normal emotions associated with returning home and change.

25. As a patient diagnosed with a mental illness is being discharged from a facility, a nurse invites the patient to the annual staff picnic. What is the best analysis of this scenario? a. The invitation facilitates dependency on the nurse. b. The nurse's action blurs the boundaries of the therapeutic relationship. c. The invitation is therapeutic for the patient's diversional activity deficit. d. The nurse's action assists the patient's integration into community living.

ANS: B The invitation creates a social relationship rather than a therapeutic relationship.

23. A patient in a detoxification unit asks, "What good it will do to go to Alcoholics Anonymous and talk to other people with the same problem?" The nurse's best response would be to explain that self-help groups such as AA provide opportunities for a. newly discharged alcoholics to learn about the disease of alcoholism. b. people with common problems to share their experiences with alcoholism and recovery. c. patients with alcoholism to receive insight-oriented treatment about the etiology of their disease. d. professional counselors to provide guidance to individuals recovering from alcoholism.

ANS: B The patient needs basic information about the purpose of a self-help group. The basis of self-help groups is sharing by individuals with similar problems. Self-help is based on the belief that an individual with a problem can be truly understood and helped only by others who have the same problem. The other options fail to address this or provide incorrect information.

14. The gas pedal on a person's car became stuck on a busy interstate highway, causing the car to accelerate rapidly. For 20 minutes, the car was very difficult to control. In the months after this experience, afterward, which assessment finding would the nurse expect? a. Weight gain b. Flashbacks c. Headache d. Diuresis

ANS: B The scenario depicts a frightening, traumatic, and stressful situation. Severe dissociation or "mind flight" may occur for those who have suffered significant trauma. The episodic failure of dissociation causes intrusive symptoms such as flashbacks. The problems identified in the distracters may or may not occur.

25. A wife believes her husband is having an affair. Lately, he has been disinterested in romance and working late. The husband has an important, demanding project at work. The mother asks her teen, "What have you noticed about your father?" The teen later mentions this to the father, who says, "Tell your mother that I can't deal with her insecurities right now." Which family dynamic is evident? a. Multigenerational dysfunction b. Triangulation c. Enmeshment d. Blaming

ANS: B Triangulation is a family dynamic wherein a pair relationship (usually the parents) is under stress and copes by drawing in a third person (usually a child) to align with one or the other members of the pair relationship. Multigenerational dysfunction is any dysfunction that exists within or across multiple generations of a family, such as child abuse or alcoholism. Blaming is distracting attention from one's own dysfunction or reducing one's own anxiety by blaming another person. Enmeshment refers to blurred family boundaries or blending together of the thoughts, feelings, or family roles of the individuals so that clear distinctions among members fail to emerge.

1. A patient comes to the crisis clinic after an unexpected job termination. The patient paces, sobs, cringes when approached, and responds to questions with only shrugs or monosyllables. Choose the nurse's best initial comment to this patient. a. "Everything is going to be all right. You are here at the clinic and the staff will keep you safe." b. "I see you are feeling upset. I'm going to stay and talk with you to help you feel better." c. "You need to try to stop crying and pacing so we can talk about your problems." d. "Let's set some guidelines and goals for your visit here."

ANS: B A crisis exists for this patient. The two primary thrusts of crisis intervention are to provide for the safety of the individual and use anxiety-reduction techniques to facilitate use of inner resources. The nurse offers therapeutic presence, which provides caring, ongoing observation relative to the patient's safety, and interpersonal reassurance.

2. Which comment is most likely from a patient with chronic sleep deprivation? a. "I turn on the television every night to get to sleep. I set the timer so it goes off in 30 minutes." b. "I have diarrhea frequently and not much energy, so I stay at home most of the time." c. "I only sleep about 7 hours a night, but I know I should sleep 8 or 9 hours." d. "When my alarm clock goes off every morning, it seems like I am dreaming."

ANS: B A discrepancy between hours of sleep obtained and hours required leads to sleep deprivation. Adults with less than 6 hours of sleep per night often suffer from chronic sleep deprivation. Common complaints include poor general health, physical and mental distress, limitations in ADLs, depressive or anxious symptoms, and pain. One distracter indicates a problem with sleep hygiene (television). The remaining distracters do not indicate a problem.

7. An intramuscular dose of antipsychotic medication needs to be administered to a patient who is becoming increasingly more aggressive and refused to leave the day room. The nurse should enter the day room a. and say, "Would you like to come to your room and take some medication your health care provider prescribed for you?" b. accompanied by three staff members and say, "Please come to your room so I can give you some medication that will help you regain control." c. and place the patient in a basket-hold and then say, "I am going to take you to your room to give you an injection of medication to calm you." d. accompanied by a male security guard and tell the patient, "Come to your room willingly so I can give you this medication, or the guard and I will take you there."

ANS: B A patient gains feelings of security if he or she sees others are present to help with control. The nurse gives a simple direction, honestly states what is going to happen, and reassures the patient that the intervention will be helpful. This positive approach assumes the patient can act responsibly and will maintain control. Physical control measures are used only as a last resort.

8. A patient diagnosed with anorexia nervosa is resistant to weight gain. What is the rationale for establishing a contract with the patient to participate in measures designed to produce a specified weekly weight gain? a. Because severe anxiety concerning eating is expected, objective and subjective data may be unreliable. b. Patient involvement in decision making increases sense of control and promotes adherence to the plan of care. c. Because of increased risk of physical problems with refeeding, the patient's permission is needed. d. A team approach to planning the diet ensures that physical and emotional needs will be met.

ANS: B A sense of control for the patient is vital to the success of therapy. A diet that controls weight gain can allay patient fears of too-rapid weight gain. Data collection is not the reason for contracting. A team approach is wise but is not a guarantee that needs will be met. Permission for treatment is a separate issue. The contract for weight gain is an additional aspect of treatment.

16. A victim of a sexual assault comes to the hospital for treatment but abruptly decides to decline treatment and leaves the facility. While respecting the person's rights, the nurse should a. say, "You may not leave until you receive prophylactic treatment for sexually transmitted diseases." b. provide written information about physical and emotional reactions the person may experience. c. explain the need and importance of infectious disease and pregnancy tests. d. give verbal information about legal resources in the community.

ANS: B All information given to a patient before he or she leaves the emergency department should be in writing. Patients who are anxious are unable to concentrate and therefore cannot retain much of what is verbally imparted. Written information can be read and referred to later. Patients may not be kept against their will or coerced into treatment. This constitutes false imprisonment.

17. An unconscious teenager is treated in the emergency department. The teenager's friends suspect the teenager was drugged and raped at a party. Priority action by the nurse should focus on a. preserving rape evidence. b. maintaining physiological stability. c. determining what drugs were ingested. d. obtaining a description of the rape from a friend.

ANS: B Because the patient is unconscious, the risk for airway obstruction is present. The nurse's priority will focus on maintaining physiological stability. The distracters are of lower priority than preserving physiological functioning.

7. A person says, "I often feel like I have been dreaming just before I awake in the morning." Which rationale correctly explains the comment? a. Sleep architecture changes during the sleep period, resulting in increased slow-wave sleep at the end of the cycle. b. Cycles of REM sleep increase in the second half of sleep and occupy longer periods. c. Dreams occur more frequently when a person is experiencing unresolved conflicts or depression. d. Dream content relates directly to developmental tasks. The person is likely feeling autonomous.

ANS: B Cycles of REM sleep increase in the second half of sleep and occupy longer periods, up to 1 hour. Dreaming occurs during REM sleep. The question relates to sleep architecture rather than dream content.

5. A patient needs diagnostic evaluation of sleep problems. Which test will evaluate the patient for possible sleep-related problems? a. Skull x-rays b. Electroencephalogram (EEG) c. Positron emission tomography (PET) d. Single photon emission computed tomography (SPECT)

ANS: B EEG measures nonrapid eye movement (NREM) and rapid eye movement (REM) sleep. The distracters represent ways to diagnose structural and metabolic problems.

11. A nurse interviews a 17-year-old male victim of sexual assault. The victim is reluctant to talk about the experience. Which comment should the nurse offer to this victim? a. "Male victims of sexual assault are usually better equipped than women to deal with the emotional pain that occurs." b. "Male victims of sexual assault often experience physical injuries and are assaulted by more than one person." c. "Do you have any male friends who have also been victims of sexual assault?" d. "Why do you think you became a victim of sexual assault?"

ANS: B Few rape survivors seek help, even with serious injury; so, it is important for the nurse to help the victim discuss the experience. The correct response therapeutically gives information to this victim. A male rape victim is more likely to experience physical trauma and to have been victimized by several assailants. Males experience the same devastation, physical injury, and emotional consequences as females. Although they may cover their responses, they too benefit from care and treatment. "Why" questions represent probing, which is a nontherapeutic communication technique. The victim may or may not have friends who have had this experience, but it is important to talk about his feelings rather than theirs.

18. A patient has a history of impulsively acting-out anger by striking others. Select the most appropriate intervention for avoiding similar incidents. a. Teach the patient about herbal preparations that reduce anger. b. Help the patient identify incidents that trigger impulsive anger. c. Explain that restraint and seclusion will be used if violence occurs. d. Offer one-on-one supervision to help the patient maintain control.

ANS: B Identification of trigger incidents allows the patient and nurse to plan interventions to reduce irritation and frustration, which lead to acting-out anger, and eventually to put into practice more adaptive coping strategies.

3. A patient comes to the crisis center saying, "I'm in a terrible situation. I don't know what to do." The triage nurse can initially assume that the patient is a. suicidal. b. anxious and fearful. c. misperceiving reality. d. potentially homicidal.

ANS: B Individuals in crisis are universally anxious. They are often frightened and may be mildly confused. Perceptions are often narrowed with anxiety.

3. What feelings are most commonly experienced by nurses working with abusive families? a. Outrage toward the victim and discouragement regarding the abuser b. Helplessness regarding the victim and anger toward the abuser c. Unconcern for the victim and dislike for the abuser d. Vulnerability for self and empathy with the abuser

ANS: B Intense protective feelings, helplessness, and sympathy for the victim are common emotions of a nurse working with an abusive family. Anger and outrage toward the abuser are common emotions of a nurse working with an abusive family.

9. A person is prescribed lorazepam 2 mg PO bid as needed for anxiety. When the person takes this medication, which change in sleep is anticipated? The patient will a. have fewer dreams. b. have less slow-wave sleep. c. experience extended sleep latency. d. enter sleep through REM sleep.

ANS: B Lorazepam is a benzodiazepine, which reduces slow-wave sleep. REM sleep would likely increase. Persons with narcolepsy often enter sleep through REM.

17. A patient reports, "Nearly every night I awaken feeling frightened after a bad dream. The dream usually involves being hunted by people trying to hurt me. It usually happens between 4 and 5 AM." The nurse assesses this disorder as most consistent with criteria for which problem? a. Sleep deprivation b. Nightmare disorder c. Night terror disorder d. REM sleep behavior disorder

ANS: B Nightmares are long, frightening dreams from which people awaken in a frightened state. They occur during REM sleep late in the night. Night terror disorder occurs as arousal in the first third of the night during NREM sleep and is accompanied by feelings of panic. REM sleep behavior disorder involves acting out a violent dream during REM sleep. Nightmare disorder may lead to sleep deprivation.

16. An older adult diagnosed with Alzheimer's disease lives with family in a rural area. During the week, this adult attends a day care center while the family is at work. In the evenings, members of the family provide care. Which factor makes this adult most vulnerable to abuse? a. Multiple caregivers b. Alzheimer's disease c. Living in a rural area d. Being part of a busy family

ANS: B Older adults are at high risk for violence, particularly when there is significant dependency such as would be expected with dementia or other cognitive impairments. The incorrect responses are not identified as placing an individual at high risk.

7. A woman tells the nurse, "My partner is frustrated with me. I don't have any natural lubrication when we have sex." What type of sexual disorder is evident? a. Genito-pelvic pain/penetration disorder b. Female sexual interest/arousal disorder c. Hypoactive sexual desire disorder d. Female orgasmic disorder

ANS: B One feature of female sexual interest/arousal disorder relates to inability to maintain physiological requirements for intercourse. For women, this includes problems with lubrication and swelling. The patient's description does not meet criteria for diagnoses in the distracters.

11. An appropriate intervention for a patient diagnosed with bulimia nervosa who binges and purges is to teach the patient a. to eat a small meal after purging. b. not to skip meals or restrict food. c. to increase oral intake after 4 PM daily. d. the value of reading journal entries aloud to others.

ANS: B One goal of health teaching is normalization of eating habits. Food restriction and skipping meals lead to rebound bingeing. Teaching the patient to eat a small meal after purging will probably perpetuate the need to induce vomiting. Teaching the patient to eat a large breakfast but no lunch and increase intake after 4 PM will lead to late-day bingeing. Journal entries are private.

5. An emergency department nurse prepares to assist with evidence collection for a sexual assault victim. Prior to photographs and pelvic examination, what documentation is important? a. The patient's vital signs b. Consent signed by the patient c. Supervision and credentials of the examiner d. Storage location of the patient's personal effects

ANS: B Patients have the right to refuse legal and medical examination. Consent forms are required to proceed with these steps.

17. A respected school coach was arrested after a student reported the coach attempted to have sexual contact. Which nursing action has priority in the period immediately following the coach's arrest? a. Determine the nature and extent of the coach's sexual disorder. b. Assess the coach's potential for suicide or other self-harm. c. Assess the coach's self-perception of problem and needs. d. Determine whether other children were harmed.

ANS: B Pedophiles and other persons with paraphilic disorders can be at increased risk of self-harm associated with the guilt, shame, and anger they feel about their behavior and its effect on their families, victims, and victims' families. They also face considerable losses, such as the end of their careers or the loss of freedom to imprisonment. Thus, safety is the priority issue for assessment. Determining the nature and extent of the patient's disorder and related patient perceptions would be appropriate but not the highest priority for assessment. Investigating whether other victims exist is a matter for law enforcement rather than health care personnel. See relationship to audience response question.

14. Which situation demonstrates use of primary intervention related to crisis? a. Implementation of suicide precautions for a depressed patient b. Teaching stress-reduction techniques to a first-year college student c. Assessing coping strategies used by a patient who attempted suicide d. Referring a patient diagnosed with schizophrenia to a partial hospitalization program

ANS: B Primary care-related crisis intervention promotes mental health and reduces mental illness. The incorrect options are examples of secondary or tertiary interventions.

3. After an abduction and rape at gunpoint by an unknown assailant, which assessment finding best indicates that a patient is in the acute phase of the rape-trauma syndrome? a. Decreased motor activity b. Confusion and disbelief c. Flashbacks and dreams d. Fears and phobias

ANS: B Reactions of the acute phase of the rape-trauma syndrome are shock, emotional numbness, confusion, disbelief, restlessness, and agitated motor activity. Flashbacks, dreams, fears, and phobias are seen in the long-term reorganization phase of the rape-trauma syndrome. Decreased motor activity by itself is not indicative of any particular phase.

21. Which personality characteristic is a nurse most likely to assess in a patient diagnosed with anorexia nervosa? a. Carefree flexibility b. Rigidity, perfectionism c. Open displays of emotion d. High spirits and optimism

ANS: B Rigid thinking, inability to demonstrate flexibility, and difficulty changing cognitions are characteristic of patients with eating disorders. The incorrect options are rare in a patient with an eating disorder. Inflexibility, controlled emotions, and pessimism are more the rule.

6. A nurse in the emergency department assesses an unresponsive victim of rape. The victim's friend reports, "That guy gave her salty water before he raped her." Which question is most important for the nurse to ask of the victim's friend? a. "Does the victim have any kidney disease?" b. "Has the victim consumed any alcohol?" c. "What time was she given salty water?" d. "Did you witness the rape?"

ANS: B Salty water is a slang/street name for GHB (-hydroxy-butyric acid), a Schedule III central nervous system depressant associated with rape. Use of alcohol would produce an increased risk for respiratory depression. GHB has a duration of 1 to 12 hours, but the duration is less important than the potential for respiratory depression. Seeking evidence is less important than the victim's physiologic stability.

10. An adult consulted a nurse practitioner because of an inability to achieve orgasm for 2 years, despite having been sexually active. This adult was frustrated and expressed concerns about the relationship with the sexual partner. Which nursing diagnosis is most appropriate for this scenario? a. Defensive coping b. Sexual dysfunction c. Ineffective sexuality pattern d. Disturbed sensory perception, tactile

ANS: B Sexual dysfunction is the most appropriate nursing diagnosis for a patient who is experiencing a problem affecting one or more phases of arousal. This is the primary problem reported by this patient. Ineffective sexuality pattern, since it is due to sexual dysfunction, is secondary to the absence of orgasms. The patient has not indicated she does not become aroused, just that she cannot achieve orgasm. Disturbed sensory perception may be part of the etiology, but the problem is sexual dysfunction. There is no evidence of defensive coping.

14. A home care nurse assesses a very demanding patient with chronic obstructive pulmonary disease (COPD). Afterward, the nurse talks with the spouse who has provided this patient's care for 6 years. The spouse says, "I don't need much sleep anymore. I might need to help him during the night." Select the nurse's most therapeutic response. a. "It sounds like you are very devoted to your spouse." b. "I noticed you fell asleep while I was assessing your spouse. I'm concerned about you." c. "Your spouse is lucky to have you to provide care rather than being placed in a nursing home." d. "If you keep going like this, your health will be impaired also. Then who will take care of both of you?"

ANS: B Sleep deprivation can cause accidents. The correct answer makes an observation, gives important information about safety, and communicates care and compassion for the spouse. The distracters do not invite further dialogue with the spouse.

13. A nurse counsels a patient on ways to determine the person's total sleep requirement. Which instruction would produce the most accurate results? a. "For 1 full week, record what you remember about your dream content and related feelings as soon as you wake up. Bring the record to your next appointment." b. "While off work for 1 week, go to bed at your usual time and wake up without an alarm. Record how many hours you sleep and then average the findings." c. "For 2 full weeks, record how much time you sleep each night and rate your daytime alertness on a scale of 1 to 10. Calculate your average alertness score." d. "All adults need 7 or 8 hours of sleep to function properly. Let's design ways to help you reach that goal."

ANS: B Sleep requirements are most accurately determined by going to bed at the usual time and waking up without an alarm for several nights, ideally on vacation. The average of these findings indicates the estimated requirements. Two distracters relate to dream content and daytime alertness. Some adults are long sleepers or short sleepers with different requirements for sleep from the general population.

4. Which rationale best explains why a nurse should be aware of personal feelings while working with a family experiencing family violence? a. Self-awareness enhances the nurse's advocacy role. b. Strong negative feelings interfere with assessment and judgment. c. Strong positive feelings lead to healthy transference with the victim. d. Positive feelings promote the development of sympathy for patients.

ANS: B Strong negative feelings cloud the nurse's judgment and interfere with assessment and intervention, no matter how well the nurse tries to cover or deny feelings. Strong positive feelings lead to overinvolvement with victims rather than healthy transference.

9. Six months ago, a woman had a prophylactic double mastectomy because of a family history of breast cancer. One week ago, this woman learned her husband was involved in an extramarital affair. The woman says tearfully, "What else can happen?" If the woman's immediate family is unable to provide sufficient support, the nurse should a. suggest hospitalization for a short period. b. ask what other relatives or friends are available for support. c. tell the patient, "You are a strong person. You can get through this crisis." d. foster insight by relating the present situation to earlier situations involving loss.

ANS: B The assessment of situational supports should continue. Even though the patient's nuclear family may not be supportive, other situational supports may be available. If they are adequate, admission to an inpatient unit will be unnecessary. Psychotherapy is not appropriate for crisis intervention. Advice is usually nontherapeutic.

1. A new staff nurse tells the clinical nurse specialist, "I am unsure about my role when patients bring up sexual problems." The clinical nurse specialist should give clarification by saying, "All nurses a. qualify as sexual counselors. Nurses have knowledge about the biopsychosocial aspects of sexuality throughout the life cycle." b. should be able to screen for sexual dysfunction and give basic information about sexual feelings, behaviors, and myths." c. should defer questions about sex to other health care professionals because of their limited knowledge of sexuality." d. who are interested in sexual dysfunction can provide sex therapy for individuals and couples."

ANS: B The basic education of nurses provides information sufficient to qualify the generalist to assess for sexual dysfunction and perform health teaching. Taking a detailed sexual history and providing sex therapy requires additional training in sex education and counseling. Nurses with basic education are not qualified to be sexual counselors. Additional education is necessary. A registered nurse may provide basic information about sexual function, but complex questions may require referral.

1. A nurse cares for these four patients. Which patient has the highest risk for problems with sleep physiology? a. Retiree who volunteers twice a week at Habitat for Humanity b. Corporate accountant who travels frequently c. Parent with three teenagers d. Lawn care worker

ANS: B The corporate accountant is likely to work long hours and have significant stress associated with work demands. Compounded by travel, these factors are likely to precipitate unstable sleep patterns and inadequate sleep time. The retiree and lawn care worker engage in physical activity during the day, which will promote natural fatigue and sleep. The parent's sleep is unlikely to be disturbed; teenagers sleep through the night.

10. An adult patient assaulted another patient and was then restrained. One hour later, which NURSINGTB.COM statement by the restrained patient requires the nurse's immediate attention? a. "I hate all of you!" b. "My fingers are tingly." c. "You wait until I tell my lawyer." d. "The other patient started the fight."

ANS: B The correct response indicates impaired circulation and necessitates the nurse's immediate attention. The incorrect responses indicate the patient has continued aggressiveness and agitation.

13. Which communication technique will the nurse use more in crisis intervention than traditional counseling? a. Role modeling b. Giving direction c. Information giving d. Empathic listening

ANS: B The nurse working in crisis intervention must be creative and flexible in looking at the patient's situation and suggesting possible solutions to the patient. Giving direction is part of the active role a crisis intervention therapist takes. The other options are used equally in crisis intervention and traditional counseling roles.

7. Which nursing intervention has the highest priority as a patient diagnosed with anorexia nervosa begins to gain weight? a. Assess for depression and anxiety. b. Observe for adverse effects of refeeding. c. Communicate empathy for the patient's feelings. d. Help the patient balance energy expenditures with caloric intake.

ANS: B The nursing intervention of observing for adverse effects of refeeding most directly relates to weight gain and is a priority. Assessing for depression and anxiety, as well as communicating empathy, relates to coping. Helping the patient achieve balance between energy expenditure and caloric intake is an inappropriate intervention.

14. A survivor of physical spousal abuse was treated in the emergency department for a broken wrist. This patient said, "I've considered leaving, but I made a vow and I must keep it no matter what happens." Which outcome should be met before discharge? The patient will a. facilitate counseling for the abuser. b. name two community resources for help. c. demonstrate insight into the abusive relationship. d. reexamine cultural beliefs about marital commitment.

ANS: B The only outcome indicator clearly attainable within this time is for staff to provide the victim with information about community resources that can be contacted. Development of insight into the abusive relationship and reexamining cultural beliefs will require time. Securing a restraining order can be accomplished quickly but not while the patient is in the emergency department. Facilitating the abuser's counseling may require weeks or months.

23. An adult comes to the crisis clinic after termination from a job of 15 years. The patient says, "I don't know what to do. How can I get another job? Who will pay the bills? How will I feed my family?" Which nursing diagnosis applies? a. Hopelessness b. Powerlessness c. Chronic low self-esteem d. Interrupted family processes

ANS: B The patient describes feelings of lack of control over life events. No direct mention is made of hopelessness or chronic low self-esteem. The patient's family processes are not interrupted at this point.

18. An older woman diagnosed with Alzheimer's disease lives with family and attends day care. After observing poor hygiene, the nurse talked with the caregiver. This caregiver became defensive and said, "It takes all my energy to care for my mother. She's awake all night. I never get any sleep." Which nursing intervention has priority? a. Teach the caregiver about the effects of sundowner's syndrome. b. Secure additional resources for the mother's evening and night care. c. Support the caregiver to grieve the loss of the mother's cognitive abilities. d. Teach the family how to give physical care more effectively and efficiently.

ANS: B The patient's caregivers were coping with care until the patient began to stay awake at night. The family needs assistance with evening and night care to resume their precrisis state of functioning. Secondary prevention calls for the nurse to mobilize community resources to relieve overwhelming stress. The other interventions may then be accomplished.

4. A nurse interviews a patient abducted and raped at gunpoint by an unknown assailant. The patient says, "I shouldn't have been there alone. I knew it was a dangerous area." What is the patient's present coping strategy? a. Projection b. Self-blame c. Suppression d. Rationalization

ANS: B The patient's statements reflect self-blame, an unhealthy coping mechanism. The patient's statements do not reflect rationalization, suppression, or projection.

9. A woman consults the nurse practitioner because she has not achieved orgasm for 2 years, despite having been sexually active. This is an example of a. Paraphilic disorder. b. Female orgasmic disorder. c. Genito-pelvic pain/penetration disorder. d. Female sexual interest/arousal disorder.

ANS: B The persistent inhibition of orgasm is a form of sexual dysfunction called female orgasmic disorder. Genito-pelvic pain/penetration disorder applies to painful intercourse. The patient has not indicated that her interest in sexual activity is diminished, so female sexual interest/arousal disorder does not apply. Paraphilic disorder is not applicable.

22. A man who reports frequently experiencing premature ejaculation tells the nurse, "I feel like such a failure. It's so awful for both me and my partner. Can you help me?" Select the nurse's best response. a. "Have you discussed this problem with your partner?" b. "I can refer you to a practitioner who can help you with this problem." c. "Have you asked your health care provider for prescription medication?" d. "There are several techniques described in this pamphlet that might be helpful."

ANS: B The primary role of the nurse is to perform basic assessment and make appropriate referrals. The other options do not clarify the nurse's role.

18. A nurse who works night shift says, "I am exhausted most of the time. I sleep through my alarm. Sometimes my brain does not seem to work right. I am worried that I might make a practice error." Which question should the nursing supervisor ask first? a. "What stress are you experiencing in your life?" b. "How much sleep do you get in a 24-hour period?" c. "Would it help if you do some exercises just before going to bed?" d. "Have you considered using a hypnotic medication to help you sleep?"

ANS: B Total sleep hours should be ascertained before seeking to correct a sleep disorder. In this case, the patient describes sleep deprivation symptoms rather than a sleep disorder. The correct response is the only option that addresses total sleep hours.

17. A patient with a history of anger and impulsivity was hospitalized after an accident resulting in multiple injuries. The patient loudly scolds nursing staff, "I'm in pain all the time but you don't give me medicine until YOU think it's time." Which nursing intervention would best address this problem? a. Teach the patient to use coping strategies such as deep breathing and progressive relaxation to reduce the pain. b. Talk with the health care provider about changing the pain medication from prn to patient-controlled analgesia. c. Tell the patient that verbal assaults on nurses will not shorten the wait for analgesic medication. d. Talk with the patient about the risks of dependency associated with overuse of analgesic medication.

ANS: B Use of patient-controlled analgesia will help the patient manage the pain. This intervention will help reduce the patient's anxiety and anger. Dependency is not an important concern related to acute pain.

20. Select the best desired outcome for a patient who uses valerian. The patient will report a. stress level is lower. b. undisturbed sleep throughout the night. c. increased interest in recreational activities. d. early morning waking without an alarm clock.

ANS: B Valerian decreases sleep latency, nocturnal waking, and leads to a subjective sense of good sleep. Sleeping through the night is the best indicator the herb was effective. Although the patient's stress level may be lowered by use of valerian, the problem is insomnia; outcomes should relate to the problem. Early morning waking is indicative of depression or anxiety.

2. A team of nurses report to the community after a category 5 hurricane devastates many homes and businesses. The nurses provide emergency supplies of insulin to persons with diabetes and help transfer patients in skilled nursing facilities to sites that have electrical power. Which aspects of disaster management have these nurses fulfilled? (Select all that apply.) a. Preparedness b. Mitigation c. Response d. Recovery e. Evaluation

ANS: B, C This community has experienced a catastrophic event. There are five phases of the disaster management continuum. The nurses' activities applied to mitigation (attempts to limit a disaster's impact on human health and community function) and response (actual implementation of a disaster plan). Preparedness occurs before an event. Recovery actions focus on stabilizing the community and returning it to its previous status. Evaluation of the response efforts apply to the future.

3. A new patient at the sleep disorders clinic tells the nurse, "I have not slept well in a year, so I never feel good. I do not expect things will ever improve or be any different." Interventions the nurse should consider include (Select all that apply) a. suggesting use of alcohol as a sedative. b. providing instruction in relaxation techniques. c. counseling the patient to address cognitive distortions. d. health teaching regarding factors that influence sleep. e. teaching fatigue-producing activities to become overtired. f. encouraging long daytime naps to compensate for sleep deprivation.

ANS: B, C, D Interventions that could be helpful include teaching relaxation techniques, such as meditation or progressive relaxation, to relieve the tension that sometimes prevents initiation of sleep. Reviewing factors that influence sleep can assist the patient to diagnose and remove barriers to sleep. Cognitive therapy could be helpful in combating the hopelessness verbalized by the patient. Alcohol consumption actually disrupts sleep. Becoming overtired may be a barrier to nighttime sleep. Naps may help replace lost sleep, but lengthy daytime sleep will prevent the patient from sleeping well at night.

1. Which neurotransmitters are most responsible for wakefulness? (Select all that apply.) a. -aminobutyric acid (GABA) b. Norepinephrine c. Acetylcholine d. Dopamine e. Galanin

ANS: B, C, D The neurotransmitters responsible for wakefulness are dopamine, norepinephrine, serotonin, acetylcholine, histamine, glutamate, and hypocretin. GABA and galanin are sleep-promoting neurotransmitters.

1. The next-to-last meeting of an interpersonal therapy group is taking place. The leader should take which actions? (Select all that apply.) a. Support appropriate expressions of disagreement by the group's members. b. Facilitate discussion and resolution of feelings about the end of the group. c. Encourage members to reflect on their progress and that of the group itself. d. Remind members of the group's norms and rules, emphasizing confidentiality. e. Help members identify goals they would like to accomplish after the group ends. f. Promote the identification and development of new options for solving problems.

ANS: B, C, E The goals for the termination phase of groups are to prepare the group for separation, resolve related feelings, and prepare each member for the future. Contributions and accomplishments of members are elicited, post-group goals are identified, and feelings about the group's ending are discussed. Group norms are the focus of the orientation phase, and conflict and problem solving are emphasized in the working phase.

5. A rape victim tells the emergency nurse, "I feel so dirty. Help me take a shower before I get examined." The nurse should (Select all that apply.) a. arrange for the victim to shower. b. explain that bathing destroys evidence. c. give the victim a basin of water and towels. d. offer the victim a shower after evidence is collected. e. explain that bathing facilities are not available in the emergency department.

ANS: B, D As uncomfortable as the victim may be, she should not bathe until the examination is completed. Collection of evidence is critical for prosecution of the attacker. Showering after the examination will provide comfort to the victim. The distracters will result in destruction of evidence or are untrue.

3. A novice nurse tells a mentor, "I want to convey to my patients that I am interested in them and that I want to listen to what they have to say." Which behaviors will be helpful in meeting the nurse's goal? (Select all that apply.) a. Sitting behind a desk, facing the patient b. Introducing self to a patient and identifying own role c. Maintaining control of discussions by asking direct questions d. Using facial expressions to convey interest and encouragement e. Assuming an open body posture and sometimes mirror imaging

ANS: B, D, E Trust is fostered when the nurse gives an introduction and identifies his or her role. Facial expressions that convey interest and encouragement support the nurse's verbal statements to that effect and strengthen the message. An open body posture conveys openness to listening to what the patient has to say. Mirror imaging enhances patient comfort. A desk would place a physical barrier between the nurse and patient. A face-to-face stance should be avoided when possible and a less intense 90- or 120-degree angle used to permit either party to look away without discomfort.

2. A leader begins the discussion at the first meeting of a new group. Which comments should be included? (Select all that apply.) a. "We use groups to provide treatment because it's a more cost-effective use of staff in this time of budget constraints." b. "When someone shares a personal experience, it's important to keep the information confidential." c. "Talking to family members about our group discussions will help us achieve our goals." d. "Everyone is expected to share a personal experience at each group meeting." e. "It is important for everyone to arrive on time for our group."

ANS: B, E The leader must set ground rules for the group before members can effectively participate. Confidentiality of personal experiences should be maintained. Arriving on time is important to the group process. Talking to family members would jeopardize confidentiality. While groups are cost-effective, blaming the budget would not help members feel valued. Setting an expectation to share may be intimidating for a withdrawn patient.

11. An older adult drove to a nearby store but was unable to remember how to get home or state an address. When police intervened, they found that this adult was wearing a heavy coat and hat, even though it was July. Which stage of Alzheimer's disease is evident? a. Preclinical Alzheimer's disease b. Mild cognitive decline c. Moderately severe cognitive decline d. Severe cognitive decline

ANS: C In the moderately severe stage, deterioration is evident. Memory loss may include the inability to remember addresses or the date. Activities such as driving may become hazardous, and frustration by the increasing difficulty of performing ordinary tasks may be experienced. The individual has difficulty with clothing selection. Mild cognitive decline (early-stage) Alzheimer's can be diagnosed in some, but not all, individuals. Symptoms include misplacing items and misuse of words. In the stage of severe cognitive decline, personality changes may take place, and the patient needs extensive help with daily activities. This patient has symptoms, so the preclinical stage does not apply.

9. Which medication prescribed to patients diagnosed with Alzheimer's disease antagonizes N-Methyl-D-Aspartate (NMDA) channels rather than cholinesterase? a. Donepezil (Aricept) b. Rivastigmine (Exelon) c. Memantine (Namenda) d. Galantamine (Razadyne)

ANS: C Memantine blocks the NMDA channels and is used in moderate-to-late stages of the disease. Donepezil, rivastigmine, and galantamine are all cholinesterace inhibitors. These drugs increase the availability of acetylcholine and are most often used to treat mild-to-moderate Alzheimer's disease.

21. Which behavior shows that a nurse values autonomy? The nurse a. suggests one-on-one supervision for a patient who has suicidal thoughts. b. informs a patient that the spouse will not be in during visiting hours. c. discusses options and helps the patient weigh the consequences. d. sets limits on a patient's romantic overtures toward the nurse.

ANS: C A high level of valuing is acting on one's belief. Autonomy is supported when the nurse helps a patient weigh alternatives and their consequences before the patient makes a decision. Autonomy or self-determination is not the issue in any of the other behaviors.

11. As a nurse escorts a patient being discharged after treatment for major depression, the patient gives the nurse a necklace with a heart pendant and says, "Thank you for helping mend my broken heart." Which is the nurse's best response? a. "Accepting gifts violates the policies and procedures of the facility." b. "I'm glad you feel so much better now. Thank you for the beautiful necklace." c. "I'm glad I could help you, but I can't accept the gift. My reward is seeing you with a renewed sense of hope." d. "Helping people is what nursing is all about. It's rewarding to me when patients recognize how hard we work."

ANS: C Accepting a gift creates a social rather than therapeutic relationship with the patient and blurs the boundaries of the relationship. A caring nurse will acknowledge the patient's gesture of appreciation, but the gift should not be accepted. See relationship to audience response question.

27. Which comment best indicates that a patient perceived the nurse was caring? "My nurse a. always asks me which type of juice I want to help me swallow my medication." b. explained my treatment plan to me and asked for my ideas about how to make it better." c. spends time listening to me talk about my problems. That helps me feel like I am not alone." d. told me that if I take all the medicines the doctor prescribes, then I will get discharged sooner."

ANS: C Caring evidences empathetic understanding as well as competency. It helps change pain and suffering into a shared experience, creating a human connection that alleviates feelings of isolation. The distracters give examples of statements that demonstrate advocacy or giving advice.

15. A soldier returns to the United States from active duty in a combat zone. The soldier is diagnosed with PTSD. The nurse's highest priority is to screen this soldier for a. bipolar disorder. b. schizophrenia. c. depression. d. dementia.

ANS: C Comorbidities for adults with PTSD include depression, anxiety disorders, sleep disorders, and dissociative disorders. Incidence of the disorders identified in the distracters is similar to the general population.

10. A patient states, "I feel detached and weird all the time. It is as though I am looking at life through a cloudy window. Everything seems unreal. It really messes up things at work and school." This scenario is most suggestive of which health problem? a. Acute stress disorder b. Dissociative amnesia c. Depersonalization disorder d. Disinhibited social engagement disorder

ANS: C Depersonalization disorder involves a persistent or recurrent experience of feeling detached from and outside oneself. Although reality testing is intact, the experience causes significant impairment in social or occupational functioning and distress to the individual. Dissociative amnesia involves memory loss. Children with disinhibited social engagement disorder demonstrate no normal fear of strangers and are unusually willing to go off with strangers. Individuals with ASD (Acute Stress Disorder) experience three or more dissociative symptoms associated with a traumatic event, such as a subjective sense of numbing, detachment, or absence of emotional responsiveness; a reduction in awareness of surroundings; derealization; depersonalization or dissociative amnesia. In the scenario, the patient experiences only one symptom.

7. What is the desirable outcome for the orientation stage of a nurse-patient relationship? The patient will demonstrate behaviors that indicate a. self-responsibility and autonomy. b. a greater sense of independence. c. rapport and trust with the nurse. d. resolved transference.

ANS: C Development of rapport and trust is necessary before the relationship can progress to the working phase. Behaviors indicating a greater sense of independence, self-responsibility, and resolved transference occur in the working phase.

5. A college student who failed two tests cried for hours and then tried to telephone a parent but got no answer. The student then gave several expensive sweaters to a roommate and asked to be left alone for a few hours. Which behavior provides the strongest clue of an impending suicide attempt? a. Calling parents b. Excessive crying c. Giving away sweaters d. Staying alone in dorm room

ANS: C Giving away prized possessions may signal that the individual thinks he or she will have no further need for the item, such as when a suicide plan has been formulated. Calling parents, remaining in a dorm, and crying do not provide direct clues to suicide.

23. A patient says, "I'm still on restriction, but I want to attend some off-unit activities. Would you ask the doctor to change my privileges?" What is the nurse's best response? a. "Why are you asking me when you're able to speak for yourself?" b. "I will be glad to address it when I see your doctor later today." c. "That's a good topic for you to discuss with your doctor." d. "Do you think you can't speak to a doctor?"

ANS: C Nurses should encourage patients to work at their optimal level of functioning, which in turn promotes autonomy. A nurse does not act for the patient unless it is necessary. Acting for a patient increases feelings of helplessness and dependency.

21. A soldier who served in a combat zone returned to the United States. The soldier's spouse complains to the nurse, "We had planned to start a family, but now he won't talk about it. He won't even look at children." The spouse is describing which symptom associated with PTSD? a. Reexperiencing b. Hyperarousal c. Avoidance d. Psychosis

ANS: C Physiological reactions to reminders of the event that include persistent avoidance of stimuli associated with the trauma results in the individual's avoiding talking about the event or avoiding activities, people, or places that arouse memories of the trauma. Avoidance is exemplified by a sense of foreshortened future and estrangement. There is no evidence this soldier is having hyperarousal or reexperiencing war-related traumas. Psychosis is not evident.

19. Which issues should a nurse address during the first interview with a patient with a psychiatric disorder? a. Trust, congruence, attitudes, and boundaries b. Goals, resistance, unconscious motivations, and diversion c. Relationship parameters, the contract, confidentiality, and termination d. Transference, countertransference, intimacy, and developing resources

ANS: C Relationship parameters, the contract, confidentiality, and termination are issues that should be considered during the orientation phase of the relationship. The remaining options are issues that are dealt with later.

1. A patient has been prescribed a selective serotonin reuptake inhibitor (SSRI) antidepressant. After taking the new medication, the patient states, "This medication isn't working. I don't feel any different." What is the best response by the nurse? a. "I will call your care provider. Perhaps you need a different medication." b. "Don't worry. You can try taking it at a different time of day to help it work better." c. "It usually takes a few weeks for you to notice improvement from this medication." d. "Your life is much better now. You will feel better soon."

ANS: C Seeing a response to antidepressants takes 3 to 6 weeks. No change in medications is indicated at this point of treatment because there is no report of adverse effects from the medication. If nausea is present, taking the medication with food may help, but this is not reported by the patient, so a change in administration time is not needed. Telling a depressed patient that their life is better does not acknowledge their feelings.

3. After several therapeutic encounters with a patient who recently attempted suicide, which occurrence should cause the nurse to consider the possibility of countertransference? a. The patient's reactions toward the nurse seem realistic and appropriate. b. The patient states, "Talking to you feels like talking to my parents." c. The nurse feels unusually happy when the patient's mood begins to lift. d. The nurse develops a trusting relationship with the patient.

ANS: C Strong positive or negative reactions toward a patient or over-identification with the patient indicate possible countertransference. Nurses must carefully monitor their own feelings and reactions to detect countertransference and then seek supervision. Realistic and appropriate reactions from a patient toward a nurse are desirable. One incorrect response suggests transference. A trusting relationship with the patient is desirable. See relationship to audience response question.

6. Termination of a therapeutic nurse-patient relationship has been successful when the nurse a. avoids upsetting the patient by shifting focus to other patients before the discharge. b. gives the patient a personal telephone number and permission to call after discharge. c. discusses with the patient changes that happened during the relationship and evaluates outcomes. d. offers to meet the patient for coffee and conversation three times a week after discharge.

ANS: C Summarizing and evaluating progress help validate the experience for the patient and the nurse and facilitate closure. Termination must be discussed; avoiding discussion by spending little time with the patient promotes feelings of abandonment. Successful termination requires that the relationship be brought to closure without the possibility of dependency-producing ongoing contact.

4. A woman just received notification that her husband died. She approaches the nurse who cared for him during his last hours and says angrily, "If you had given him your undivided attention, he would still be alive." How should the nurse analyze this behavior? a. The comment suggests potential allegations of malpractice. b. In some cultures, grief is expressed solely through anger. c. Anger is an expected emotion in an adjustment disorder. d. The patient had ambivalent feelings about her husband.

ANS: C Symptoms of adjustment disorder run the gamut of all forms of distress including guilt, depression, and anger. Anger may protect the bereaved from facing the devastating reality of loss.

6. A nurse uses the SAD PERSONS scale to interview a patient. This tool provides data relevant to a. current stress level. b. mood disturbance. c. suicide potential. d. level of anxiety.

ANS: C The SAD PERSONS tool evaluates 10 major risk factors in suicide potential: sex, age, depression, previous attempt, ethanol use, rational thinking loss, social supports lacking, organized plan, no spouse, and sickness. The tool does not have categories to provide information on the other options listed.

13. A person runs from a crowded nightclub after a pyrotechnics show causes the building to catch fire. Which division of the autonomic nervous system will be stimulated in response to this experience? a. Limbic system b. Peripheral nervous system c. Sympathetic nervous system d. Parasympathetic nervous system

ANS: C The autonomic nervous system is comprised of the sympathetic (fight or flight response) and parasympathetic nervous system (relaxation response). In times of stress, the sympathetic nervous system is stimulated. A person would experience stress associated with the experience of being in danger. The peripheral nervous system responds to messages from the sympathetic nervous system. The limbic system processes emotional responses but is not specifically part of the autonomic nervous system.

12. A nurse and patient construct a no-suicide contract. Select the preferable wording. a. "I will not try to harm myself during the next 24 hours." b. "I will not make a suicide attempt while I am hospitalized." c. "For the next 24 hours, I will not in any way attempt to harm or kill myself." d. "I will not kill myself until I call my primary nurse or a member of the staff."

ANS: C The correct answer leaves no loopholes. The wording about not harming oneself and not making an attempt leaves loopholes or can be ignored by the patient who thinks "I am not going to harm myself, I am going to kill myself" or "I am not going to attempt suicide, I am going to commit suicide." A patient may call a therapist and leave the telephone to carry out the suicidal plan.

18. Select the best question for the nurse to ask to assess a family's ability to cope. a. "What strengths does your family have?" b. "Do you think your family copes effectively?" c. "Describe how you successfully handled one family problem." d. "How do you think the current family problem should be resolved?"

ANS: C The correct option is the only statement addressing coping strategies used by the family. The distracters seek opinions or use closed-ended communication techniques.

14. A nurse interacts with an outpatient who has a history of multiple suicide attempts. Select the most helpful response for a nurse to make when the patient states, "I am considering committing suicide." a. "I'm glad you shared this. Please do not worry. We will handle it together." b. "I think you should admit yourself to the hospital to keep you safe." c. "Bringing up these feelings is a very positive action on your part." d. "We need to talk about the good things you have to live for."

ANS: C The correct response gives the patient reinforcement, recognition, and validation for making a positive response rather than acting out the suicidal impulse. It gives neither advice nor false reassurance, and it does not imply stereotypes such as "You have a lot to live for." It uses the patient's ambivalence and sets the stage for more realistic problem solving.

17. A group is in the working phase. One member says, "That is the stupidest thing I've ever heard. Everyone whines and tells everyone else what to do. This group is a total waste of my time." Which comment by the group leader would be most therapeutic? a. "You seem to think you know a lot already. Since you know so much, perhaps you can tell everyone why you are back in the hospital?" b. "I think you have made your views clear, but I wonder if others feel the same way. How does everyone else feel about our group?" c. "It must be hard to be so angry." Direct this comment to another group member, "You were also angry at first but not now. What has helped you?" d. "I would like to remind you that one of our group rules is that everyone is to offer only positive responses to the comments of others."

ANS: C The member's comments demean the group and its members and suggest that the member is very angry. Labeling the emotion and conveying empathy would be therapeutic. Focusing on members who are likely to be more positive can balance the influence of demoralizing members. "You seem to know a lot ..." conveys hostility from the leader, who confronts and challenges the member to explain how he came to be readmitted if he was so knowledgeable, implying that he is less knowledgeable than he claims. This comment suggests countertransference and is non-therapeutic. Shifting away from the complaining member to see if others agree seeks to have others express disagreement with this member, but that might not happen. In the face of his anger, they might be quiet or afraid to oppose him, or they could respond in kind by expressing hostility themselves. A rule that only positive exchanges are permitted would suppress conflict, reducing the effectiveness of the therapy group.

7. A person intentionally overdosed on antidepressants. Which nursing diagnosis has the highest priority? a. Powerlessness b. Social isolation c. Risk for suicide d. Compromised family coping

ANS: C This diagnosis is the only one with life-or-death ramifications and is therefore of higher priority than the other options.

17. A nurse assesses a patient who reports a 3-week history of depression and periods of uncontrolled crying. The patient says, "My business is bankrupt, and I was served with divorce papers." Which subsequent statement by the patient alerts the nurse to a concealed suicidal message? a. "I wish I were dead." b. "Life is not worth living." c. "I have a plan that will fix everything." d. "My family will be better off without me."

ANS: C Verbal clues to suicide may be overt or covert. The incorrect options are overt references to suicide. The correct option is more veiled. It alludes to the patient's suicide as being a way to "fix everything" but does not say it outright.

4. An adolescent comes to the crisis clinic and reports sexual abuse by an uncle. The adolescent told both parents about the uncle's behavior, but the parents did not believe the adolescent. What type of crisis exists? a. Maturational b. Tertiary c. Situational d. Organic

ANS: C A situational crisis arises from events that are extraordinary, external rather than internal, and often unanticipated. Sexual molestation falls within this classification. Maturational crisis occurs as an individual arrives at a new stage of development, when old coping styles may be ineffective. "Organic" and "Tertiary" are not types of crisis.

3. A patient referred to the eating disorders clinic has lost 35 pounds during the past 3 months. To assess eating patterns, the nurse should ask the patient: a. "Do you often feel fat?" b. "Who plans the family meals?" c. "What do you eat in a typical day?" d. "What do you think about your present weight?"

ANS: C Although all the questions might be appropriate to ask, only "What do you eat in a typical day?" focuses on the eating patterns. Asking if the patient often feels fat focuses on distortions in body image. Questions about family meal planning are unrelated to eating patterns. Asking for the patient's thoughts on present weight explores the patient's feelings about weight.

20. Which referral will be most helpful for a woman who was severely beaten by intimate partner, has no relatives or friends in the community, is afraid to return home, and has limited financial resources? a. A support group b. A mental health center c. A women's shelter d. Vocational counseling

ANS: C Because the woman has no safe place to go, referral to a shelter is necessary. The shelter will provide other referrals as necessary.

6. What is a nurse's legal responsibility if child abuse or neglect is suspected? a. Discuss the findings with the child's parent and health care provider. b. Document the observation and suspicion in the medical record. c. Report the suspicion according to state regulations. d. Continue the assessment.

ANS: C Each state has specific regulations for reporting child abuse that must be observed. The nurse is a mandated reporter. The reporter does not need to be sure that abuse or neglect occurred, only that it is suspected. Speculation should not be documented, only the facts.

19. A nurse conducting group therapy on the eating-disorder unit schedules the sessions immediately after meals for the primary purpose of a. maintaining patients' concentration and attention. b. shifting the patients' focus from food to psychotherapy. c. promoting processing of anxiety associated with eating. d. focusing on weight control mechanisms and food preparation.

ANS: C Eating produces high anxiety for patients with eating disorders. Anxiety levels must be lowered if the patient is to be successful in attaining therapeutic goals. Shifting the patients' focus from food to psychotherapy and focusing on weight control mechanisms and food preparation are not desirable. Maintaining patients' concentration and attention is important, but not the primary purpose of the schedule.

2. An 11-year-old reluctantly tells the nurse, "My parents don't like me. They said they wish I was never born." Which type of abuse is likely? a. Sexual b. Physical c. Emotional d. Economic

ANS: C Examples of emotional abuse include having an adult demean a child's worth, frequently criticize, or belittle the child. No data support physical battering or endangerment, sexual abuse, or economic abuse.

11. An adult consulted a nurse practitioner because of an inability to achieve orgasm for 2 years, despite having been sexually active. This adult was frustrated and expressed concerns about the relationship with the sexual partner. Which documentation best indicates the treatment was successful? a. "No complaints related to sexual function; to return next week." b. "Patient reports achieving orgasm last week; seems very happy." c. "Reports satisfaction with sexual encounters; feels partner is supportive." d. "Reports achieving orgasm occasionally; relationship with partner is adequate."

ANS: C Human sexuality, sexual expression, and expectations related to sexuality vary tremendously from person to person and across cultures. Therefore, the best indication of satisfactory treatment is that the patient is satisfied with what has been achieved. In this instance, "Patient reports satisfaction with sexual encounters; feels partner is supportive" best indicates that the patient is satisfied, and both presenting issues are progressing in a positive manner. Achieving orgasm once or occasionally may or may not represent satisfactory progress to the patient. "No complaints" does not necessarily mean that satisfaction exists.

16. A nurse provides health teaching for a patient diagnosed with bulimia nervosa. Priority information the nurse should provide relates to a. self-monitoring of daily food and fluid intake. b. establishing the desired daily weight gain. c. how to recognize hypokalemia. d. self-esteem maintenance.

ANS: C Hypokalemia results from potassium loss associated with vomiting. Physiological integrity can be maintained if the patient can self-diagnose potassium deficiency and adjust the diet or seek medical assistance. Self-monitoring of daily food and fluid intake is not useful if the patient purges. Daily weight gain may not be desirable for a patient with bulimia nervosa. Self-esteem is an identifiable problem but is of lesser priority than the dangers associated with hypokalemia.

7. Several children are seen in the emergency department for treatment of various illnesses and injuries. Which assessment finding would create the most suspicion for child abuse? The child who has a. complaints of abdominal pain. b. repeated middle ear infections. c. bruises on extremities. d. diarrhea.

ANS: C Injuries such as immersion or cigarette burns, facial fractures, whiplash, bite marks, traumatic injuries, bruises, and fractures in various stages of healing suggest the possibility of abuse. In older children, vague complaints such as back pain may also be suspicious. Ear infections, diarrhea, and abdominal pain are problems that were unlikely to have resulted from violence.

5. A patient is pacing the hall near the nurses' station, swearing loudly. An appropriate initial intervention for the nurse would be to address the patient by name and say: a. "What is going on?" b. "Please be quiet and sit down in this chair immediately." c. "I'd like to talk with you about how you're feeling right now." d. "You must go to your room and try to get control of yourself."

ANS: C Intervention should begin with analysis of the patient and the situation. When anger is escalating, a patient's ability to process decreases. It is important to speak to the patient slowly and in short sentences, using a low and calm voice. Use open-ended statements designed to hear the patient's feelings and concerns. This leads to the next step of planning an intervention.

16. After celebrating the fortieth birthday, an individual becomes concerned with the loss of youthful appearance. What type of crisis has occurred? a. Reactive b. Situational c. Maturational d. Body image

ANS: C Maturational crises occur when a person arrives at a new stage of development and finds that old coping styles are ineffective but has not yet developed new strategies. Situational crises arise from sources external to the individual. "Reactive" and "body image" are not types of crisis.

20. Which medication from the medication administration record should a nurse administer to provide immediate intervention for a psychotic patient whose aggressive behavior continues to escalate despite verbal intervention? a. Lithium b. Trazodone c. Olanzapine d. Valproic acid

ANS: C Olanzapine is a short-acting antipsychotic useful in calming angry, aggressive patients regardless of diagnosis. The other drugs listed require long-term use to reduce anger. Lithium is for bipolar patients. Trazodone is commonly prescribed for patients experiencing depression, insomnia, or chronic pain. Valproic acid is for bipolar or borderline patients.

1. Over the past year, a woman has cooked gourmet meals for her family but eats only tiny servings. This person wears layered loose clothing. Her current weight is 95 pounds, a loss of 35 pounds. Which medical diagnosis is most likely? a. Binge eating b. Bulimia nervosa c. Anorexia nervosa d. Eating disorder not otherwise specified

ANS: C Overly controlled eating behaviors, extreme weight loss, preoccupation with food, and wearing several layers of loose clothing to appear larger are part of the clinical picture of an individual with anorexia nervosa. The individual with bulimia usually is near normal weight. The binge eater is often overweight. The patient with eating disorder not otherwise specified may be obese.

2. Which clinical scenario predicts the highest risk for directing violent behavior toward others? a. Major depressive disorder with delusions of worthlessness b. Obsessive-compulsive disorder; performs many rituals c. Paranoid delusions of being followed by alien monsters d. Completed alcohol withdrawal; beginning a rehabilitation program

ANS: C Patients who are delusional, hyperactive, impulsive, or predisposed to irritability are at higher risk for violence. The patient in the correct response has the greatest disruption of ability to perceive reality accurately. People who feel persecuted may strike out against those believed to be persecutors. The other patients have better reality-testing ability.

23. Family members describe the patient as "a difficult person who finds fault with others." The patient verbally abuses nurses for their poor care. The most likely explanation lies in a. poor childrearing that did not teach respect for others. b. automatic thinking leading to cognitive distortions. c. a personality style that externalizes problems. d. delusions that others wish to deliver harm.

ANS: C Patients whose personality style causes them to externalize blame see the source of their discomfort and anxiety as being outside themselves. They displace anger and are often unable to self-soothe. The incorrect options are less likely to have a bearing on this behavior.

12. Which characteristic fits the usual profile of an individual diagnosed with pedophilic disorder? a. Homosexual b. Ritualistic behaviors c. Seeks access to children d. Self-confident professional

ANS: C Persons with pedophilic disorder usually place themselves in jobs, activities, or relationships that provide easy access to children. They often become trusted by both parents and children. The other characteristics have no particular relationship to pedophilic disorder.

10. A rape victim visited a rape crisis counselor weekly for 8 weeks. At the end of this counseling period, which comment by the victim best demonstrates that reorganization was successful and the victim is now in recovery? a. "I have a rash on my buttocks. It itches all the time." b. "Now I know what I did that triggered the attack on me." c. "I'm sleeping better although I still have an occasional nightmare." d. "I have lost 8 pounds since the attack, but I needed to lose some weight."

ANS: C Rape-trauma syndrome is a variant of posttraumatic stress disorder. The absence of signs and symptoms of posttraumatic stress disorder suggest that the long-term reorganization phase was successfully completed. The victim's sleep has stabilized; occasional nightmares occur, even in reorganization. The distracters suggest somatic symptoms, appetite disturbances, and self-blame, all of which are indicators that the process is ongoing.

4. A nurse provides health education for an adult with sleep deprivation. It is most important for the nurse to encourage caution when the patient engages in a. using a vacuum cleaner. b. cooking a meal. c. driving a car. d. bathing.

ANS: C Safety is the highest priority for this patient. Sleep deprivation causes psychomotor deficits. Driver drowsiness and fatigue lead to many automobile injuries and fatalities. The distracters are less likely to be associated with serious injury.

7. Six months ago, a woman had a prophylactic double mastectomy because of a family history of breast cancer. One week ago, this woman learned her husband was involved in an extramarital affair. The woman tearfully says to the nurse, "What else can happen?" What type of crisis is this person experiencing? a. Maturational b. Mitigation c. Situational d. Recurring

ANS: C Severe physical or mental illness is a potential cause of a situational crisis. The potential loss of a loved one also serves as a potential cause of a situational crisis. Maturational crisis occurs as an individual arrives at a new stage of development, when old coping styles may be ineffective. No classification of recurring crisis exists. Mitigation refers to attempts to limit a disaster's impact on human health and community function.

19. A patient's medical record documents sexual masochism. This patient derives sexual pleasure a. from inanimate objects. b. by inflicting pain on a partner. c. when sexually humiliated by a partner. d. from touching a nonconsenting person.

ANS: C Sexual masochism is sexual pleasure derived from being humiliated, beaten, or otherwise made to suffer. The distracters refer to fetishism, sexual sadism, and frotteurism.

6. A patient says, "It takes me about 15 minutes to go to sleep each night." This comment describes a. delta sleep. b. parasomnia. c. sleep latency. d. REM sleep.

ANS: C Sleep latency refers to the amount of time it takes a person to fall asleep. The distracters represent other phases of the sleep cycle.

21. Which patient would most likely benefit from taking St. John's wort? A patient with a. mood swings. b. hypomanic symptoms. c. mild depressive symptoms. d. panic disorder with agoraphobia.

ANS: C St. John's wort may be effective in treating mild to moderate depression. St John's wort has not been found to be effective in treatment of cyclothymic, bipolar, or anxiety disorders.

15. A patient with multi-infarct dementia lashes out and kicks at people who walk past in the hall of a skilled nursing facility. Intervention by the nurse should begin by a. gently touching the patient's arm. b. asking the patient, "What do you need?" c. saying to the patient, "This is a safe place." d. directing the patient to cease the behavior.

ANS: C Striking out usually signals fear or that the patient perceives the environment to be out of control. Getting the patient's attention is fundamental to intervention. The nurse should make eye contact and assure the patient of safety. Once the nurse has the patient's attention, gently touching the patient, asking what he or she needs, or directing the patient to discontinue the behavior may be appropriate.

25. At the last contracted visit in the crisis intervention clinic, an adult says, "I've emerged from this a stronger person. You helped me get my life back in balance." The nurse responds, "I think we should have two more sessions to explore why your reactions were so intense." Which analysis applies? a. The patient is experiencing transference. b. The patient demonstrates need for continuing support. c. The nurse is having difficulty terminating the relationship. d. The nurse is empathizing with the patient's feelings of dependency.

ANS: C Termination is indicated; however, the nurse's remark is clearly an invitation to work on other problems and prolong contact with the patient. The focus of crisis intervention is the problem that precipitated the crisis, not other issues. The scenario does not describe transference. The patient shows no need for continuing support. The scenario does not describe dependency needs.

8. After an assault by a patient, a nurse has difficulty sleeping, startles easily, and is preoccupied with the incident. The nurse said, "That patient should not be allowed to get away with that behavior." Which response poses the greatest barrier to the nurse's ability to provide therapeutic care? a. Startle reactions b. Difficulty sleeping c. A wish for revenge d. Preoccupation with the incident

ANS: C The desire for revenge signals an urgent need for professional supervision to work through anger and counter the aggressive feelings. Feelings of revenge create a risk for harm to the patient. The distracters are normal in a person who was assaulted. They usually are relieved with crisis intervention, help the individual regain a sense of control, and make sense of the event.

11. A patient who is visiting the crisis clinic for the first time asks, "How long will I be coming here?" The nurse's reply should consider that the usual duration of crisis intervention is a. 1 to 2 weeks. b. 3 to 4 weeks. c. 4 to 6 weeks. d. 8 to 12 weeks.

ANS: C The disorganization associated with crisis is so distressing that it usually cannot be tolerated for more than 4 to 6 weeks. If it is not resolved by that time, the individual usually adopts dysfunctional behaviors that reduce anxiety without solving the problem. Crisis intervention can shorten the duration.

17. As a patient admitted to the eating-disorder unit undresses, a nurse observes that the patient's body is covered by fine, downy hair. The patient weighs 70 pounds and is 5'4" tall. Which term should be documented? a. Amenorrhea b. Alopecia c. Lanugo d. Stupor

ANS: C The fine, downy hair noted by the nurse is called lanugo. It is frequently seen in patients with anorexia nervosa. None of the other conditions can be supported by the data the nurse has gathered.

12. An adult tells the nurse, "My partner abuses me when I make mistakes, but I always get an apology and a gift afterward. I've considered leaving but haven't been able to bring myself to actually do it." Which phase in the cycle of violence prevents this adult from leaving? a. Tension-building b. Acute battering c. Honeymoon d. Stabilization

ANS: C The honeymoon stage is characterized by kind, loving behaviors toward the abused spouse when the perpetrator feels remorseful. The victim believes the promises and drops plans to leave or seek legal help. The tension-building stage is characterized by minor violence in the form of abusive verbalization or pushing. The acute battering stage involves the abuser beating the victim. The violence cycle does not include a stabilization stage.

16. A cognitively impaired patient has been a widow for 30 years. This patient frantically tries to leave the facility, saying, "I have to go home to cook dinner before my husband arrives from work." To intervene with validation therapy, the nurse will say: a. "You must come away from the door." b. "You have been a widow for many years." c. "You want to go home to prepare your husband's dinner?" d. "Your husband gets angry if you do not have dinner ready on time?"

ANS: C Validation therapy meets the patient "where she or he is at the moment" and acknowledges the patient's wishes. Validation does not seek to redirect, reorient, or probe. The distracters do not validate the patient's feelings.

23. Which complementary and alternative therapy may be safely combined with traditional Western medicine in the treatment of anxiety disorder? a. Electroconvulsive therapy b. Mega doses of vitamins c. Meditative practices d. Herbal therapy

ANS: C Yoga, meditation, and prayer are considered to be beneficial adjuncts to treatment for anxiety disorder. Research supports this with findings of lower catecholamine levels following meditation. Patient self-reports suggest patient satisfaction, with increased ability to relax. Meditation and spiritual practices have no associated untoward side effects. Herbal therapy and megadoses of vitamins have potential associated side effects and interactions. Electroconvulsive therapy is not CAM.

3. Which nursing diagnoses are most applicable for a patient diagnosed with severe Alzheimer's disease? Select all that apply. a. Acute confusion b. Anticipatory grieving c. Urinary incontinence d. Disturbed sleep pattern e. Risk for caregiver role strain

ANS: C, D, E The correct answers are consistent with problems frequently identified for patients with late-stage Alzheimer's disease. Confusion is chronic, not acute. The patient's cognition is too impaired to grieve.

4. Which experiences are most likely to precipitate PTSD? (Select all that apply). a. A young adult bungee jumped from a bridge with a best friend. b. An 8-year-old child watched an R-rated movie with both parents. c. An adolescent was kidnapped and held for 2 years in the home of a sexual predator. d. A passenger was in a bus that overturned on a sharp curve and tumbled down an embankment. e. An adult was trapped for 3 hours at an angle in an elevator after a portion of the supporting cable breaks.

ANS: C, D, E PTSD usually occurs after a traumatic event that is outside the range of usual experience. Examples are childhood physical abuse, torture/kidnap, military combat, sexual assault, and natural disasters, such as floods, tornados, earthquakes, tsunamis; human disasters, such as a bus or elevator accident; or crime-related events, such being taken hostage. The common element in these experiences is the individual's extraordinary helplessness or powerlessness in the face of such stressors. Bungee jumps by adolescents are part of the developmental task and might be frightening, but in an exhilarating way rather than a harmful way. A child may be disturbed by an R-rated movie, but the presence of the parents would modify the experience in a positive way.

1. A nurse is developing a plan of care for a patient admitted with a diagnosis of bipolar disorder, manic phase. Which nursing diagnoses address priority needs for the patient? (Select all that apply.) a. Risk for caregiver strain b. Impaired verbal communication c. Risk for injury d. Imbalanced nutrition, less than body requirements e. Ineffective coping f. Sleep deprivation

ANS: C, D, F Risk for injury, poor nutrition, and impaired sleep are priority needs of the patient experiencing mania related to their impulsivity, inability to attend to activities of daily living such as diet and hygiene, and disruption of sleep. Caregiver strain is important to be addressed but is not a priority need on admission for the patient. Verbal communication improves when the mania is managed, and racing thoughts return to normal patterns. Ineffective coping will require stabilization of the acute phase along with cognitive therapy over time.

17. A patient diagnosed with Alzheimer's disease calls the fire department saying, "My smoke detectors are going off." Firefighters investigate and discover that the patient misinterpreted the telephone ringing. Which problem is this patient experiencing? a. Hyperorality b. Aphasia c. Apraxia d. Agnosia

ANS: D Agnosia is the inability to recognize familiar objects, parts of one's body, or one's own reflection in a mirror. Hyperorality refers to placing objects in the mouth. Aphasia refers to the loss of language ability. Apraxia refers to the loss of purposeful movements, such as being unable to dress.

24. What is the priority need for a patient with late-stage dementia? a. Promotion of self-care activities b. Meaningful verbal communication c. Preventing the patient from wandering d. Maintenance of nutrition and hydration

ANS: D In late-stage dementia, the patient often seems to have forgotten how to eat, chew, and swallow. Nutrition and hydration needs must be met if the patient is to live. The patient is incapable of self-care, ambulation, or verbal communication.

3. A patient with fluctuating levels of consciousness, disturbed orientation, and perceptual alteration begs, "Someone get these bugs off me." What is the nurse's best response? a. "No bugs are on your legs. You are having hallucinations." b. "I will have someone stay here and brush off the bugs for you." c. "Try to relax. The crawling sensation will go away sooner if you can relax." d. "I don't see any bugs, but I can tell you are frightened. I will stay with you."

ANS: D When hallucinations are present, the nurse should acknowledge the patient's feelings and state the nurse's perception of reality, but not argue. Staying with the patient increases feelings of security, reduces anxiety, offers the opportunity for reinforcing reality, and provides a measure of physical safety. Denying the patient's perception without offering help does not support the patient emotionally. Telling the patient to relax makes the patient responsible for self-soothing. Telling the patient that someone will brush the bugs away supports the perceptual distortions.

5. A patient who is taking prescribed lithium carbonate is exhibiting signs of diarrhea, blurred vision, frequent urination, and an unsteady gait. Which serum lithium level would the nurse expect for this patient? a. 0 to 0.5 mEq/L b. 0.6 to 0.9 mEq/L c. 1.0 to 1.4 mEq/L d. 1.5 or higher mEq/L

ANS: D Diarrhea, blurred vision, ataxia, and polyuria are all signs of lithium toxicity, which generally occurs at serum levels above 1.5 mEq/L. Serum levels within the normal range of 0.8 to 1.4 mEq/L are not likely to cause signs of toxicity.

21. A parent says, "My son and I argue constantly since he started using drugs. When I talk to him about not using drugs, he tells me to stay out of his business." What is the nurse's first most appropriate action? a. Educate the parent about stages of family development. b. Report the son to law enforcement authorities. c. Refer the son for substance abuse treatment. d. Make a referral for family therapy.

ANS: D Family therapy is indicated, and the nurse should provide a referral. Reporting the child to law enforcement would undermine trust and violate confidentiality. The other distracters may occur later.

22. A soldier returned home last year after deployment to a war zone. The soldier's spouse complains, "We were going to start a family, but now he won't talk about it. He will not look at children. I wonder if we're going to make it as a couple." Select the nurse's best response. a. "Posttraumatic stress disorder (PTSD) often changes a person's sexual functioning." b. "I encourage you to continue to participate in social activities where children are present." c. "Have you talked with your spouse about these reactions? Sometimes we just need to confront behavior." d. "Posttraumatic stress disorder often strains relationships. Here are some community resources for help and support."

ANS: D PTSD precipitates changes that can lead to divorce. It is important to provide support to both the veteran and spouse. Confrontation will not be effective. While it is important to provide information, on-going support will be more effective.

6. A child drowned while swimming in a local lake 2 years ago. Which behavior indicates the child's parents have adapted to their loss? The parents a. visit their child's grave daily. b. maintain their child's room as the child left it 2 years ago. c. keep a place set for the dead child at the family dinner table. d. throw flowers on the lake at each anniversary date of the accident.

ANS: D Resilience refers to positive adaptation or the ability to maintain or regain mental health despite adversity. Loss of a child is among the highest risk situations for an adjustment disorder and maladaptive grieving. The parents who throw flowers on the lake on each anniversary date of the accident are openly expressing their feelings. The other behaviors are maladaptive because of isolating themselves and/or denying their feelings. After 2 years, the frequency of visiting the grave should have decreased.

22. Which scenario best demonstrates a healthy family? a. One parent takes care of children. The other parent earns income and maintains the home. b. A family has strict boundaries that require members to address problems within the family. c. A couple requires their adolescent children to attend church services three times a week. d. A couple renews their marital relationship after their children become adults.

ANS: D Revamping the marital relationship after children move out of the family of origin indicates the family is moving through its stages of development. Strict family boundaries or roles interfere with flexibility and use of outside resources. Adolescents should have some input into deciding their activities.

15. A nurse caring for a withdrawn, suspicious patient recognizes development of feelings of anger toward the patient. The nurse should a. suppress the angry feelings. b. express the anger openly and directly with the patient. c. tell the nurse manager to assign the patient to another nurse. d. discuss the anger with a clinician during a supervisory session.

ANS: D The nurse is accountable for the relationship. Objectivity is threatened by strong positive or negative feelings toward a patient. Supervision is necessary to work through countertransference feelings.

7. A store clerk was killed during a robbery 2 weeks ago. His widow, who has a long history of schizoaffective disorder, cries spontaneously when talking about his death. Select the nurse's most therapeutic response. a. "Are you taking your medications the way they are prescribed?" b. "This loss is harder to accept because of your mental illness. Do you think you should be hospitalized?" c. "I'm worried about how much you are crying. Your grief over your husband's death has gone on too long." d. "The unexpected death of your husband is very painful. I'm glad you are able to talk about your feelings."

ANS: D The patient is expressing feelings related to the loss, and this is an expected and healthy behavior. This patient is at risk for a maladaptive response because of the history of a serious mental illness, but the nurse's priority intervention is to form a therapeutic alliance and support the patient's expression of feelings. Crying at 2 weeks after his death is expected and normal.

18. A patient says, "People should be allowed to commit suicide without interference from others." A nurse replies, "You're wrong. Nothing is bad enough to justify death." What is the best analysis of this interchange? a. The patient is correct. b. The nurse is correct. c. Neither person is correct. d. Differing values are reflected in the two statements.

ANS: D Values guide beliefs and actions. The individuals stating their positions place different values on life and autonomy. Nurses must be aware of their own values and be sensitive to the values of others.

5. A nurse is talking with a patient, and 5 minutes remain in the session. The patient has been silent most of the session. Another patient comes to the door of the room, interrupts, and says to the nurse, "I really need to talk to you." The nurse should a. invite the interrupting patient to join in the session with the current patient. b. say to the interrupting patient, "I am not available to talk with you at the present time." c. end the unproductive session with the current patient and spend time with the interrupting patient. d. tell the interrupting patient, "This session is 5 more minutes; then I will talk with you."

ANS: D When a specific duration for sessions has been set, the nurse must adhere to the schedule. Leaving the first patient would be equivalent to abandonment and would destroy any trust the patient had in the nurse. Adhering to the contract demonstrates that the nurse can be trusted and that the patient and the sessions are important. The incorrect responses preserve the nurse-patient relationship with the silent patient but may seem abrupt to the interrupting patient, abandon the silent patient, or fail to observe the contract with the silent patient.

23. A nurse finds a patient diagnosed with anorexia nervosa vigorously exercising before gaining the agreed-upon weekly weight. Which response by the nurse is appropriate? a. "You and I will have to sit down and discuss this problem." b. "It bothers me to see you exercising. I am afraid you will lose more weight." c. "Let's discuss the relationship between exercise, weight loss, and the effects on your body." d. "According to our agreement, no exercising is permitted until you have gained a specific amount of weight."

ANS: D A matter-of-fact statement that the nurse's perceptions are different will help to avoid a power struggle. Treatment plans have specific goals for weight restoration. Exercise is limited to promote weight gain. Patients must be held accountable for required behaviors. 24. Which nursing diagnosis is more appropriate for a patient diagnosed with anorexia nervosa who restricts intake and is 20% below normal weight than for a 130-pound patient diagnosed with bulimia nervosa who purges? a. Powerlessness b. Ineffective coping c. Disturbed body image d. Imbalanced nutrition: less than body requirements* ANS: D The patient with bulimia nervosa usually maintains a close to normal weight, whereas the patient with anorexia nervosa may approach starvation. The incorrect options may be appropriate for patients with either anorexia nervosa or bulimia nervosa.

8. A rape victim tells the nurse, "I should not have been out on the street alone." Select the nurse's most therapeutic response. a. "Rape can happen anywhere." b. "Blaming yourself increases your anxiety and discomfort." c. "You are right. You should not have been alone on the street at night." d. "You feel as though this would not have happened if you had not been alone."

ANS: D A reflective communication technique is most helpful. Looking at one's role in the event serves to explain events that the victim would otherwise find incomprehensible. The distracters discount the victim's perceived role and interfere with further discussion.

2. A woman was found confused and disoriented after being abducted and raped at gunpoint by an unknown assailant. The emergency department nurse makes these observations about the woman: talking rapidly in disjointed phrases, unable to concentrate, indecisive when asked to make simple decisions. What is the woman's level of anxiety? a. Weak b. Mild c. Moderate d. Severe

ANS: D Acute anxiety results from the personal threat to the victim's safety and security. In this case, the patient's symptoms of rapid, dissociated speech, inability to concentrate, and indecisiveness indicate severe anxiety. Weak is not a level of anxiety. Mild and moderate levels of anxiety would allow the patient to function at a higher level.

9. The nursing diagnosis Rape-trauma syndrome applies to a rape victim in the emergency department. Select the most appropriate outcome to achieve before discharging the patient. a. The memory of the rape will be less vivid and less frightening. b. The patient is able to describe feelings of safety and relaxation. c. Symptoms of pain, discomfort, and anxiety are no longer present. d. The patient agrees to a follow-up appointment with a rape victim advocate.

ANS: D Agreeing to keep a follow-up appointment is a realistic short-term outcome. The victim is in the acute phase; the distracters are unlikely to be achieved during the limited time the victim is in an emergency department.

1. The nurse at a university health center leads a dialogue with female freshmen about rape and sexual assault. One student says, "If I avoid strangers or situations where I am alone outside at night, I'll be safe from sexual attacks." Choose the nurse's best response. a. "Your plan is not adequate. You could still be raped or sexually assaulted." b. "I am glad you have this excellent safety plan. Would others like to comment?" c. "It's better to walk with someone or call security when you enter or leave a building." d. "Sexual assaults are more often perpetrated by acquaintances. Let's discuss ways to prevent that."

ANS: D Almost half of female victims have been raped by an acquaintance. The nurse should share this information along with encouraging discussion of safety measures. The distracters fail to provide adequate information or encourage discussion.

4. Which nursing action should occur first regarding a patient who has a problem of sexual dysfunction or sexual disorder? The nurse should a. develop an understanding of human sexual response. b. assess the patient's sexual functioning and needs. c. acquire knowledge of the patient's sexual roles. d. clarify own personal values about sexuality.

ANS: D Before one can be helpful to patients with sexual dysfunctions or disorders, the nurse must be aware of his or her own feelings and values about sex and sexuality. Nurses must keep their personal beliefs separate from their patient care in order to remain objective, professional, and effective. Nurses must be comfortable with the idea that patients have a right to their own values and must avoid criticism and censure. The other options are indicated as well, but self-awareness must precede them to provide the best care.

2. Disturbed body image is a nursing diagnosis established for a patient diagnosed with an eating disorder. Which outcome indicator is most appropriate to monitor? a. Weight, muscle, and fat congruence with height, frame, age, and sex b. Calorie intake is within required parameters of treatment plan c. Weight reaches established normal range for the patient d. Patient expresses satisfaction with body appearance

ANS: D Body image disturbances are considered improved or resolved when the patient is consistently satisfied with his or her own appearance and body function. This is a subjective consideration. The other indicators are more objective but less related to the nursing diagnosis.

24. A troubled adolescent pulled out a gun in a school cafeteria, fatally shot three people and injured many others. Hundreds of parents come to the school after hearing news reports. After police arrest the shooter, which action should occur next? a. Ask police to encircle the school campus with yellow tape to prevent parents from entering. b. Announce over the loudspeakers, "The campus is now secure. Please return to your classrooms." c. Require parents to pass through metal detectors and then allow them to look for their children in the school. d. Designate zones according to the alphabet and direct students to the zones based on their surnames to facilitate reuniting them with their parents.

ANS: D Chaos is likely among students and desperate parents. A directive approach is best. Once the scene is secure, creative solutions are needed. Creating zones by letters of the alphabet will assist anxious parents and their children to unite. Preventing parents from uniting with their children will further incite the situation.

4. A confused older adult patient in a skilled nursing facility was asleep when unlicensed assistive personnel (UAP) entered the room quietly and touched the bed to see if it was wet. The patient awakened and hit the UAP in the face. Which statement best explains the patient's action? a. Older adult patients often demonstrate exaggerations of behaviors used earlier in life. b. Crowding in skilled nursing facilities increases an individual's tendency toward violence. c. The patient learned violent behavior by watching other patients act out. d. The patient interpreted the UAP's behavior as potentially harmful.

ANS: D Confused patients are not always able to evaluate the actions of others accurately. This patient behaved as though provoked by the intrusive actions of the staff.

3. A patient was arrested for breaking windows in the home of a former domestic partner. The patient's history also reveals childhood abuse by a punitive parent, torturing family pets, and an arrest for disorderly conduct. Which nursing diagnosis has priority? a. Risk for injury b. Ineffective coping c. Impaired social interaction d. Risk for other-directed violence

ANS: D Defining characteristics for risk for other-directed violence include a history of being abused as a child, having committed other violent acts, and demonstrating poor impulse control. There is no indicator that the patient will experience injury. Ineffective coping and impaired social interaction have lower priorities.

10. A young adult has recently had multiple absences from work. After each absence, this adult returned to work wearing dark glasses and long-sleeved shirts. During an interview with the occupational health nurse, this adult says, "My partner beat me, but it was because I did not do the laundry." What is the nurse's next action? a. Call the police. b. Arrange for hospitalization. c. Call the adult protective agency. d. Document injuries with a body map.

ANS: D Documentation of injuries provides a basis for possible legal intervention. In most states, the abused adult would need to make the decision to involve the police. Because the worker is not an older adult and is competent, the adult protective agency is unable to assist. Admission to the hospital is not necessary.

8. A woman said, "I can't take anymore! Last year my husband had an affair and now we don't communicate. Three months ago, I found a lump in my breast. Yesterday my daughter said she's quitting college." What is the nurse's priority assessment? a. Identify measures useful to help improve the couple's communication. b. The patient's feelings about the possibility of having a mastectomy c. Whether the husband is still engaged in an extramarital affair d. Clarify what the patient means by "I can't take anymore."

ANS: D During crisis intervention, the priority concern is patient safety. This question helps assess personal coping skills. The other options are incorrect because the focus of crisis intervention is on the event that occurred immediately before the patient sought help.

14. Before a victim of sexual assault is discharged from the emergency department, the nurse should a. notify the victim's family to provide emotional support. b. offer to stay with the patient until stability is regained. c. advise the patient to try not to think about the assault. d. provide referral information verbally and in writing.

ANS: D Immediately after the assault, rape victims are often disorganized and unable to think well or remember instructions. Written information acknowledges this fact and provides a solution. The distracters violate the patient's right to privacy, evidence a rescue fantasy, and offer a platitude that is neither therapeutic nor effective.

12. A student falsely accused a college professor of sexual intimidation. The professor tells the nurse, "I cannot teach nor do any research. My mind is totally preoccupied with these false accusations." What is the priority nursing diagnosis? a. Ineffective denial related to threats to professional identity b. Deficient knowledge related to sexual harassment protocols c. Impaired social interaction related to loss of teaching abilities d. Ineffective coping related to distress from false accusations

ANS: D Ineffective coping may be evidenced by inability to meet basic needs, inability to meet role expectations. This nursing diagnosis is the priority because it reflects the consequences of the precipitating event associated with the professor's crisis. There is no evidence of denial. Deficient knowledge may apply, but it is not the priority. Data are not present to diagnose impaired social interaction.

6. While performing an assessment, the nurse says to a patient, "While growing up, most of us heard some half-truths about sexual matters that continue to puzzle us as adults. Do any come to your mind now?" The purpose of this question is to a. identify areas of sexual dysfunction for treatment. b. determine possible homosexual urges. c. introduce the topic of masturbation. d. identify sexual misinformation.

ANS: D Misinformation about normal sex and sexuality is common. Lack of knowledge may affect an individual's sexual adjustment. Once myths have been identified, the nurse can give information to dispel the myth.

6. Outpatient treatment is planned for a patient diagnosed with anorexia nervosa. Select the most important desired outcome related to the nursing diagnosis Imbalanced nutrition: less than body requirements. Within 1 week, the patient will a. weigh self accurately using balanced scales. b. limit exercise to less than 2 hours daily. c. select clothing that fits properly. d. gain 1 to 2 pounds.

ANS: D Only the outcome of a gain of 1 to 2 pounds can be accomplished within 1 week when the patient is an outpatient. The focus of an outcome would not be on the patient weighing self. Limiting exercise and selecting proper clothing are important, but weight gain takes priority.

16. Which statement about paraphilic disorders is accurate? a. Paraphilic behavior is controllable by willpower, but most persons with these disorders fail to do so. b. Persons with paraphilic disorders rarely experience shame and are not distressed by their acts. c. Persons with paraphilic disorders prey primarily on female children between the ages of 12 and 15 years. d. Acts of paraphilia are common because persons with the disorders commit the acts repeatedly, but paraphilic disorders are uncommon.

ANS: D Paraphilic disorders are uncommon; however, because persons with these disorders repeatedly enact behaviors associated with their disorders, paraphilic acts are relatively common. The majority of victims of pedophiles are males in early adolescence; those pedophiles who prefer females usually prefer prepubescent children. Some persons with paraphilic disorders experience shame and are at higher risk for suicide due to the stigma, shame, and embarrassment. Biological and psychological drives underlying paraphilic behavior can be very strong and often are not controllable by willpower alone. Persons with paraphilic disorders have difficulty controlling their behavior, even when very motivated to do so.

15. A patient with rheumatoid arthritis reports, "For the past month I've had trouble falling asleep. When I finally get to sleep, I wake up several times during the night." Which information should the nurse seek initially? a. "What have you done to try to improve your sleep?" b. "What would be a good sleep pattern for you?" c. "How much exercise are you getting?" d. "Do you have pain at night?"

ANS: D Patients with diseases such as arthritis may have sleep disturbance related to nightly pain. Because the pain is chronic, the patient may fail to realize it is the reason for the inability to sleep. The other options do not follow the patient's lead or begin problem solving without an adequate baseline.

23. A young adult says to the nurse, "I go to sleep without any problem, but I often wake up during the night because it feels like there are rubber bands in my legs." Which assessment question should the nurse ask to assess for restless legs syndrome (RLS)? a. "What type of birth control do you use?" b. "How much caffeine do you use every day?" c. "How much exercise do you get in a typical day?" d. "Does anyone else in your family have this problem?"

ANS: D RLS is a sensory and movement disorder characterized by an unpleasant, uncomfortable sensation in the legs accompanied by an urge to move. Symptoms begin or worsen during periods of inactivity, such as sleep. Symptoms can have a significant impact on the individual's ability to fall asleep and stay asleep. There is likely to be a strong genetic component, especially when seen in individuals less than 40 years old.

9. The nursing care plan for a patient diagnosed with anorexia nervosa includes the intervention "monitor for complications of refeeding." Which system should a nurse closely monitor for dysfunction? a. Renal b. Endocrine c. Integumentary d. Cardiovascular

ANS: D Refeeding resulting in too-rapid weight gain can overwhelm the heart, resulting in cardiovascular collapse. Focused assessment is a necessity to ensure the patient's physiological integrity. The other body systems are not initially involved in the refeeding syndrome.

10. A person is prescribed sertraline 100 mg PO daily. Which change in sleep is likely secondary to this medication? The patient will have a. more dreams. b. excessive sleepiness. c. less slow-wave sleep. d. less REM sleep.

ANS: D Sertraline (Zoloft) is an SSRI antidepressant medication, which suppresses REM sleep. Dreams would decrease because they occur during REM. Benzodiazepines reduce slow-wave sleep. SSRIs have a side effect of insomnia.

21. A patient tells the nurse, "Everyone says we should sleep 8 hours a night. I can only sleep 6 hours, no matter how hard I try. Am I doing harm to my body?" Select the nurse's best response. a. "Tell me about strategies you have tried to increase your total sleep hours." b. "Lack of sleep acts as a stressor on the body and can cause physical changes." c. "If you have really tried to sleep more, maybe you should consult your health care provider." d. "If you function well with 6 hours of sleep, you are a short sleeper. That's normal for some people."

ANS: D Some individuals require less sleep than others do. Those who need less are called "short sleepers," compared with "long sleepers," who require more than 8 hours. The distracters do not provide information the patient is seeking or are untrue.

22. Which assessment finding for a patient diagnosed with an eating disorder meets criteria for hospitalization? a. Urine output 40 mL/hour b. Pulse rate 58 beats/min c. Serum potassium 3.4 mEq/L d. Systolic blood pressure 62 mm Hg

ANS: D Systolic blood pressure less than 70 mm Hg is an indicator for inpatient care. Many people without eating disorders have bradycardia (pulse less than 60 beats/min). Urine output should be more than 30 mL/hour. A potassium level of 3.4 mEq/L is within the normal range.

10. A psychiatric clinical nurse specialist uses cognitive-behavioral therapy for a patient diagnosed with anorexia nervosa. Which statement by the staff nurse supports this type of therapy? a. "What are your feelings about not eating foods that you prepare?" b. "You seem to feel much better about yourself when you eat something." c. "It must be difficult to talk about private matters to someone you just met." d. "Being thin doesn't seem to solve your problems. You are thin now but still unhappy."

ANS: D The correct response is the only strategy that questions the patient's distorted thinking.

14. A patient sat in silence for 20 minutes after a therapy appointment, appearing tense and vigilant. The patient abruptly stood, paced back and forth, clenched and unclenched fists, and then stopped and stared in the face of a staff member. The patient is a. demonstrating withdrawal. b. working though angry feelings. c. attempting to use relaxation strategies. d. exhibiting clues to potential aggression.

ANS: D The description of the patient's behavior shows the classic signs of someone whose potential for aggression is increasing.

10. A woman says, "I can't take anymore. Last year my husband had an affair and now we do not communicate. Three months ago, I found a lump in my breast. Yesterday my daughter said she's quitting college and moving in with her boyfriend." Which issue should the nurse focus on during crisis intervention? a. The possible mastectomy b. The disordered family communication c. The effects of the husband's extramarital affair d. Coping with the reaction to the daughter's events

ANS: D The focus of crisis intervention is on the most recent problem: "the straw that broke the camel's back." The patient had coped with the breast lesion, the husband's infidelity, and the disordered communication. Disequilibrium occurred only with the introduction of the daughter leaving college and moving.

6. An adult seeks counseling after the spouse was murdered. The adult angrily says, "I hate the beast that did this. It has ruined my life. During the trial, I don't know what I'll do if the jury doesn't return a guilty verdict." What is the nurse's highest priority response? a. "Would you like to talk to a psychiatrist about some medication to help you cope during the trial?" b. "What resources do you need to help you cope with this situation?" c. "Do you have enough support from your family and friends?" d. "Are you having thoughts of hurting yourself or others?"

ANS: D The highest nursing priority is safety. The nurse should assess suicidal and homicidal potential. The distracters are options, but the highest priority is safety.

9. An adult has recently been absent from work for 3-day periods on several occasions. Each time, the individual returned wearing dark glasses. Facial and body bruises were apparent. What is occupational health nurse's priority assessment? a. Interpersonal relationships b. Work responsibilities c. Socialization skills d. Physical injuries

ANS: D The individual should be assessed for possible battering. Physical injuries are abuse indicators and are the primary focus for assessment. No data support the other options.

19. A patient with severe burn injuries is irritable, angry, and belittles the nurses. As a nurse changes a dressing, the patient screams, "Don't touch me! You are so stupid. You will make it worse!" Which action by the nurse will best help to diffuse the patient's anger? a. Stop the dressing change and say, "I will leave the supplies so that you can change your own dressing." b. Continue the dressing change and say, "This dressing change is necessary because you were careless with fire." c. Discontinue the dressing change, tell the patient, "I will return when you gain control of yourself," and leave the room. d. Continue the dressing change and say, "Dressing changes are needed to prevent infection. What are your ideas about how to make it less painful?"

ANS: D The nurse should not respond personally to the patient's comments. The correct answer objectively gives the patient information that may lead to lowering his anger and engages the patient in problem solving. The incorrect options will escalate the patient's anger by belittling or escalating the patient's sense of powerlessness. Dressing changes are needed for the patient's physiological integrity; therefore, the nurse should not abandon the responsibility to perform them.

22. Which information from a patient's record would indicate marginal coping skills and the need for careful assessment of the risk for violence? A history of a. academic problems. b. family involvement. c. childhood trauma. d. substance abuse.

ANS: D The nurse should suspect marginal coping skills in a patient with substance abuse. They are often anxious, may be concerned about inadequate pain relief, and may have personality styles that externalize blame. The incorrect options do not signal as high a degree of risk as substance abuse.

13. A nursing diagnosis for a patient diagnosed with bulimia nervosa is Ineffective coping related to feelings of loneliness as evidenced by overeating to comfort self, followed by self-induced vomiting. The best outcome related to this diagnosis is that within 2 weeks the patient will a. appropriately express angry feelings. b. verbalize two positive things about self. c. verbalize the importance of eating a balanced diet. d. identify two alternative methods of coping with loneliness.

ANS: D The outcome of identifying alternative coping strategies is most directly related to the diagnosis of Ineffective coping. Verbalizing positive characteristics of self and verbalizing the importance of eating a balanced diet are outcomes that might be used for other nursing diagnoses. Appropriately expressing angry feelings is not measurable.

5. A patient was diagnosed with anorexia nervosa. The history shows the patient virtually stopped eating 5 months ago and lost 25% of body weight. The serum potassium is currently 2.7 mg/dL. Which nursing diagnosis applies? a. Adult failure to thrive related to abuse of laxatives as evidenced by electrolyte imbalances and weight loss b. Disturbed energy field related to physical exertion in excess of energy produced through caloric intake as evidenced by weight loss and hyperkalemia c. Ineffective health maintenance related to self-induced vomiting as evidenced by swollen parotid glands and hyperkalemia d. Imbalanced nutrition: less than body requirements related to reduced oral intake as evidenced by loss of 25% of body weight and hypokalemia

ANS: D The patient's history and lab result support the nursing diagnosis Imbalanced nutrition: less than body requirements. Data are not present that the patient uses laxatives, induces vomiting, or exercises excessively. The patient has hypokalemia rather than hyperkalemia.

13. An emergency code was called after a patient pulled a knife from a pocket and threatened, "I will kill anyone who tries to get near me." The patient was safely disarmed and placed in seclusion. Justification for use of seclusion was that the patient a. was threatening to others. b. was experiencing psychosis. c. presented an undeniable escape risk. d. presented a clear and present danger to others.

ANS: D The patient's threat to kill self or others with the knife he possessed constituted a clear and present danger to self and others. The distracters are not sufficient reasons for seclusion.

18. A patient being admitted to the eating-disorder unit has a yellow cast to the skin and fine, downy hair over the trunk. The patient weighs 70 pounds; height is 5'4". The patient says, "I won't eat until I look thin." Select the priority initial nursing diagnosis. a. Anxiety related to fear of weight gain b. Disturbed body image related to weight loss c. Ineffective coping related to lack of conflict resolution skills d. Imbalanced nutrition: less than body requirements related to self-starvation

ANS: D The physical assessment shows cachexia, which indicates imbalanced nutrition. Addressing the patient's self-starvation is the priority.

17. Which scenario is an example of a situational crisis? a. The death of a child from sudden infant death syndrome b. Development of a heroin addiction c. Retirement of a 55-year-old person d. A riot at a rock concert

ANS: D The rock concert riot is unplanned, accidental, violent, and not a part of everyday life. The incorrect options are examples of maturational crises.

16. A 76-year-old man tells the nurse at the sleep disorder clinic, "I awaken almost nightly in the midst of violent dreams in which I am defending myself against multiple attackers. Then I realize I have been hitting and kicking my wife. She has bruises." Which health problem is most likely? a. Sleep paralysis b. Night terror disorder c. Sleep-related bruxism d. REM Sleep Behavior Disorder

ANS: D The scenario describes REM sleep behavior disorder in which the patient engages in violent and complex behaviors during REM sleep as though acting out his dreams. Older men have a higher incidence of this problem. Sleep paralysis refers to the sudden inability to perform voluntary movement at either sleep onset or awakening from sleep. Bruxism refers to grinding teeth during stage 2 sleep. Night terror disorder occurs as arousal in the first third of the night during NREM sleep, accompanied by feelings of panic.

20. A man with hypospadias tells the nurse, "Intercourse with my new bride is painful." Which term applies to the patient's complaint? a. Delayed ejaculation b. Erectile dysfunction c. Premature ejaculation d. Genito-pelvic pain/penetration disorder

ANS: D This sexual pain is genito-pelvic pain/penetration disorder and may occur in men or women. The individual feels pain in the genitals during intercourse. Erectile or ejaculation problems are not evident. See relationship to audience response question.

5. A patient tells the nurse that his sexual functioning is normal when his wife wears short, red camisole-style nightgowns. He states, "Without the red teddies, I am not interested in sex." The nurse can assess this as consistent with a. exhibitionism. b. voyeurism. c. frotteurism. d. fetishism.

ANS: D To be sexually satisfied, a person with a sexual fetish finds it necessary to have some external object present, in fantasy or in reality. Frotteurism involves deriving sexual pleasure from rubbing against others surreptitiously. Exhibitionism is the intentional display of the genitalia in a public place. Voyeurism refers to viewing others in intimate situations.

8. The male manager of a health club placed a hidden video camera in the women's locker room and recorded several women as they showered and dressed. The disorder most likely represented by this behavior is a. homosexuality. b. exhibitionism. c. pedophilia. d. voyeurism.

ANS: D Voyeurism is achieving sexual pleasure through the viewing of others in intimate situations, such as undressing, bathing, or having sexual relations. A homosexual individual would be interested in watching members of the same sex, and homosexuality is not typically associated with voyeurism. Exhibitionists are interested in exposing their genitals to others. Pedophiles seek sexual contact with children.

A nurse works with a child who is sad and irritable because the child's parents are divorcing. Why is establishing a therapeutic alliance with this child a priority? a. Therapeutic relationships provide an outlet for tension. b. Focusing on the strengths increases a person's self-esteem. c. Acceptance and trust convey feelings of security to the child. d. The child should express feelings rather than internalize them.

Acceptance and trust convey feelings of security to the child.

A client is admitted to an acute care mental health facility. The following medical diagnoses and psychosocial information are available at the time of admission: hypothyroidism, mild mental retardation, bipolar I disorder. The client's highest level of functioning from a global assessment of functioning (GAF) performed a year ago was 45. Today, the highest level of functioning on the same scale is 15. The client has been fighting with other clients frequently at the group home. How should the nurse enter this information into the multiaxial system of the DSM- IV- TR? Axis I: Bipolar I disorder Axis II: Mild mental retardation Axis III: Hypothyroidism Axis IV: Has been fighting with other clients frequently at group home Axis V: GAF 15/45

Axis I includes most mental health clinical disorders, except those placed on Axis II. Axis II disorders include personality disorders and mental retardation. Axis III includes general medical disorders and problems. Axis IV includes pertinent psychosocial information or problems with living conditions. Axis V includes GAF for present assessment and previous assessment within 1 year of present.

A nurse wants to find a description of diagnostic criteria for anxiety disorders. Which resource would have the most complete information? a. Nursing Outcomes Classification (NOC) b. DSM-V c. The ANA's Psychiatric-Mental Health Nursing Scope and Standards of Practice d. ICD-10

B

After a Category 5 tornado hits a community and destroys many homes and businesses, a community mental health nurse encourages victims to describe their memories and feelings about the event. This action by the nurse best demonstrates a. triage. b. primary prevention. c. psychosocial rehabilitation. d. psychiatric case management.

B

Complete this analogy. NANDA : clinical judgment :: NIC : _________________ a. patient outcomes b. nursing actions c. diagnosis d. symptoms

B

Which component of treatment of mental illness is specifically recognized by Quality and Safety Education for Nurses (QSEN)? a. All genomes are unique. b. Care is centered on the patient. c. Healthy development is vital to mental health. d. Recovery occurs on a continuum from illness to health.

B

A nurse wants to teach alternative coping strategies to a patient experiencing severe anxiety. Which action should the nurse perform first? a.Verify the patient's learning style. b.Lower the patient's current anxiety. c.Create outcomes and a teaching plan. d.Assess how the patient uses defense mechanisms.

B A patient experiencing severe anxiety has a markedly narrowed perceptual field and difficulty attending to events in the environment. A patient experiencing severe anxiety will not learn readily. Determining preferred modes of learning, devising outcomes, and constructing teaching plans are relevant to the task but are not the priority measure. The nurse has already assessed the patient's anxiety level. Use of defense mechanisms does not apply.

A patient is experiencing moderate anxiety. The nurse encourages the patient to talk about feelings and concerns. What is the rationale for this intervention? a.Offering hope allays and defuses the patient's anxiety. b.Concerns stated aloud become less overwhelming and help problem solving begin. c.Anxiety is reduced by focusing on and validating what is occurring in the environment. d.Encouraging patients to explore alternatives increases the sense of control and lessens anxiety.

B All principles listed are valid, but the only rationale directly related to the intervention of assisting the patient to talk about feelings and concerns is the one that states that concerns spoken aloud become less overwhelming and help problem solving begin.

A patient performs ritualistic hand washing. Which action should the nurse implement to help the patient develop more effective coping? a.Allow the patient to set a hand-washing schedule. b.Encourage the patient to participate in social activities. c.Encourage the patient to discuss hand-washing routines. d.Focus on the patient's symptoms rather than on the patient.

B Because obsessive-compulsive patients become overly involved in the rituals, promotion of involvement with other people and activities is necessary to improve coping. Daily activities prevent constant focus on anxiety and symptoms. The other interventions focus on the compulsive symptom. See relationship to audience response question.

A woman is 5'7", 160 lbs, and wears a size 8 shoe. She says, "My feet are huge. I've asked three orthopedists to surgically reduce my feet." This person tries to buy shoes to make her feet look smaller and, in social settings, conceals both feet under a table or chair. Which health problem is likely? a.Social anxiety disorder b.Body dysmorphic disorder c.Separation anxiety disorder d.Obsessive-compulsive disorder due to a medical condition

B Body dysmorphic disorder refers to a preoccupation with an imagined defect in appearance in a normal-appearing person. The patient's feet are proportional to the rest of the body. In obsessive-compulsive or related disorder due to a medical condition, the individual's symptoms of obsessions and compulsions are a direct physiological result of a medical condition. Social anxiety disorder, also called social phobia, is characterized by severe anxiety or fear provoked by exposure to a social or a performance situation that will be evaluated negatively by others. People with separation anxiety disorder exhibit developmentally inappropriate levels of concern over being away from a significant other.

A patient preparing for surgery has moderate anxiety and is unable to understand preoperative information. Which nursing intervention is most appropriate? a. Reassure the patient that all nurses are skilled in providing postoperative care. b. Present the information again in a calm manner using simple language. c. Tell the patient that staff is prepared to promote recovery. d. Encourage the patient to express feelings to family.

B Giving information in a calm, simple manner will help the patient grasp the important facts. Introducing extraneous topics as described in the distracters will further scatter the patient's attention.

The nurse assesses a patient who complains of loneliness and episodes of anxiety. Which statement by the patient is mostly likely if this patient also has agoraphobia? a."I'm sure I will get over not wanting to leave home soon. It takes time." b."Being afraid to go out seems ridiculous, but I can't go out the door." c."My family says they like it now that I stay home most of the time." d."When I have a good incentive to go out, I can do it."

B Individuals who are agoraphobic generally acknowledge that the behavior is not constructive and that they do not really like it. The symptom is ego dystonic. However, patients will state they are unable to change the behavior. Agoraphobics are not optimistic about change. Most families are dissatisfied when family members refuse to leave the house.

A patient with an abdominal mass is scheduled for a biopsy. The patient has difficulty understanding the nurse's comments and asks, "What do you mean? What are they going to do?" Assessment findings include tremulous voice, respirations 28, and pulse 110. What is the patient's level of anxiety? a.Mild b.Moderate c.Severe d.Panic

B Moderate anxiety causes the individual to grasp less information and reduces problem-solving ability to a less-than-optimal level. Mild anxiety heightens attention and enhances problem solving. Severe anxiety causes great reduction in the perceptual field. Panic-level anxiety results in disorganized behavior.

Which characteristic would be more applicable to a community mental health nurse than to a nurse working in an operating room? a. Kindness b. Autonomy c. Compassion d. Professionalism

B A community mental health nurse often works autonomously. Kindness, compassion, and professionalism apply to both nurses.

A patient diagnosed with depression repeatedly tells staff, "I have cancer. It's my punishment for being a bad person." Diagnostic tests reveal no cancer. Select the priority nursing diagnosis. a. Powerlessness b. Risk for suicide c. Stress overload d. Spiritual distress

B A patient diagnosed with depression who feels so worthless as to believe cancer is deserved is at risk for suicide. Safety concerns take priority over the other diagnoses listed.

A nurse wants to engage an interpreter for a severely anxious 21-year-old male who immigrated to the United States 2 years ago. Of the four interpreters below who are available and fluent in the patient's language, which one should the nurse call? a. 65-year-old female professional interpreter b. 24-year-old male professional interpreter c. A member of the patient's family d. The patient's best friend

B A professional interpreter will be most effective because he/she will be able to interpret both language and culture. When an interpreter is engaged, the interpreter should be matched to the patient as closely as possible in gender, age, social status, and religion. Interpreters should not be relatives or friends of the patient. The stigma of mental illness may prevent the openness needed during the encounter.

Select the example of a tort. a. The plan of care for a patient is not completed within 24 hours of the patient's admission. b. A nurse gives a prn dose of an antipsychotic drug to an agitated patient because the unit is short-staffed. c. An advanced practice nurse recommends hospitalization for a patient who is dangerous to self and others. d. A patient's admission status changed from involuntary to voluntary after the patient's hallucinations subside.

B A tort is a civil wrong against a person that violates his or her rights. Giving unnecessary medication for the convenience of staff controls behavior in a manner similar to secluding a patient-- thus, false imprisonment is a possible charge.

Which statement by a patient would lead the nurse to suspect unsuccessful completion of the psychosocial developmental task of infancy? a. "I know how to do things right, so I prefer jobs where I work alone rather than on a team." b. "I do not allow other people to truly get to know me." c. "I depend on frequent praise from others to feel good about myself." d. "I usually need to do things several times before I get them right."

B According to Erikson, the developmental task of infancy is the development of trust. The correct response is the only statement clearly showing lack of ability to trust others. An inability to work with others, coupled with a sense of superiority, suggests unsuccessful completion of the task of intimacy versus isolation. Relying on praise from others suggests unsuccessful completion of the task of identity versus role confusion. Shame suggests failure to resolve the crisis of initiative versus guilt.

Which level of prevention activities would a nurse in an emergency department employ most often? a. Primary b. Secondary c. Tertiary

B An emergency department nurse would generally see patients in crisis or with acute illness, so secondary prevention is used. Primary prevention involves preventing a health problem from developing, and tertiary prevention applies to rehabilitative activities.

Priority interventions for a patient diagnosed with major depression and feelings of worthlessness should include: a. distracting the patient from self-absorption. b. careful unobtrusive observation around the clock. c. allowing the patient to spend long periods alone in meditation. d. opportunities to assume a leadership role in the therapeutic milieu.

B Approximately two-thirds of people with depression contemplate suicide. Patients with depression who exhibit feelings of worthlessness are at higher risk. Regular planned observations of the patient diagnosed with depression may prevent a suicide attempt on the unit.

A patient says, "All my life I've been surrounded by stupidity. Everything I buy breaks because the entire American workforce is incompetent." This patient is experiencing a a. self-esteem deficit. b. cognitive distortion. c. deficit in motivation. d. deficit in love and belonging.

B Automatic thoughts, or cognitive distortions, are irrational and lead to false assumptions and misinterpretations. See related audience response question.

A patient says, "I always feel good when I wear a size 2 petite." Which type of cognitive distortion is evident? a. Disqualifying the positive b. Overgeneralization c. Catastrophizing d. Personalization

B Automatic thoughts, or cognitive distortions, are irrational and lead to false assumptions and misinterpretations. The stem offers an example of overgeneralization.

The psychiatric nurse addresses axis I of the DSM as the focus of treatment but must also consider physical health problems that may affect treatment. Which axis contains the desired information? a. II b. III c. IV d. V

B Axis III indicates any relevant general medical conditions. Axis II refers to personality disorders and mental retardation. Together they constitute the classification of abnormal behavior diagnosed in the individual. Axis IV reports psychosocial and environmental problems that may affect the diagnosis, treatment, and prognosis. Axis V is the global assessment of functioning.

An adolescent hospitalized after a violent physical outburst tells the nurse, "I'm going to kill my father, but you can't tell anyone." Select the nurse's best response. a. "You are right. Federal law requires me to keep clinical information private." b. "I am obligated to share that information with the treatment team." c. "Those kinds of thoughts will make your hospitalization longer." d. "You should share this thought with your psychiatrist."

B Breach of nurse-patient confidentiality does not pose a legal dilemma for nurses in these circumstances because a team approach to delivery of psychiatric care presumes communication of patient information to other staff members to develop treatment plans and outcome criteria. The patient should also know that the team has a duty to warn the father of the risk for harm.

A patient diagnosed with major depression tells the nurse, "Bad things that happen are always my fault." Which response by the nurse will best assist the patient to reframe this overgeneralization? a. "I really doubt that one person can be blamed for all the bad things that happen." b. "Let's look at one bad thing that happened to see if another explanation exists." c. "You are being extremely hard on yourself. Try to have a positive focus." d. "Are you saying that you don't have any good things happen?"

B By questioning a faulty assumption, the nurse can help the patient look at the premise more objectively and reframe it as a more accurate representation of fact. The incorrect responses cast doubt but do not require the patient to evaluate the statement.

Which instruction has priority when teaching a patient about clozapine? a. "Avoid unprotected sex." b. "Report sore throat and fever immediately." c. "Reduce foods high in polyunsaturated fats." d. "Use over-the-counter preparations for rashes."

B Clozapine therapy may produce agranulocytosis... therefore, signs of infection should be immediately reported to the health care provider. In addition, the patient should have white blood cell levels measured weekly. The other options are not relevant to clozapine.

By which mechanism do SSRI medications improve depression? a. Destroying increased amounts of serotonin b. Making more serotonin available at the synaptic gap c. Increasing production of acetylcholine and dopamine d. Blocking muscarinic and á1 norepinephrine receptors

B Depression is thought to be related to lowered availability of the neurotransmitter serotonin. SSRIs act by blocking reuptake of serotonin, leaving a higher concentration available at the synaptic cleft. SSRIs prevent destruction of serotonin. SSRIs have little or no effect on acetylcholine and dopamine production. SSRIs do not produce muscarinic or á1 norepinephrine blockade.

What is the focus of priority nursing interventions for the period immediately after electroconvulsive therapy treatment? a. Nutrition and hydration b. Supporting physiological stability c. Reducing disorientation and confusion d. Assisting the patient to identify and test negative thoughts

B During the immediate post-treatment period, the patient is recovering from general anesthesia; hence, the priority need is to establish and support physiological stability. Reducing disorientation and confusion is an acceptable intervention but not the priority. Assisting the patient in identifying and testing negative thoughts is inappropriate in the immediate post-treatment period because the patient may be confused.

Which nursing intervention demonstrates false imprisonment? a. A confused and combative patient says, "I'm getting out of here, and no one can stop me." The nurse restrains this patient without a health care provider's order and then promptly obtains an order. b. A patient has been irritating and attention seeking much of the day. A nurse escorts the patient down the hall saying, "Stay in your room, or you'll be put in seclusion." c. An involuntarily hospitalized patient with suicidal ideation runs out of the psychiatric unit. The nurse rushes after the patient and convinces the patient to return to the unit. d. An involuntarily hospitalized patient with homicidal ideation attempts to leave the facility. A nurse calls the security team and uses established protocols to prevent the patient from leaving.

B False imprisonment involves holding a competent person against his or her will. Actual force is not a requirement of false imprisonment. The individual needs only to be placed in fear of imprisonment by someone who has the ability to carry out the threat. If a patient is not competent (confused), then the nurse should act with beneficence. Patients admitted involuntarily should not be allowed to leave without permission of the treatment team.

The therapeutic action of neurotransmitter inhibitors that block reuptake cause a. decreased concentration of the blocked neurotransmitter in the central nervous system. b. increased concentration of the blocked neurotransmitter in the synaptic gap. c. destruction of receptor sites specific to the blocked neurotransmitter. d. limbic system stimulation.

B If the reuptake of a substance is inhibited, it accumulates in the synaptic gap, and its concentration increases, permitting ease of transmission of impulses across the synaptic gap. Normal transmission of impulses across synaptic gaps is consistent with normal rather than depressed mood. The other options are not associated with blocking neurotransmitter reuptake.

A nurse would anticipate that treatment for a patient with memory difficulties might include medications designed to a. inhibit GABA. b. prevent destruction of acetylcholine. c. reduce serotonin metabolism. d. increase dopamine activity.

B Increased acetylcholine plays a role in learning and memory. Preventing destruction of acetylcholine by acetylcholinesterase would result in higher levels of acetylcholine, with the potential for improved memory. GABA affects anxiety rather than memory. Increased dopamine would cause symptoms associated with schizophrenia or mania rather than improve memory. Decreasing dopamine at receptor sites is associated with Parkinson's disease rather than improving memory.

A patient diagnosed with schizophrenia tells the nurse, "The Central Intelligence Agency is monitoring us through the fluorescent lights in this room. The CIA is everywhere, so be careful what you say." Which response by the nurse is most therapeutic? a. "Let's talk about something other than the CIA." b. "It sounds like you're concerned about your privacy." c. "The CIA is prohibited from operating in health care facilities." d. "You have lost touch with reality, which is a symptom of your illness."

B It is important not to challenge the patient's beliefs, even if they are unrealistic. Challenging undermines the patient's trust in the nurse. The nurse should try to understand the underlying feelings or thoughts the patient's message conveys. The correct response uses the therapeutic technique of reflection. The other comments are nontherapeutic. Asking to talk about something other than the concern at hand is changing the subject. Saying that the CIA is prohibited from operating in health care facilities gives false reassurance. Stating that the patient has lost touch with reality is truthful, but uncompassionate.

A patient diagnosed with schizophrenia believes a local minister stirred evil spirits. The patient threatens to bomb a local church. The psychiatrist notifies the minister. Select the answer with the correct rationale. The psychiatrist a. released information without proper authorization. b. demonstrated the duty to warn and protect. c. violated the patient's confidentiality. d. avoided charges of malpractice.

B It is the health care professional's duty to warn or notify an intended victim after a threat of harm has been made. Informing a potential victim of a threat is a legal responsibility of the health care professional. It is not a violation of confidentiality.

A patient taking medication for mental illness develops restlessness and an uncontrollable need to be in motion. Which drug action causes these symptoms to develop? a. Anticholinergic effects b. Dopamine-blocking effects c. Endocrine-stimulating effects d. Ability to stimulate spinal nerves

B Medication that blocks dopamine often produces disturbances of movement, such as akathisia, because dopamine affects neurons involved in both thought processes and movement regulation. Anticholinergic effects include dry mouth, blurred vision, urinary retention, and constipation. Akathisia is not caused by endocrine stimulation or spinal nerve stimulation.

The nurse prepares to assess a patient diagnosed with major depressive disorder for disturbances in circadian rhythms. Which question should the nurse ask this patient? a. "Have you ever seen or heard things that others do not?" b. "What are your worst and best times of the day?" c. "How would you describe your thinking?" d. "Do you think your memory is failing?"

B Mood changes throughout the day may be related to circadian rhythm disturbances. Questions about sleep pattern are also relevant tocircadian rhythms.

During a psychiatric assessment, the nurse observes a patient's facial expression is without emotion. The patient says, "Life feels so hopeless to me. I've been feeling sad for several months." How will the nurse document the patient's affect and mood? a. Affect depressed; mood flat b. Affect flat; mood depressed c. Affect labile; mood euphoric d. Affect and mood are incongruent.

B Mood refers to a person's self-reported emotional feeling state. Affect is the emotional feeling state that is outwardly observable by others. When there is no evidence of emotion in a person's expression, the affect is flat.

A fearful patient has an increased heart rate and blood pressure. The nurse suspects increased activity of which neurotransmitter? a. GABA b. Norepinephrine c. Acetylcholine d. Histamine

B Norepinephrine is the neurotransmitter associated with sympathetic nervous system stimulation, preparing the individual for "fight or flight." GABA is a mediator of anxiety level. A high concentration of histamine is associated with an inflammatory response. A high concentration of acetylcholine is associated with parasympathetic nervous system stimulation.

The nurse schedules the activities for the patient. Which role has the nurse fulfilled? a. Advocate b. Case manager c. Milieu manager d. Provider of care

B Nurses on psychiatric units routinely coordinate patient services, serving as case managers as described in this scenario. The role of advocate would require the nurse to speak out on the patient's behalf. The role of milieu manager refers to maintaining a therapeutic environment. Provider of care refers to giving direct care to the patient.

A nurse prepares to administer a scheduled intramuscular injection of an antipsychotic medication to an outpatient diagnosed with schizophrenia. As the nurse swabs the site, the patient shouts, "Stop! I don't want to take that medicine anymore. I hate the side effects." Select the nurse's best action. a. Assemble other staff for a show of force and proceed with the injection, using restraint if necessary. b. Stop the medication administration procedure and say to the patient, "Tell me more about the side effects you've been having." c. Proceed with the injection but explain to the patient that there are medications that will help reduce the unpleasant side effects. d. Say to the patient, "Since I've already drawn the medication in the syringe, I'm required to give it, but let's talk to the doctor about delaying next month's dose."

B Patients diagnosed with mental illness retain their civil rights unless there is clear, cogent, and convincing evidence of dangerousness. The patient in this situation presents no evidence of dangerousness. The nurse, as an advocate and educator, should seek more information about the patient's decision and not force the medication.

Two hospitalized patients fight whenever they are together. During a team meeting, a nurse asserts that safety is of paramount importance, so treatment plans should call for both patients to be secluded to keep them from injuring each other. This assertion a. reinforces the autonomy of the two patients. b. violates the civil rights of both patients. c. represents the intentional tort of battery. d. correctly places emphasis on safety.

B Patients have a right to treatment in the least restrictive setting. Safety is important, but less restrictive measures should be tried first. Unnecessary seclusion may result in a charge of false imprisonment. Seclusion violates the patient's autonomy. The principle by which the nurse is motivated is beneficence, not justice. The tort represented is false imprisonment.

A patient experiencing psychosis asks a psychiatric technician, "What's the matter with me?" The technician replies, "Nothing is wrong with you. You just need to use some self-control." The nurse who overheard the exchange should take action based on a. the technician's unauthorized disclosure of confidential clinical information. b. violation of the patient's right to be treated with dignity and respect. c. the nurse's obligation to report caregiver negligence. d. the patient's right to social interaction.

B Patients have the right to be treated with dignity and respect. The technician's comment disregards the seriousness of the patient's illness. The Code of Ethics for Nurses requires intervention. Patient emotional abuse has been demonstrated, not negligence. An interaction with the technician is not an aspect of social interaction.

A nurse instructs a patient taking a medication that inhibits the action of monoamine oxidase (MAO) to avoid certain foods and drugs because of the risk of: a. hypotensive shock. b. hypertensive crisis. c. cardiac dysrhythmia. d. cardiogenic shock.

B Patients taking MAO-inhibiting drugs must be on a tyramine-free diet to prevent hypertensive crisis. In the presence of MAOIs, tyramine is not destroyed by the liver and in high levels produces intense vasoconstriction, resulting in elevated blood pressure.

Which assessment finding for a patient diagnosed with serious and persistent mental illness and living in the community merits priority intervention by the psychiatric nurse? The patient a. receives social security disability income plus a small check from a trust fund every month. b. was absent from two of six planned Alcoholics Anonymous meetings in the past 2 weeks. c. lives in an apartment with two patients who attend partial hospitalization programs. d. has a sibling who was recently diagnosed with a mental illness.

B Patients who use alcohol or illegal substances often become medication non-adherent. Medication non-adherence, along with the disorganizing influence of substances on cellular brain function, promotes relapse. The distracters do not suggest problems.

A patient diagnosed with major depression says, "No one cares about me anymore. I'm not worth anything." Today the patient is wearing a new shirt and has neat, clean hair. Which remark by the nurse supports building a positive self-esteem for this patient? a. "You look nice this morning." b. "You're wearing a new shirt." c. "I like the shirt you are wearing." d. "You must be feeling better today."

B Patients with depression usually see the negative side of things. The meaning of compliments may be altered to "I didn't look nice yesterday" or "They didn't like my other shirt." Neutral comments such as making an observation avoid negative interpretations. Saying, "You look nice" or "I like your shirt" gives approval (non-therapeutic techniques). Saying "You must be feeling better today" is an assumption, which is non-therapeutic.

A patient reports fears of having cervical cancer and says to the nurse, "I've had Pap smears by six different doctors. The results were normal, but I'm sure that's because of errors in the laboratory." Which disorder would the nurse suspect? a. Conversion (functional neurological) disorder b. Illness anxiety disorder (hypochondriasis) c. Somatic symptom disorder d. Factitious disorder

B Patients with illness anxiety disorder have fears of serious medical problems, such as cancer or heart disease. These fears persist despite medical evaluations and interfere with daily functioning. There are no complaints of pain. There is no evidence of factitious or conversion disorder.

A psychoeducational session will discuss medication management for a culturally diverse group of patients. Group participants are predominantly members of minority cultures. Of the four staff nurses below, which nurse should lead this group? a. Very young registered nurse b. Older, mature registered nurse c. Newly licensed registered nurse d. A registered nurse who is very thin

B Persons of minority cultures value age and wisdom. Persons with a Western worldview tend to value youth. An older, mature registered nurse would be the most credible leader of this group. The nurse's size has no bearing on credibility.

Which activity is appropriate for a nurse engaged exclusively in community-based primary prevention? a. Medication follow-up b. Teaching parenting skills c. Substance abuse counseling d. Making a referral for family therapy

B Primary prevention activities are directed to healthy populations to provide information for developing skills that promote mental health. The distracters represent secondary or tertiary prevention activities.

Select the example of primary prevention. a. Assisting a person diagnosed with a serious mental illness to fill a pill-minder b. Helping school-age children identify and describe normal emotions c. Leading a psychoeducational group in a community care home d. Medicating an acutely ill patient who assaulted a staff person

B Primary preventions are directed at healthy populations with a goal of preventing health problems from occurring. Helping school-age children describe normal emotions people experience promotes coping, a skill that is needed throughout life.

A nurse caring for a patient taking a SSRI will develop outcome criteria related to a. coherent thought processes. b. improvement in depression. c. reduced levels of motor activity. d. decreased extrapyramidal symptoms.

B SSRIs affect mood, relieving depression in many cases. SSRIs do not act to reduce thought disorders. SSRIs reduce depression but have little effect on motor hyperactivity. SSRIs do not produce extrapyramidal symptoms.

Which comment most clearly shows a speaker views mental illness with stigma? a. "Some mental illnesses are inherited." b. "Most people with mental illness are unmotivated." c. "Severe environmental stress sometimes causes mental illness." d. "Some mental illnesses are brain disorders resulting from changes in how impulses are transmitted."

B Stigma refers to stereotypical, negative beliefs. With respect to mental health and mental illness, stigma often leads to discrimination and uncaring attitudes. Mental illness has multiple causes, including stress, changes in brain structure or function, and genetic transmission.

A drug blocks the attachment of norepinephrine to a1 receptors. The patient may experience a. hypertensive crisis. b. orthostatic hypotension. c. severe appetite disturbance. d. an increase in psychotic symptoms.

B Sympathetic-mediated vasoconstriction is essential for maintaining normal blood pressure in the upright position. Blockage of a1 receptors leads to vasodilation and orthostatic hypotension. Orthostatic hypotension may cause fainting and falls. Teach patients ways of minimizing this phenomenon.

A patient is fearful of riding on elevators. The therapist first rides an escalator with the patient. The therapist and patient then stand in an elevator with the door open for 5 minutes and later with the elevator door closed for 5 minutes. Which technique has the therapist used? a. Classic psychoanalytic therapy b. Systematic desensitization c. Rational emotive therapy d. Biofeedback

B Systematic desensitization is a form of behavior modification therapy that involves the development of behavior tasks customized to the patient's specific fears. These tasks are presented to the patient while using learned relaxation techniques. The patient is incrementally exposed to the fear.

Which intervention best demonstrates that a nurse correctly understands the cultural needs of a hospitalized Asian American patient diagnosed with a mental illness? a. Encouraging the family to attend community support groups b. Involving the patient's family to assist with activities of daily living c. Providing educational pamphlets to explain the patient's mental illness d. Restricting homemade herbal remedies the family brings to the hospital

B The Asian community values the family in caring for each other. The Asian community uses traditional medicines and healers, including herbs for mental symptoms. The Asian community describes illness in somatic terms. The Asian community attaches a stigma to mental illness, so interfacing with the community would not be appealing.

A nurse taught a patient about a tyramine-restricted diet. Which menu selection would the nurse approve? a. Macaroni and cheese, hot dogs, banana bread, caffeinated coffee b. Mashed potatoes, ground beef patty, corn, green beans, apple pie c. Avocado salad, ham, creamed potatoes, asparagus, chocolate cake d. Noodles with cheddar cheese sauce, smoked sausage, lettuce salad, yeast rolls

B The correct answer describes a meal that contains little tyramine. Vegetables and fruits contain little or no tyramine. Fresh ground beef and apple pie are safe. The other meals contain various amounts of tyramine-rich foods or foods that contain vasopressors: avocados, ripe bananas (banana bread), sausages/hot dogs, smoked meat (ham), cheddar cheese, yeast, caffeine drinks, and chocolate.

Which finding best indicates that a patient has a mental illness? The patient: a.responds to rules, routines, and customs of a group. b. reports mood is consistently sad, discouraged, and hopeless. c. performs tasks attempted within the limits set by own abilities. d. is able to see the difference between the "as if" and the "for real."

B The correct response describes a mood alteration, which further reflects mental illness. The distracters describe mentally healthy behaviors.

A patient has disorganized thinking associated with schizophrenia. Neuroimaging would likely show dysfunction in which part of the brain? a. Hippocampus b. Frontal lobe c. Cerebellum d. Brainstem

B The frontal lobe is responsible for intellectual functioning. The hippocampus is involved in emotions and learning. The cerebellum regulates skeletal muscle coordination and equilibrium. The brainstem regulates internal organs.

Consider these medications: carbamazepine, lamotrigine, gabapentin. Which medication below also belongs to this group? a. Galantamine b. Valproate c. Buspirone d. Tacrine

B The medications listed in the stem are mood stabilizers, anticonvulsant types. Valproate (Depakote) is also a member of this group. The distracters are drugs for treatment of Alzheimer's disease and anxiety.

A nurse finds a psychiatric advance directive in the medical record of a patient currently experiencing psychosis. The directive was executed during a period when the patient was stable and competent. The nurse should a. review the directive with the patient to ensure it is current. b. ensure that the directive is respected in treatment planning. c. consider the directive only if there is a cardiac or respiratory arrest. d. encourage the patient to revise the directive in light of the current health problem.

B The nurse has an obligation to honor the right to self-determination. An advanced psychiatric directive supports that goal. Since the patient is currently psychotic, the terms of the directive now apply

An aide in a psychiatric hospital says to the nurse, "We don't have time every day to help each patient complete a menu selection. Let's tell dietary to prepare popular choices and send them to our unit." Select the nurse's best response. a. "Thanks for the suggestion, but that idea may not work because so many patients take MAOI (monoamine oxidase inhibitor) antidepressants." b. "Thanks for the idea, but it's important to treat patients as individuals. Giving choices is one way we can respect patients' individuality." c. "Thank you for the suggestion, but the patients' bill of rights requires us to allow patients to select their own diet." d. "Thank you. That is a very good idea. It will make meal preparation easier for the dietary department."

B The nurse's response to the aide should recognize patients' rights to be treated with dignity and respect as well as promote autonomy. This response also shows respect for the aide and fulfills the nurse's obligation to provide supervision of unlicensed personnel. The incorrect responses have flawed rationale or do not respect patients as individuals.

A student nurse says, "I don't need to interact with my patients. I learn what I need to know by observation." An instructor can best interpret the nursing implications of Sullivan's theory to this student by responding: a. "Interactions are required in order to help you develop therapeutic communication skills." b. "Nurses cannot be isolated. We must interact to provide patients with opportunities to practice interpersonal skills." c. "Observing patient interactions will help you formulate priority nursing diagnoses and appropriate interventions." d. "It is important to pay attention to patients' behavioral changes, because these signify adjustments in personality."

B The nurse's role includes educating patients and assisting them in developing effective interpersonal relationships. Mutuality, respect for the patient, unconditional acceptance, and empathy are cornerstones of Sullivan's theory. The nurse who does not interact with the patient cannot demonstrate these cornerstones. Observations provide only objective data. Priority nursing diagnoses usually cannot be accurately established without subjective data from the patient. The other distracters relate to Maslow and behavioral theory. This item relates to an audience response question.

A nurse influenced by Peplau's interpersonal theory works with an anxious, withdrawn patient. Interventions should focus on a. rewarding desired behaviors. b. use of assertive communication. c. changing the patient's self-concept. d. administering medications to relieve anxiety.

B The nurse-patient relationship is structured to provide a model for adaptive interpersonal relationships that can be generalized to others. Helping the patient learn to use assertive communication will improve the patient's interpersonal relationships. The distracters apply to theories of cognitive, behavioral, and biological therapy.

The parent of a child diagnosed with schizophrenia tearfully asks the nurse, "What could I have done differently to prevent this illness?" Select the nurse's best response. a. "Although schizophrenia results from impaired family relationships, try not to feel guilty. No one can predict how a child will respond to parental guidance." b. "Schizophrenia is a biological illness resulting from changes in how the brain and nervous system function. You are not to blame for your child's illness." c. "There is still hope. Changing your parenting style can help your child learn to cope effectively with the environment." d. "Most mental illnesses result from genetic inheritance. Your genes are more at fault than your parenting."

B The parent's comment suggests feelings of guilt or inadequacy. The nurse's response should address these feelings as well as provide information. Patients and families need reassurance that the major mental disorders are biological in origin and are not the "fault" of parents. One distracter places the burden of having faulty genes on the shoulders of the parents. The other distracters are neither wholly accurate nor reassuring.

A patient has blindness related to conversion (functional neurological) disorder. To help the patient eat, the nurse should: a. establish a "buddy" system with other patients who can feed the patient at each meal. b. expect the patient to feed self after explaining arrangement of the food on the tray. c. direct the patient to locate items on the tray independently and feed self. d. address needs of other patients in the dining room, then feed this patient.

B The patient is expected to maintain some level of independence by feeding self, while the nurse is supportive in a matter-of-fact way. The distracters support dependency or offer little support.

A patient with blindness related to conversion (functional neurological) disorder says, "All the doctors and nurses in the hospital stop by often to check on me. Too bad people outside the hospital don't find me as interesting." Which nursing diagnosis is most relevant? a. Social isolation b. Chronic low self-esteem c. Interrupted family processes d. Ineffective health maintenance

B The patient mentions that the symptoms make people more interested. This indicates that the patient feels uninteresting and unpopular without the symptoms, thus supporting the nursing diagnosis of chronic low self-esteem. Defining characteristics for the other nursing diagnoses are not present in the scenario.

A patient with a somatic symptom disorder has the nursing diagnosis Interrupted family processes related to patient's disabling symptoms as evidenced by spouse and children assuming roles and tasks that previously belonged to patient. An appropriate outcome is that the patient will: a. assume roles and functions of other family members. b. demonstrate performance of former roles and tasks. c. focus energy on problems occurring in the family. d. rely on family members to meet personal needs.

B The patient with a somatic symptom disorder has typically adopted a sick role in the family, characterized by dependence. Increasing independence and resumption of former roles are necessary to change this pattern. The distracters are inappropriate outcomes.

The patient says, "My marriage is just great. My spouse and I always agree." The nurse observes the patient's foot moving continuously as the patient twirls a shirt button. The conclusion the nurse can draw is that the patient's communication is a. clear. b. distorted. c. incongruous. d. inadequate.

B The patient's verbal and nonverbal communication in this scenario are incongruous. Incongruous messages involve transmission of conflicting messages by the speaker. The patient's verbal message that all was well in the relationship was modified by the nonverbal behaviors denoting anxiety. Data are not present to support the choice of the verbal message being clear, explicit, or inadequate.

Two nursing students discuss their career plans after graduation. One student wants to enter psychiatric nursing. The other asks, "Why would you want to be a psychiatric nurse? The only thing they do is talk. You'll lose all your skills." Select the best response. a. "Psychiatric nurses practice in safer environments than other specialties. Nurse-to-patient ratios must be better because of the nature of the patients' problems." b. "Psychiatric nurses use complex communication skills as well as critical thinking to solve multidimensional problems. I am challenged by those situations." c. "I think I will be good in the mental health field. I did not like clinical rotations in school, so I do not want to continue them after I graduate." d. "Psychiatric nurses do not have to deal with as much pain and suffering as medical-surgical nurses do. That appeals to me."

B The practice of psychiatric nursing requires a different set of skills than medical-surgical nursing, though there is substantial overlap. Psychiatric nurses must be able to help patients with medical as well as mental health problems, reflecting the holistic perspective these nurses must have. Nurse-patient ratios and workloads in psychiatric settings have increased, just like other specialties. Psychiatric nursing involves clinical practice, not just documentation. Psychosocial pain and suffering is as real as physical.

A patient diagnosed with bipolar disorder displays aggressiveness, agitation, talkativeness, and irritability. The nurse expects the health care provider to prescribe a medication from which group? a. Psychostimulants b. Mood stabilizers c. Anticholinergics d. Antidepressants

B The symptoms describe mania, which is effectively treated by mood stabilizers, such as lithium, and selected anticonvulsants (carbamazepine, valproic acid, and lamotrigine). Drugs from the other classifications listed are not effective in the treatment of mania.

A patient participated in psychotherapy weekly for 5 months. The therapist used free association, dream analysis, and facilitated transference to help the patient understand conflicts and foster change. Select the term that applies to this method. a. Rational-emotive behavior therapy b. Psychodynamic psychotherapy c. Cognitive-behavioral therapy d. Operant conditioning

B The techniques are aspects of psychodynamic psychotherapy. The distracters use other techniques.

A parent says, "My 2-year-old child refuses toilet training and shouts 'No!' when given directions. What do you think is wrong?" Select the nurse's best reply. a. "Your child needs firmer control. It is important to set limits now." b. "This is normal for your child's age. The child is striving for independence." c. "There may be developmental problems. Most children are toilet trained by age 2." d. "Some undesirable attitudes are developing. A child psychologist can help you develop a plan."

B This behavior is conventional of a child around the age of 2 years, whose developmental task is to develop autonomy. The distracters indicate the child's behavior is abnormal.

A professional football player is seen in the emergency department after losing consciousness from an illegal block. Prior to discharge, the nurse assists the patient to schedule an outpatient computed tomography (CT) scan for the next day. Which strategy should the nurse use to ensure the patient remembers the appointment? a. Write the appointment day, time, and location on a piece of paper and give it to the player. b. Log the appointment day, time, and location into the player's cell phone calendar feature. c. Ask the health care provider to admit the patient to the hospital overnight. d. Verbally inform the patient of the appointment day, time, and location.

B This player may have suffered repeated head injuries with damage to the hippocampus. The hippocampus has significant role in maintaining memory. Logging the appointment into the player's cell phone calendar will remind him of the appointment the next day. Paper will be lost, and the patient is unlikely to remember verbal instruction. Hospitalization is unnecessary.

During the first interview with a parent whose child died in a car accident, the nurse feels empathic and reaches out to take the patient's hand. Select the correct analysis of the nurse's behavior. a. It shows empathy and compassion. It will encourage the patient to continue to express feelings. b. The gesture is premature. The patient's cultural and individual interpretation of touch is unknown. c. The patient will perceive the gesture as intrusive and overstepping boundaries. d. The action is inappropriate. Psychiatric patients should not be touched.

B Touch has various cultural and individual interpretations. Nurses should refrain from using touch until an assessment is completed regarding the way in which the patient will perceive touch. The incorrect options present prematurely drawn conclusions.

Transcranial Magnetic Stimulation (TCM) is scheduled for a patient diagnosed with major depression. Which comment by the patient indicates teaching about the procedure was effective? a. "They will put me to sleep during the procedure so I won't know what is happening." b. "I might be a little dizzy or have a mild headache after each procedure." c. "I will be unable to care for my children for about 2 months." d. "I will avoid eating foods that contain tyramine."

B Transcranial Magnetic Stimulation (TCM) treatments take about 30 minutes. Treatments are usually 5 days a week. Patients are awake and alert during the procedure. After the procedure, patients may experience a headache and lightheadedness. No neurological deficits or memory problems have been noted. The patient will be able to care for children.

When asked, the nurse explains that a client's id is a. the control over the emotional frustration he feels over the loss of his job. b. the source of his instincts to save himself from hurting himself. c. not in place since he was abused after the age of 5 months. d. able to differentiate his believed experiences and reality.

B the source of his instincts to save himself from hurting himself

A Native American patient describes a difficult childhood and dropping out of high school. The patient abused alcohol as a teenager to escape feelings of isolation but stopped 10 years ago. The patient now says, "I feel stupid. I've never had a good job. I don't help my people." Which nursing diagnosis applies? a. Risk for other-directed violence b. Chronic low self-esteem c. Deficient knowledge d. Social isolation

B The patient has given several indications of chronic low self-esteem.

In order to best differentiate whether an Asian client is demonstrating a mental illness when attempting suicide is to A. ask the client whether he views himself as being depressed. B. identify his culture's view regarding suicide. C. explain to him that suicide is often regarded as a desperate act. D. assess the client for other examples of depressive behaviors.

B identify his culture's view regarding suicide

These severe mental illnesses are recognized across cultures: A antisocial and borderline personality disorders. B schizophrenia and bipolar disorder. C bulimia and anorexia nervosa. D amok and social phobia.

B schizophrenia & bipolar disorder

A patient cries as the nurse explores the patient's feelings about the death of a close friend. The patient sobs, "I shouldn't be crying like this. It happened a long time ago." Which responses by the nurse facilitate communication? (Select all that apply.) a. "Why do you think you are so upset?" b. "I can see that you feel sad about this situation." c. "The loss of a close friend is very painful for you." d. "Crying is a way of expressing the hurt you are experiencing." e. "Let's talk about something else because this subject is upsetting you."

B, C, D Reflecting ("I can see that you feel sad," "This is very painful for you") and giving information ("Crying is a way of expressing hurt") are therapeutic techniques. "Why" questions often imply criticism or seem intrusive or judgmental. They are difficult to answer. Changing the subject is a barrier to communication.

Questions the nurse could ask that would be nonjudgmental when obtaining information about a patient's use of complementary and herbal remedies include (Select all that apply) a. "You don't regularly take herbal remedies, do you?" b. "What herbal medicines have you used to relieve your symptoms?" c. "What over-the-counter medicines, vitamins, and nutritional supplements do you use?" d. "What differences in your symptoms do you notice when you take herbal supplements?" e. "Have you experienced problems from using herbal and prescription drugs at the same time?"

B, C, D, E The correct responses are neutral in tone and do not express bias for or against the use of complementary or herbal medicines. The distracter, worded in a negative way, makes the nurse's bias evident.

Which activities represent the art of nursing? (Select all that apply.) a. Administering medications on time to a group of patients b. Listening to a new widow grieve her husband's death c. Helping a patient obtain groceries from a food bank d. Teaching a patient about a new medication e. Holding the hand of a frightened patient

B, C, E Peplau described the science and art of professional nursing practice. The art component of nursing consists of the care, compassion, and advocacy nurses provide to enhance patient comfort and well-being. The science component of nursing involves the application of knowledge to understand a broad range of human problems and psychosocial phenomena, intervening to relieve patients' suffering and promote growth. See related audience response question.

A patient in the emergency department says, "The voices say someone is stalking me. They want to kill me because I developed a cure for cancer. I have a knife and will stab anyone who is a threat." Which aspect(s) of mental health should be of greatest immediate concern to the nurse? Select all that apply. a. Happiness b. Appraisal of reality c. Control over behavior d. Effectiveness in work e.Healthy self-concept

B, C, E The aspects of mental health of greatest concern are the patient's appraisal of and control over behavior. The appraisal of reality is inaccurate. There are auditory hallucinations, delusions of persecution, and delusions of grandeur. In addition, the patient's control over behavior is tenuous, as evidenced by the plan to stab anyone who seems threatening. A healthy self-concept is lacking, as evidenced by the delusion of grandeur. Data are not present to suggest that the other aspects of mental health (happiness and effectiveness in work) are of immediate concern.

Which assessment questions would be most appropriate for the nurse to ask a patient with possible obsessive-compulsive disorder? Select all that apply. a."Are there certain social situations that cause you to feel especially uncomfortable?" b."Are there others in your family who must do things in a certain way to feel comfortable?" c."Have you been a victim of a crime or seen someone badly injured or killed?" d."Is it difficult to keep certain thoughts out of your awareness?" e."Do you do certain things over and over again?"

B, D, E The correct questions refer to obsessive thinking and compulsive behaviors. There is likely a genetic correlation to the disorder. The incorrect responses are more pertinent to a patient with suspected posttraumatic stress disorder or with suspected social phobia. See relationship to audience response question.

A child has a history of multiple hospitalizations for recurrent systemic infections. The child is not improving in the hospital, despite aggressive treatment. Factitious disorder by proxy is suspected. Which nursing interventions are appropriate? Select all that apply. a. Increase private visiting time for the parents to improve bonding. b. Keep careful, detailed records of visitation and untoward events. c. Place mittens on the child to reduce access to ports and incisions. d. Encourage family members to visit in groups of two or three. e. Interact with the patient frequently during visiting hours.

B, D, E Factitious disorder by proxy is a rare condition wherein a person intentionally causes or perpetuates the illness of a loved one (e.g., by periodically contaminating IV solutions with fecal material). When this disorder is suspected, the child's life could be at risk. Depending on the evidence supporting this suspicion, interventions could range from minimizing unsupervised visitation to blocking visitation altogether. Frequently checking on the child during visitation and minimizing unobserved access to the child (by encouraging small group visits) reduces the opportunity to take harmful action and increases the collection of data that can help determine whether this disorder is at the root of the child's illness.

When alprazolam (Xanax) is prescribed for a patient who experiences acute anxiety, health teaching should include instructions to: a.report drowsiness. b.eat a tyramine-free diet. c.avoid alcoholic beverages. d.adjust dose and frequency based on anxiety level.

C Drinking alcohol or taking other anxiolytics along with the prescribed benzodiazepine should be avoided because depressant effects of both drugs will be potentiated. Tyramine-free diets are necessary only with monoamine oxidase inhibitors (MAOIs). Drowsiness is an expected effect and needs to be reported only if it is excessive. Patients should be taught not to deviate from the prescribed dose and schedule for administration.

A patient experiencing moderate anxiety says, "I feel undone." An appropriate response for the nurse would be: a."What would you like me to do to help you?" b."Why do you suppose you are feeling anxious?" c."I'm not sure I understand. Give me an example." d."You must get your feelings under control before we can continue."

C Increased anxiety results in scattered thoughts and an inability to articulate clearly. Clarifying helps the patient identify thoughts and feelings. Asking the patient why he or she feels anxious is non-therapeutic; the patient likely does not have an answer. The patient may be unable to determine what he or she would like the nurse to do in order to help. Telling the patient to get his or her feelings under control is a directive the patient is probably unable to accomplish.

A patient diagnosed with obsessive-compulsive disorder has this nursing diagnosis: Anxiety related to __________ as evidenced by inability to control compulsive cleaning. Which phrase correctly completes the etiological portion of the diagnosis? a.feelings of responsibility for the health of family members b.approval-seeking behavior from friends and family c.persistent thoughts about bacteria, germs, and dirt d.needs to avoid interactions with others

C Many compulsive rituals accompany obsessive thoughts. The patient uses these rituals for anxiety relief. Unfortunately, the anxiety relief is short lived, and the patient must frequently repeat the ritual. The other options are unrelated to the dynamics of compulsive behavior. See relationship to audience response question.

A patient tells a nurse, "My best friend is a perfect person. She is kind, considerate, good-looking, and successful with every task. I could have been like her if I had the opportunities, luck, and money she's had." This patient is demonstrating a. denial. b. projection. c. rationalization. d. compensation.

C Rationalization consists of justifying illogical or unreasonable ideas, actions, or feelings by developing acceptable explanations that satisfy the teller as well as the listener. Denial is an unconscious process that would call for the nurse to ignore the existence of the situation. Projection operates unconsciously and would result in blaming behavior. Compensation would result in the nurse unconsciously attempting to make up for a perceived weakness by emphasizing a strong point.

A patient experiencing panic suddenly began running and shouting, "I'm going to explode!" Select the nurse's best action. a.Ask, "I'm not sure what you mean. Give me an example." b.Capture the patient in a basket-hold to increase feelings of control. c.Tell the patient, "Stop running and take a deep breath. I will help you." d.Assemble several staff members and say, "We will take you to seclusion to help you regain control."

C Safety needs of the patient and other patients are a priority. Comments to the patient should be simple, neutral, and give direction to help the patient regain control. Running after the patient will increase the patient's anxiety. More than one staff member may be needed to provide physical limits, but using seclusion or physically restraining the patient prematurely is unjustified. Asking the patient to give an example would be futile; a patient in panic processes information poorly.

A person has minor physical injuries after an auto accident. The person is unable to focus and says, "I feel like something awful is going to happen." This person has nausea, dizziness, tachycardia, and hyperventilation. What is the person's level of anxiety? a. Mild b. Moderate c. Severe d. Panic

C The person whose anxiety is severe is unable to solve problems and may have a poor grasp of what is happening in the environment. Somatic symptoms such as those described are usually present. The individual with mild anxiety is only mildly uncomfortable and may even find his or her performance enhanced. The individual with moderate anxiety grasps less information about a situation and has some difficulty with problem solving. The individual in panic will demonstrate markedly disturbed behavior and may lose touch with reality.

A voluntarily hospitalized patient tells the nurse, "Get me the forms for discharge. I want to leave now." Select the nurse's best response. a. "I will get the forms for you right now and bring them to your room." b. "Since you signed your consent for treatment, you may leave if you desire." c. "I will get them for you, but let's talk about your decision to leave treatment." d. "I cannot give you those forms without your health care provider's permission."

C A voluntarily admitted patient has the right to demand and obtain release in most states. However, as a patient advocate, the nurse is responsible for weighing factors related to the patient's wishes and best interests. By asking for information, the nurse may be able to help the patient reconsider the decision. Facilitating discharge without consent is not in the patient's best interests before exploring the reason for the request.

A patient discloses several concerns and associated feelings. If the nurse wants to seek clarification, which comment would be appropriate? a. "What are the common elements here?" b. "Tell me again about your experiences." c. "Am I correct in understanding that." d. "Tell me everything from the beginning."

C Asking, "Am I correct in understanding that ..." permits clarification to ensure that both the nurse and patient share mutual understanding of the communication. Asking about common elements encourages comparison rather than clarification. The remaining responses are implied questions that suggest the nurse was not listening.

A suspicious, socially isolated patient lives alone, eats one meal a day at a local shelter, and spends the remaining daily food allowance on cigarettes. Select a community psychiatric nurse's best initial action. a. Explore ways to help the patient stop smoking. b. Report the situation to the manager of the shelter. c. Assess the patient's weight, determine foods and amounts eaten. d. Arrange hospitalization for the patient in order to formulate a new treatment plan.

C Assessment of biopsychosocial needs and general ability to live in the community is called for before any other action is taken. Both nutritional status and income adequacy are critical assessment parameters. A patient may be able to maintain adequate nutrition while eating only one meal a day. The rule is to assess before taking action. Hospitalization may not be necessary. Smoking cessation strategies can be pursued later.

A patient tells the nurse, "I don't think I'll ever get out of here." Select the nurse's most therapeutic response. a. "Don't talk that way. Of course you will leave here!" b. "Keep up the good work, and you certainly will." c. "You don't think you're making progress?" d. "Everyone feels that way sometimes."

C By asking if the patient does not believe that progress has been made, the nurse is reflecting or paraphrasing by putting into words what the patient is hinting. By making communication more explicit, issues are easier to identify and resolve. The remaining options are nontherapeutic techniques. Telling the patient not to "talk that way" is disapproving. Saying that everyone feels that way at times minimizes feelings. Telling the patient that good work will always result in success is falsely reassuring.

When counseling patients diagnosed with major depression, an advanced practice nurse will address the negative thought patterns by using: a. psychoanalytic therapy. b. desensitization therapy. c. cognitive behavioral therapy. d. alternative and complementary therapies

C Cognitive behavioral therapy attempts to alter the patient's dysfunctional beliefs by focusing on positive outcomes rather than negative attributions. The patient is also taught the connection between thoughts and resultant feelings. Research shows that cognitive behavioral therapy involves the formation of new connections between nerve cells in the brain and that it is at least as effective as medication. Evidence is not present to support superior outcomes for the other psychotherapeutic modalities mentioned.

A Mexican American patient puts a picture of the Virgin Mary on the bedside table. What is the nurse's best action? a. Move the picture so it is beside a window. b. Send the picture to the business office safe. c. Leave the picture where the patient placed it. d. Send the picture home with the patient's family.

C Cultural heritage is expressed through language, works of art, music, dance, customs, traditions, diet, and expressions of spirituality. This patient's prominent placement of the picture is an example of expression of cultural heritage and spirituality. The nurse should not move it unless the patient's safety is jeopardized.

A group activity on an inpatient psychiatric unit is scheduled to begin at 1000. A patient, who was recently discharged from U.S. Marine Corps, arrives at 0945. Which analysis best explains this behavior? a. The patient wants to lead the group and give directions to others. b. The patient wants to secure a chair that will be close to the group leader. c. The military culture values timeliness. The patient does not want to be late. d. The behavior indicates feelings of self-importance that the patient wants others to appreciate.

C Culture is more than ethnicity and social norms. it includes religious, geographic, socioeconomic, occupational, ability- or disability-related, and sexual orientation-related beliefs and behaviors. In this instance, the patient's military experience represents an aspect of the patient's behavior. The military culture values timeliness. The distracters represent misinterpretation of the patient's behavior and have no bearing on the situation.

A patient diagnosed with major depression began taking a tricyclic antidepressant 1 week ago. Today the patient says, "I don't think I can keep taking these pills. They make me so dizzy, especially when I stand up." The nurse will: a. limit the patient's activities to those that can be performed in a sitting position. b. withhold the drug, force oral fluids, and notify the health care provider. c. teach the patient strategies to manage postural hypotension. d. update the patient's mental status examination

C Drowsiness, dizziness, and postural hypotension usually subside after the first few weeks of therapy with tricyclic antidepressants. Postural hypotension can be managed by teaching the patient to stay well hydrated and rise slowly. Knowing this information may convince the patient to continue the medication. Activity is an important aspect of the patient's treatment plan and should not be limited to activities that can be done in a sitting position. Withholding the drug, forcing oral fluids, and notifying the health care provider are unnecessary actions. Independent nursing action is called for. Updating a mental status examination is unnecessary.

After leaving work, a nurse realizes documentation of administration of a prn medication was omitted. This off-duty nurse phones the nurse on duty and says, "Please document administration of the medication for me. My password is alpha1." The nurse receiving the call should a. fulfill the request promptly. b. document the caller's password. c. refer the matter to the charge nurse to resolve. d. report the request to the patient's health care provider.

C Fraudulent documentation may be grounds for discipline by the state board of nursing. Referring the matter to the charge nurse will allow observance of hospital policy while ensuring that documentation occurs. Notifying the health care provider would be unnecessary when the charge nurse can resolve the problem. Nurses should not provide passwords to others.

A nurse can anticipate anticholinergic side effects are likely when a patient takes a. lithium. b. buspirone. c. imipramine. d. risperidone.

C Imipramine (Tofranil) is a tricyclic antidepressant with strong anticholinergic properties, resulting in dry mouth, blurred vision, constipation, and urinary retention.

A category 5 tornado occurred in a community of 400 people. Many homes and businesses were destroyed. In the 2 years following the disaster, 140 individuals were diagnosed with posttraumatic stress disorder (PTSD). Which term best applies to these newly diagnosed cases? a. Prevalence b. Comorbidity c. Incidence d. Parity

C Incidence refers to the number of new cases of mental disorders in a healthy population within a given period of time.

Which individual diagnosed with a mental illness may need involuntary hospitalization? An individual a. who has a panic attack after her child gets lost in a shopping mall. b. with visions of demons emerging from cemetery plots throughout the community. c. who takes 38 acetaminophen tablets after the person's stock portfolio becomes worthless. d. diagnosed with major depression who stops taking prescribed antidepressant medication.

C Involuntary hospitalization protects patients who are dangerous to themselves or others and cannot care for their own basic needs. Involuntary hospitalization also protects other individuals in society. An overdose of acetaminophen indicates dangerousness to self. The behaviors described in the other options are not sufficient to require involuntary hospitalization.

Operant conditioning is part of the treatment plan to encourage speech in a child who is nearly mute. Which technique applies? a. Encourage the child to observe others talking. b. Include the child in small group activities. c. Give the child a small treat for speaking. d. Teach the child relaxation techniques.

C Operant conditioning involves giving positive reinforcement for a desired behavior. Treats are rewards and reinforce speech through positive reinforcement.

A nurse consistently encourages patient to do his or her own activities of daily living. If the patient is unable to complete an activity, the nurse helps until the patient is once again independent. This nurse's practice is most influenced by which theorist? a. Betty Neuman b. Patricia Benner c. Dorothea Orem d. Joyce Travelbee

C Orem emphasizes the role of the nurse in promoting self-care activities of the patient, this has relevance to the seriously and persistently mentally ill patient.

A patient diagnosed with schizophrenia had an exacerbation related to medication non-adherence and was hospitalized for 5 days. The patient's thoughts are now more organized and discharge is planned. The patient's family says, "It's too soon for discharge. We will just go through all this again." The nurse should a. ask the case manager to arrange a transfer to a long-term care facility. b. notify hospital security to handle the disturbance and escort the family off the unit. c. explain that the patient will continue to improve if the medication is taken regularly. d. contact the health care provider to meet with the family and explain the discharge rationale.

C Patients do not stay in a hospital until every symptom disappears. The nurse must assume responsibility to advocate for the patient's right to the least restrictive setting as soon as the symptoms are under control and for the right of citizens to control health care costs. The health care provider will use the same rationale. Shifting blame will not change the discharge. Security is unnecessary. The nurse can handle this matter.

A nurse instructs a patient taking a drug that inhibits MAO to avoid certain foods and drugs because of the risk of a. cardiac dysrhythmia. b. hypotensive shock. c. hypertensive crisis. d. hypoglycemia.

C Patients taking MAO-inhibiting drugs must be on a low tyramine diet to prevent hypertensive crisis. In the presence of MAO inhibitors, tyramine is not destroyed by the liver and in high levels produces intense vasoconstriction, resulting in elevated blood pressure.

A nurse cares for a group of patients receiving various medications, including haloperidol, carbamazepine, trazodone, and phenalgine. The nurse will order a special diet for the patient who takes a. carbamazepine. b. haloperidol. c. phenelzine. d. trazodone

C Patients taking phenelzine, an MAO inhibitor, must be on a low tyramine diet to prevent hypertensive crisis. There are no specific dietary precautions associated with the distracters.

A psychotherapist works with an anxious, dependent patient. Which strategy is most consistent with psychoanalytic psychotherapy? a. Identifying the patient's strengths and assets b. Praising the patient for describing feelings of isolation c. Focusing on feelings developed by the patient toward the therapist d. Providing psychoeducation and emphasizing medication adherence

C Positive or negative feelings of the patient toward the therapist indicate transference. Transference is a psychoanalytic concept that can be used to explore previously unresolved conflicts. The distracters relate to biological therapy and supportive psychotherapy. Use of psychoeducational materials is a common "homework" assignment used in cognitive therapy.

A patient diagnosed with major depression received six electroconvulsive therapy sessions and aggressive doses of antidepressant medication. The patient owns a small business and was counseled not to make major decisions for a month. Select the correct rationale for this counseling. a. Antidepressant medications alter catecholamine levels, which impairs decision-making abilities. b. Antidepressant medications may cause confusion related to limitation of tyramine in the diet. c. Temporary memory impairments and confusion may occur with electroconvulsive therapy. d. The patient needs time to readjust to a pressured work schedule.

C Recent memory impairment and/or confusion is often present during and for a short time after electroconvulsive therapy. An inappropriate business decision might be made because of forgotten important details. The rationales are untrue statements in the incorrect responses. The patient needing time to reorient to a pressured work schedule is less relevant than the correct rationale.

A patient diagnosed with a somatic symptom disorder says, "My pain is from an undiagnosed injury. I can't take care of myself. I need pain medicine six or seven times a day. I feel like a baby because my family has to help me so much." It is important for the nurse to assess: a. mood. b. cognitive style. c. secondary gains. d. identity and memory.

C Secondary gains should be assessed. The patient's dependency needs may be met through care from the family. When secondary gains are prominent, the patient is more resistant to giving up the symptom. The scenario does not allude to a problem of mood. Cognitive style and identity and memory assessment are of lesser concern because the patient's diagnosis has been established.

To assist patients diagnosed with somatic symptom disorders, nursing interventions of high priority: a. explain the pathophysiology of symptoms. b. help these patients suppress feelings of anger. c. shift focus from somatic symptoms to feelings. d. investigate each physical symptom as it is reported.

C Shifting the focus from somatic symptoms to feelings or to neutral topics conveys interest in the patient as a person rather than as a condition. The need to gain attention with the use of symptoms is reduced over the long term. A desired outcome would be that the patient would express feelings, including anger if it is present. Once physical symptoms are investigated, they do not need to be reinvestigated each time the patient reports them.

A college student received an invitation to attend the wedding of a close friend who lives across the country. The student is afraid of flying. Which type of therapy would be most helpful for this patient? a. Psychoanalysis b. Aversion therapy c. Systematic desensitization d. Short-term dynamic therapy

C Systematic desensitization is a type of therapy aimed at extinguishing a specific behavior, such as the fear of flying. Psychoanalysis and short-term dynamic therapy seek to uncover conflicts. Aversion therapy involves use of a noxious stimulus, punishment, and avoidance.

A patient who has been hospitalized for 3 days with a serious mental illness says, "I've got to get out of here and back to my job. I get 60 to 80 messages a day, and I'm getting behind on my email correspondence." What is this patient's perspective about health and illness? a. Fateful, magical b. Eastern, holistic c. Western, biomedical d. Harmonious, religious

C The Western biomedical perspective holds the belief that sick people should be as independent and self-reliant as possible. Self-care is encouraged.

The case manager plans to discuss the treatment plan with a patient's family. Select the case manager's first action. a. Determine an appropriate location for the conference. b. Support the discussion with examples of the patient's behavior. c. Obtain the patient's permission for the exchange of information. d. Determine which family members should participate in the conference.

C The case manager must respect the patient's right to privacy, which extends to discussions with family. Talking to family members is part of the case manager's role. Actions identified in the distracters occur after the patient has given permission.

A patient was hospitalized for 24 hours after a reaction to a psychotropic medication. While planning discharge, the case manager learned that the patient received a notice of eviction immediately prior to admission. Select the case manager's most appropriate action. a. Postpone the patient's discharge from the hospital. b. Contact the landlord who evicted the patient to further discuss the situation. c. Arrange a temporary place for the patient to stay until new housing can be arranged. d. Determine whether the adverse medication reaction was genuine because the patient had nowhere to live.

C The case manager should intervene by arranging temporary shelter for the patient until an apartment can be found. This activity is part of the coordination and delivery of services that falls under the case manager role. None of the other options is a viable alternative.

A nurse presents a community education program about mental illness. Which comment by a participant best demonstrates a correct understanding of mental illness from a biological perspective? a. "Some people experience life events so traumatic that they cannot be overcome." b. "Disturbed and conflicted family relationships are usually a starting place for mental illness." c. "My friend has had bipolar disorder for years and many problems have resulted. It's not her fault." d. "Mental illness is the result of developmental complications that cause a person not to grow to their full potential."

C The correct response demonstrates an understanding that mental illness is physical in origin. The physical origins of mental illness are aspects of the biological model. The incorrect responses assign the origins of mental illness to interpersonal relationships and traumatic events.

An adult dies in a tragic accident. Afterward, the siblings plan a funeral service. Which statement by a sibling best indicates a sense of self-actualization? a. "Of all of us, I am the most experienced with planning these types of events." b. "Funerals are supposed to be conducted quietly, respectfully, and according to a social protocol." c. "This death was unfair but I hope we can plan a service that everyone feels is a celebration of life." d. "This death was probably the consequence of years of selfish and inconsiderate behavior by our sibling."

C The correct response shows an accurate perception of reality as well as a focus on solving the problem in a way that involves others. These factors are characteristic of self-actualization. The incorrect responses demonstrate self-centeredness, rigidity, and blaming which are characteristic of a failure to achieve self-actualization.

A new antidepressant is prescribed for an elderly patient diagnosed with major depressive disorder, but the dose is more than the usual geriatric dose. The nurse should a. consult a reliable drug reference. b. teach the patient about possible side effects and adverse effects. c. withhold the medication and confer with the health care provider. d. encourage the patient to increase oral fluids to reduce drug concentration.

C The dose of antidepressants for elderly patients is often less than the usual adult dose. The nurse should withhold the medication and consult the health care provider who wrote the order. The nurse's duty is to practice according to professional standards as well as intervene and protect the patient.

A patient being treated for depression has taken 300 mg amitriptyline (Elavil) daily for a year. The patient calls the case manager at the clinic and says, "I stopped taking my antidepressant 2 days ago. Now I am having cold sweats, nausea, a rapid heartbeat, and nightmares." The nurse will advise the patient to: a. "Go to the nearest emergency department immediately." b. "Do not to be alarmed. Take two aspirin and drink plenty of fluids." c. "Take a dose of your antidepressant now and come to the clinic to see the health care provider." d. "Resume taking your antidepressants for 2 more weeks and then discontinue them again."

C The patient has symptoms associated with abrupt withdrawal of the tricyclic antidepressant. Taking a dose of the drug will ameliorate the symptoms. Seeing the health care provider will allow the patient to discuss the advisability of going off the medication and to be given a gradual withdrawal schedule if discontinuation is the decision. This situation is not a medical emergency, although it calls for medical advice. Resuming taking the antidepressant for 2 more weeks and then discontinuing again would produce the same symptoms the patient is experiencing.

Major depression resulted after a patient's employment was terminated. The patient now says to the nurse, "I'm not worth the time you spend with me. I am the most useless person in the world." Which nursing diagnosis applies? a. Powerlessness b. Defensive coping c. Situational low self-esteem d. Disturbed personal identity

C The patient's statements express feelings of worthlessness and most clearly relate to the nursing diagnosis of situational low self-esteem. Insufficient information exists to lead to other diagnoses.

The parent of a 4-year-old rewards and praises the child for helping a sibling, being polite, and using good manners. These qualities are likely to be internalized and become part of which system of the personality? a. Id b. Ego c. Superego d. Preconscious

C The superego contains the "shoulds," or moral standards internalized from interactions with significant others. Praise fosters internalization of desirable behaviors. The id is the center of basic instinctual drives, and the ego is the mediator. The ego is the problem-solving and reality-testing portion of the personality that negotiates solutions with the outside world. The preconscious is a level of awareness from which material can be retrieved easily with conscious effort. This item relates to an audience response question.

An adult says, "I never know the answers," and "My opinion does not count." Which psychosocial crisis was unsuccessfully resolved for this adult? a. Initiative versus guilt b. Trust versus mistrust c. Autonomy versus shame and doubt d. Generativity versus self-absorption

C These statements show severe self-doubt, indicating that the crisis of gaining control over the environment was not met successfully. Unsuccessful resolution of the crisis of initiative versus guilt results in feelings of guilt. Unsuccessful resolution of the crisis of trust versus mistrust results in poor interpersonal relationships and suspicion of others. Unsuccessful resolution of the crisis of generativity versus self-absorption results in self-absorption that limits the ability to grow as a person.

A nurse's identification badge includes the term, "Psychiatric Mental Health Nurse." A client with a history of paranoia asks, "What does that title mean?" The nurse responds best by answering: A. "Don't be afraid; it means I'm here to help, not hurt, you." B. "Psychiatric mental health nurses care for people with mental illnesses." C. "We have the specialized skills needed to care for those with mental illnesses." D. "The nurses who work in mental health facilities have that title."

C "We have the specialized skills needed to care for those mental illnesses."

A nurse explains the multiaxial DSM-IV-TR to a psychiatric technician and includes information that it: a. focuses on plans for treatment. b. includes nursing and medical diagnoses. c. classifies problems in multiple areas of functioning. d. uses the framework of a specific biopsychosocial theory.

C The use of five axes requires assessment beyond diagnosis of a mental disorder and includes relevant medical conditions, psychosocial and environmental problems, and global assessment of functioning. The DSM-IV-TR does not include treatment plans or nursing diagnoses. It does not use specific biopsychosocial theories

In which situations would a nurse have the duty to intervene and report? (Select all that apply.) a. A peer has difficulty writing measurable outcomes. b. A health care provider gives a telephone order for medication. c. A peer tries to provide patient care in an alcohol-impaired state. d. A team member violates relationship boundaries with a patient. e. A patient refuses medication prescribed by a licensed health care provider.

C, D Both keyed answers are events that jeopardize patient safety. The distracters describe situations that may be resolved with education or that are acceptable practices.

Which comments by a nurse demonstrate use of therapeutic communication techniques? (Select all that apply.) a. "Why do you think these events have happened to you?" b. "There are people with problems much worse than yours." c. "I'm glad you were able to tell me how you felt about your loss." d. "I noticed your hands trembling when you told me about your accident." e. "You look very nice today. I'm proud you took more time with your appearance."

C, D The correct responses demonstrate use of the therapeutic techniques making an observation and showing empathy. The incorrect responses demonstrate minimizing feelings, probing, and giving approval, which are nontherapeutic techniques.

Why is the study of culture so important for psychiatric nurses in the United States? (Select all that apply.) a. Psychiatric nurses often practice in other countries. b. Psychiatric nurses must advocate for the traditions of the Western culture. c. Cultural competence helps protect patients from prejudice and discrimination. d. Patients should receive information about their illness and treatment in terms they understand. e. Psychiatric nurses often interface with patients and their significant others over a long period of time.

C, D, E One purpose of cultural competence is for the psychiatric nurse to relate and explain information about the patient's illness and treatment in an understandable way, incorporating the patient's own beliefs and values. A fundamental aspect of nursing practice is advocacy. Cultural competence promotes recognition of prejudices in care, such as stigma and misdiagnosis. Psychiatric nurses often interface with patients and families over years and in community settings.

Which statements most clearly reflect the stigma of mental illness? (Select all that apply.) a. "Many mental illnesses are hereditary." b. "Mental illness can be evidence of a brain disorder." c. "People claim mental illness so they can get disability checks." d. "Mental illness results from the breakdown of American families." e. "If people with mental illness went to church, their symptoms would disappear."

C, D, E Stigma is represented by judgmental remarks that discount the reality and validity of mental illness. Many mental illnesses are genetically transmitted. Neuroimaging can show changes associated with some mental illnesses.

Which actions violate the civil rights of a psychiatric patient? The nurse (Select all that apply) a. performs mouth checks after overhearing a patient say, "I've been spitting out my medication." b. begins suicide precautions before a patient is assessed by the health care provider. c. opens and reads a letter a patient left at the nurse's station to be mailed. d. places a patient's expensive watch in the hospital business office safe. e. restrains a patient who uses profanity when speaking to the nurse.

C, E The patient has the right to send and receive mail without interference. Restraint is not indicated because a patient uses profanity

Which statement about diagnosis of a mental disorder is true?

Culture may cause variations in symptoms for each clinical disorder.

Select the best response for the nurse to a question from another health professional regarding the difference between a diagnosis in DSM-V and a nursing diagnosis. a. "There is no functional difference between the two. Both identify human disorders." b. "The DSM-V diagnosis disregards culture, whereas the nursing diagnosis takes culture into account." c. "The DSM-V diagnosis describes causes of disorders whereas a nursing diagnosis does not explore etiology." d. "The DSM-V diagnosis guides medical treatment, whereas the nursing diagnosis offers a framework for identifying interventions for issues a patient is experiencing."

D

A patient checks and rechecks electrical cords related to an obsessive thought that the house may burn down. The nurse and patient explore the likelihood of an actual fire. The patient states this event is not likely. This counseling demonstrates principles of: a.flooding. b.desensitization. c.relaxation technique. d.cognitive restructuring.

D Cognitive restructuring involves the patient in testing automatic thoughts and drawing new conclusions. Desensitization involves graduated exposure to a feared object. Relaxation training teaches the patient to produce the opposite of the stress response. Flooding exposes the patient to a large amount of an undesirable stimulus in an effort to extinguish the anxiety response.

A person who feels unattractive repeatedly says, "Although I'm not beautiful, I am smart." This is an example of: a.repression. b.devaluation. c.identification. d.compensation.

D Compensation is an unconscious process that allows us to make up for deficits in one area by excelling in another area to raise self-esteem. Repression unconsciously puts an idea, event, or feeling out of awareness. Identification is an unconscious mechanism calling for imitation of mannerisms or behaviors of another. Devaluation occurs when the individual attributes negative qualities to self or others.

A patient undergoing diagnostic tests says, "Nothing is wrong with me except a stubborn chest cold." The spouse reports the patient smokes, coughs daily, lost 15 pounds, and is easily fatigued. Which defense mechanism is the patient using? a.Displacement c.Projection b.Regression d.Denial

D Denial is an unconscious blocking of threatening or painful information or feelings. Regression involves using behaviors appropriate at an earlier stage of psychosexual development. Displacement shifts feelings to a more neutral person or object. Projection attributes one's own unacceptable thoughts or feelings to another.

A student says, "Before taking a test, I feel very alert and a little restless." The nurse can correctly assess the student's experience as: a.culturally influenced. b.displacement. c.trait anxiety. d.mild anxiety.

D Mild anxiety is rarely obstructive to the task at hand. It may be helpful to the patient because it promotes study and increases awareness of the nuances of questions. The incorrect responses have different symptoms. See relationship to audience response question.

A person who has been unable to leave home for more than a week because of severe anxiety says, "I know it does not make sense, but I just can't bring myself to leave my apartment alone." Which nursing intervention is appropriate? a.Help the person use online video calls to provide interaction with others. b.Advise the person to accept the situation and use a companion. c.Ask the person to explain why the fear is so disabling. d.Teach the person to use positive self-talk techniques.

D Positive self-talk, a form of cognitive restructuring, replaces negative thoughts such as "I can't leave my apartment" with positive thoughts such as "I can control my anxiety." This technique helps the patient gain mastery over the symptoms. The other options reinforce the sick role.

As a result of Harry Stack Sullivan's work, the mental health nurse is involved in providing clients with a. security operations. b. psychoanalysis. c. analysis of behavior patterns. d. a psychotherapeutic environment.

D a psychotherapeutic environment

A nurse interacts with a newly hospitalized patient. Select the nurse's comment that applies the communication technique of "offering self." a. "I've also had traumatic life experiences. Maybe it would help if I told you about them." b. "Why do you think you had so much difficulty adjusting to this change in your life?" c. "I hope you will feel better after getting accustomed to how this unit operates." d. "I'd like to sit with you for a while to help you get comfortable talking to me."

D "Offering self" is a technique that should be used in the orientation phase of the nurse-patient relationship. Sitting with the patient, an example of "offering self," helps to build trust and convey that the nurse cares about the patient. Two incorrect responses are ineffective and nontherapeutic. The other incorrect response is therapeutic but is an example of "offering hope."

Which belief will best support a nurse's efforts to provide patient advocacy during a multidisciplinary patient care planning session? a. All mental illnesses are culturally determined. b. Schizophrenia and bipolar disorder are cross-cultural disorders. c. Symptoms of mental disorders are unchanged from culture to culture. d. Assessment findings in mental disorders reflect a person's cultural patterns.

D A nurse who understands that a patient's symptoms are influenced by culture will be able to advocate for the patient to a greater degree than a nurse who believes that culture is of little relevance. The distracters are untrue statements.

Which patient would be most appropriate to refer for assertive community treatment (ACT)? A patient diagnosed with a. a phobic fear of crowded places. b. a single episode of major depressive disorder. c. a catastrophic reaction to a tornado in the community. d. schizophrenia and four hospitalizations in the past year.

D ACT provides intensive case management for persons with serious persistent mental illness who live in the community. Repeated hospitalization is a frequent reason for this intervention. The distracters identify mental health problems of a more episodic nature.

The patients below were evaluated in the emergency department. The psychiatric unit has one bed available. Which patient should be admitted? The patient a. feeling anxiety and a sad mood after separation from a spouse of 10 years. b. who self-inflicted a superficial cut on the forearm after a family argument. c. experiencing dry mouth and tremor related to taking antipsychotic medication. d. who is a new parent and hears voices saying, "Smother your baby."

D Admission to the hospital would be justified by the risk of patient danger to self or others. The other patients have issues that can be handled with less restrictive alternatives than hospitalization.

A patient diagnosed with depression is receiving imipramine (Tofranil) 200 mg qhs. Which assessment finding would prompt the nurse to collaborate with the health care provider regarding potentially hazardous side effects of this drug? a. Dry mouth b. Blurred vision c. Nasal congestion d. Urinary retention

D All the side effects mentioned are the result of the anticholinergic effects of the drug. Only urinary retention and severe constipation warrant immediate medical attention. Dry mouth, blurred vision, and nasal congestion may be less troublesome as therapy continues.

A newly admitted acutely psychotic patient is a private patient of the medical director and a private-pay patient. To whom does the psychiatric nurse assigned to the patient owe the duty of care? a. Medical director b. Hospital c. Profession d. Patient

D Although the nurse is accountable to the health care provider, the agency, the patient, and the profession, the duty of care is owed to the patient. This duty reflects both legal and ethical standards of nursing practice.

The nurse assigned to ACT should explain the program's treatment goal as a. assisting patients to maintain abstinence from alcohol and other substances of abuse. b. providing structure and a therapeutic milieu for mentally ill patients whose symptoms require stabilization. c. maintaining medications and stable psychiatric status for incarcerated inmates who have a history of mental illness. d. providing services for mentally ill individuals who require intensive treatment to continue to live in the community.

D An ACT program provides intensive community services to persons with serious, persistent mental illness who live in the community but require aggressive services to prevent repeated hospitalizations.

Which aspect of direct care is an experienced, inpatient psychiatric nurse most likely to provide for a patient? a. Hygiene assistance b. Diversional activities c. Assistance with job hunting d. Building assertiveness skills

D Assertiveness training relies on the counseling and psychoeducational skills of the nurse. Assistance with personal hygiene would usually be accomplished by a psychiatric technician or nursing assistant. Diversional activities are usually the province of recreational therapists. The patient would probably be assisted in job hunting by a social worker or vocational therapist.

To plan effective care for patients diagnosed with somatic symptom disorders, the nurse should understand that patients have difficulty giving up the symptoms because the symptoms: a. are generally chronic. b. have a physiological basis. c. can be voluntarily controlled. d. provide relief from health anxiety.

D At the unconscious level, the patient's primary gain from the symptoms is anxiety relief. Considering that the symptoms actually make the patient more psychologically comfortable and may also provide secondary gain, patients frequently fiercely cling to the symptoms. The symptoms tend to be chronic, but that does not explain why they are difficult to give up. The symptoms are not under voluntary control or physiologically based.

What is the legal significance of a nurse's action when a patient verbally refuses medication and the nurse gives the medication over the patient's objection? The nurse a. has been negligent. b. committed malpractice. c. fulfilled the standard of care. d. can be charged with battery.

D Battery is an intentional tort in which one individual violates the rights of another through touching without consent. Forcing a patient to take medication after the medication was refused constitutes battery. The charge of battery can be brought against the nurse. The medication may not necessarily harm the patient-- harm is a component of malpractice.

A nurse listens to a group of recent retirees. One says, "I volunteer with Meals on Wheels, coach teen sports, and do church visitation." Another laughs and says, "I'm too busy taking care of myself to volunteer to help others." Which psychosocial developmental task do these statements contrast? a. Trust and mistrust b. Intimacy and isolation c. Industry and inferiority d. Generativity and self-absorption

D Both retirees are in middle adulthood, when the developmental crisis to be resolved is generativity versus self-absorption. One exemplifies generativity the other embodies self-absorption.

An informal group of patients discusses their perceptions of nursing care. Which comment best indicates a patient perceived the nurse was caring? "My nurse: a. always asks me which type of juice I want to help me swallow my medication." b. explained my treatment plan to me and asked for my ideas about how to make it better." c. told me that if I take all the medicines the doctor prescribes, then I will get discharged soon." d. spends time listening to me talk about my problems. That helps me feel like I'm not alone."

D Caring evidences empathetic understanding as well as competency. It helps change pain and suffering into a shared experience, creating a human connection that alleviates feelings of isolation. The distracters give examples of statements that demonstrate advocacy or giving advice.

Clinical pathways are used in managed care settings to a. stabilize aggressive patients. b. identify obstacles to effective care. c. relieve nurses of planning responsibilities. d. streamline the care process and reduce costs.

D Clinical pathways provide guidelines for assessments, interventions, treatments, and outcomes as well as a designated timeline for accomplishment. Deviations from the timeline must be reported and investigated. Clinical pathways streamline the care process and save money.

A patient experiencing psychosis became aggressive, struck another patient, and required seclusion. Select the best documentation. a. Patient struck another patient who attempted to leave day room to go to bathroom. Seclusion necessary at 1415. Plan: Maintain seclusion for 8 hours and keep these two patients away from each other for 24 hours. b. Seclusion ordered by physician at 1415 after command hallucinations told the patient to hit another patient. Careful monitoring of patient maintained during period of seclusion. c. Seclusion ordered by MD for aggressive behavior. Begun at 1415. Maintained for 2 hours without incident. Outcome: Patient calmer and apologized for outburst. d. Patient pacing, shouting. Haloperidol 5 mg given PO at 1300. No effect by 1315. At 1415 patient yelled, "I'll punch anyone who gets near me," and struck another patient with fist. Physically placed in seclusion at 1420. Seclusion order obtained from MD at 1430.

D Documentation must be specific and detail the key aspects of care. It should demonstrate implementation of the least restrictive alternative. Justification for why a patient was secluded should be recorded, along with interventions attempted in an effort to avoid seclusion. Documentation should include a description of behavior and verbalizations, interventions tried and their outcomes, and the name of the health care provider ordering the use of seclusion.

A patient expresses a desire to be cared for by others and often behaves in a helpless fashion. Which stage of psychosexual development is most relevant to the patient's needs? a. Latency b. Phallic c. Anal d. Oral

D Fixation at the oral stage sometimes produces dependent infantile behaviors in adults. Latency fixations often result in difficulty identifying with others and developing social skills, resulting in a sense of inadequacy and inferiority. Phallic fixations result in having difficulty with authority figures and poor sexual identity. Anal fixation sometimes results in retentiveness, rigidity, messiness, destructiveness, and cruelty. This item relates to an audience response question.

A patient with fears of serious heart disease was referred to the mental health center by a cardiologist. Extensive diagnostic evaluation showed no physical illness. The patient says, "My chest is tight, and my heart misses beats. I'm often absent from work. I don't go out much because I need to rest." Which health problem is most likely? a. Dysthymic disorder b. Somatic symptom disorder c. Antisocial personality disorder d. Illness anxiety disorder (hypochondriasis)

D Illness anxiety disorder (hypochondriasis) involves preoccupation with fears of having a serious disease even when evidence to the contrary is available. The preoccupation causes impairment in social or occupational functioning. Somatic symptom disorder involves fewer symptoms. Dysthymic disorder is a disorder of lowered mood. Antisocial disorder applies to a personality disorder in which the individual has little regard for the rights of others. See relationship to audience response question.

A person says, "I was the only survivor in a small plane crash. Three business associates died. I got depressed and saw a counselor twice a week for 4 weeks. We talked about my feelings related to being a survivor, and I'm better now." Which type of therapy was used? a. Systematic desensitization b. Psychoanalysis c. Behavior modification d. Interpersonal psychotherapy

D Interpersonal psychotherapy returned the patient to his former level of functioning by helping him come to terms with the loss of friends and guilt over being a survivor. Systematic desensitization is a type of therapy aimed at extinguishing a specific behavior, such as the fear of flying. Psychoanalysis would call for a long period of exploration of unconscious material. Behavior modification would focus on changing a behavior rather than helping the patient understand what is going on in his life.

Which patient meets criteria for involuntary hospitalization for psychiatric treatment? The patient who a. is noncompliant with the treatment regimen. b. fraudulently files for bankruptcy. c. sold and distributed illegal drugs. d. threatens to harm self and others.

D Involuntary hospitalization protects patients who are dangerous to themselves or others and cannot care for their own basic needs. Involuntary commitment also protects other individuals in society. The behaviors described in the other options are not sufficient to require involuntary hospitalization

A Puerto Rican American patient uses dramatic body language when describing emotional discomfort. Which analysis most likely explains the patient's behavior? The patient a. has a histrionic personality disorder. b. believes dramatic body language is sexually appealing. c. wishes to impress staff with the degree of emotional pain. d. belongs to a culture in which dramatic body language is the norm.

D Members of Hispanic American subcultures tend to use high affect and dramatic body language as they communicate. The other options are more remote possibilities.

A patient diagnosed with major depression refuses solid foods. In order to meet nutritional needs, which beverage will the nurse offer to this patient? a. Tomato juice b. Orange juice c. Hot tea d. Milk

D Milk is the only beverage listed that provides protein, fat, and carbohydrates. In addition, milk is fortified with vitamins.

A Native American patient sadly describes a difficult childhood. The patient abused alcohol as a teenager but stopped 10 years ago. The patient now says, "I feel stupid and good for nothing. I don't help my people." How should the treatment team focus planning for this patient? a. Psychopharmacological and somatic therapies should be central techniques. b. Apply a psychoanalytical approach, focused on childhood trauma. c. Depression and alcohol abuse should be treated concurrently. d. Use a holistic approach, including mind, body, and spirit.

D Native Americans, because of their beliefs in the interrelatedness of parts and about being in harmony with nature, respond best to a holistic approach.

A nurse is caring for a patient with low self-esteem. Which nonverbal communication should the nurse anticipate from this patient? a. Arms crossed b. Staring at the nurse c. Smiling inappropriately d. Eyes pointed downward

D Nonverbal communication is usually considered more powerful than verbal communication. Downward casted eyes suggest feelings of worthlessness or hopelessness.

A nurse worked with a patient diagnosed with major depression, severe withdrawal, and psychomotor retardation. After 3 weeks, the patient did not improve. The nurse is most at risk for feelings of: a. guilt and despair. b. over-involvement. c. interest and pleasure. d. ineffectiveness and frustration.

D Nurses may have expectations for self and patients that are not wholly realistic, especially regarding the patient's progress toward health. Unmet expectations result in feelings of ineffectiveness, anger, or frustration. Nurses rarely become over-involved with patients with depression because of the patient's resistance. Guilt and despair might be seen when the nurse experiences the patient's feelings because of empathy. Interest is possible, but not the most likely result.

A nurse provided medication education for a patient diagnosed with major depression who began a new prescription for phenelzine (Nardil). Which behavior indicates effective learning? The patient: a. monitors sodium intake and weight daily. b. wears support stockings and elevates the legs when sitting. c. can identify foods with high selenium content that should be avoided. d. confers with a pharmacist when selecting over-the-counter medications.

D Over-the-counter medicines may contain vasopressor agents or tyramine, a substance that must be avoided when the patient takes MAOI antidepressants. Medications for colds, allergies, or congestion or any preparation that contains ephedrine or phenylpropanolamine may precipitate a hypertensive crisis. MAOI antidepressant therapy is unrelated to the need for sodium limitation, support stockings, or leg elevation. MAOIs interact with tyramine-containing foods, not selenium, to produce dangerously high blood pressure.

A patient tells the nurse, "My doctor prescribed paroxetine for my depression. I assume I'll have side effects like I had when I was taking imipramine." The nurse's reply should be based on the knowledge that paroxetine is a(n) a. selective norepinephrine reuptake inhibitor. b. tricyclic antidepressant. c. monoamine oxidase (MAO) inhibitor. d. SSRI.

D Paroxetine is an SSRI and will not produce the same side effects as imipramine, a tricyclic antidepressant. The patient will probably not experience dry mouth, constipation, or orthostatic hypotension.

A patient in the emergency department shows a variety of psychiatrical symptoms, including restlessness and anxiety. The patient says, "I feel sad because evil spirits have overtaken my mind." Which worldview is most applicable to this individual? a. Eastern/balance b. Southern/holistic c. Western/scientific d. Indigenous/harmony

D Persons of an indigenous worldview believe disease results from a lack of personal, interpersonal, environmental, or spiritual harmony and that evil spirits exist. The holism of body-mind-spirit is a key component of this view. If one believes an evil spirit has taken control, distress results. Western and Eastern worldviews do not embrace spirits.

A patient begins therapy with a phenothiazine medication. What teaching should the nurse provide related to the drug's strong dopaminergic effect? a. Chew sugarless gum. b. Increase dietary fiber. c. Arise slowly from bed. d. Report changes in muscle movement.

D Phenothiazines block dopamine receptors in both the limbic system and basal ganglia. Movement disorders and motor abnormalities (extrapyramidal side effects), such as parkinsonism, akinesia, akathisia, dyskinesia, and tardive dyskinesia, are likely to occur early in the course of treatment. They are often heralded by sensations of muscle stiffness. Early intervention with antiparkinsonism medication can increase the patient's comfort and prevent dystonic reactions. The distracters are related to anticholinergic effects.

A new staff nurse completes orientation to a psychiatric unit. This nurse may expect an advanced practice nurse to perform which additional interventions? a. Conduct mental health assessments b. Establish therapeutic relationships c. Individualize nursing care plans d. Prescribe psychotropic medication

D Prescriptive privileges are granted to masters-prepared nurse practitioners who have taken special courses on prescribing medication. The nurse prepared at the basic level is permitted to perform mental health assessments, establish relationships, and provide individualized care planning

Which nursing intervention below is part of the scope of an advanced practice psychiatric/mental health nurse rather than a basic level registered nurse? a. Coordination of care b. Health teaching c. Milieu therapy d. Psychotherapy

D Psychotherapy is part of the scope of practice of an advanced practice nurse.

Which individual is demonstrating the highest level of resilience? One who a. is able to repress stressors. b. becomes depressed after the death of a spouse. c. lives in a shelter for 2 years after the home is destroyed by fire. d. takes a temporary job to maintain financial stability after loss of a permanent job.

D Resilience is closely associated with the process of adapting and helps people facing tragedy, loss, trauma and severe stress.

A black patient, originally from Haiti, has a diagnosis of major depressive disorder. A colleague tells the nurse, "This patient often looks down and is reluctant to share feelings. However, I've observed the patient spontaneously interacting with other black patients." Select the nurse's best response. a. "Black patients depend on the church for support. Have you consulted the patient's pastor?" b. "Encourage the patient to talk in a group setting. It will be less intimidating than one-to-one interaction." c. "Don't take it personally. Black patients often have a resentful attitude that takes a long time to overcome." d. "The patient may have difficulty communicating in English. Have you considered using a cultural broker?"

D Society expects a culturally diverse patient to accommodate and use English. Feelings are abstract, which requires a greater command of the language. This may be especially difficult during episodes of high stress or mental illness. Cultural brokers can be helpful with language and helping the nurse to understand the Haitian worldview and cultural nuances.

A patient became severely depressed when the last of the family's six children moved out of the home 4 months ago. The patient repeatedly says, "No one cares about me. I'm not worth anything." Which response by the nurse would be the most helpful? a. "Things will look brighter soon. Everyone feels down once in a while." b. "Our staff members care about you and want to try to help you get better." c. "It is difficult for others to care about you when you repeatedly say the same negative things." d. "I'll sit with you for 10 minutes now and 10 minutes after lunch to help you feel that I care about you."

D Spending time with the patient at intervals throughout the day shows acceptance by the nurse and will help the patient establish a relationship with the nurse. The therapeutic technique is "offering self." Setting definite times for the therapeutic contacts and keeping the appointments show predictability on the part of the nurse, an element that fosters trust building. The incorrect responses would be difficult for a person with profound depression to believe, provide false reassurance, and are counterproductive. The patient is unable to say positive things at this point.

A nurse wants to find information on current evidence-based research, programs, and practices regarding mental illness and addictions. Which resource should the nurse consult? a. American Psychiatric Association b. American Psychological Association (APA) c. Clinician's Quick Guide to Interpersonal Psychotherapy d. Substance Abuse and Mental Health Services Administration (SAMHSA)

D The SAMHSA maintains a National Registry of Evidence-based Practices and Programs. New therapies are entered into the database on a regular basis. The incorrect responses are resources but do not focus on evidence-based information.

A health care provider prescribed long acting antipsychotic medication injections every 3 weeks at the clinic for a patient with a history of medication nonadherence. For this plan to be successful, which factor will be of critical importance? a. The attitude of significant others toward the patient b. Nutrition services in the patient's neighborhood c. The level of trust between the patient and nurse d. The availability of transportation to the clinic

D The ability of the patient to get to the clinic is of paramount importance to the success of the plan. The long acting antipsychotic medication injections relieve the patient of the necessity to take medication daily, but if he or she does not receive the injection at 3-week intervals, non-adherence will again be the issue. Attitude toward the patient, trusting relationships, and nutrition are important but not fundamental to this particular problem.

A Filipino American patient had a nursing diagnosis of situational low self-esteem related to poor social skills as evidenced by lack of eye contact. Interventions were applied to increase the patient's self-esteem but after 3 weeks, the patient's eye contact did not improve. What is the most accurate analysis of this scenario? a. The patient's eye contact should have been directly addressed by role playing to increase comfort with eye contact. b. The nurse should not have independently embarked on assessment, diagnosis, and planning for this patient. c. The patient's poor eye contact is indicative of anger and hostility that were unaddressed. d. The nurse should have assessed the patient's culture before making this diagnosis and plan.

D The amount of eye contact a person engages in is often culturally determined. In some cultures, eye contact is considered insolent, whereas in others eye contact is expected. Asian Americans, including persons from the Philippines, often prefer not to engage in direct eye contact.

A nurse prepares to assess a newly hospitalized patient who moved to the United States 6 months ago from Somalia. The nurse should first determine a. if the patient's immunizations are current. b. the patient's religious preferences. c. the patient's specific ethnic group. d. whether an interpreter is needed.

D The assessment depends on communication. The nurse should first determine whether an interpreter is needed. The other information can be subsequently assessed.

To provide culturally competent care, the nurse should a. accurately interpret the thinking of individual patients. b. predict how a patient may perceive treatment interventions. c. formulate interventions to reduce the patient's ethnocentrism. d. identify strategies that fit within the cultural context of the patient.

D The correct answer is the most global response.

A school age child tells the school nurse, "Other kids call me mean names and will not sit with me at lunch. Nobody likes me." Select the nurse's most therapeutic response. a. "Just ignore them and they will leave you alone." b. "You should make friends with other children." c. "Call them names if they do that to you." d. "Tell me more about how you feel."

D The correct response uses exploring, a therapeutic technique. The distracters give advice, a nontherapeutic technique.

A 26-month-old displays negative behavior, refuses toilet training, and often says, "No!" Which psychosocial crisis is evident? a. Trust versus mistrust b. Initiative versus guilt c. Industry versus inferiority d. Autonomy versus shame and doubt

D The crisis of autonomy versus shame and doubt relates to the developmental task of gaining control of self and environment, as exemplified by toilet training. This psychosocial crisis occurs during the period of early childhood. Trust versus mistrust is the crisis of the infant. Initiative versus guilt is the crisis of the preschool and early-school-aged child. Industry versus inferiority is the crisis of the 6- to 12-year-old child.

Which documentation of a patient's behavior best demonstrates a nurse's observations? a. Isolates self from others. Frequently fell asleep during group. Vital signs stable. b. Calmer-- more cooperative. Participated actively in group. No evidence of psychotic thinking. c. Appeared to hallucinate. Frequently increased volume on television, causing conflict with others. d. Wore four layers of clothing. States, "I need protection from evil bacteria trying to pierce my skin."

D The documentation states specific observations of the patient's appearance and the exact statements made. The other options are vague or subjective statements and can be interpreted in different ways.

A family member of a patient with delusions of persecution asks the nurse, "Are there any circumstances under which the treatment team is justified in violating a patient's right to confidentiality?" The nurse should reply that confidentiality may be breached a. under no circumstances. b. at the discretion of the psychiatrist. c. when questions are asked by law enforcement. d. if the patient threatens the life of another person.

D The duty to warn a person whose life has been threatened by a psychiatric patient overrides the patient's right to confidentiality. The right to confidentiality is not suspended at the discretion of the therapist or for legal investigations.

A patient's history shows drinking 4 to 6 L of fluid and eating more than 6,000 calories per day. Which part of the central nervous system is most likely dysfunctional for this patient? a. Amygdala b. Parietal lobe c. Hippocampus d. Hypothalamus

D The hypothalamus, a small area in the ventral superior portion of the brainstem, plays a vital role in such basic drives as hunger, thirst, and sex. See relationship to audience response question.

A nurse supports a parent for praising a child who behaves in helpful ways to others. When this child behaves with politeness and helpfulness in adulthood, which feeling will most likely result? a. Guilt b. Anxiety c. Humility d. Self-esteem

D The individual will be living up to the ego ideal, which will result in positive feelings about self. The other options are incorrect because each represents a negative feeling.

What is an essential difference between somatic symptom disorders and factitious disorders? a. Somatic symptom disorders are under voluntary control, whereas factitious disorders are unconscious and automatic. b. Factitious disorders are precipitated by psychological factors, whereas somatic symptom disorders are related to stress. c. Factitious disorders are individually determined and related to childhood sexual abuse, whereas somatic symptom disorders are culture bound. d. Factitious disorders are under voluntary control, whereas somatic symptom disorders involve expression of psychological stress through somatization.

D The key is the only fully accurate statement. Somatic symptom disorders involve expression of stress through bodily symptoms and are not under voluntary control or culture bound. Factitious disorders are under voluntary control. See relationship to audience response question.

Select the best response for the nurse who receives a query from another mental health professional seeking to understand the difference between a DSM-IV-TR diagnosis and a nursing diagnosis. a. "There is no functional difference between the two. Both identify human disorders." b. "The DSM-IV-TR diagnosis disregards culture, whereas the nursing diagnosis takes culture into account." c. "The DSM-IV-TR diagnosis is associated with present distress or disability, whereas a nursing diagnosis considers past and present responses to actual mental health problems." d. "The DSM-IV-TR diagnosis affects the choice of medical treatment, whereas the nursing diagnosis offers a framework for identifying interventions for phenomena a patient is experiencing."

D The medical diagnosis is concerned with the patient's disease state, causes, and cures, whereas the nursing diagnosis focuses on the patient's response to stress and possible caring interventions. Both tools consider culture. The DSM-IV-TR is multiaxial. Nursing diagnoses also consider potential problems.

In a team meeting a nurse says, "I'm concerned about whether we are behaving ethically by using restraint to prevent one patient from self-mutilation, while the care plan for another self-mutilating patient requires one-on-one supervision." Which ethical principle most clearly applies to this situation? a. Beneficence b. Autonomy c. Fidelity d. Justice

D The nurse is concerned about justice, that is, fair distribution of care, which includes treatment with the least restrictive methods for both patients. Beneficence means promoting the good of others. Autonomy is the right to make one's own decisions. Fidelity is the observance of loyalty and commitment to the patient.

A nurse assessing a patient diagnosed with a somatic symptom disorder is most likely to note that the patient: a. sees a relationship between symptoms and interpersonal conflicts. b. has little difficulty communicating emotional needs to others. c. rarely derives personal benefit from the symptoms. d. has altered comfort and activity needs.

D The patient frequently has altered comfort and activity needs associated with the symptoms displayed (fatigue, insomnia, weakness, tension, pain, etc.). In addition, hygiene, safety, and security needs may also be compromised. The patient is rarely able to see a relation between symptoms and events in his or her life, which is readily discernible to health professionals. Patients with somatic symptom disorders often derive secondary gain from their symptoms and/or have considerable difficulty identifying feelings and conveying emotional needs to others.

A patient says to the nurse, "I dreamed I was stoned. When I woke up, I felt emotionally drained, as though I hadn't rested well." Which response should the nurse use to clarify the patient's comment? a. "It sounds as though you were uncomfortable with the content of your dream." b. "I understand what you're saying. Bad dreams leave me feeling tired, too." c. "So you feel as though you did not get enough quality sleep last night?" d. "Can you give me an example of what you mean by 'stoned'?"

D The technique of clarification is therapeutic and helps the nurse examine the meaning of the patient's statement. Asking for a definition of "stoned" directly asks for clarification. Restating that the patient is uncomfortable with the dream's content is parroting, a nontherapeutic technique. The other responses fail to clarify the meaning of the patient's comment.

An individual hiking in the forest encounters a large poisonous snake on the path. Which change in this individual's vital signs is most likely? a. Pulse rate changes from 90 to 72. b. Respiratory rate changes from 22 to 18. c. Complaints of intestinal cramping begin. d. Blood pressure changes from 114/62 to 136/78.

D This frightening experience would stimulate the sympathetic nervous system, causing a release of norepinephrine, an excitatory neurotransmitter. It prepares the body for fight or flight. Increased blood pressure, pupil size, respiratory rate, and pulse rate signify release of norepinephrine. Intestinal cramping would be associated with stimulation of the parasympathetic nervous system.

The nurse should refer which of the following patients to a partial hospitalization program? A patient who a. has a therapeutic lithium level and reports regularly for blood tests and clinic follow-up. b. needs psychoeducation for relaxation therapy related to agoraphobia and panic episodes. c. spent yesterday in a supervised crisis care center and continues to have active suicidal ideation. d. states, "I'm not sure I can avoid using alcohol when my spouse goes to work every morning."

D This patient could profit from the structure and supervision provided by spending the day at the partial hospitalization program. During the evening, at night, and on weekends, the spouse could assume responsibility for supervision. A suicidal patient needs inpatient hospitalization. The other patients can be served in the community or with individual visits.

Which individual diagnosed with mental illness may need emergency or involuntary admission? The individual who a. resumes using heroin while still taking naltrexone (ReVia). b. reports hearing angels playing harps during thunderstorms. c. does not keep an outpatient appointment with the mental health nurse. d. throws a heavy plate at a waiter at the direction of command hallucinations.

D Throwing a heavy plate is likely to harm the waiter and is evidence of dangerousness to others. This behavior meets the criteria for emergency or involuntary hospitalization for mental illness. The behaviors in the other options evidence mental illness but not dangerousness. See related audience response question.

During an interview, a patient attempts to shift the focus from self to the nurse by asking personal questions. The nurse should respond by saying: a. "Why do you keep asking about me?" b. "Nurses direct the interviews with patients." c. "Do not ask questions about my personal life." d. "The time we spend together is to discuss your concerns."

D When a patient tries to focus on the nurse, the nurse should refocus the discussion back onto the patient. Telling the patient that interview time should be used to discuss patient concerns refocuses discussion in a neutral way. Telling patients not to ask about the nurse's personal life shows indignation. Saying that nurses prefer to direct the interview reflects superiority. "Why" questions are probing and nontherapeutic.

The nurse providing anticipatory guidance to the mother of a toddler should advise that childhood temper tantrums are best handled by a. giving the child what he is asking for. b. scolding the child when he displays tantrum behaviors. c. spanking the child at the onset of the tantrum behaviors. d. ignoring the tantrum and giving attention when the child acts appropriately.

D ignoring the tantrum and giving attention when the child acts appropriately

Which client problem would be most suited to the use of interpersonal therapy? a. Disturbed sensory perception b. Impaired social interaction c. Medication noncompliance d. Dysfunctional grieving

D Dysfunctional grieving

The spouse of a patient with schizophrenia says, "I don't understand how nurturing or toilet training in childhood has anything to do with this incredibly disabling illness." Which response by the nurse will best help the spouse understand this disorder? a. "This illness is the result of genetic factors." b. "Psychological stress is at the root of most mental disorders." c. "It must be frustrating for you that your spouse is sick so much of the time." d. "New findings show that this condition more likely has biological rather than psychological origins."

D Many of the most prevalent and disabling mental disorders have strong biological influences. Genetics are only one part of biological factors. Empathy does not address increasing the spouse's level of knowledge about the cause of the disorder. The other distracters are not established facts.

Which documentation of diagnosis would a nurse expect in a psychiatric treatment setting? a. I Acute renal failure II 75 III Bipolar disorder I, mixed IV Loss of disability benefits 2 months ago V None b. I Schizophrenia, paranoid type II Death of spouse last year III 60 IV None V Diabetes, type 2 c. I Polysubstance dependence II Narcissistic Personality Disorder III 90 IV Hyperlipidemia V Charges pending for assault d. I Major Depression II Avoidant Personality Disorder III Hypertension IV Home destroyed by hurricane last year V 80

D The DSM-IV-TR profiles psychiatric diagnoses on five axes. Each axis defines a specific aspect of the diagnosis. Axis I identifies major clinical disorders. Axis II details personality and developmental disorders. Axis III identifies general medical conditions. Axis IV details psychosocial and environmental problems. Axis V rates the Global Assessment of Functioning

A patient's relationships are intense and unstable. The patient initially idealizes the significant other and then devalues them, resulting in frequent feelings of emptiness. This patient will benefit from interventions to develop which aspect of mental health? a. Effectiveness in work b. Communication skills c. Productive activities d. Fulfilling relationships

D The information given centers on relationships with others, which are described as intense and unstable. The relationships of mentally healthy individuals are stable, satisfying, and socially integrated. Data are not present to describe work effectiveness, communication skills, or activities.

A patient asks, "What are neurotransmitters? My doctor said mine are imbalanced." Select the nurse's best response. a. "How do you feel about having imbalanced neurotransmitters?" b. "Neurotransmitters protect us from harmful effects of free radicals." c. "Neurotransmitters are substances we consume that influence memory and mood." d. "Neurotransmitters are natural chemicals that pass messages between brain cells."

D The patient asked for information, and the correct response is most accurate. Neurotransmitters are chemical substances that function as messengers in the central nervous system. They are released from the axon terminal, diffuse across the synapse, and attach to specialized receptors on the postsynaptic neuron. The distracters either do not answer the patient's question or provide untrue, misleading information.

A patient is depressed, mute, and motionless. According to family members, the patient has refused to bathe or eat for a week. The patient's global assessment of functioning score is: a. 100 b. 50 c. 25 d. 10

D The patient is unable to maintain personal hygiene, oral intake, or verbal communication. The patient is a danger to self because of not eating. The distracters represent higher levels of functioning.

A newly admitted patient is uncommunicative about recent life events. The nurse suspects marital and economic problems, but the social worker's assessment is not yet available. Select the nurse's best action. a. Focus assessment questions on these two topics. b. Ask another patient who shares a room with this patient. c. Avoid seeking information on these topics at this time. d. Refer to axis IV of the DSM-IV-TR in the medical record.

D The admitting physician would use axis IV to record psychosocial and environmental problems pertinent to the patient's situation, providing another source of information for the nurse. Persistent questioning may cause the patient to withdraw. The other distracters demonstrate violation of the patient's privacy rights and are not an effective solution.

Which therapies involve electrical brain stimulation for treatment of mental illness? (Select all that apply.) a. Aversion therapy b. Operant conditioning c. Systematic desensitization d. Electroconvulsive therapy (ECT) e. Transcranial magnetic stimulation (TMS)

D, E ECT and TMS are therapies that use electrical stimulation of the brain as a form of treatment for mental illness. The incorrect responses are therapies that are interpersonal in nature.

A nurse is caring for a client diagnosed with paranoid schizophrenia, asthma, generalized anxiety disorder, and borderline personality disorder. Which of the following diagnoses should the nurse expect to find included in Axis II of this client's DSM-IV-TR axis diagnosis? D. Borderline personality disorder

D. Borderline personality disorder Personality disorders and mental retardation are included in Axis II. Paranoid schizophrenia and generalized anxiety disorder are found in Axis I. Asthma and other general medical conditions are found in Axis III.

Which nursing diagnosis would most likely apply to both a patient diagnosed with major depression as well as one experiencing acute mania?

Disturbed sleep pattern

A person's spouse filed charges after repeatedly being battered. The person sarcastically says, "I'm sorry for what I did. I need psychiatric help." Which statement by this person supports an antisocial personality disorder? a."I have a quick temper, but I can usually keep it under control." b."I've done some stupid things in my life, but I've learned a lesson." c."I'm feeling terrible about the way my behavior has hurt my family." d."I hit because I am tired of being nagged. My spouse deserves the beating."

I hit because im tired of being nagged. my spouse deserves the beating

A patient says, "The other nurses won't give me my medication early, but you know what it's like to be in pain and don't let your patients suffer. Could you get me my pill now? I won't tell anyone." Which response by the nurse would be most therapeutic? a."I'm not comfortable doing that," and then ignore subsequent requests for early medication. b."I understand that you have pain, but giving medicine too soon would not be safe." c."I'll have to check with your doctor about that; I will get back to you after I do." d."It would be unsafe to give the medicine early; none of us will do that."

I understand that you have pain, but giving medicine too soon would not be safe

A child diagnosed with attention deficit hyperactivity disorder (ADHD) shows hyperactivity, aggression, and impaired play. The health care provider prescribed amphetamine salts (Adderall). The nurse should monitor for which desired behavior? a. Increased expressiveness in communication with others b. Abilities to identify anxiety and implement self-control strategies c. Improved abilities to participate in cooperative play with other children d. Tolerates social interactions for short periods without disruption or frustration

Improved abilities to participate in cooperative play with other children

Which suggestions are appropriate for the family of a patient diagnosed with bipolar disorder who is being treated as an outpatient during a hypomanic episode? Select all that apply.

Limit credit card access. Provide a structured environment. Monitor the patient's sleep patterns

A person was online continuously for over 24 hours, posting rhymes on official government websites and inviting politicians to join social networks. The person has not slept or eaten for 3 days. What features of mania are evident?

Poor judgment and hyperactivity

A patient tells the nurse, "I'm ashamed of being bipolar. When I'm manic, my behavior embarrasses everyone. Even if I take my medication, there are no guarantees. I'm a burden to my family." These statements support which nursing diagnoses? Select all that apply.

Powerlessness Chronic low self-esteem

Soon after parents announced they were divorcing, a child stopped participating in sports, sat alone at lunch, and avoided former friends. The child told the school nurse, "If my parents loved me, they would work out their problems." Which nursing diagnosis has the highest priority? a. Social isolation b. Decisional conflict c. Chronic low self-esteem d. Disturbed personal identity

Social isolation

Joel is a 43-year-old patient being seen in the mental health clinic with depression. Joel states, "I have always been a practicing Jew, but in the past few months I am questioning everything. I just don't know if I believe in it anymore." Which of the following nursing diagnoses best describes Joel's comment? Spiritual distress

Spiritual distress Correct Joel is expressing distress regarding his religion and spiritual well-being. Joel could be experiencing ineffective coping, but this does not directly relate to his comment. There is nothing in Joel's comment that would lead to the conclusion the patient is having thoughts of harming himself. Joel's comment does not describe hopelessness.

A patient has had difficulty keeping a job because of arguing with co-workers and accusing them of conspiracy. Today this patient shouts, "They're all plotting to destroy me. Isn't that true?" Select the nurse's most therapeutic response.

a. "Everyone here is trying to help you. No one wants to harm you." b. "Feeling that people want to destroy you must be very frightening." c. "That is not true. People here are trying to help you if you will let them." d. "Staff members are health care professionals who are qualified to help you." ANSWER:B

A patient diagnosed with schizophrenia anxiously tells the nurse, "The voice is telling me to do things." Select the nurse's priority assessment question

a. "How long has the voice been directing your behavior?" b. "Does what the voice tell you to do frighten you?" c. "Do you recognize the voice speaking to you?' d. "What is the voice telling you to do?" ANSWER: D

Which hallucination necessitates the nurse to implement safety measures? The patient says,

a. "I hear angels playing harps." b. "The voices say everyone is trying to kill me." c. "My dead father tells me I am a good person." d. "The voices talk only at night when I'm trying to sleep." ANSWER:B

A nurse educates a patient about the antipsychotic medication regime. Afterward, which comment by the patient indicates the teaching was effective?

a. "I will need higher and higher doses of my medication as time goes on." b. "I need to store my medication in a cool dark place, such as the refrigerator." c. "Taking this medication regularly will reduce the severity of my symptoms." d. "If I run out or stop taking my medication, I will experience withdrawal symptoms." ANSWER: C

A nurse's neighbor says, "My sister has been diagnosed with bipolar disorder but will not take her medication. I have tried to help her for over 20 years, but it seems like everything I do fails. Do you have any suggestions?" Select the nurse's best response. a. "NAMI offers a family education series that you might find helpful." b. "Since your sister is noncompliant, perhaps it's time for her to be changed to injectable medication." c. "You have done all you can. Now it's time to put yourself first and move on with your life." d. "You cannot help her. Would it be better for you to discontinue your relationship?"

a. "NAMI offers a family education series that you might find helpful." NAMI offers a family education series that assists with the stress caregivers and other family members often experience. The nurse should not give advice about injectable medication or encourage the family member to give up on the patient.

A patient diagnosed with schizophrenia tells the nurse, "I eat skiller. Tend to end. Easter. It blows away. Get it?" Select the nurse's most therapeutic response.

a. "Nothing you are saying is clear." b. "Your thoughts are very disconnected." c. "Try to organize your thoughts and then tell me again." d. "I am having difficulty understanding what you are saying." ANSWER: D

A nurse asks a patient diagnosed with schizophrenia, "What is meant by the old saying 'You can't judge a book by looking at the cover.'?" Which response by the patient indicates concrete thinking?

a. "The table of contents tells what a book is about." b. "You can't judge a book by looking at the cover." c. "Things are not always as they first appear." d. "Why are you asking me about books?" ANSWER: A

Which statements most clearly indicate the speaker views mental illness with stigma? (Select all that apply.) a. "We are all a little bit crazy." b. "If people with mental illness would go to church, their problems would be solved." c. "Many mental illnesses are genetically transmitted. It's no one's fault that the illness occurs." d. "Anyone can have a mental illness. War or natural disasters can be too stressful for healthy people." e. "People with mental illness are lazy. They get government disability checks instead of working."

a. "We are all a little bit crazy." b. "If people with mental illness would go to church, their problems would be solved." e. "People with mental illness are lazy. They get government disability checks instead of working." Stigma is represented by judgmental remarks that discount the reality and validity of mental illness. It is evidenced in stereotypical statements, by oversimplification, and by multiple other messages of guilt or shame. See related audience response question.

An acutely violent patient diagnosed with schizophrenia received several doses of haloperidol. Two hours later the nurse notices the patient's head rotated to one side in a stiffly fixed position, the lower jaw thrust forward, and drooling. Which intervention by the nurse is indicated?

a. Administer diphenhydramine 50 mg IM from the prn medication administration record. b. Reassure the patient that the symptoms will subside. Practice relaxation exercises with the patient. c. Give trihexyphenidyl 5 mg orally at the next regularly scheduled medication administration time. d. Administer atropine sulfate 2 mg subcut from the prn medication administration record. ANSWER: A

A patient diagnosed with schizophrenia is very disturbed and violent. After several doses of haloperidol, the patient is calm. Two hours later the nurse sees the patient's head rotated to one side in a stiff position, the lower jaw thrust forward, and drooling. Which problem is most likely?

a. An acute dystonic reaction b. Tardive dyskinesia c. Waxy flexibility d. Akathisia ANSWER: A

The nurse assesses a patient diagnosed with schizophrenia. Which assessment finding would the nurse regard as a negative symptom of schizophrenia?

a. Auditory hallucinations b. Delusions of grandeur c. Poor personal hygiene d. Psychomotor agitation ANSWER: C

A health care provider considers which antipsychotic medication to prescribe for a patient diagnosed with schizophrenia who has auditory hallucinations and poor social function. The patient is also overweight and hypertensive. Which drug should the nurse advocate?

a. Clozapine b. Ziprasidone c. Olanzapine d. Aripiprazole ANSWER: D

A patient diagnosed with SMI was living successfully in a group home but wanted an apartment. The prospective landlord said, "People like you have trouble getting along and paying their rent." The patient and nurse meet for a problem-solving session. Which options should the nurse endorse? (Select all that apply.) a. Coach the patient in ways to control symptoms effectively. b. Seek out landlords less affected by the stigma associated with mental illness. c. Threaten the landlord with legal action because of the discriminatory actions. d. Encourage the patient to remain in the group home until the illness is less obvious. e. Suggest that the patient list a false current address in the rental application. f. Have the case manager meet with the landlord to provide education about mental illness.

a. Coach the patient in ways to control symptoms effectively. b. Seek out landlords less affected by the stigma associated with mental illness. f. Have the case manager meet with the landlord to provide education about mental illness. Managing symptoms so that they are less obvious or socially disruptive can reduce negative reactions and reduce rejection due to stigma. Seeking a more receptive landlord might be the most expeditious route to housing for this patient. Educating the landlord to reduce stigma might make him more receptive and give the case manager an opportunity to address some of his concerns (e.g., the case manager could arrange a payee to assure that the rent is paid each month). However, threatening a lawsuit would increase the landlord's defensiveness and would likely be a long and expensive undertaking. Delaying the patient's efforts to become more independent is not clinically necessary according to the data noted here; the problem is the landlord's bias and response, not the patient's illness. It would be unethical to encourage falsification and poor role modeling to do so; further, if falsification is discovered, it could permit the landlord to refuse or cancel her lease. See related audience response question.

A nurse leads a psychoeducational group about first-generation antipsychotic medications with six adult men diagnosed with schizophrenia. The nurse will monitor for concerns regarding body image with respect to which potential side effect of these medications?

a. Constipation b. Gynecomastia c. Visual changes d. Photosensitivity ANSWER: B

A patient's care plan includes monitoring for auditory hallucinations. Which assessment findings suggest the patient may be hallucinating?

a. Detachment and overconfidence b. Darting eyes, tilted head, mumbling to self c. Euphoric mood, hyperactivity, distractibility d. Foot tapping and repeatedly writing the same phrase ANSWER:B

A patient diagnosed with schizophrenia says, "My co-workers are out to get me. I also saw two doctors plotting to kill me." How does this patient perceive the environment?

a. Disorganized b. Dangerous c. Supportive d. Bizarre ANSWER:B

A nurse observes a catatonic patient standing immobile, facing the wall with one arm extended in a salute. The patient remains immobile in this position for 15 minutes, moving only when the nurse gently lowers the arm. What is the name of this phenomenon?

a. Echolalia b. Waxy flexibility c. Depersonalization d. Thought withdrawal ANSWER: B

A consumer at a rehabilitative psychosocial program says to the nurse, "People are not cleaning up behind themselves in the bathrooms. The building is dirty and cluttered." How should the nurse respond? a. Encourage the consumer to discuss it at a meeting with everyone. b. Hire a professional cleaning service to clean the restrooms. c. Address the complaint at the next staff meeting. d. Tell the consumer, "That's not my problem."

a. Encourage the consumer to discuss it at a meeting with everyone. Consumer-run programs range from informal "clubhouses," which offer socialization and recreation, to competitive businesses, such as snack bars or janitorial services, which provide needed services and consumer employment while encouraging independence and building vocational skills. Consumers engage in problem solving under the leadership of staff. See related audience response question.

A patient diagnosed with schizophrenia has taken a conventional antipsychotic medication for a year. Hallucinations are less intrusive, but the patient continues to have apathy, poverty of thought, and social isolation. The nurse would expect a change to which medication?

a. Haloperidol b. Olanzapine c. Chlorpromazine d. Diphenhydramine ANSWER: B

Which finding constitutes a negative symptom associated with schizophrenia?

a. Hostility b. Bizarre behavior c. Poverty of thought d. Auditory hallucinations ANSWER: C

A patient diagnosed with a SMI lives independently and attends a psychosocial rehabilitation program. The patient presents at the emergency department seeking hospitalization. The patient has no acute symptoms but says, "I have no money to pay my rent or refill my prescription." Select the nurse's best action. a. Involve the patient's case manager to provide crisis intervention. b. Send the patient to a homeless shelter until housing can be arranged. c. Arrange for a short in-patient admission and begin discharge planning. d. Explain that one must have active psychiatric symptoms to be admitted.

a. Involve the patient's case manager to provide crisis intervention. Impaired stress tolerance and problem-solving abilities can cause persons with SMI to experience relatively minor stressors as crises. This patient has run out of money, and this has overwhelmed her ability to cope, resulting in a crisis for which crisis intervention would be an appropriate response. Inpatient care is not clinically indicated nor is the patient homeless (although she may fear she is). Telling the patient that she is not symptomatic enough to be admitted may prompt malingering.

The nurse manager of a mental health center wants to improve medication adherence among the seriously mentally ill persons treated there. Which interventions are likely to help achieve this goal? (Select all that apply.) a. Maintain stable and consistent staff. b. Increase the length of medication education groups. c. Stress that without treatment, illnesses will worsen. d. Prescribe drugs in smaller but more frequent dosages. e. Make it easier to access prescribers and pay for drugs. f. Require adherence in order to participate in programming.

a. Maintain stable and consistent staff. e. Make it easier to access prescribers and pay for drugs. Trust in one's providers is a key factor in treatment adherence, and mentally ill persons can sometimes take a very long time to develop such trust; therefore, interventions which stabilize staffing allow patients to have more time with staff to develop these bonds. Ready access to prescribers allows medicine-related concerns to be addressed quickly, reducing obstacles to adherence such as side effects or ineffective dosages. Medication costs can be obstacles to adherence as well. Many SMI patients have anosognosia and do not adhere to treatment because they believe they are not ill, so telling them nonadherence will worsen an illness they do not believe they have is unlikely to be helpful. Increasing medication education is helpful only when the cause of nonadherence is a knowledge deficit. Other issues that reduce adherence, particularly anosognosia and side effects, are seldom helped by longer medication education. Requiring medication adherence to participate in other programs is coercive and unethical. Smaller, more frequent doses do not reduce side effects and make the regimen more difficult for the patient to remember.

A patient diagnosed with schizophrenia says, "It's beat. Time to eat. No room for the cat." What type of verbalization is evident?

a. Neologism b. Idea of reference c. Thought broadcasting d. Associative looseness ANSWER: D

A patient diagnosed with schizophrenia has taken fluphenazine 5 mg po bid for 3 weeks. The nurse now observes a shuffling propulsive gait, a mask-like face, and drooling. Which term applies to these symptoms?

a. Neuroleptic malignant syndrome b. Hepatocellular effects c. Pseudoparkinsonism d. Akathisia ANSWER: C

A patient diagnosed with schizophrenia says, "Contagious bacteria are everywhere. When they get in your body, you will be locked up with other infected people." Which problem is evident?

a. Poverty of content b. Concrete thinking c. Neologisms d. Paranoia ANSWER: D

The family of a patient diagnosed with schizophrenia is unfamiliar with the illness and family's role in recovery. Which type of therapy should the nurse recommend?

a. Psychoeducational b. Psychoanalytic c. Transactional d. Family ANSWER: A

An adult patient tells the case manager, "I don't have bipolar disorder anymore, so I don't need medicine. After I was in the hospital last year, you helped me get an apartment and disability checks. Now I'm bored and don't have any friends." Where should the nurse refer the patient? (Select all that apply.) a. Psychoeducational classes b. Vocational rehabilitation c. Social skills training d. A homeless shelter e. Crisis intervention

a. Psychoeducational classes b. Vocational rehabilitation c. Social skills training The patient does not understand the illness and need for adherence to the medication regimen. Psychoeducation for the patient (and family) can address this lack of knowledge. The patient, who considers himself friendless, could also profit from social skills training to improve the quality of interpersonal relationships. Many patients with SMI have such poor communication skills that others are uncomfortable interacting with them. Interactional skills can be effectively taught by breaking the skill down into smaller verbal and nonverbal components. Work gives meaning and purpose to life, so vocational rehabilitation can assist with this aspect of care. The nurse case manager will function in the role of crisis stabilizer, so no related referral is needed. The patient presently has a home and does not require a homeless shelter.

A patient diagnosed with schizophrenia was hospitalized after arguing with co-workers and threatening to harm them. The patient is aloof, suspicious, and says, "Two staff members I saw talking were plotting to kill me." Based on data gathered at this point, which nursing diagnoses relate? (Select all that apply.)

a. Risk for other-directed violence b. Disturbed thought processes c. Risk for loneliness d. Spiritual distress e. Social isolation ANSWERS: A and B

A nurse leads a psychoeducational group about problem solving with six adults diagnosed with schizophrenia. Which teaching strategy is likely to be most effective?

a. Suggest analogies that might apply to a common daily problem. b. Assign each participant a problem to solve independently and present to the group. c. Ask each patient to read aloud a short segment from a book about problem solving. d. Invite participants to come up with solution to getting incorrect change for a purchase. ANSWER: D

A client says, "Facebook has a new tracking capacity. If I use the Internet, Homeland Security will detain me as a terrorist." Select the nurse's best initial action.

a. Tell the client, "Facebook is a safe website. You don't need to worry about Homeland Security." b. Tell the client, "You are in a safe place where you will be helped." c. Administer a prn dose of an antipsychotic medication. d. Tell the client, "You don't need to worry about that." ANSWER: B

What assessment findings mark the prodromal stage of schizophrenia?

a. Withdrawal, misinterpreting, poor concentration, and preoccupation with religion b. Auditory hallucinations, ideas of reference, thought insertion, and broadcasting c. Stereotyped behavior, echopraxia, echolalia, and waxy flexibility d. Loose associations, concrete thinking, and echolalia neologisms ANSWER:A

A newly hospitalized patient experiencing psychosis says, "Red chair out town board." Which term should the nurse use to document this finding?

a. Word salad b. Neologism c. Anhedonia d. Echolalia ANSWER: A

A patient diagnosed with schizophrenia begins to talks about "macnabs" hiding in the warehouse at work. The term "macnabs" should be documented as

a. a neologism. b. concrete thinking. c. thought insertion. d. an idea of reference. ANSWER: A

A patient diagnosed with schizophrenia demonstrates little spontaneous movement and has waxy flexibility. The patient's activities of daily living are severely compromised. An appropriate outcome would be that the patient will

a. demonstrate increased interest in the environment by the end of week 1. b. perform self-care activities with coaching by the end of day 3. c. gradually take the initiative for self-care by the end of week 2. d. accept tube feeding without objection by day 2. ANSWER: B

A newly admitted patient diagnosed with schizophrenia is hypervigilant and constantly scans the environment. The patient states, "I saw two doctors talking in the hall. They were plotting to kill me." The nurse may correctly assess this behavior as

a. echolalia. b. an idea of reference. c. a delusion of infidelity. d. an auditory hallucination. ANSWER:B

The nurse is developing a plan for psychoeducational sessions for a small group of adults diagnosed with schizophrenia. Which goal is best for this group? Members will

a. gain insight into unconscious factors that contribute to their illness. b. explore situations that trigger hostility and anger. c. learn to manage delusional thinking. d. demonstrate improved social skills. ANSWER: D

A patient diagnosed with schizophrenia anxiously says, "I can see the left side of my body merging with the wall, then my face appears and disappears in the mirror." While listening, the nurse should

a. sit close to the patient. b. place an arm protectively around the patient's shoulders. c. place a hand on the patient's arm and exert light pressure. d. maintain a normal social interaction distance from the patient. ANSWER: D

A patient diagnosed with schizophrenia has been stable for a year; however, the family now reports the patient is tense, sleeps 3 to 4 hours per night, and has difficulty concentrating. The patient says, "My computer is sending out infected radiation beams." The nurse can correctly assess this information as an indication of

a. the need for psychoeducation. b. medication nonadherence. c. chronic deterioration. d. relapse. ANSWER: D

Which characteristic of personality disorders makes it most necessary for staff to schedule frequent team meetings in order to address the patient's needs and maintain a therapeutic milieu? a.Ability to achieve true intimacy b.Flexibility and adaptability to stress c.Ability to provoke interpersonal conflict d.Inability to develop trusting relationships

ability to provoke interpersonal conflict

You are conducting an admission interview with Callie, who was raped 2 weeks ago. When you ask Callie about the rape, she becomes very anxious and upset and begins to sob. Your best course of actions would be to: acknowledge that the topic of the rape is upsetting to Callie and reassure her that it can be discussed at another time when she feels more comfortable.

acknowledge that the topic of the rape is upsetting to Callie and reassure her that it can be discussed at another time when she feels more comfortable. Correct The best atmosphere for conducting an assessment is one with minimal anxiety on the patient's part. If a topic causes distress, it is best to abandon the topic at that time. It is important not to pry or push for information that is difficult for the patient to discuss. The use of silence continues to expect the patient to discuss the topic now. Reassurance of confidentiality continues to expect the patient to discuss the topic now.

he history shows that a newly admitted patient is impulsive. The nurse would expect behavior characterized by a.adherence to a strict moral code. b.manipulative, controlling strategies. c.acting without thought on urges or desires. d.postponing gratification to an appropriate time.

acting without thought on urges or desires

A patient with acute mania has disrobed in the hall three times in 2 hours. The nurse should:

arrange for one-on-one supervision.

Using Maslow's model of needs, the nurse providing care for an anxious client identifies the priority intervention to be a. assessing the client's success at fulfilling her appropriate developmental level tasks. b. assessing the client for her strengths upon which a nurse-client relationship can be based. c. planning one-on-one time with the client to assist in identifying the fears behind her anxiety. d. evaluating the client's ability to learn and retain essential information regarding her condition.

assessing the client for her strengths upon which a nurse-client relationship can be based

A nurse is about to interview a client whose glasses and hearing aid were placed in safe-keeping when she was admitted. Before beginning the interview, the nursing intervention that will best facilitate data collection is to assist the client in putting on glasses and hearing aid.

assist the client in putting on glasses and hearing aid. Correct A client whose hearing or sight is impaired may have difficulty providing information if these items have been removed from his or her possession. Assisting the client in wearing these assistive devices is the best initial intervention. REF: Page 118-119

One month ago, a patient diagnosed with borderline personality disorder and a history of self-mutilation began dialectical behavior therapy. Today the patient phones to say, "I feel empty and want to hurt myself." The nurse should a.arrange for emergency inpatient hospitalization. b.send the patient to the crisis intervention unit for 8 to 12 hours. c.assist the patient to choose coping strategies for triggering situations. d.advise the patient to take an antianxiety medication to decrease the anxiety level.

assist the patient to choose coping strategies for triggering situations

The sibling of a patient who was diagnosed with a SMI asks why a case manager has been assigned. The nurse's reply should cite the major advantage of the use of case management as: a. "The case manager can modify traditional psychotherapy for homeless patients so that it is more flexible." b. "Case managers coordinate services and help with accessing them, making sure the patient's needs are met." c. "The case manager can focus on social skills training and esteem building in the real world where the patient lives." d. "Having a case manager has been shown to reduce hospitalizations, which prevents disruption and saves money."

b. "Case managers coordinate services and help with accessing them, making sure the patient's needs are met." The case manager helps the patient gain entrance into the system of care, can coordinate multiple referrals that so often confuse the seriously mentally ill person and his family, and can help overcome obstacles to access and treatment participation. Case managers do not usually possess the credentials needed to provide psychotherapy or function as therapists. Case management promotes efficient use of services in general, but only ACT programming has been shown to reduce hospitalization (which the sibling might see as a disadvantage). Case managers operate in the community, but this is not the primary advantage of their services.

A homeless patient diagnosed with a SMI became suspicious and delusional. Depot antipsychotic medication began and housing was obtained in a local shelter. One month later, which statement by the patient indicates significant improvement? a. "They will not let me drink. They have many rules in the shelter." b. "I feel comfortable here. Nobody bothers me." c. "Those shots make my arm very sore." d. "Those people watch me a lot."

b. "I feel comfortable here. Nobody bothers me." Evaluation of a patient's progress is made based on patient satisfaction with the new health status and the health care team's estimation of improvement. For a formerly delusional patient to admit to feeling comfortable and free of being "bothered" by others denotes improvement in the patient's condition. The other options suggest that the patient is in danger of relapse.

A patient diagnosed with a SMI died suddenly at age 52. The patient lived in the community for 5 years without relapse and held supported employment the past 6 months. The distressed family asks, "How could this happen?" Which response by the nurse accurately reflects research and addresses the family's question? a. "A certain number of people die young from undetected diseases, and it's just one of those sad things that sometimes happen." b. "Mentally ill people tend to die much younger than others, perhaps because they do not take as good care of their health, smoke more, or are overweight." c. "We will have to wait for the autopsy to know what happened. There were some medical problems, but we were not expecting death." d. "We are all surprised. The patient had been doing so well and saw the nurse every other week."

b. "Mentally ill people tend to die much younger than others, perhaps because they do not take as good care of their health, smoke more, or are overweight." The family is in distress. Because they do not understand his death, they are less able to accept it and seek specific information to help them understand what happened. Persons with SMI die an average of 25 years prematurely. Contributing factors include failing to provide for their own health needs (e.g., forgetting to take medicine), inability to access or pay for care, higher rates of smoking, poor diet, criminal victimization, and stigma. The most accurate answer indicates that seriously mentally ill people are at much higher risk of premature death for a variety of reasons. Staff would not have been surprised that the patient died prematurely, and they would not attribute his death to random, undetected medical problems. Although the cause of death will not be reliably established until the autopsy, this response fails to address the family's need for information.

Many persons brought before a criminal court have mental illness, have committed minor offenses, and are off medications. The judge consults the nurse at the local community mental health center for guidance about how to respond when handling such cases. Which advice from the nurse would be most appropriate? a. "Sometimes a little time in jail makes a person rethink what they've been doing and puts them back on the right track." b. "Sentencing such persons to participate in treatment instead of incarcerating them has been shown to reduce repeat offenses." c. "Arresting these people helps them in the long run. Sometimes we cannot hospitalize them, but in jail they will get their medication." d. "Research suggests that special mental health courts do not make much difference so far, but outpatient commitment does seem to help."

b. "Sentencing such persons to participate in treatment instead of incarcerating them has been shown to reduce repeat offenses." Research supports the use of special mental health courts that can sentence mentally ill persons to treatment instead of jail. Jail exposes vulnerable mentally ill persons to criminals, victimization, and high levels of stimulation and stress. Incarceration can also interrupt eligibility for benefits or lead to the loss of housing and often provides lower-quality mental health treatment in other settings. Recidivism rates for both mentally ill and non-mentally ill offenders are relatively high, so it does not appear that incarceration necessarily leads people to behave more appropriately. In addition, a criminal record can leave them more desperate and with fewer options after release. Research indicates that outpatient commitment is less effective at improving the mental health of mentally ill persons than was expected.

A nurse prepares to lead a discussion at a community health center regarding children's health problems. The nurse wants to use current terminology when discussing these issues. Which terms are appropriate for the nurse to use? Select all that apply. a. Autism b. Bullying c. Mental retardation d. Autism spectrum disorder e. Intellectual development disorder

b. Bullying d. Autism spectrum disorder e. Intellectual development disorder

A person diagnosed with a SMI enters a shelter for the homeless. Which intervention should be the nurse's initial priority? a. Find supported employment. b. Develop a trusting relationship. c. Administer prescribed medication. d. Teach appropriate health care practices.

b. Develop a trusting relationship. Basic psychosocial needs do not change because a person is homeless. The first step in caring for health care needs is establishing rapport. Once a trusting relationship is established, the nurse pursues other interventions.

A person diagnosed with SMI has frequent relapses, usually precipitated by situational stressors such as running out of money or the absence of key staff at the mental health center. Which interventions would the nurse suggest to reduce the risk of stressors to cause relapse? (Select all that apply.) a. Discourage potentially stressful activities such as groups or volunteer work. b. Develop written plans that will help the patient remember what to do in a crisis. c. Help the patient identify and anticipate events that are likely to be overwhelming. d. Encourage health-promoting activities such as exercise and getting adequate rest. e. Accompany the patient to a NAMI support group.

b. Develop written plans that will help the patient remember what to do in a crisis. c. Help the patient identify and anticipate events that are likely to be overwhelming. d. Encourage health-promoting activities such as exercise and getting adequate rest. e. Accompany the patient to a NAMI support group. Basic interventions for coping with crises involve anticipating crises where possible and then developing a plan with specific actions to take when faced with an overwhelming stressor. Written plans are helpful; it can be difficult for anyone, especially a person with cognitive or memory impairments, to develop or remember steps to take when under overwhelming stress. Health-promoting activities enhance a person's ability to cope with stress. As the name suggests, support groups help a person develop a support system, and they provide practical guidance from peers who learned from experience how to deal with issues the patient may be facing. Groups and volunteer work may involve a measure of stress but also provide benefits that help persons cope and should not be discouraged unless they are being done to excess.

At the time of a home visit, the nurse notices that each parent and child in a family has his or her own personal online communication device. Each member of the family is in a different area of the home. Which nursing actions are appropriate? Select all that apply. a. Report the finding to the official child protection social services agency. b. Educate all members of the family about risks associated with cyberbullying. c. Talk with the parents about parental controls on the children's communication devices. d. Encourage the family to schedule daily time together without communication devices. e. Obtain the family's network password and examine online sites family members have visited.

b. Educate all members of the family about risks associated with cyberbullying. c. Talk with the parents about parental controls on the children's communication devices. d. Encourage the family to schedule daily time together without communication devices.

A patient living independently had command hallucinations to shout warnings to neighbors. After a short hospitalization, the patient was prohibited from returning to the apartment. The landlord said, "You cause too much trouble." What problem is the patient experiencing? a. Grief b. Stigma c. Homelessness d. Nonadherence

b. Stigma The inability to obtain shelter because of negative attitudes about mental illness is an example of stigma. Stigma is defined as damage to reputation, shame, and ridicule society places on mental illness. Data are not present to identify grief as a patient problem. Data do not suggest that the patient is actually homeless. See relationship to audience response question.

For patients diagnosed with SMI, what is the major advantage of case management? a. The case manager can modify traditional psychotherapy. b. With one coordinator of services, resources can be more efficiently used. c. The case manager can focus on social skills training and esteem building. d. Case managers bring groups of patients together to discuss common problems.

b. With one coordinator of services, resources can be more efficiently used. The case manager coordinates the care and multiple referrals that so often confuse the seriously mentally ill patient and the patient's family. Case management promotes efficient use of services. The other options are lesser advantages or are irrelevant

SMI is characterized as a. any mental illness of more than 2 weeks' duration. b. a major long-term mental illness marked by significant functional impairments. c. a mental illness accompanied by physical impairment and severe social problems. d. a major mental illness that cannot be treated to prevent deterioration of cognitive and social abilities.

b. a major long-term mental illness marked by significant functional impairments. "Serious mental illness" has replaced the term "chronic mental illness." Global impairments in function are evident, particularly social. Physical impairments may be present. SMI can be treated, but remissions and exacerbations are part of the course of the illness.

After 5 years in a state hospital, an adult diagnosed with schizophrenia was discharged to the community. This patient now requires continual direction to accomplish activities of daily living and expects others to provide meals and do laundry. The nurse assesses this behavior as the probable result of a. side effects of antipsychotic medications. b. dependency caused by institutionalization. c. cognitive deterioration from schizophrenia. d. stress associated with acclimation to the community.

b. dependency caused by institutionalization Institutions tend to impede independent functioning; for example, daily activities are planned and directed by staff; others provide meals and only at set times. Over time, patients become dependent on the institution to meet their needs and adapt to being cared for rather than caring for themselves. When these patients return to the community, many continue to demonstrate passive behaviors despite efforts to promote. Cognitive dysfunction and antipsychotic side effects can make planning and carrying out activities more difficult, but the question is more suggestive of adjustment to institutional care and difficulty readjusting to independence instead.

A patient demonstrating characteristics of acute mania relapsed after discontinuing lithium. New orders are written to resume lithium twice daily and begin olanzapine (Zyprexa). What is the rationale for the addition of olanzapine to the medication regimen? It will:

bring hyperactivity under rapid control.

Which service would be expected to provide resources 24 hours a day, 7 days a week if needed for persons with SMI? a. Clubhouse model b. Cognitive-behavioral therapy (CBT) c. Assertive community treatment (ACT) d. Cognitive enhancement therapy (CET)

c. Assertive community treatment (ACT) ACT involves consumers working with a multidisciplinary team that provides a comprehensive array of services. At least one member of the team is available 24 hours a day for crisis needs, and the emphasis is on treating the patient within his own environment.

Which nursing diagnosis is likely to apply to an individual diagnosed with a SMI who is homeless? a. Insomnia b. Substance abuse c. Chronic low self-esteem d. Impaired environmental interpretation syndrome

c. Chronic low self-esteem Many individuals with SMI do not live with their families and become homeless. Life on the street or in a shelter has a negative influence on the individual's self-esteem, making this nursing diagnosis one that should be considered. Substance abuse is not an approved North American Nursing Diagnosis Association (NANDA)-International diagnosis. Insomnia may be noted in some patients but is not a universal problem. Impaired environmental interpretation syndrome refers to persistent disorientation, which is not seen in a majority of the homeless.

The nurse wants to enroll a patient with poor social skills in a training program for patients diagnosed with schizophrenia. Which description accurately describes social skills training? a. Patients learn to improve their attention and concentration. b. Group leaders provide support without challenging patients to change. c. Complex interpersonal skills are taught by breaking them into simpler behaviors. d. Patients learn social skills by practicing them in a supported employment setting.

c. Complex interpersonal skills are taught by breaking them into simpler behaviors. In social skills training, complex interpersonal skills are taught by breaking them down into component behaviors that are covered in a stepwise fashion. Social skills training is not based in employment settings, although such skills can be addressed as part of supported employment services. The other distracters are less relevant to social skills training.

A person diagnosed with a SMI living in the community was punched, pushed to the ground, and robbed of $7 during the day on a public street. Which statements about violence and SMI in general are accurate? (Select all that apply.) a. Persons with SMI are more likely to be violent. b. SMI persons are more likely to commit crimes than to be the victims of crime. c. Impaired judgment and social skills can provoke hostile or assaultive behavior. d. Lower incomes force SMI persons to live in high-crime areas, increasing risk. e. SMI persons experience higher rates of sexual assault and victimization than others. f. Criminals may believe SMI persons are less likely to resist or testify against them.

c. Impaired judgment and social skills can provoke hostile or assaultive behavior. d. Lower incomes force SMI persons to live in high-crime areas, increasing risk. e. SMI persons experience higher rates of sexual assault and victimization than others. f. Criminals may believe SMI persons are less likely to resist or testify against them. Mentally ill persons are more likely to be victims of crime than perpetrators of criminal acts. They are often victims of criminal behavior, including sexual crimes, at a higher rate than others. When a mentally ill person commits a crime, it is usually nonviolent. Mental illnesses interfere with employment and are associated with poverty, limiting SMI persons to living in inexpensive areas that also tend to be higher-crime areas. SMI persons may inadvertently provoke others because of poor judgment or socially inappropriate behavior, or they may be victimized because they are perceived as passive, less likely to resist, and less likely to be believed as witnesses. See related audience response question.

An adult diagnosed with a serious mental illness (SMI) says, "I do not need help with money management. I have excellent ideas about investments." This patient usually does not have money to buy groceries by the middle of the month. The nurse assesses the patient as demonstrating a. rationalization. b. identification. c. anosognosia. d. projection.

c. anosognosia. The patient scenario describes anosognosia, the inability to recognize one's deficits due to one's illness. The patient is not projecting an undesirable thought or emotion from himself onto others. He is not justifying his behavior via rationalization and is not identifying with another.

A nurse prepares the plan of care for a 15-year-old diagnosed with moderate intellectual developmental disorder. What are the highest outcomes that are realistic for this patient? Within 5 years, the patient will: (select all that apply) a. graduate from high school. b. live independently in an apartment. c. independently perform own personal hygiene. d. obtain employment in a local sheltered workshop. e. correctly use public buses to travel in the community.

c. independently perform own personal hygiene. d. obtain employment in a local sheltered workshop. e. correctly use public buses to travel in the community.

A patient diagnosed with bipolar disorder has rapidly changing mood cycles. The health care provider prescribes an anticonvulsant medication. To prepare teaching materials, which drug should the nurse anticipate will be prescribed?

carbamazepine (Tegretol)

The primary source for data collection during a psychiatric nursing assessment is the client's own words and actions.

client's own words and actions. Correct The client should always be considered the primary data source. At times, however, the client will be unable to fulfill this role

A nurse assesses a 3-year-old diagnosed with an autism spectrum disorder. Which finding is most associated with the child's disorder? The child:* a. has occasional toileting accidents. b. is unable to read children's books. c. cries when separated from a parent. d. continuously rocks in place for 30 minutes.

continuously rocks in place for 30 minutes.

An outpatient diagnosed with schizophrenia attends programming at a community mental health center. The patient says, "I threw away the pills because they keep me from hearing God." Which response by the nurse would most likely to benefit this patient? a. "You need your medicine. Your schizophrenia will get worse without it." b. "Do you want to be hospitalized again? You must take your medication." c. "I would like you to come to the medication education group every Thursday." d. "I noticed that when you take the medicine, you are able to keep the job you wanted."

d. "I noticed that when you take the medicine, you are able to keep the job you wanted." The patient appears not to understand that he has an illness. He has stopped his medication because it interferes with a symptom that he finds desirable (auditory hallucinations—the voice of God). Connecting medication adherence to one of the patient's goals (the job) can serve to motivate the patient to take the medication and override concerns about losing the hallucinations. Exhorting a patient to take medication because it is needed to control his illness is unlikely to be successful; he does not believe he has an illness. Medication psychoeducation would be appropriate if the cause of nonadherence was a knowledge deficit.

The parent of a seriously mentally ill adult asks the nurse, "Why are you making a referral to a vocational rehabilitation program? My child won't ever be able to hold a job." Which is the nurse's best reply? a. "We make this referral to continue eligibility for federal funding." b. "Are you concerned that we're trying to make your child too independent?" c. "If you think the program would be detrimental, we can postpone it for a time." d. "Most patients are capable of employment at some level, competitive or supported."

d. "Most patients are capable of employment at some level, competitive or supported." Studies have shown that most patients who complete vocational rehabilitation programs are capable of some level of employment. They also demonstrate significant improvement in assertiveness and work behaviors as well as decreased depression.

A family discusses the impact of a seriously mental ill member. Insurance partially covers treatment expenses, but the family spends much of their savings for care. The patient's sibling says, "My parents have no time for me." The parents are concerned that when they are older, there will be no one to care for the patient. Which response by the nurse would be most helpful? a. Acknowledge their concerns and consult with the treatment team about ways to bring the patient's symptoms under better control. b. Give them names of financial advisors that could help them save or borrow sufficient funds to leave a trust fund to care for their loved one. c. Refer them to crisis intervention services to learn ways to manage caregiver stress and provide titles of some helpful books for families. d. Discuss benefits of participating in National Alliance on Mental Illness (NAMI) programs and ways to help the patient become more independent.

d. Discuss benefits of participating in National Alliance on Mental Illness (NAMI) programs and ways to help the patient become more independent. The family has raised a number of concerns, but the major issues appear to be the effects caregiving has had on the family and their concerns about the patient's future. NAMI offers support, education, resources, and access to other families who have experience with the issues now facing this family. NAMI can help address caregiver burden and planning for the future needs of SMI persons. Improving the patient's symptom control and general functioning can help reduce caregiver burden but would likely be a slow process, whereas NAMI involvement could benefit them on a number of fronts, possibly in a shorter time period. The family will need more than financial planning; their issues go beyond financial. The family is distressed but not in crisis. Crisis intervention is not an appropriate resource for the longer-term issues and needs affecting this family.

A homeless individual diagnosed with SMI and a history of persistent treatment nonadherence plans to begin attending the day program at a community mental health center. Which intervention should be the team's initial focus? a. Teach appropriate health maintenance and prevention practices. b. Educate the patient about the importance of treatment adherence. c. Help the patient obtain employment in a local sheltered workshop. d. Interact regularly and supportively without trying to change the patient.

d. Interact regularly and supportively without trying to change the patient. Given the history of treatment nonadherence and the difficulty achieving other goals until psychiatrically stable and adherent, getting the patient to accept and adhere to treatment is the fundamental goal to address. The intervention most likely to help meet that goal at this stage is developing a trusting relationship with the patient. Interacting regularly, supportively, and without demands is likely to build the necessary trust and relationships that will be the foundation for all other interventions later on. No data here suggest the patient is in crisis, so it is possible to proceed slowly and build this foundation of trust.

A hospitalized patient diagnosed with schizophrenia has a history of multiple relapses. The patient usually responds quickly to antipsychotic medication but soon discontinues the medication. Discharge plans include follow-up at the mental health center, group home placement, and a psychosocial day program. Which strategy should apply first as the patient transitions from hospital to community? a. Administer a second-generation antipsychotic to help negative symptoms. b. Use a quick-dissolving medication formulation to reduce "cheeking." c. Prescribe a long-acting intramuscular antipsychotic medication. d. Involve the patient in decisions about which medication is best.

d. Involve the patient in decisions about which medication is best. Persons with schizophrenia are at high risk for treatment nonadherence, so the strategy needs primarily to address that risk. Of the options here, involving the patient in the decision is best because it will build trust and help establish a therapeutic alliance with care providers, an essential foundation to adherence. Intramuscular depot medications can be helpful for promoting adherence if other alternatives have been unsuccessful, but IM medications are painful and may jeopardize the patient's acceptance. All of the other strategies also apply but are secondary to trust and bonding with providers.

A patient diagnosed with schizophrenia tells the community mental health nurse, "I threw away my pills because they interfere with God's voice." The nurse identifies the etiology of the patient's ineffective management of the medication regime as a. inadequate discharge planning. b. poor therapeutic alliance with clinicians. c. dislike of antipsychotic medication side effects. d. impaired reasoning secondary to the schizophrenia.

d. impaired reasoning secondary to the schizophrenia. The patient's ineffective management of the medication regime is most closely related to impaired reasoning associated with the thought disturbances of schizophrenia. The patient believes in being an exalted personage who hears God's voice, rather than an individual with a serious mental disorder who needs medication to control symptoms. Data do not suggest any of the other factors often related to medication nonadherence.

An outpatient diagnosed with schizophrenia tells the nurse, "I am here to save the world. I threw away the pills because they make God go away." The nurse identifies the patient's reason for medication nonadherence as a. poor alliance with clinicians. b. inadequate discharge planning. c. dislike of medication side effects. d. thought disturbances associated with the illness.

d. thought disturbances associated with the illness. The patient's nonadherence is most closely related to thought disturbances associated with the illness. The patient believes he is an exalted personage who hears God's voice rather than an individual with a serious mental disorder who needs medication to control his symptoms. While the distracters may play a part in the patient's nonadherence, the correct response is most likely.

Freud believed that individuals cope with anxiety by using ...

defense mechanisms

A nurse determines desired outcomes for a patient diagnosed with schizotypal personality disorder. Select the best outcome. The patient will a.adhere willingly to unit norms. b.report decreased incidence of self-mutilative thoughts. c.demonstrate fewer attempts at splitting or manipulating staff. d.demonstrate ability to introduce self to a stranger in a social situation.

demonstrate ability to introduce self to a stranger ins a social situation

A parent diagnosed with schizophrenia and 13-year-old child live in a homeless shelter. The child formed a trusting relationship with a shelter volunteer. The child says, "My three friends and I got an A on our school science project." The nurse can assess that the child: a. displays resiliency. b. has a passive temperament. c. is at risk for posttraumatic stress disorder. d. uses intellectualization to deal with problems.

displays resiliency.

This nursing diagnosis applies to a patient with acute mania: Imbalanced nutrition: less than body requirements related to insufficient caloric intake and hyperactivity as evidenced by 5-pound weight loss in 4 days.Select an appropriate outcome. The patient will:

drink six servings of a high-calorie, high-protein drink each day.

*When a 5-year-old diagnosed with attention deficit hyperactivity disorder (ADHD) bounces out of a chair and runs over and slaps another child, what is the nurse's best action?* a. Instruct the parents to take the aggressive child home. b. Direct the aggressive child to stop immediately. c. Call for emergency assistance from other staff. d. Take the aggressive child to another room.

engages in cooperative play with other children.

A child diagnosed with attention deficit hyperactivity disorder had this nursing diagnosis: impaired social interaction related to excessive neuronal activity as evidenced by aggression and demanding behavior with others. Which finding indicates the plan of care was effective? The child: a. has an improved ability to identify anxiety and use self-control strategies. b. has increased expressiveness in communication with others. c. shows increased responsiveness to authority figures. d. engages in cooperative play with other children.

engages in cooperative play with other children..

The nurse is working with a client experiencing both post-partum depression and very low self-esteem. The client is distrustful of unit staff and "just wants to go home." Initially, the nurse's priority is to...

establish trust with the client

When a patient diagnosed with a personality disorder uses manipulation to get needs met, the staff applies limit-setting interventions. What is the correct rationale for this action? a.It provides an outlet for feelings of anger and frustration. b.It respects the patient's wishes, so assertiveness will develop. c.External controls are necessary due to failure of internal control. d.Anxiety is reduced when staff assumes responsibility for the patient's behavior.

external controls are necessary due to failure of internal control

A patient diagnosed with borderline personality disorder self-inflicted wrist lacerations after gaining new privileges on the unit. In this case, the self-mutilation may have been due to a.an inherited disorder that manifests itself as an incapacity to tolerate stress. b.use of projective identification and splitting to bring anxiety to manageable levels. c.a constitutional inability to regulate affect, predisposing to psychic disorganization. d.fear of abandonment associated with progress toward autonomy and independence.

fear of abandonment with progress towards autonomy and independence

When preparing to interview a patient diagnosed with narcissistic personality disorder, a nurse can anticipate the assessment findings will include a.preoccupation with minute details; perfectionist. b.charm, drama, seductiveness; seeking admiration. c.difficulty being alone; indecisive, submissiveness. d.grandiosity, self-importance, and a sense of entitlement.

grandiosity, self importance, and a sense of entitlement

The prevalence rate over a 12-month period for major depressive disorder is...

greater than the prevalence rate for generalized anxiety

The nurse planning care for a mentally ill client bases interventions on the concept that the client ....

has areas of strength on which to build

An outpatient diagnosed with bipolar disorder is prescribed lithium. The patient telephones the nurse to say, "I've had severe diarrhea for 4 days. I feel very weak and unsteady when I walk. My usual hand tremor has gotten worse. What should I do?" The nurse will advise the patient to:

have someone bring the patient to the clinic immediately.

Which behavior indicates that the treatment plan for a child diagnosed with an autism spectrum disorder was effective? The child: a. plays with one toy for 30 minutes. b. repeats words spoken by a parent. c. holds the parent's hand while walking. d. spins around and claps hands while walking.

holds the parent's hand while walking.

Maslow's theory of humanistic psychology has provided nursing with a framework for...

holistic assessment

A nursing diagnosis appropriate to consider for a patient diagnosed with any of the personality disorders is a.nonadherence. b.impaired social interaction. c.disturbed personal identity. d.diversional activity deficit.

impaired social interaction

During the initial assessment interview with a psychiatric client, the nurse should regard the spiritual assessment as important to complete.

important to complete. Correct For many clients, religious or spiritual practices are an important part of the quality of their lives. Nurses should support the spiritual dimension of the person. To do so, assessment is necessary.

High levels of anxiety and maladaptive behavior are seen in all areas in the health care setting.

in all areas in the health care setting. Correct Anxiety occurs whenever individuals are faced with unfamiliar circumstances or other threats to the self. The health care setting presents many possible threats to the self, such as illness, disability, surgery, and pain.

A patient waves a newspaper and says, "I must have my credit card and use the computer right now. A store is having a big sale, and I need to order 10 dresses and four pairs of shoes." Select the nurse's appropriate intervention. The nurse:

invites the patient to sit together and look at new fashion magazines.

An outpatient diagnosed with bipolar disorder takes lithium carbonate 300 mg three times daily. The patient reports nausea. To reduce the nausea most effectively, the nurse suggests that the lithium be taken with:

meals.

A patient diagnosed with borderline personality disorder has a history of self-mutilation and suicide attempts. The patient reveals feelings of depression and anger with life. Which type of medication would the nurse expect to be prescribed? a.Benzodiazepine b.Mood stabilizing medication c.Monoamine oxidase inhibitor (MAOI) d.Cholinesterase inhibitor

mood stabilizing medication

An individual is found to consistently wear only a bathrobe and neglect the cleanliness of his apartment. When neighbors ask him to stop his frequent outbursts of operatic arias, he acts outraged and tells them he must sing daily and will not promise to be quieter. This behavior supports that he is .....

not demonstrating any definitive signs of mental illness

Personality traits most likely to be documented regarding a patient demonstrating characteristics of an obsessive-compulsive personality disorder are a.affable, generous. b.perfectionist, inflexible. c.suspicious, holds grudges. d.dramatic speech, impulsive.

perfectionist, inflexible

The mental health status of a particular client can best be assessed by considering ...

placement on a continuum from health to illness

A patient experiencing acute mania undresses in the group room and dances. The nurse intervenes initially by:

putting a blanket around the patient and walking with the patient to a quiet room.

A kindergartner is disruptive in class. This child is unable to sit for expected lengths of time, inattentive to the teacher, screams while the teacher is talking, and is aggressive toward other children. The nurse plans interventions designed to:* a. promote integration of self-concept. b. provide inpatient treatment for the child. c. reduce loneliness and increase self-esteem. d. improve language and communication skills.

reduce loneliness and increase self-esteem.

Which intervention is appropriate for an individual diagnosed with an antisocial personality disorder who frequently manipulates others? a.Refer requests and questions related to care to the case manager. b.Encourage the patient to discuss feelings of fear and inferiority. c.Provide negative reinforcement for acting-out behavior. d.Ignore, rather than confront, inappropriate behavior.

refer requests and questions related to care to the case manager

The client's priority nursing diagnosis has been established as risk for self-directed violence: suicide related to multiple losses. The priority outcome would be that the client will refrain from attempting suicide.

refrain from attempting suicide. Correct Refraining from suicidal attempts is the only outcome that addresses the risk for self-directed violence. The absence of a feeling of powerlessness is not appropriate for the stated nursing diagnosis. The remaining options are interventions. REF: Page 124-125

What is the priority intervention for a nurse beginning to work with a patient diagnosed with a schizotypal personality disorder? a.Respect the patient's need for periods of social isolation. b.Prevent the patient from violating the nurse's rights. c.Teach the patient how to select clothing for outings. d.Engage the patient in community activities.

respect the patients need for periods of social isolation

As a nurse prepares to administer medication to a patient diagnosed with a borderline personality disorder, the patient says, "Just leave it on the table. I'll take it when I finish combing my hair." What is the nurse's best response? a.Reinforce this assertive action by the patient. Leave the medication on the table as requested. b.Respond to the patient, "I'm worried that you might not take it. I'll come back later." c.Say to the patient, "I must watch you take the medication. Please take it now." d.Ask the patient, "Why don't you want to take your medication now?"

say to the patient, i must watch you take the medication. please take it now

A patient diagnosed with borderline personality disorder was hospitalized several times after multiple episodes of head banging and carving on both wrists. The patient remains impulsive. Which nursing diagnosis is the initial focus of this patient's care? a.Self-mutilation b.Impaired skin integrity c.Risk for injury d.Powerlessness

self mutilation

The exact cause of bipolar disorder has not been determined; however, for most patients:

several factors, including genetics, are implicated.

When a 5-year-old is disruptive, the nurse says, "You must take a time-out." The expectation is that the child will: a. go to a quiet room until called for the next activity. b. slowly count to 20 before returning to the group activity. c. sit on the edge of the activity until able to regain self-control. d. sit quietly on the lap of a staff member until able to apologize for the behavior.

sit on the edge of the activity until able to regain self-control.

The nurse caring for an individual demonstrating symptoms of schizotypal personality disorder would expect assessment findings to include a.arrogant, grandiose, and a sense of self-importance. b.attention seeking, melodramatic, and flirtatious. c.impulsive, restless, socially aggressive behavior. d.socially anxious, rambling stories, peculiar ideas.

socially anxious, rambling stories, peculiar ideas

A nurse set limits while interacting with a patient demonstrating behaviors associated with borderline personality disorder. The patient tells the nurse, "You used to care about me. I thought you were wonderful. Now I can see I was wrong. You're evil." This outburst can be assessed as a.denial. b.splitting. c.defensive. d.reaction formation.

splitting

The nurse best ensures appropriate client care when choosing an intervention from a Nursing Interventions Classification that matches both the defining data and the nursing diagnosis.

the defining data and the nursing diagnosis. Correct When choosing nursing interventions from the Nursing Interventions Classification or some other source, the nurse selects interventions that fit the nursing diagnosis (e.g., risk for suicide) and that match the defining data.

For which behavior would limit setting be most essential? The patient who a.clings to the nurse and asks for advice about inconsequential matters. b.is flirtatious and provocative with staff members of the opposite sex. c.is hypervigilant and refuses to attend unit activities. d.urges a suspicious patient to hit anyone who stares.

urges a suspicious patient to hit anyone who stares

A nurse reports to the treatment team that a patient diagnosed with an antisocial personality disorder has displayed the behaviors below. This patient is detached and superficial during counseling sessions. Which behavior by the patient most clearly warrants limit setting? a.Flattering the nurse b.Lying to other patients c.Verbal abuse of another patient d.Detached superficiality during counseling

verbal abuse of another patient

A new psychiatric technician says, "Schizophrenia ... schizotypal! What's the difference?" The nurse's response should include which information? a.A patient diagnosed with schizophrenia is not usually overtly psychotic. b.In schizotypal personality disorder, the patient remains psychotic much longer. c.With schizotypal personality disorder, the person can be made aware of misinterpretations of reality. d.Schizotypal personality disorder causes more frequent and more prolonged hospitalizations than schizophrenia.

with schizotypal personality disorder, the person can be made aware of misinterpretations of reality

The nurse receives a laboratory report indicating a patient's serum level is 1 mEq/L. The patient's last dose of lithium was 8 hours ago. This result is:

within therapeutic limits

Which documentation indicates that the treatment plan for a patient diagnosed with acute mania has been effective?

"Converses with few interruptions; clothing matches; participates in activities."

A nurse is interviewing a new client who is angry and highly suspicious. When asked about sexual orientation, the client becomes highly distressed and threatens to walk out of the interview. The nurse responds "I can see that this topic makes you uncomfortable. We can defer discussion of it today."

"I can see that this topic makes you uncomfortable. We can defer discussion of it today." Correct A cardinal rule of interviewing is "Don't probe sensitive areas." Clients are allowed to take the lead. REF: Page 117-118

You are interviewing Jamie, a 17-year-old female patient. She confides that she has been thinking of ways to kill a female peer who is Jamie's rival for the volleyball team captain position. She asks you if you can keep it a secret. The most appropriate response for you to make is: "Jamie, issues of this kind have to be shared with the treatment team and your parents."

"Jamie, issues of this kind have to be shared with the treatment team and your parents." Correct Although adolescent patients request confidentiality, issues of sexual abuse, threats of suicide or homicide, or issues that put the patient at risk for harm must be shared with the treatment team and the parents. A threat of this nature must be discussed with the treatment team and the parents. Confidentiality laws do not protect information that would lead to harm to the patient or others. This information would be shared with both the team and the parents

A patient diagnosed with bipolar disorder is in the maintenance phase of treatment. The patient asks, "Do I have to keep taking this lithium even though my mood is stable now?" Select the nurse's appropriate response.

"Taking the medication every day helps reduce the risk of a relapse."

Which response to a patient's question of why you need to conduct an assessment interview best explains its purpose? "We will be able to form a relationship together where we can discuss the current problems and come up with goals and a plan for treatment."

"We will be able to form a relationship together where we can discuss the current problems and come up with goals and a plan for treatment." Correct Some of the purposes of the assessment interview are to establish rapport, learn more about the presenting issues, and form mutual goals and a plan for treatment. The other options do not appropriately explain the assessment purpose.

The nurse best assesses the client's spiritual life by asking, "What role does religion play in your life?"

"What role does religion play in your life?" Correct Asking the client to define the role of religion in their life allows for discussion related to the other topics. REF: Page 121-122

You are working in the emergency department when a 26-year-old male patient is brought in suffering from psychosis. The patient is unable to give any coherent history. The patient's best friend is with him and offers to give you information regarding the patient. Which of the following responses is appropriate? "Yes, I will be happy to get any information and history that you can provide."

"Yes, I will be happy to get any information and history that you can provide." Correct The friend is a secondary source of information that will be helpful since the patient is not able to give any history or information at this time. Confidentiality laws do not prohibit obtaining information from a secondary source. The friend can provide information and/or history immediately and may be able to relate events that happened just before coming to the hospital. A release would not be necessary to take information about the patient from a secondary source, and a psychotic patient would not be competent to sign a release.

A cognitive therapist would help a client restructure the thought "I am stupid!" to...

-- "What I did was stupid."

According to Freud, the nurse recognizes that a client experiencing dysfunction of the conscious as part of the mind will have problems with...

-- all material that the person is aware of at any one time

Match the group member's comment to the role it most closely represents. a. Playboy b. Energizer c. Organizer d. Follower 1. "We aren't getting much done; let's speed things up and make a decision." 2. "Last week we finished our first goal and today we are starting on our second." 3. "Everyone has had good ideas so far; so, whatever you say is fine with me." 4. "It doesn't really matter what we do. I'm just here because they make me attend."

1. ANS: B 2. ANS: C 3. ANS: D 4. ANS: A

Which finding best indicates that the goal "Demonstrate mentally healthy behavior" was achieved? A patient: a. sees self as approaching ideals and capable of meeting demands. b. seeks others to assume responsibility for major areas of own life. c. behaves without considering the consequences of personal actions. d. aggressively meets own needs without considering the rights of others.

A

Two staff nurses applied for promotion to nurse manager. The nurse not promoted initially had feelings of loss but then became supportive of the new manager by helping make the transition smooth and encouraging others. Which term best describes the nurse's response? a.Altruism b.Suppression c.Intellectualization d.Reaction formation

A Altruism is the mechanism by which an individual deals with emotional conflict by meeting the needs of others and receiving gratification vicariously or from the responses of others. The nurse's reaction is conscious rather than unconscious. There is no evidence of suppression. Intellectualization is a process in which events are analyzed based on remote, cold facts and without passion, rather than incorporating feeling and emotion into the processing. Reaction formation is when unacceptable feelings or behaviors are controlled and kept out of awareness by developing the opposite behavior or emotion.

A patient in the emergency department shows disorganized behavior and incoherence after a friend suggested a homosexual encounter. In which room should the nurse place the patient? a.An interview room furnished with a desk and two chairs b.A small, empty storage room with no windows or furniture c.A room with an examining table, instrument cabinets, desk, and chair d.The nurse's office, furnished with chairs, files, magazines, and bookcases

A Individuals experiencing severe to panic-level anxiety require a safe environment that is quiet, non-stimulating, structured, and simple. A room with a desk and two chairs provides simplicity, few objects with which the patient could cause self-harm, and a small floor space in which the patient can move about. A small, empty storage room without windows or furniture would feel like a jail cell. The nurse's office or a room with an examining table and instrument cabinets may be over-stimulating and unsafe.

An individual experiences sexual dysfunction and blames it on a partner by calling the person unattractive and unromantic. Which defense mechanism is evident? a.Rationalization b.Compensation c.Introjection d.Regression

A Rationalization involves unconsciously making excuses for one's behavior, inadequacies, or feelings. Regression involves the unconscious use of a behavior from an earlier stage of emotional development. Compensation involves making up for deficits in one area by excelling in another area. Introjection is an unconscious, intense identification with another person.

A person speaking about a rival for a significant other's affection says in an emotional, syrupy voice, "What a lovely person. That's someone I simply adore." The individual is demonstrating: a.reaction formation. b.repression. c.projection. d.denial.

A Reaction formation is an unconscious mechanism that keeps unacceptable feelings out of awareness by using the opposite behavior. Instead of expressing hatred for the other person, the individual gives praise. Denial operates unconsciously to allow an anxiety-producing idea, feeling, or situation to be ignored. Projection involves unconsciously disowning an unacceptable idea, feeling, or behavior by attributing it to another. Repression involves unconsciously placing an idea, feeling, or event out of awareness.

A nurse assesses an individual who commonly experiences anxiety. Which comment by this person indicates the possibility of obsessive-compulsive disorder? a."I check where my car keys are eight times." b."My legs often feel weak and spastic." c."I'm embarrassed to go out in public." d."I keep reliving a car accident."

A Recurring doubt (obsessive thinking) and the need to check (compulsive behavior) suggest obsessive-compulsive disorder. The repetitive behavior is designed to decrease anxiety but fails and must be repeated. Stating "My legs feel weak most of the time" is more in keeping with a somatic disorder. Being embarrassed to go out in public is associated with an avoidant personality disorder. Reliving a traumatic event is associated with posttraumatic stress disorder. See relationship to audience response question.

A patient fearfully runs from chair to chair crying, "They're coming! They're coming!" The patient does not follow the staff's directions or respond to verbal interventions. The initial nursing intervention of highest priority is to: a.provide for the patient's safety. b.encourage clarification of feelings. c.respect the patient's personal space. d.offer an outlet for the patient's energy.

A Safety is of highest priority because the patient experiencing panic is at high risk for self-injury related to increased non-goal-directed motor activity, distorted perceptions, and disordered thoughts. Offering an outlet for the patient's energy can occur when the current panic level subsides. Respecting the patient's personal space is a lower priority than safety. Clarification of feelings cannot take place until the level of anxiety is lowered.

A student says, "Before taking a test, I feel very alert and a little restless." Which nursing intervention is most appropriate to assist the student? a.Explain that the symptoms result from mild anxiety and discuss the helpful aspects. b.Advise the student to discuss this experience with a health care provider. c.Encourage the student to begin antioxidant vitamin supplements. d.Listen attentively, using silence in a therapeutic way.

A Teaching about symptoms of anxiety, their relation to precipitating stressors, and, in this case, the positive effects of anxiety will serve to reassure the patient. Advising the patient to discuss the experience with a health care provider implies that the patient has a serious problem. Listening without comment will do no harm but deprives the patient of health teaching. Antioxidant vitamin supplements are not useful in this scenario.

A patient repeatedly stated, "I'm stupid." Which statement by that patient would show progress resulting from cognitive-behavioral therapy? a. "Sometimes I do stupid things." b. "Things always go wrong for me." c. "I always fail when I try new things." d. "I'm disappointed in my lack of ability."

A "I'm stupid" is a cognitive distortion. A more rational thought is "Sometimes I do stupid things." The latter thinking promotes emotional self-control. The distracters reflect irrational or distorted thinking. This item relates to an audience response question.

A nurse makes an initial visit to a homebound patient diagnosed with a serious mental illness. A family member offers the nurse a cup of coffee. Select the nurse's best response. a. "Thank you. I would enjoy having a cup of coffee with you." b. "Thank you, but I would prefer to proceed with the assessment." c. "No, but thank you. I never accept drinks from patients or families." d. "Our agency policy prohibits me from eating or drinking in patients' homes."

A Accepting refreshments or chatting informally with the patient and family represent therapeutic use of self and help to establish rapport. The distracters fail to help establish rapport.

The nurse should assess a patient taking a drug with anticholinergic properties for inhibited function of the a. parasympathetic nervous system. b. sympathetic nervous system. c. reticular activating system. d. medulla oblongata.

A Acetylcholine is the neurotransmitter found in high concentration in the parasympathetic nervous system. When anticholinergic drugs inhibit acetylcholine action, blurred vision, dry mouth, constipation, and urinary retention commonly occur.

An experienced psychiatric nurse plans to begin a new job in a community-based medication clinic. The clinic sees culturally diverse patients. Which action should the nurse take first to prepare for this position? a. Investigate cultural differences in patients' responses to psychotropic medications. b. Contact the clinical nurse specialist for guidelines regarding cultural competence. c. Examine the literature on various health beliefs of members of diverse cultures. d. Complete an online continuing education offering about psychopharmacology.

A An experienced nurse working on a mental health inpatient unit would be familiar with the action and side effects of most commonly prescribed psychotropic medications. However, because the clinic serves a culturally diverse population, reviewing cultural differences in patients' responses to these medications is helpful and vital to patient safety. The distracters identify actions the nurse would take later.

Which viewpoint of an Asian American family will most affect decision making about care? a. The father is the authority figure. b. The mother is head of the household. c. Women should make their own decisions. d. Emotional communication styles are desirable.

A Asian American families traditionally place the father in the position of power as the head of the household. Mothers, as well as other women, are usually subservient to fathers in these cultures. Asian Americans are more likely to be reserved

The sibling of an Asian American patient tells the nurse, "My sister needs help for pain. She cries from the hurt." Which understanding by the nurse will contribute to culturally competent care for this patient? Persons of an Asian American heritage a. often express emotional distress with physical symptoms. b. will probably respond best to a therapist who is impersonal. c. will require prolonged treatment to stabilize these symptoms. d. should be given direct information about the diagnosis and prognosis

A Asian Americans commonly express psychological distress as a physical problem. The patient may believe psychological problems are caused by a physical imbalance. Treatment will likely be short. The patient will probably respond best to a therapist who is perceived as giving. Asian Americans usually have strong family ties and value hope more than truth.

Which action by a psychiatric nurse best applies the ethical principle of autonomy? a. Exploring alternative solutions with the patient, who then makes a choice. b. Suggesting that two patients who were fighting be restricted to the unit. c. Intervening when a self-mutilating patient attempts to harm self. d. Staying with a patient demonstrating a high level of anxiety.

A Autonomy is the right to self-determination, that is, to make one's own decisions. By exploring alternatives with the patient, the patient is better equipped to make an informed, autonomous decision. The distracters demonstrate beneficence, fidelity, and justice.

Which technique is most applicable to aversion therapy? a. Punishment b. Desensitization c. Role modeling d. Positive reinforcement

A Aversion therapy is akin to punishment. Aversive techniques include pairing of a maladaptive behavior with a noxious stimulus, punishment, and avoidance training.

Which comment by a patient who recently experienced a myocardial infarction indicates use of maladaptive, ineffective coping strategies? a. "My employer should have paid for a health club membership for me." b. "My family will see me through this. It won't be easy, but I will never be alone." c. "My heart attack was no fun, but it showed me up the importance of a good diet and more exercise." d. "I accept that I have heart disease. Now I need to decide if I will be able to continue my work daily."

A Blaming someone else and rationalizing one's failure to exercise are not adaptive coping strategies. Seeing the glass as half full, using social and religious supports, and confronting one's situation are seen as more effective strategies. The distracters demonstrate effective coping associated with a serious medical condition.

A nurse speaks with family members of a Chinese American parent recently diagnosed with major depressive disorder. Which comment by the nurse will the family find most comforting? "The nursing staff will a. take good care of your parent." b. pray with your parent several times a day." c. teach your parent important self-care strategies." d. educate your parent about safety information regarding medication."

A Chinese Americans hold an Eastern (balance) worldview. Persons who are ill or need health care are vulnerable and need protection. The family will find comfort in a nurse's statement that good care will be provided. The distracters apply to persons with a Western or indigenous worldview.

The nurse administers medications to a culturally diverse group of patients on a psychiatric unit. What expectation should the nurse have about pharmacokinetics? a. Patients of different cultural groups may metabolize medications at different rates. b. Metabolism of psychotropic medication is consistent among various cultural groups. c. Differences in hepatic enzymes will influence the rate of elimination of psychotropic medications. d. It is important to provide patients with oral and written literature about their psychotropic medications.

A Cytochrome enzyme systems, which vary among different cultural groups, influence the rate of metabolism of psychoactive drugs. Renal function influences elimination of psychotropic medication: hepatic function influences metabolism rates.

While talking with a patient diagnosed with major depressive disorder, a nurse notices the patient is unable to maintain eye contact. The patient's chin lowers to the chest. The patient looks at the floor. Which aspect of communication has the nurse assessed? a. Nonverbal communication b. A message filter c. A cultural barrier d. Social skills

A Eye contact and body movements are considered nonverbal communication. There are insufficient data to determine the level of the patient's social skills or an existing cultural barrier.

A patient is having difficulty making a decision. The nurse has mixed feelings about whether to provide advice. Which principle usually applies? Giving advice a. is rarely helpful. b. fosters independence. c. lifts the burden of personal decision making. d. helps the patient develop feelings of personal adequacy.

A Giving advice fosters dependence on the nurse and interferes with a patient's right to make personal decisions. It robs the patient of the opportunity to weigh alternatives and develop problem-solving skills. Furthermore, it may contribute to a patient's feelings of personal inadequacy. Giving advice also keeps the nurse in control and feeling powerful.

An obese patient has a diagnosis of schizophrenia. Medications that block which receptors would contribute to further weight gain? a. H1 b. 5 HT2 c. Acetylcholine d. GABA

A H1 receptor blockade results in weight gain, which is undesirable for an obese patient. Blocking of the other receptors would have little or no effect on the patient's weight.

A nurse in the clinic has a full appointment schedule. A Hispanic American patient arrives at 1230 for a 1000 appointment. A Native American patient does not keep an appointment at all. What understanding will improve the nurse's planning? These patients are a. members of cultural groups that have a different view of time. b. immature and irresponsible in health care matters. c. acting-out feelings of anger toward the system. d. displaying passive-aggressive tendencies.

A Hispanic Americans and Native Americans traditionally treat time in a way unlike the Western culture. They tend to be present-oriented- that is, they value the current interaction more than what is to be done in the future. If engaged in an activity, for example, they may simply continue the activity and appear later for an appointment. Understanding this, the nurse can avoid feelings of frustration and anger when the nurse's future orientation comes into conflict with the patient's present orientation.

A psychiatric nurse leads a medication education group for Hispanic patients. This nurse holds a Western worldview and uses pamphlets as teaching tools. Groups are short and concise. After the group, the patients are most likely to believe a. the nurse was uncaring. b. the session was effective. c. the teaching was efficient. d. they were treated respectfully.

A Hispanic individuals usually value relationship behaviors. Their needs are for learning through verbal communication rather than reading and for having time to chat before approaching the task.

A community mental health nurse has worked for months to establish a relationship with a delusional, suspicious patient. The patient recently lost employment and could no longer afford prescribed medications. The patient says, "Only a traitor would make me go to the hospital." Select the nurse's best initial intervention. a. With the patient's consent, contact resources to provide medications without charge temporarily. b. Arrange a bed in a local homeless shelter with nightly on-site supervision. c. Hospitalize the patient until the symptoms have stabilized. d. Ask the patient, "Do you feel like I am a traitor?"

A Hospitalization may damage the nurse-patient relationship, even if it provides an opportunity for rapid stabilization. If medication is restarted, the patient may possibly be stabilized in the home setting, even if it takes a little longer. Programs are available to help patients who are unable to afford their medications. A homeless shelter is inappropriate and unnecessary. Hospitalization may be necessary later, but a less restrictive solution should be tried first, since the patient is not dangerous. A yes/no question is non-therapeutic communication.

The nurse administers a medication that potentiates the action of ã-aminobutyric acid (GABA). Which effect would be expected? a.Reduced anxiety b.Improved memory c.More organized thinking d.Fewer sensory perceptual alterations

A Increased levels of GABA reduce anxiety. Acetylcholine and substance P are associated with memory enhancement. Thought disorganization is associated with dopamine. GABA is not associated with sensory perceptual alterations.

A nurse prepares to teach important medication information to a patient of Mexican heritage. How should the nurse manage the teaching environment? a. Stand very close to the patient while teaching. b. Maintain direct eye contact with the patient while teaching. c. Maintain a neutral emotional tone during the teaching session. d. Sit 4 feet or more from the patient during the teaching session.

A Latin American cultures use close personal space, closer than many other minority groups. Standing very close to the patient frequently indicates acceptance. Direct eye contact should not be prolonged with this patient. Persons of this cultural heritage have high emotionality.

A patient in alcohol rehabilitation reveals to the nurse, "I feel terrible guilt for sexually abusing my 6-year-old before I was admitted." Select the nurse's most important action. a. Anonymously report the abuse by phone to the local child protection agency. b. Reply, "I'm glad you feel comfortable talking to me about it." c. File a written report with the agency's ethics committee. d. Respect nurse-patient relationship confidentiality.

A Laws regarding child abuse reporting discovered by a professional during the suspected abuser's alcohol or drug treatment differ by state. Federal law supersedes state law and prohibits disclosure without a court order except in instances in which the report can be made anonymously or without identifying the abuser as a patient in an alcohol or drug treatment facility.

A nurse cares for four patients who are receiving clozapine, lithium, fluoxetine, and venlafaxine, respectively. With which patient should the nurse be most alert for problems associated with fluid and electrolyte imbalance? The patient receiving a. lithium. b. clozapine. c. fluoxetine. d. venlafaxine.

A Lithium is a salt and known to alter fluid and electrolyte balance, producing polyuria, edema, and other symptoms of imbalance. Patients receiving clozapine should be monitored for agranulocytosis. Patients receiving fluoxetine should be monitored for acetylcholine block. Patients receiving venlafaxine should be monitored for heightened feelings of anxiety.

A patient became depressed after the last of the family's six children moved out of the home 4 months ago. Select the best initial outcome for the nursing diagnosis Situational low self-esteem related to feelings of abandonment. The patient will: a. verbalize realistic positive characteristics about self by (date). b. agree to take an antidepressant medication regularly by (date). c. initiate social interaction with another person daily by (date). d. identify two personal behaviors that alienate others by (date).

A Low self-esteem is reflected by making consistently negative statements about self and self-worth. Replacing negative cognitions with more realistic appraisals of self is an appropriate intermediate outcome. The incorrect options are not as clearly related to the nursing diagnosis. Outcomes are best when framed positively; identifying two personal behaviors that might alienate others is a negative concept.

A patient diagnosed with major depression does not interact with others except when addressed, and then only in monosyllables. The nurse wants to show nonjudgmental acceptance and support for the patient. Which communication technique will be effective? a. Make observations. b. Ask the patient direct questions. c. Phrase questions to require yes or no answers. d. Frequently reassure the patient to reduce guilt feelings.

A Making observations about neutral topics such as the environment draws the patient into the reality around him or her but places no burdensome expectations for answers on the patient. Acceptance and support are shown by the nurse's presence. Direct questions may make the patient feel that the encounter is an interrogation. Open-ended questions are preferable if the patient is able to participate in dialogue. Platitudes are never acceptable. They minimize patient feelings and can increase feelings of worthlessness.

During a one-on-one interaction with the nurse, a patient frequently looks nervously at the door. Select the best comment by the nurse regarding this nonverbal communication. a. "I notice you keep looking toward the door." b. "This is our time together. No one is going to interrupt us." c. "It looks as if you are eager to end our discussion for today." d. "If you are uncomfortable in this room, we can move someplace else."

A Making observations and encouraging the patient to describe perceptions are useful therapeutic communication techniques for this situation. The other responses are assumptions made by the nurse.

A drug causes muscarinic receptor blockade. The nurse will assess the patient for a. dry mouth. b. gynecomastia. c. pseudoparkinsonism. d. orthostatic hypotension.

A Muscarinic receptor blockade includes atropine-like side effects, such as dry mouth, blurred vision, and constipation. Gynecomastia is associated with decreased prolactin levels. Movement defects are associated with dopamine blockade. Orthostatic hypotension is associated with á1 antagonism.

A nurse is concerned that an agency's policies are inadequate. Which understanding about the relationship between substandard institutional policies and individual nursing practice should guide nursing practice? a. Agency policies do not exempt an individual nurse of responsibility to practice according to professional standards of nursing care. b. Agency policies are the legal standard by which a professional nurse must act and therefore override other standards of care. c. Faced with substandard policies, a nurse has a responsibility to inform the supervisor and discontinue patient care immediately. d. Interpretation of policies by the judicial system is rendered on an individual basis and therefore cannot be predicted.

A Nurses are professionally bound to uphold standards of practice regardless of lesser standards established by a health care agency or a state. Conversely, if the agency standards are higher than standards of practice, the agency standards must be upheld. The Courts may seek to establish the standard of care through the use of expert witnesses when the issue is clouded.

In order to release information to another health care facility or third party regarding a patient diagnosed with a mental illness, the nurse must obtain a. a signed consent by the patient for release of information stating specific information to be released. b. a verbal consent for information release from the patient and the patient's guardian or next of kin. c. permission from members of the health care team who participate in treatment planning. d. approval from the attending psychiatrist to authorize the release of information.

A Nurses have an obligation to protect patients' privacy and confidentiality. Clinical information should not be released without the patient's signed consent for the release.

Which technique will best communicate to a patient that the nurse is interested in listening? a. Restating a feeling or thought the patient has expressed. b. Asking a direct question, such as "Did you feel angry?" c. Making a judgment about the patient's problem. d. Saying, "I understand what you're saying."

A Restating allows the patient to validate the nurse's understanding of what has been communicated. Restating is an active listening technique. Judgments should be suspended in a nurse-patient relationship. Close-ended questions such as "Did you feel angry?" ask for specific information rather than showing understanding. When the nurse simply states that he or she understands the patient's words, the patient has no way of measuring the understanding.

Which assessment question could a nurse ask to help identify secondary gains associated with a somatic symptom disorder? a. "What are you unable to do now but were previously able to do?" b. "How many doctors have you seen in the last year?" c. "Who do you talk to when you're upset?" d. "Did you experience abuse as a child?"

A Secondary gains should be assessed. Secondary gains reinforce maladaptive behavior. The patient's dependency needs may be evident through losses of abilities. When secondary gains are prominent, the patient is more resistant to giving up the symptom. There may be a history of abuse or doctor shopping, but the question does not assess the associated gains.

Which documentation for a patient diagnosed with major depression indicates the treatment plan was effective? a. Slept 6 hours uninterrupted. Sang with activity group. Anticipates seeing grandchild. b. Slept 10 hours uninterrupted. Attended craft group; stated "project was a failure, just like me." c. Slept 5 hours with brief interruptions. Personal hygiene adequate with assistance. Weight loss of 1 pound. d. Slept 7 hours uninterrupted. Preoccupied with perceived inadequacies. States, "I feel tired all the time."

A Sleeping 6 hours, participating with a group, and anticipating an event are all positive events. All the other options show at least one negative finding.

An adult diagnosed with major depression was treated with medication and cognitive behavioral therapy. The patient now recognizes how passivity contributed to the depression. Which intervention should the nurse suggest? a. Social skills training b. Desensitization techniques c. Relaxation training classes d. Use of complementary therapy

A Social skill training is helpful in treating and preventing the recurrence of depression. Training focuses on assertiveness and coping skills that lead to positive reinforcement from others and development of a patient's support system. Use of complementary therapy refers to adjunctive therapies such as herbals, which would be less helpful than social skill training. Assertiveness would be of greater value than relaxation training because passivity was a concern. Desensitization is used in treatment of phobias.

Select the example of tertiary prevention. a. Helping a person diagnosed with a serious mental illness learn to manage money b. Restraining an agitated patient who has become aggressive and assaultive c. Teaching school-age children about the dangers of drugs and alcohol d. Genetic counseling with a young couple expecting their first child

A Tertiary prevention involves services that address residual impairments, with a goal of improved independent functioning. Restraint is a secondary prevention. Genetic counseling and teaching school-age children about substance abuse and dependence are examples of primary prevention.

A patient is suspicious and frequently manipulates others. To which psychosexual stage do these traits relate? a. Oral b. Anal c. Phallic d. Genital

A The behaviors in the stem develop as the result of attitudes formed during the oral stage, when an infant first learns to relate to the environment. Anal-stage traits include stinginess, stubbornness, orderliness, or their opposites. Phallic-stage traits include flirtatiousness, pride, vanity, difficulty with authority figures, and difficulties with sexual identity. Genital-stage traits include the ability to form satisfying sexual and emotional relationships with members of the opposite sex, emancipation from parents, a strong sense of personal identity, or the opposites of these traits.

A person in the community asks, "Why aren't people with mental illness kept in state institutions anymore?" Select the nurse's best response. a. "Less restrictive settings are available now to care for individuals with mental illness." b. "There are fewer persons with mental illness, so less hospital beds are needed." c. "Most people with mental illness are still in psychiatric institutions." d. "Psychiatric institutions violated patients' rights."

A The community is a less restrictive alternative than hospitals for treatment of persons with mental illness. The distracters are incorrect and part of the stigma of mental illness.

A Chinese American patient diagnosed with an anxiety disorder says, "My problems began when my energy became imbalanced." The nurse asks for the patient's ideas about how to treat the imbalance. Which comment would the nurse expect from this patient? a. "My family will bring special foods to help me get well." b. "I hope my health care provider will prescribe some medication to help me." c. "I think I would benefit from talking to other patients with a similar problem." d. "I would like to have a native healer perform a ceremony to balance my energy."

A The concept of energy imbalance as a source of illness is an explanatory model familiar to Asian cultures.

A patient was diagnosed with seasonal affective disorder (SAD). During which month would this patient's symptoms be most acute? a. January b. April c. June d. September

A The days are short in January, so the patient would have the least exposure to sunlight. Seasonal affective disorder is associated with disturbances in circadian rhythm. Days are longer in spring, summer, and fall.

A 4-year-old grabs toys from other children and says, "I want that now!" From a psychoanalytic perspective, this behavior is a product of impulses originating in which system of the personality? a. Id b. Ego c. Superego d. Preconscious

A The id operates on the pleasure principle, seeking immediate gratification of impulses. The ego acts as a mediator of behavior and weighs the consequences of the action, perhaps determining that taking the toy is not worth the mother's wrath. The superego would oppose the impulsive behavior as "not nice." The preconscious is a level of awareness rather than an aspect of personality.

When a female Mexican American patient and a female nurse sit together, the patient often holds the nurse's hand. The patient also links arms with the nurse when they walk. The nurse is uncomfortable with this behavior. Which analysis is most accurate? a. The patient is accustomed to touch during conversation, as are members of many Hispanic subcultures. b. The patient understands that touch makes the nurse uncomfortable and controls the relationship based on that factor. c. The patient is afraid of being alone. When touching the nurse, the patient is reassured and comforted. d. The patient is trying to manipulate the nurse using nonverbal techniques.

A The most likely answer is that the patient's behavior is culturally influenced. Hispanic women frequently touch women they consider to be their friends. Although the other options are possible, they are less likely.

A nurse uses Maslow's hierarchy of needs to plan care for a patient diagnosed with mental illness. Which problem will receive priority? The patient a. refuses to eat or bathe. b. reports feelings of alienation from family. c. is reluctant to participate in unit social activities. d. is unaware of medication action and side effects.

A The need for food and hygiene are physiological and therefore take priority over psychological or meta-needs in care planning.

A patient with acute depression states, "God is punishing me for my past sins." What is the nurse's most therapeutic response? a. "You sound very upset about this." b. "God always forgives us for our sins." c. "Why do you think you are being punished?" d. "If you feel this way, you should talk to your minister."

A The nurse reflects the patient's comment, a therapeutic technique to encourage sharing for perceptions and feelings. The incorrect responses reflect probing, closed-ended comments, and giving advice, all of which are nontherapeutic.

A patient diagnosed with schizophrenia has been stable for 2 months. Today the patient's spouse calls the nurse to report the patient has not taken prescribed medication and is having disorganized thinking. The patient forgot to refill the prescription. The nurse arranges a refill. Select the best outcome to add to the plan of care. a. The patient's spouse will mark dates for prescription refills on the family calendar. b. The nurse will obtain prescription refills every 90 days and deliver to the patient. c. The patient will call the nurse weekly to discuss medication-related issues. d. The patient will report to the clinic for medication follow-up every week.

A The nurse should use the patient's support system to meet patient needs whenever possible. Delivery of medication by the nurse should be unnecessary for the nurse to do if patient or a significant other can be responsible. The patient may not need more intensive follow-up as long as medication is taken as prescribed.

A nurse surveys medical records. Which finding signals a violation of patients' rights? a. A patient was not allowed to have visitors. b. A patient's belongings were searched at admission. c. A patient with suicidal ideation was placed on continuous observation. d. Physical restraint was used after a patient was assaultive toward a staff member.

A The patient has the right to have visitors. Inspecting patients' belongings is a safety measure. Patients have the right to a safe environment, including the right to be protected against impulses to harm self.

Insurance will not pay for continued private hospitalization of a mentally ill patient. The family considers transferring the patient to a public hospital but expresses concern that the patient will not get any treatment if transferred. Select the nurse's most helpful reply. a. "By law, treatment must be provided. Hospitalization without treatment violates patients' rights." b. "All patients in public hospitals have the right to choose both a primary therapist and a primary nurse." c. "You have a justifiable concern because the right to treatment extends only to provision of food, shelter, and safety." d. "Much will depend on other patients, because the right to treatment for a psychotic patient takes precedence over the right to treatment of a patient who is stable."

A The right to medical and psychiatric treatment is conferred on all patients hospitalized in public mental hospitals under federal law.

Which principle has the highest priority when addressing a behavioral crisis in an inpatient setting? a. Resolve the crisis with the least restrictive intervention possible. b. Swift intervention is justified to maintain the integrity of a therapeutic milieu. c. Rights of an individual patient are superseded by the rights of the majority of patients. d. Patients should have opportunities to regain control without intervention if the safety of others is not compromised.

A The rule of using the least restrictive treatment or intervention possible to achieve the desired outcome is the patient's legal right. Planned interventions are nearly always preferable. Intervention may be necessary when the patient threatens harm to self.

Consider this comment from a therapist: "The patient is homosexual but has kept this preference secret. Severe anxiety and depression occur when the patient anticipates family reactions to this sexual orientation." Which perspective is evident in the speaker? a. Theory of interpersonal relationships b. Classical conditioning theory c. Psychosexual theory d. Behaviorism theory

A The theory of interpersonal relationships recognizes the anxiety and depression as resulting from unmet interpersonal security needs. Behaviorism and classical conditioning theories do not apply. A psychosexual formulation would focus on uncovering unconscious material that relates to the patient problem.

The laboratory report for a patient taking clozapine (Clozaril) shows a white blood cell count of 3000 mm3. Select the nurse's best action. a. Report the results to the health care provider immediately. b. Administer the next dose as prescribed. c. Give aspirin and force fluids. d. Repeat the laboratory test.

A These laboratory values indicate the possibility of agranulocytosis, a serious side effect of clozapine therapy. These results must be immediately reported to the health care provider, and the drug should be withheld. The health care provider may repeat the test, but in the meantime, the drug should be withheld.

Which scenario best depicts a behavioral crisis? A patient is a. waving fists, cursing, and shouting threats at a nurse. b. curled up in a corner of the bathroom, wrapped in a towel. c. crying hysterically after receiving a phone call from a family member. d. performing push-ups in the middle of the hall, forcing others to walk around.

A This behavior constitutes a behavioral crisis because the patient is threatening harm to another individual. Intervention is called for to defuse the situation. The other options speak of behaviors that may require intervention of a less urgent nature because the patients in question are not threatening harm to self or others.

A patient who experienced a myocardial infarction was transferred from critical care to a step-down unit. The patient then used the call bell every 15 minutes for minor requests and complaints. Staff nurses reported feeling inadequate and unable to satisfy the patient's needs. When the nurse manager intervenes directly with this patient, which comment is most therapeutic? a. "I'm wondering if you are feeling anxious about your illness and being left alone." b. "The staff are concerned that you are not satisfied with the care you are receiving." c. "Let's talk about why you use your call light so frequently. It is a problem." d. "You frustrate the staff by calling them so often. Why are you doing that?"

A This patient is experiencing anxiety associated with a serious medical condition. Verbalization is an effective outlet for anxiety. "I'm wondering if you are anxious..." focuses on the emotions underlying the behavior rather than the behavior itself. This opening conveys the nurse's willingness to listen to the patient's feelings and an understanding of the commonly seen concern about not having a nurse always nearby as in the intensive care unit. The other options focus on the behavior or its impact on nursing and do not help the patient with her emotional needs.

Inpatient hospitalization for persons with mental illness is generally reserved for patients who a. present a clear danger to self or others. b. are noncompliant with medication at home. c. have limited support systems in the community. d. develop new symptoms during the course of an illness.

A Hospitalization is justified when the patient is a danger to self or others, has dangerously decompensated, or needs intensive medical treatment. The distracters do not necessarily describe patients who require inpatient treatment.

In the majority culture of the United States, which individual has the greatest risk to be labeled mentally ill? One who: a. describes hearing God's voice speaking. b. is usually pessimistic but strives to meet personal goals. c. is wealthy and gives away $20 bills to needy individuals. d. always has an optimistic viewpoint about life and having own needs met.

A The question asks about risk. Hearing voices is generally associated with mental illness, but in charismatic religious groups, hearing the voice of God or a prophet is a desirable event. Cultural norms vary, which makes it more difficult to make an accurate diagnosis. The individuals described in the other options are less likely to be labeled mentally ill.

A 14-year-old belongs to a neighborhood gang, engages in sexually promiscuous behavior, and has a history of school truancy but reports that her parents are just old- fashioned and don't understand her. The assessment data supports that the client A. is displaying deviant behavior. B. cannot accurately appraise reality. C. is seriously and persistently mentally ill. D. should be considered for group home placement.

A is displaying deviant behavior

One implication of Freud's theory of the unconscious on psychiatric mental health nursing is related to the consideration that conscious and unconscious influences can help nurses better understand a. the root causes of client suffering. b. the client's immature behavior. c. the client's interpersonal interactions. d. the client's psychological ability to reason.

A the root causes of client suffering

An experienced nurse says to a new graduate, "When you've practiced as long as I have, you'll instantly know how to take care of psychotic patients." Which information should the new graduate consider when analyzing this comment? You may select more than one answer. a. The experienced nurse may have lost sight of patients' individuality, which may compromise the integrity of practice. b. New research findings should be integrated continuously into a nurse's practice to provide the most effective care. c. Experience provides mental health nurses with the essential tools and skills needed for effective professional practice. d. Experienced psychiatric nurses have learned the best ways to care for mentally ill patients through trial and error. e. An intuitive sense of patients' needs guides effective psychiatric nurses.

A & B Evidence-based practice involves using research findings and standards of care to provide the most effective nursing care. Evidence is continuously emerging, so nurses cannot rely solely on experience. The effective nurse also maintains respect for each patient as an individual. Overgeneralization compromises that perspective. Intuition and trial and error are unsystematic approaches to care.

The parent of a child diagnosed with Tourette's disorder says to the nurse, "I think my child is faking the tics because they come and go." Which response by the nurse is accurate? a. "Perhaps your child was misdiagnosed." b. "Your observation indicates the medication is effective." c. "Tics often change frequency or severity. That doesn't mean they aren't real." d. "This finding is unexpected. How have you been administering your child's medication?"

A 3-year-old who is mute, passive toward adults, and twirls while walking

Which presentations suggest the possibility of a factitious disorder, self-directed type? Select all that apply. a. History of multiple hospitalizations without findings of physical illness b. History of multiple medical procedures or exploratory surgeries c. Going from one doctor to another seeking the desired response d. Claims illness to obtain financial benefit or other incentive e. Difficulty describing symptoms

A, B Persons with factitious disorders, self-directed type, typically have a history of multiple hospitalizations and medical workups, with negative findings from workups. Sometimes they have even had multiple surgeries seeking the origin of the physical complaints. If they do not receive the desired response from a hospitalization, they may elope or accuse staff of incompetence. Such persons usually seek treatment through a consistent health care provider rather than doctor-shopping, are not motivated by financial gain or other external incentives, and present symptoms in a very detailed, plausible manner indicating considerable understanding of the disorder or presentation they are mimicking. See relationship to audience response question.

A patient's sibling says, "My brother has a mental illness, but the doctor ordered a functional magnetic resonance image (fMRI) test. That test is too expensive and will just increase the hospital bill." Select the nurse's best responses. (Select all that apply.) a. "Sometimes there are physical causes for psychiatric symptoms. This test will help us understand whether that is the situation." b. "Some mental illnesses are evident on fMRIs. This test will give information to help us plan the best care for your brother." c. "This test will indicate whether your brother has been taking his psychotropic medications as prescribed." d. "It sounds like you do not truly believe your brother had a mental illness." e. "It would be better for you to discuss your concerns with the health care provider."

A, B The correct responses provide information to the sibling. Modern imaging techniques are important tools in assessing molecular changes in mental disease and marking the receptor sites of drug action, which can help in treatment planning. Psychiatric symptoms can be caused by anatomical or physiologic abnormalities. There is no evidence of denial in the sibling's comment. The nurse can answer this question rather than referring it to the physician/health care provider. An fMRI does not demonstrate adherence to the medication regime.

Which benefits are most associated with use of telehealth technologies? (Select all that apply.) a. Cost savings for patients b. Maximize care management c. Access to services for patients in rural areas d. Prompt reimbursement by third-party payers e. Rapid development of trusting relationships with patients

A, B, C Telehealth has shown that it can maximize health and improve disease management skills and confidence with the disease process. Many rural parents have felt disconnected from services- telehealth technologies can solve those problems. Although telehealth's improved health outcomes regularly show cost savings for payers, one significant barrier is the current lack of reimbursement for remote patient monitoring by third-party payers. Telehealth technologies have not shown rapid development of trusting relationships.

A patient diagnosed with major depression shows vegetative signs of depression. Which nursing actions should be implemented? Select all that apply. a. Offer laxatives if needed. b. Monitor food and fluid intake. c. Provide a quiet sleep environment. d. Eliminate all daily caffeine intake. e. Restrict intake of processed foods.

A, B, C The correct options promote a normal elimination pattern. Although excessive intake of stimulants such as caffeine may make the patient feel jittery and anxious, small amounts may provide useful stimulation. No indication exists that processed foods should be restricted. See relationship to audience response question.

A nurse can best address factors of critical importance to successful community treatment by including making assessments relative to (Select all that apply) a. housing adequacy. b. family and support systems. c. income adequacy and stability. d. early psychosocial development. e. substance abuse history and current use.

A, B, C, E Early psychosocial developmental history is less relevant to successful outcomes in the community than the assessments listed in the other options. If a patient is homeless or fears homelessness, focusing on other treatment issues is impossible. Sufficient income for basic needs and medication is necessary. Adequate support is a requisite to community placement. Substance abuse undermines medication effectiveness and interferes with community adjustment.

Which statements by patients diagnosed with a serious mental illness best demonstrate that the case manager has established an effective long-term relationship? "My case manager (Select all that apply) a. talks in language I can understand." b. helps me keep track of my medication." c. gives me little gifts from time to time." d. looks at me as a whole person with many needs." e. let me do whatever I choose without interfering."

A, B, D Each correct answer is an example of appropriate nursing foci..communicating at a level understandable to the patient, providing medication supervision, and using holistic principles to guide care.

A psychiatric nurse discusses rules of the therapeutic milieu and patients' rights with a newly admitted patient. Which rights should be included? The right to (Select all that apply) a. have visitors. b. confidentiality. c. a private room. d. complain about inadequate care. e. select the nurse assigned to their care.

A, B, D Patients' rights should be discussed shortly after admission. Patients have rights related to receiving/refusing visitors, privacy, filing complaints about inadequate care, and accepting/refusing treatments (including medications). Patients do not have a right to a private room or selecting which nurse will provide care.

The nurse should be particularly alert to expression of psychological distress through physical symptoms among patients whose cultural beliefs include (Select all that apply) a. mental illness reflects badly on the family. b. mental illness shows moral weakness. c. intergenerational conflict is common. d. the mind, body, and spirit are merged. e. food choices influence one's health.

A, B, D Physical symptoms are seen as more acceptable in cultural groups in which interdependence and harmony of the group are emphasized. Mental illness is often perceived as reflecting a failure of the entire family. In groups in which mental illness is seen as a moral weakness and both the individual and family are stigmatized, somatization of mental distress is better accepted. In groups in which mind, body, and spirit are holistically perceived, somatization of psychological distress is common. Somatization and food are not commonly related. Intergenerational conflict has not been noted as a risk factor for somatization.

Which questions should the nurse ask to determine an individual's worldview? (Select all that apply.) a. What is more important: the needs of an individual or the needs of a community? b. How would you describe an ideal relationship between individuals? c. How long have you lived at your present residence? d. Of what importance are possessions in your life? e. Do you speak any foreign languages?

A, B, D The answers provide information about cultural values related to the importance of individuality, material possessions, relational connectedness, community needs versus individual needs, and interconnectedness between humans and nature. These will assist the nurse to determine a patient's worldview.

An individual is experiencing problems with memory. Which of these structures are most likely to be involved in this deficit? (Select all that apply.) a. Amygdala b. Hippocampus c. Occipital lobe d. Temporal lobe e. Basal ganglia

A, B, D The frontal and temporal lobes of the cerebrum play a key role in the storage and processing of memories. The amygdala and hippocampus also play roles in memory. The occipital lobe is predominantly involved with vision. The basal ganglia influence integration of physical movement, as well as some thoughts and emotions.

A child was placed in a foster home after being removed from abusive parents. The child is apprehensive and overreacts to environmental stimuli. The foster parents ask the nurse how to help the child. Which interventions should the nurse suggest? Select all that apply. a.Use a calm manner and low voice. b.Maintain simplicity in the environment. c.Avoid repetition in what is said to the child. d.Minimize opportunities for exercise and play. e.Explain and reinforce reality to avoid distortions.

A, B, E The child has moderate anxiety. A calm manner will calm the child. A simple, structured, predictable environment is desirable to decrease anxiety provoking and reduce stimuli. Calm, simple explanations that reinforce reality validate the environment. Repetition is often needed when the individual is unable to concentrate because of elevated levels of anxiety. Opportunities for play and exercise should be provided as avenues to reduce anxiety. Physical movement helps channel and lower anxiety. Play helps by allowing the child to act out concerns.

A person in the community asks, "People with mental illnesses went to state hospitals in earlier times. Why has that changed?" Select the nurse's accurate responses. (Select all that apply.) a. "Science has made significant improvements in drugs for mental illness, so now many persons may live in their communities." b. "There's now a better selection of less restrictive treatment options available in communities to care for people with mental illness." c. "National rates of mental illness have declined significantly. There actually is not a need for state institutions anymore." d. "Most psychiatric institutions were closed because of serious violations of patients' rights and unsafe conditions." e. "Federal legislation and payment for treatment of mental illness has shifted the focus to community rather than institutional settings."

A, B, E The community is a less restrictive alternative than hospitals for treatment of persons with mental illness. Funding for treatment of mental illness remains largely inadequate but now focuses on community rather than institutional care. Antipsychotic medications improve more symptoms of mental illness

A nurse assesses a patient diagnosed with conversion (functional neurological) disorder. Which comment is most likely from this patient? a. "Since my father died, I've been short of breath and had sharp pains that go down my left arm, but I think it's just indigestion." b. "I have daily problems with nausea, vomiting, and diarrhea. My skin is very dry, and I think I'm getting seriously dehydrated." c. "Sexual intercourse is painful. I pretend as if I'm asleep so I can avoid it. I think it's starting to cause problems with my marriage." d. "I get choked very easily and have trouble swallowing when I eat. I think I might have cancer of the esophagus."

A Patients with conversion (functional neurological) disorder demonstrate a lack of concern regarding the seriousness of symptoms. This lack of concern is termed "la belle indifférence." There is also a specific, identifiable cause for the development of the symptoms; in this instance, the death of a parent would precipitate stress. The distracters relate to sexual dysfunction and illness anxiety disorder.

Which patient is the best candidate for brief psychodynamic therapy? a. An accountant with a loving family and successful career who was involved in a short extramarital affair b. An adult with a long history of major depression who was charged with driving under the influence c. A woman with a history of borderline personality disorder who recently cut both wrists d. An adult male recently diagnosed with anorexia nervosa

A The best candidates for psychodynamic therapy are relatively healthy and well-functioning individuals, sometimes referred to as the "worried well," who have a clearly circumscribed area of difficulty and are intelligent, psychologically minded, and well-motivated for change. Patients with psychosis, severe depression, borderline personality disorders, and severe character disorders are not appropriate candidates for this type of treatment.

A nurse performed these actions while caring for patients in an inpatient psychiatric setting. Which action violated patients' rights? a. Prohibited a patient from using the telephone b. In patient's presence, opened a package mailed to patient c. Remained within arm's length of patient with homicidal ideation d. Permitted a patient with psychosis to refuse oral psychotropic medication

A The patient has a right to use the telephone. The patient should be protected against possible harm to self or others. Patients have rights to send and receive mail and be present during package inspection. Patients have rights to refuse treatment.

A new bill introduced in Congress would reduce funding for care of persons with mental illness. Groups of nurses write letters to their elected representatives in opposition to the legislation. Which role have the nurses fulfilled? a. Advocacy b. Attending c. Recovery d. Evidence-based practice

A An advocate defends or asserts another's cause, particularly when the other person lacks the ability to do that for self. Examples of individual advocacy include helping patients understand their rights or make decisions. On a community scale, advocacy includes political activity, public speaking, and publication in the interest of improving the human condition. Since funding is necessary to deliver quality programming for persons with mental illness, the letter-writing campaign advocates for that cause on behalf of patients who are unable to articulate their own needs.

A nurse encounters an unfamiliar psychiatric disorder on a new patient's admission form. To determine criteria used to establish this diagnosis, the nurse should consult which resource? a. Diagnostic and Statistical Manual of Mental Disorders b. A nursing diagnosis handbook c. A psychiatric nursing textbook d. A behavioral health reference manual

A The DSM-IV-TR gives the criteria used to diagnose each mental disorder. The distracters may not contain diagnostic criteria for a psychiatric illness.

A 40-year-old who lives with parents and works at an unchallenging job says, "I'm as happy as anyone else, even though I don't socialize much outside of work. My work is routine, but when new things come up, my boss explains things a few times to make sure I catch on. At home, my parents make decisions for me, and I go along with their ideas." The nurse should identify interventions to improve this patient's: a. self-concept. b. overall happiness. c. appraisal of reality. d. control over behavior.

A The patient sees self as needing multiple explanations of new tasks at work and allows the parents to make decisions, even though she is 40 years old. These behaviors indicate a poorly developed self-concept.

Which child demonstrates behaviors indicative of a neurodevelopmental disorder? a. A 4-year-old who stuttered for 3 weeks after the birth of a sibling b. A 9-month-old who does not eat vegetables and likes to be rocked c. A 3-month-old who cries after feeding until burped and sucks a thumb d. A 3-year-old who is mute, passive toward adults, and twirls while walking

A 3-year-old who is mute, passive toward adults, and twirls while walking

Which of the following best demonstrates party related to mental health care?

A client's mental health coverage is equal to his medical/surgical coverage

A patient states, "I'm starting cognitive-behavioral therapy. What can I expect from the sessions?" Which responses by the nurse would be appropriate? (Select all that apply.) a. "The therapist will be active and questioning." b. "You will be given some homework assignments." c. "The therapist will ask you to describe your dreams." d. "The therapist will help you look at your ideas and beliefs about yourself." e. "The goal is to increase subjectivity about thoughts that govern your behavior."

A, B, D Cognitive therapists are active rather than passive during therapy sessions because they help patient's reality-test their thinking. Homework assignments are given and completed outside the therapy sessions. Homework is usually discussed at the next therapy session. The goal of cognitive therapy is to assist the patient in identifying inaccurate cognitions and in reality-testing and formulating new, accurate cognitions. One distracter applies to psychoanalysis. Increasing subjectivity is not desirable.

A nurse assesses a patient suspected of having somatic symptom disorder. Which assessment findings regarding this patient support the suspected diagnosis? Select all that apply. a. Female b. Reports frequent syncope c. Rates pain as "1" on a scale of "10" d. First diagnosed with psoriasis at age 12 e. Reports insomnia often results from back pain

A, B, E There is no chronic disease to explain the symptoms for patients with somatic symptom disorder. Patients report multiple symptoms; gastrointestinal and pseudoneurological symptoms are common. This disorder is more common in women than in men. Patients with conversion disorder would have a tendency to underrate pain

A patient being treated with paroxetine (Paxil) 50 mg po daily for depression reports to the clinic nurse, "I took a few extra tablets earlier today and now I feel bad." Which assessments are most critical? Select all that apply. a. Vital signs b. Urinary frequency c. Psychomotor retardation d. Presence of abdominal pain and diarrhea e. Hyperactivity or feelings of restlessness

A, D, E The patient is taking the maximum dose of this SSRI and has ingested an additional unknown amount of the drug. Central serotonin syndrome must be considered. Symptoms include abdominal pain, diarrhea, tachycardia, elevated blood pressure, hyperpyrexia, increased motor activity, and muscle spasms. Central serotonin syndrome may progress to a full medical emergency if not treated early. The patient may have urinary retention, but frequency would not be expected.

. During a mental status examination, a client who is hospitalized states that she is in the hospital "to help out with the other patients." The nurse should record this information as A. poor insight.

A. poor insight. The nurse's objective assessment of the client's insight reflects the client's understanding of her current situation and medical condition. Knowledge, judgment, and memory are other objective cognitive assessments. None reflect the client's understanding of the responsibility for, or analysis of, the current situation.

6. A patient diagnosed with delirium is experiencing perceptual alterations. Which environmental adjustment should the nurse make for this patient? a. Provide a well-lit room without glare or shadows. Limit noise and stimulation. b. Maintain soft lighting day and night. Keep a radio on low volume continuously. c. Light the room brightly day and night. Awaken the patient hourly to assess mental status. d. Keep the patient by the nurse's desk while awake. Provide rest periods in a room with a television on.

ANS: A A quiet, shadow-free room offers an environment that produces the fewest sensory perceptual distortions for a patient with cognitive impairment associated with delirium. The other options have the potential to produce increased perceptual alterations.

16. An older adult patient in the intensive care unit has visual and auditory illusions. Which intervention will be most helpful? a. Using the patient's glasses and hearing aids b. Placing personally meaningful objects in view c. Placing large clocks and calendars on the wall d. Assuring that the room is brightly lit but very quiet at all times

ANS: A Illusions are sensory misperceptions. Glasses and hearing aids help clarify sensory perceptions. Without glasses, clocks, calendars, and personal objects are meaningless. Round-the-clock lighting promotes sensory overload and sensory perceptual alterations.

20. Goals of care for an older adult patient diagnosed with delirium caused by fever and dehydration will focus on: a. returning to premorbid levels of function. b. identifying stressors negatively affecting self. c. demonstrating motor responses to noxious stimuli. d. exerting control over responses to perceptual distortions.

ANS: A The desired overall goal is that the delirious patient will return to the level of functioning held before the development of delirium. Demonstrating motor response to noxious stimuli is an indicator appropriate for a patient whose arousal is compromised. Identifying stressors that negatively affect the self is too nonspecific to be useful for a patient with delirium. Exerting control over responses to perceptual distortions is an unrealistic indicator for a patient with sensorium problems related to delirium.

4. A patient says, "Please don't share information about me with the other people." How should the nurse respond? a. "I will not share information with your family or friends without your permission, but I will share information about you with other staff." b. "A therapeutic relationship is just between the nurse and the patient. It is up to you to tell others what you want them to know." c. "It depends on what you choose to tell me. I will be glad to disclose at the end of each session what I will report to others." d. "I cannot tell anyone about you. It will be as though I am talking about my own problems, and we can help each other by keeping it between us."

ANS: A A patient has the right to know with whom the nurse will share information and that confidentiality will be protected. Although the relationship is primarily between the nurse and patient, other staff needs to know pertinent data. The other incorrect responses promote incomplete disclosure on the part of the patient, require daily renegotiation of an issue that should be resolved as the nurse-patient contract is established, and suggest mutual problem solving. The relationship must be patient centered. See relationship to audience response question.

24. A community mental health nurse has worked with a patient for 3 years but is moving out of the city and terminates the relationship. When a novice nurse begins work with this patient, what is the starting point for the relationship? a. Begin at the orientation phase. b. Resume the working relationship. c. Initially establish a social relationship. d. Return to the emotional catharsis phase.

ANS: A After termination of a long-term relationship, the patient and new nurse usually have to begin at ground zero, the orientation phase, to build a new relationship. If termination is successfully completed, the orientation phase sometimes progresses quickly to the working phase. Other times, even after successful termination, the orientation phase may be prolonged.

17. A patient says, "I've done a lot of cheating and manipulating in my relationships." Select a nonjudgmental response by the nurse. a. "How do you feel about that?" b. "I am glad that you realize this." c. "That's not a good way to behave." d. "Have you outgrown that type of behavior?"

ANS: A Asking a patient to reflect on feelings about his or her actions does not imply any judgment about those actions, and it encourages the patient to explore feelings and values. The remaining options offer negative judgments.

7. As a nurse in the emergency department, you are caring for a patient who is exhibiting signs of depression. What is a priority nursing intervention you should perform for this patient? a. Assess for depression and ask directly about suicide thoughts. b. Ask the care provider to prescribe blood lab work to assess for depression. c. Focus on the presenting problems and refer the patient for a mental health evaluation. d. Interview the patient's family to identify their concerns about the patient's behaviors.

ANS: A Assessing directly for thoughts of harm to self or others is a priority intervention for any patient exhibiting signs of a mental health disorder. It is estimated that 50% of individuals who succeed in suicide had visited a health care provider within the previous 24 hours. Currently there is no serum lab that identifies depression. The risk of self-harm is a priority safety issue that is monitored in all health care within the scope of the nurse. It is important to obtain information directly from the patient when possible, and then validate the information from family or other secondary sources.

2. Which statement shows a nurse has empathy for a patient who made a suicide attempt? a. "You must have been very upset when you tried to hurt yourself." b. "It makes me sad to see you going through such a difficult experience." c. "If you tell me what is troubling you, I can help you solve your problems." d. "Suicide is a drastic solution to a problem that may not be such a serious matter."

ANS: A Empathy permits the nurse to see an event from the patient's perspective, understand the patient's feelings, and communicate this to the patient. The incorrect responses are nurse-centered (focusing on the nurse's feelings rather than the patient's), belittling, and sympathetic.

26. A 16-year-old wants to drive, but the parents will not allow it. A 14-year-old sibling was invited to several sleepovers, but the parents found reasons to deny permission. Both teens are annoyed because the parents buy clothes for them that are more suitable for younger children. The parents say, "We don't want our kids to grow up too fast." Which term best describes this family's boundaries? a. Rigid b. Clear c. Enmeshed d. Differentiated

ANS: A Rigid boundaries are those that do not change or flex with changing circumstances, as indicated here by parents who are reluctant to revise their roles and expectations about their children as the children mature. Enmeshed boundaries are those that have failed to differentiate or develop individually; the family shares roles and thoughts to an excessive degree, without a healthy degree of individuality. Clear boundaries are not enmeshed; they are appropriate and well maintained.

23. Which comment by a mother during a family therapy session shows evidence of scapegoating? a. "Our youngest child always starts arguments and upsets everyone else." b. "We all express our feelings openly except when we think it might upset my husband." c. "Our oldest child knows that my husband and I are doing all we can for the others." d. "After my husband has been drinking, I have to get everyone up and ready for school."

ANS: A Scapegoating is blaming family problems on a member of the family who is not very powerful. The purpose of the blaming is to keep the focus off painful issues and off the blamers themselves. A double-bind message, such as "We all express our feelings openly except when," involves giving instructions that are inherently contradictory or that place the person in a no-win situation. "Our oldest child knows that ..." is an example of triangulation, wherein a third party is engaged to help stabilize an unstable pair within the family. A child assuming parental responsibilities (e.g., caring for siblings) because a parent fails to do so is an example of enabling.

5. A wife received news that her husband died of heart failure and called her family to come to the hospital. She angrily tells the nurse who cared for him, "He would still be alive if you had given him your undivided attention." Select the nurse's best intervention. a. Say to the wife, "I understand you are feeling upset. I will stay with you until your family comes." b. Say to the wife, "Your husband's heart was so severely damaged that it could no longer pump." c. Say to the wife, "I will call the health care provider to discuss this matter with you." d. Hold the wife's hand in silence until the family arrives.

ANS: A The nurse builds trust and shows compassion in the face of adjustment disorders. Therapeutic responses provide comfort. The nurse should show patience and tact while offering sympathy and warmth. The distracters are defensive, evasive, or placating.

4. Because an intervention was required to control a patient's aggressive behavior, the nurse plans a critical incident debriefing with staff members. Which topics should be the primary focus of this discussion? (Select all that apply.) a. Patient behaviors associated with the incident b. Genetic factors associated with aggression c. Intervention techniques used by the staff d. Effects of environmental factors e. Theories of aggression

ANS: A, C, D The patient's behavior, the intervention techniques used, and the environment in which the incident occurred are important to establish realistic outcomes and effective nursing interventions. Discussing views about the theoretical origins of aggression would be less effective and relevant.

1. A patient referred to the eating disorders clinic has lost 35 pounds in 3 months. For which physical manifestations of anorexia nervosa should a nurse assess? (Select all that apply.) a. Peripheral edema b. Parotid swelling c. Constipation d. Hypotension e. Dental caries f. Lanugo

ANS: A, C, D, F Peripheral edema is often present because of hypoalbuminemia. Constipation related to starvation is often present. Hypotension is often present because of dehydration. Lanugo is often present and is related to starvation. Parotid swelling is associated with bulimia. Dental caries are associated with bulimia.

5. What is the priority intervention for a patient diagnosed with delirium who has fluctuating levels of consciousness, disturbed orientation, and perceptual alterations? a. Distraction using sensory stimulation b. Careful observation and supervision c. Avoidance of physical contact d. Activation of the bed alarm

ANS: B Careful observation and supervision are of ultimate importance because an appropriate outcome would be that the patient will remain safe and free from injury. Physical contact during care cannot be avoided. Activating a bed alarm is only one aspect of providing for the patient's safety.

13. A patient with stage 3 Alzheimer's disease tires easily and prefers to stay home rather than attend social activities. The spouse does the grocery shopping because the patient cannot remember what to buy. Which nursing diagnosis applies at this time? a. Self-care deficit b. Impaired memory c. Caregiver role strain d. Adult failure to thrive

ANS: B Memory impairment begins at stage 2 and progresses in stage 3. This patient is able to perform most self-care activities. Caregiver role strain and adult failure to thrive occur later.

27. An elderly patient is admitted with delirium secondary to a urinary tract infection. The family asks whether the patient will ever recover. Select the nurse's best response. a. "The health care provider is the best person to answer your question." b. "The confusion will probably get better as we treat the infection." c. "Unfortunately, delirium is a progressively disabling disorder." d. "I will be glad to contact the chaplain to talk with you."

ANS: B Usually, as the underlying cause of the delirium is treated, the symptoms of delirium clear. The distracters mislead the family.

24. Which type of group is a staff nurse with 2 months' psychiatric experience best qualified to conduct? a. Psychodynamic/psychoanalytic group b. Medication education group c. Existential/Gestalt group d. Family therapy group

ANS: B All nurses receive information about patient teaching strategies and basic information about psychotropic medications, making a medication education group a logical group for a beginner to conduct. The other groups would need a leader with more education and experience.

1. An adult outpatient diagnosed with major depressive disorder has a history of several suicide attempts by overdose. Given this patient's history and diagnosis, which antidepressant medication would the nurse expect to be prescribed? a. Amitriptyline b. Fluoxetine c. Desipramine d. Tranylcypromine sulfate

ANS: B Selective serotonin reuptake inhibitor antidepressants are very safe in overdosage situations, which is not true of the other medications listed. Given this patient's history of overdosing, it is important that the medication be as safe as possible in the event of another overdose of prescribed medication.

12. Which assessment finding presents the greatest risk for violent behavior directed at others? a. Severe agoraphobia b. History of spousal abuse c. Bizarre somatic delusions d. Verbalized hopelessness and powerlessness

ANS: B A history of prior aggression or violence is the best predictor of who may become violent. Patients with anxiety disorders are not particularly prone to violence unless panic occurs. Patients experiencing hopelessness and powerlessness may have coexisting anger, but violence is uncommon. Patients with paranoid delusions are at greater risk for violence than those with bizarre somatic delusions.

10. An older adult was stopped by police for driving through a red light. When asked for a driver's license, the adult hands the police officer a pair of sunglasses. What sign of dementia is evident? a. Aphasia b. Apraxia c. Agnosia d. Anhedonia

ANS: C Agnosia refers to the loss of sensory ability to recognize objects. Aphasia refers to the loss of language ability. Apraxia refers to the loss of purposeful movement. Anhedonia refers to a loss of joy in life.

29. A nurse gives anticipatory guidance to the family of a patient diagnosed with mild cognitive decline Alzheimer's disease. Which problem common to that stage should the nurse address? a. Violent outbursts b. Emotional disinhibition c. Communication deficits d. Inability to feed or bathe self

ANS: C Families should be made aware that the patient will have difficulty concentrating and following or carrying on in-depth or lengthy conversations. The other symptoms are usually seen at later stages of the disease.

15. Two patients in a residential care facility have dementia. One shouts to the other, "Move along, you're blocking the road." The other patient turns, shakes a fist, and shouts, "You're trying to steal my car." What is the nurse's best action? a. Administer one dose of an antipsychotic medication to both patients. b. Reinforce reality. Say to the patients, "Walk along in the hall. This is not a traffic intersection." c. Separate and distract the patients. Take one to the day room and the other to an activities area. d. Step between the two patients and say, "Please quiet down. We do not allow violence here."

ANS: C Separating and distracting prevents escalation from verbal to physical acting out. Neither patient loses self-esteem during this intervention. Medication probably is not necessary. Stepping between two angry, threatening patients is an unsafe action, and trying to reinforce reality during an angry outburst will probably not be successful when the patients are cognitively impaired.

2. A patient with fluctuating levels of awareness, confusion, and disturbed orientation shouts, "Bugs are crawling on my legs. Get them off!" Which problem is the patient experiencing? a. Aphasia b. dystonia c. Tactile hallucinations d. Mnemonic disturbance

ANS: C The patient feels bugs crawling on both legs, even though no sensory stimulus is actually present. This description meets the definition of a hallucination, a false sensory perception. Tactile hallucinations may be part of the symptom constellation of delirium. Aphasia refers to a speech disorder. Dystonia refers to excessive muscle tonus. Mnemonic disturbance is associated with dementia rather than delirium.

2. A married couple has two children living in the home. Recently, the wife's mother moved in. This family should be assessed as a. nuclear. b. blended. c. extended. d. alternative.

ANS: C An extended family has members from three or more generations living together. Nuclear family refers to a couple and their children. A blended family is one made up of members from two or more unrelated families. An alternative family can consist of a same-sex couple or an unmarried couple and children.

3. A patient has been resistant to treatment with antidepressant therapy. The care provider prescribes a monoamine oxidase inhibitor (MAOI) medication. What teaching is critical for the nurse to give the patient? a. Serum blood levels must be regularly monitored to assess for toxicity. b. To prevent side effects, the medication should be administered as an intramuscular injection. c. Eating foods such as blue cheese or red wine will cause side effects. d. This medication class may only be used safely for a few days at a time.

ANS: C MAOIs have serious food interactions when ingested with tyramine-containing foods such as aged or processed foods. Serum levels are routinely monitored when mood stabilizers such as lithium carbonate are prescribed. It is not necessary to administer this class intramuscularly. This medication takes several weeks to show effectiveness and should not be stopped abruptly; short-term use will not be effective.

18. A group is in the working phase. One member states, "That is the stupidest thing I've ever heard. Everyone whines and tells everyone else what to do. This group is a waste of my time." Which initial action by the group leader would be most therapeutic? a. Advise the member that hostility is inappropriate. Remove the member if it continues. b. Keep the group's focus on this member so the person can express the anger. c. Meet privately with the member outside of group to discuss the anger. d. Change to a more positive topic of discussion in this group session.

ANS: C Meeting privately with the member can convey interest and help defuse the anger so that it is less disruptive to the group. Removing the member would be a last resort and used only when the behavior is intolerably disruptive to the group process and all other interventions have failed. Decreasing the focus on the hostile member and focusing more on positive members can help soften the anger. Angry members often hide considerable vulnerability by using anger to keep others at a distance and intimidated. Changing the subject fails to respond to the behavior.

21. An emergency department nurse realizes that the spouse of a patient is becoming increasingly irritable while waiting. Which intervention should the nurse use to prevent further escalation of the spouse's anger? a. Offer the waiting spouse a cup of coffee. b. Explain that the patient's condition is not life threatening. c. Periodically provide an update and progress report on the patient. d. Suggest that the spouse return home until the patient's treatment is complete.

ANS: C Periodic updates reduce anxiety and defuse anger. This strategy acknowledges the spouse's presence and concern. A cup of coffee is a nice gesture, but it does not address the spouse's feelings. The other incorrect options would be likely to increase anger because they imply that the anxiety is inappropriate.

2. A patient diagnosed with anorexia nervosa is hospitalized for treatment. What features should the milieu provide? (Select all that apply.) a. Flexible mealtimes b. Unscheduled weight checks c. Adherence to a selected menu d. Observation during and after meals e. Monitoring during bathroom trips f. Privileges correlated with emotional expression

ANS: C, D, E Priority milieu interventions support restoration of weight and normalization of eating patterns. This requires close supervision of the patient's eating and prevention of exercise, purging, and other activities. There is strict adherence to menus. Observe patients during and after meals to prevent throwing away food or purging. Monitor all trips to the bathroom. Mealtimes are structured, not flexible. Weighing is performed on a regular schedule. Privileges are correlated with weight gain and treatment plan compliance.

22. A older patient diagnosed with severe, late-stage dementia no longer recognizes family members. The family asks how long it will be before this patient recognizes them when they visit. What is the nurse's best reply? a. "Your family member will never again be able to identify you." b. "I think that is a question the health care provider should answer." c. "One never knows. Consciousness fluctuates in persons with dementia." d. "It is disappointing when someone you love no longer recognizes you."

ANS: D Therapeutic communication techniques can assist the family to come to terms with the losses and irreversibility dementia imposes on both the loved one and themselves. Two incorrect responses close communication. The nurse should take the opportunity to foster communication. Consciousness does not fluctuate in patients with dementia.

16. A nurse assesses a patient for inclusion in group therapy. This patient has a childhood history of neglect and ridicule by parents. The patient says to the nurse, "My boss always expects more of me than the others, but talking to him would only make it worse." Which type of group would best address the patient's needs? a. Support b. Self-Help c. Psychoeducational d. Cognitive-behavioral

ANS: D Cognitive-behavioral group therapy focuses on specific maladaptive behaviors and thought patterns. Patients often repeat patterns of behavior in a group that they learned in their families. This type of group will afford the patient an opportunity for a corrective recapitulation of the primary family group. The incorrect answers identify groups appropriate for other types of problems.

4. A patient with a diagnosis of depression and suicidal ideation was started on an antidepressant 1 month ago. When the patient comes to the community health clinic for a follow-up appointment he is cheerful and talkative. What priority assessment must the nurse consider for this patient? a. The medication dose needs to be decreased. b. Treatment is successful, and medication can be stopped. c. The patient is ready to return to work. d. Specific assessment for suicide plan must be evaluated.

ANS: D Energy levels increase as depression lifts; this may increase the risk of completing a suicide plan. An increase in mood would not indicate a decrease or discontinuation of prescribed medication. The patient may be ready to return to work, but assessment for suicide risk in a patient who has had suicidal ideation is the priority assessment.

16. A family expresses helplessness related to dealing with a mentally ill member's odd behaviors, mood swings, and argumentativeness. An effective nursing intervention for this family would be to a. express sympathy for their situation. b. involve local social service agencies. c. explain symptoms of relapse. d. role-play difficult situations.

ANS: D Helping a family learn to set limits and deal with difficult behaviors can often be accomplished by using role-playing situations, which give family members the opportunity to try new, more effective approaches. The other options would not provide learning opportunities.

26. Select the correct etiology to complete this nursing diagnosis for a patient diagnosed with dissociative identity disorder. Disturbed personal identity related to a. obsessive fears of harming self or others. b. poor impulse control and lack of self-confidence. c. depressed mood secondary to nightmares and intrusive thoughts. d. cognitive distortions associated with unresolved childhood abuse issues.

ANS: D Nearly all patients with dissociative identity disorder have a history of childhood abuse or trauma. None of the other etiology statements is relevant.

6. A patient newly diagnosed with depression states, "I have had other people in my family say that they have depression. Is this an inherited problem?" What is the nurse's best response? a. "There are a lot of mood disorders that are caused by many different causes. Inheriting these disorders is not likely." b. "Current research is focusing on fluid and electrolyte disorders as a cause for mood disorders." c. "All of your family members raised in the same area have probably learned to respond to problems in the same way." d. "Members of the same family may have the same biological predisposition to experiencing mood disorders."

ANS: D Research is showing a genetic or hereditary role in the predisposition of experiencing mood disorders. These tendencies can be inherited by family members. Fluid and electrolyte imbalances cause many problems, but neurotransmitters in the brain are more directly linked to mood disorders. Mood disorders are not a learned behavior, but are linked to neurotransmitters in the brain.

10. Guidelines followed by the leader of a therapeutic group include focusing on recognizing dysfunctional behavior and thinking patterns, followed by identifying and practicing more adaptive alternate behaviors and thinking. Which theory is evident by this approach? a. Behavioral b. Interpersonal c. Psychodynamic d. Cognitive-behavioral

ANS: D The characteristics described are those of cognitive-behavioral therapy, in which patients learn to reframe dysfunctional thoughts and extinguish maladaptive behaviors. Behavioral therapy focuses solely on changing behavior rather than thoughts, feelings, and behaviors together. Interpersonal theory focuses on interactions and relationships. Psychodynamic groups focus on developing insight to resolve unconscious conflicts.

18. A soldier returned 3 months ago from a combat zone and was diagnosed with PTSD. Which social event would be most disturbing for this soldier? a. Halloween festival with neighborhood children b. Singing carols around a Christmas tree c. A family outing to the seashore d. Fireworks display on July 4th

ANS: D The exploding noises associated with fireworks are likely to provoke exaggerated responses for this soldier. The distracters are not associated with offensive sounds.

12. Which remark by a patient indicates passage from orientation to the working phase of a nurse-patient relationship? a. "I don't have any problems." b. "It is so difficult for me to talk about problems." c. "I don't know how it will help to talk to you about my problems." d. "I want to find a way to deal with my anger without becoming violent."

ANS: D Thinking about a more constructive approach to dealing with anger indicates a readiness to make a behavioral change. Behavioral change is associated with the working phase of the relationship. Denial is often seen in the orientation phase. It is common early in the relationship, before rapport and trust are firmly established, for a patient to express difficulty in talking about problems. Stating skepticism about the effectiveness of the nurse-patient relationship is more typically a reaction during the orientation phase.

What is the common behavior shared by both client and nurse at the beginning of the initial assessment interview? Anxiety

Anxiety Correct Both parties feel at least a small amount of anxiety associated with interacting with an unknown person. REF: Page 117-118

A client tells the mental health nurse "I am terribly frightened! I hear whispering that someone is going to kill me." Which criterion of mental health can the nurse assess as lacking?

Appraisal of reality

Others describe a worker as very shy and lacking in self-confidence. This worker stays in an office cubicle all day, never coming out for breaks or lunch. Which term best describes this behavior? a.Narcissistic b.Histrionic c.Avoidant d.Paranoid

Avoidant

A nurse inspects an inpatient psychiatric unit and finds that exits are free of obstructions, no one is smoking, and the janitor's closet is locked. These observations relate to a. coordinating care of patients. b. management of milieu safety. c. management of the interpersonal climate. d. use of therapeutic intervention strategies.

B Nursing staff are responsible for all aspects of milieu management. The observations mentioned in this question directly relate to the safety of the unit. The other options, although part of the nurse's concerns, are unrelated to the observations cited.

Which assessment data would help the health care team distinguish symptoms of conversion (functional neurological) disorder from symptoms of illness anxiety disorder (hypochondriasis)? a. Voluntary control of symptoms b. Patient's style of presentation c. Results of diagnostic testing d. The role of secondary gains

B Patients with illness anxiety disorder (hypochondriasis) tend to be more anxious about their concerns and display more obsessive attention to detail, whereas the patient with conversion (functional neurological) disorder often exhibits less concern with the symptom they are presenting than would be expected. Neither disorder involves voluntary control of the symptoms. Results of diagnostic testing for both would be negative (i.e., no physiological basis would be found for the symptoms). Secondary gains can occur in both disorders but are not necessary to either. See relationship to audience response question.

A patient says, "I know I have a brain tumor despite the results of the MRI. The radiologist is wrong. People who have brain tumors vomit, and yesterday I vomited all day." Which response by the nurse fosters cognitive reframing? a. "You do not have a brain tumor. The more you talk about it, the more it reinforces your belief." b. "Let's see if there are any other possible explanations for your vomiting." c. "You seem so worried. Let's talk about how you're feeling." d. "We need to talk about something else."

B Questioning the evidence is a cognitive reframing technique. Identifying causes other than the feared disease can be helpful in changing distorted perceptions. Distraction by changing the subject will not be effective.

The parent of an adolescent diagnosed with schizophrenia asks the nurse, "My child's doctor ordered a PET. What kind of test is that?" Select the nurse's best reply. a. "This test uses a magnetic field and gamma waves to identify problem areas in the brain. Does your teenager have any metal implants?" b. "PET means positron-emission tomography. It is a special type of scan that shows blood flow and activity in the brain." c. "A PET scan passes an electrical current through the brain and shows brain-wave activity. It can help diagnose seizures." d. "It's a special x-ray that shows structures of the brain and whether there has ever been a brain injury."

B The parent is seeking information about PET scans. It is important to use terms the parent can understand, so the nurse should identify what the initials mean. The correct response is the only option that provides information relevant to PET scans. The distracters describe magnetic resonance image (MRI), computed tomography (CT) scans, and EEG. See relationship to audience response question.

A patient says to the nurse, "My father has been dead for over 10 years, but talking to you is almost as comforting as the talks he and I had when I was a child." Which term applies to the patient's comment? a. Superego b. Transference c. Reality testing d. Counter-transference

B Transference refers to feelings a patient has toward the health care workers that were originally held toward significant others in his or her life. Counter-transference refers to unconscious feelings that the health care worker has toward the patient. The superego represents the moral component of personality it seeks perfection.

Which prescription medication would the nurse expect to be prescribed for a patient diagnosed with a somatic symptom disorder? a. Narcotic analgesics for use as needed for acute pain b. Antidepressant medications to treat underlying depression c. Long-term use of benzodiazepines to support coping with anxiety d. Conventional antipsychotic medications to correct cognitive distortions

B Various types of antidepressants may be helpful in somatic disorders directly by reducing depressive symptoms and hence somatic responses, but also indirectly by affecting nerve circuits that affect not only mood, but fatigue, pain perception, GI distress, and other somatic symptoms. Patients may benefit from short-term use of anti-anxiety medication (benzodiazepines) but require careful monitoring because of risks of dependence. Conventional antipsychotic medications would not be used, although selected atypical antipsychotics may be useful. Narcotic analgesics are not indicated.

A family has a long history of conflicted relationships among the members. Which family member's comment best reflects a mentally healthy perspective? a. "I've made mistakes but everyone else in this family has also." b. "I remember joy and mutual respect from our early years together." c. "I will make some changes in my behavior for the good of the family." d. "It's best for me to move away from my family. Things will never change."

C

A cruel and abusive person often uses rationalization to explain the behavior. Which comment demonstrates use of this defense mechanism? a."I don't know why I do mean things." b."I have always had poor impulse control." c."That person should not have provoked me." d."I'm really a coward who is afraid of being hurt."

C Rationalization consists of justifying one's unacceptable behavior by developing explanations that satisfy the teller and attempt to satisfy the listener. The abuser is suggesting that the abuse is not his or her fault; it would not have occurred except for the provocation by the other person. The distracters indicate some measure of acceptance of responsibility for the behavior.

A patient diagnosed with major depression began taking escitalopram (Lexapro) 5 days ago. The patient now says, "This medicine isn't working." The nurse's best intervention would be to: a. discuss with the health care provider the need to increase the dose. b. reassure the patient that the medication will be effective soon. c. explain the time lag before antidepressants relieve symptoms. d. critically assess the patient for symptoms of improvement.

C Escitalopram is an SSRI antidepressant. One to three weeks of treatment is usually necessary before symptom relief occurs. This information is important to share with patients.

A patient usually watches television all day, seldom going out in the community or socializing with others. The patient says, "I don't know what to do with my free time." Which member of the treatment team would be most helpful to this patient? a. Psychologist b. Social worker c. Recreational therapist d. Occupational therapist

C Recreational therapists help patients use leisure time to benefit their mental health. Occupational therapists assist with a broad range of skills, including those for employment. Psychologists conduct testing and provide other patient services. Social workers focus on the patient's support system.

Which disorder is an example of a culture-bound syndrome? a. Epilepsy b. Schizophrenia c. Running amok d. Major depressive disorder

C Running amok

A patient is hospitalized for severe major depressive disorder. Of the medications listed below, the nurse can expect to provide the patient with teaching about a. chlordiazepoxide. b. clozapine. c. sertraline. d. tacrine.

C Sertraline (Zoloft) is an selective serotonin reuptake inhibitor (SSRI). This antidepressant blocks the reuptake of serotonin, with few anticholinergic and sedating side effects. Clozapine is an antipsychotic. Chlordiazepoxide is an anxiolytic. Tacrine treats Alzheimer's disease.

Which principle should guide the nurse in determining the extent of silence to use during patient interview sessions? a. A nurse is responsible for breaking silences. b. Patients withdraw if silences are prolonged. c. Silence can provide meaningful moments for reflection. d. Silence helps patients know that what they said was understood.

C Silence can be helpful to both participants by giving each an opportunity to contemplate what has transpired, weigh alternatives, and formulate ideas. A nurse breaking silences is not a principle related to silences. It is inaccurate to say that patients withdraw during long silences or that silence helps patients know that they are understood. Feedback helps patients know they have been understood.

A patient diagnosed with depression begins selective serotonin reuptake inhibitor (SSRI) antidepressant therapy. The nurse should provide information to the patient and family about: a. restricting sodium intake to 1 gram daily. b. minimizing exposure to bright sunlight. c. reporting increased suicidal thoughts. d. maintaining a tyramine-free diet.

C Some evidence indicates that suicidal ideation may worsen at the beginning of antidepressant therapy; thus, close monitoring is necessary. Avoiding exposure to bright sunlight and restricting sodium intake are unnecessary. Tyramine restriction is associated with monoamine oxidase inhibitor (MAOI) therapy.

The unit secretary receives a phone call from the health insurer for a hospitalized patient. The caller seeks information about the patient's projected length of stay. How should the nurse instruct the unit secretary to handle the request? a. Obtain the information from the patient's medical record and relay it to the caller. b. Inform the caller that all information about patients is confidential. c. Refer the request for information to the patient's case manager. d. Refer the request to the health care provider.

C The case manager usually confers with insurers and provides the treatment team with information about available resources. The unit secretary should be mindful of patient confidentiality and should neither confirm that the patient is an inpatient nor disclose other information.

Documentation in a patient's chart shows, "Throughout a 5-minute interaction, patient fidgeted and tapped left foot, periodically covered face with hands, and looked under chair while stating, 'I enjoy spending time with you.'" Which analysis is most accurate? a. The patient is giving positive feedback about the nurse's communication techniques. b. The nurse is viewing the patient's behavior through a cultural filter. c. The patient's verbal and nonverbal messages are incongruent. d. The patient is demonstrating psychotic behaviors.

C When a verbal message is not reinforced with nonverbal behavior, the message is confusing and incongruent. It is inaccurate to say that the patient is giving positive feedback about the nurse's communication techniques. The concept of a cultural filter is not relevant to the situation because a cultural filter determines what we will pay attention to and what we will ignore. Data are insufficient to draw the conclusion that the patient is demonstrating psychotic behaviors.

A Vietnamese patient's family reports that the patient has wind illness. Which menu selection will be most helpful for this patient? a. Iced tea b. Ice cream c. Warm broth d. Gelatin dessert

C Wind illness is a culture-bound syndrome found in the Chinese and Vietnamese population. It is characterized by a fear of cold, wind, or drafts. It is treated by keeping very warm and avoiding foods, drinks, and herbs that are cold. Warm broth would be most in sync with the patient's culture and provide the most comfort. The distracters are cold foods.

A suspicious client who smokes several packs of cigarettes daily and drinks large quantities of coffee and soda as he is able to afford reacts to every nursing intervention with sarcasm. When asking for advice, the nurse manager's most helpful response is a. "You are dealing with a very difficult and resistant client; just keep with your plan." b. "If you haven't been able to establish client trust by now, ask for a change of assignment." c. "Remember that sarcasm represents the oral-stage fixation of development." d. "You are attempting to work with a client who likes to keep others off-balance."

C Remember that sarcasm represents the oral-stage fixation of development

A nurse wants to find a description of diagnostic criteria for anxiety disorders. Which resource would have the most complete information? a. The ICD-10 b. Nursing Outcomes Classification c. Diagnostic and Statistical Manual of Mental Disorders d. The ANA Psychiatric-Mental Health Nursing Scope and Standards of Practice

C The DSM-IV-TR details the diagnostic criteria for psychiatric clinical conditions. The other references are good resources but do not define the diagnostic criteria.

The nurse planning care for a 14-year-old needs to take into account that the developmental task of adolescence is to: a. establish trust. b. gain autonomy. c. achieve identity. d. develop a sense of industry.

C achieve identity

The admission note indicates a patient diagnosed with major depression has anergia and anhedonia. For which measures should the nurse plan? Select all that apply. a. Channeling excessive energy b. Reducing guilty ruminations c. Instilling a sense of hopefulness d. Assisting with self-care activities e. Accommodating psychomotor retardation

C, D, E Anergia refers to a lack of energy. Anhedonia refers to the inability to find pleasure or meaning in life; thus, planning should include measures to accommodate psychomotor retardation, assist with activities of daily living, and instill hopefulness. Anergia is lack of energy, not excessive energy. Anhedonia does not necessarily imply the presence of guilty ruminations.

While performing a mental status examination on a client, the nurse notices that the client's facial expression constantly appears angry. This information should be recorded as part of the client's C. affect.

C. affect Description of the client's facial expression is described as affect. Facial expression is not described in the areas of behavior, appearance, or thought process.

A child diagnosed with attention deficit hyperactivity disorder will begin medication therapy. The nurse should prepare a plan to teach the family about which classification of medications?* a. Central nervous system stimulants b. Tricyclic antidepressants c. Antipsychotics d. Anxiolytics

Central nervous system stimulants

A patient with diagnosed bipolar disorder was hospitalized 7 days ago and has been taking lithium 600 mg tid. Staff observes increased agitation, pressured speech, poor personal hygiene, and hyperactivity. Which action demonstrates that the nurse understands the most likely cause of the patient's behavior?

Consider the need to check the lithium level. The patient may not be swallowing medications

Which comment best indicates a patient is self-actualized? a. "I have succeeded despite a world filled with evil." b. "I have a plan for my life. If I follow it, everything will be fine." c. "I'm successful because I work hard. No one has ever given me anything." d. "My favorite leisure is walking on the beach, hearing soft sounds of rolling waves."

D The self-actualized personality is associated with high productivity and enjoyment of life. Self-actualized persons experience pleasure in being alone and an ability to reflect on events.

A white patient of German descent rocks back and forth, grimaces, and rubs both temples. What is the nurse's best action? a. Assess the patient for extrapyramidal symptoms. b. Sit beside the patient and rock in sync. c. Offer to pray with the patient. d. Assess the patient for pain.

D This patient of German descent would hold a Western worldview and be stoic about pain. This patient will keep pain as silent as possible and be reluctant to disclose pain unless the nurse actively assesses for it.

The Diagnostic and Statistical Manual of Mental Disorders classifies: a. deviant behaviors. b. people with mental disorders. c. present disability or distress. d. mental disorders people have.

D The DSM-IV-TR classifies disorders people have rather than people themselves. The terminology of the tool reflects this distinction by referring to individuals with a disorder rather than as a "schizophrenic" or "alcoholic," for example. Deviant behavior is not generally considered a mental disorder. Present disability or distress is only one aspect of the diagnosis.

A nurse participating in a community health fair is asked, "What is the most prevalent mental disorder in the United States?" Select the nurse's best response. a. Schizophrenia b. "Why do you ask?" c. Bipolar disorder d. Alzheimer's disease

D The prevalence for Alzheimer's disease is 10% for persons older than 65 and 50% for persons older than 85. The prevalence of schizophrenia is 1.1% per year. The prevalence of bipolar disorder is 2.6%. It is important for the nurse to provide information rather than probe the reason for the person's question.

A patient diagnosed with bipolar disorder commands other patients, "Get me a book. Take this stuff out of here," and other similar demands. The nurse wants to interrupt this behavior without entering into a power struggle. Which initial approach should the nurse select?

Distraction: "Let's go to the dining room for a snack."

A nurse prepares the plan of care for a patient experiencing an acute manic episode. Which nursing diagnoses are most likely? Select all that apply.

Disturbed thought processes Sleep deprivation

A college student said, "Most of the time I'm happy and feel good about myself. I have learned that what I get out of something is proportional to the effort I put into it." Which number on this mental health continuum should the nurse select? Mental Illness Mental Health 1 2 3 4 5

E The student is happy and has an adequate self-concept. The student is reality oriented, works effectively, and has control over own behavior. Mental health does not mean that a person is happy all the time.

A child reports to the school nurse of being verbally bullied by an aggressive classmate. What is the nurse's best first action? a. Give notice to the chief administrator at the school regarding the events. b. Encourage the victimized child to share feelings about the experience. c. Encourage the victimized child to ignore the bullying behavior. d. Discuss the events with the aggressive classmate.

Encourage the victimized child to share feelings about the experience.

An important difference between the developmental theories of Freud and Erikson is ...

Erikson viewed individual growth in terms of social setting.

A patient diagnosed with bipolar disorder is dressed in a red leotard and bright scarves. The patient twirls and shadow boxes. The patient says gaily, "Do you like my scarves? Here; they are my gift to you." How should the nurse document the patient's mood?

Euphoric

Which activity is NOT considered a purpose of the initial psychiatric assessment? Evaluating the results of intervention

Evaluating the results of intervention Correct At an initial assessment, no interventions would have taken place; hence evaluation is not a purpose of the initial contact.

Which nursing diagnosis for a psychiatric client is correctly structured and worded? Imbalanced nutrition: less than body requirements related to poor self-concept as evidenced by reporting "I'm not worthy of eating"

Imbalanced nutrition: less than body requirements related to poor self-concept as evidenced by reporting "I'm not worthy of eating" Correct This diagnosis contains all the required components: problem statement, the etiology, and supporting data. REF: Page 123

Which nursing diagnosis is universally applicable for children diagnosed with autism spectrum disorders? a. Impaired social interaction related to difficulty relating to others b. Chronic low self-esteem related to excessive negative feedback c. Deficient fluid volume related to abnormal eating habits d. Anxiety related to nightmares and repetitive activities

Impaired social interaction related to difficulty relating to others

Assessment data for a 7-year-old reveals an inability to take turns, blurting out answers to questions before a question is complete, and frequently interrupting others' conversations. How should the nurse document these behaviors? a. Disobedience b. Hyperactivity c. Impulsivity d. Anxiety

Impulsivity

Which factor presents the highest risk for a child to develop a psychiatric disorder? a. Having an uncle with schizophrenia b. Being the oldest child in a family c. Living with an alcoholic parent d. Being an only child

Living with an alcoholic parent

What is the most challenging nursing intervention with patients diagnosed with personality disorders who use manipulation? a.Supporting behavioral change b.Maintaining consistent limits c.Monitoring suicide attempts d.Using aversive therapy

Maintaining constant limits

The mental status examination aids in the collection of what type of data? Objective

Objective Correct The mental status exam mostly aids in the collection of objective data.

The plan of care for a patient in the manic state of bipolar disorder should include which interventions? Select all that apply.

Provide a structured environment for the patient. Ensure that the patient's nutritional needs are met.

A patient experiencing acute mania dances around the unit, seldom sits, monopolizes conversations, interrupts, and intrudes. Which nursing intervention will best assist the patient with energy conservation?

Provide a subdued environment.

A patient diagnosed with acute mania has distributed pamphlets about a new business venture on a street corner for 2 days. Which nursing diagnosis has priority?

Risk for injury

*What is the nurse's priority focused assessment for side effects in a child taking methylphenidate (Ritalin) for attention deficit hyperactivity disorder (ADHD)?* a. Dystonia, akinesia, and extrapyramidal symptoms b. Bradycardia and hypotensive episodes c. Sleep disturbances and weight loss d. Neuroleptic malignant syndrome

Sleep disturbances and weight loss

Which of the following are examples of subjective assessment data? (Select all that apply.) ● ● "Client states he has no reason to live." ● ● "Client meditates for relaxation." ● ● "Client states that he drinks three beers a day."

Subjective data includes psychosocial information about the client's thoughts, actions and feelings that can only be described by the client. Objective data is based on observable or verifiable facts.

Which statement best clarifies the difference between the art and the science of nursing?

The art is the care, compassion, and advocacy component, and the science is the applied knowledge base.

Which assessment finding would cause the nurse to consider a child to be most at risk for the development of mental illness? a. The child has been raised by a parent with chronic major depression. b. The child's best friend was absent from the child's birthday party. c. The child was not promoted to the next grade one year. d. The child moved to three new homes over a 2-year period.

The child has been raised by a parent with chronic major depression.

A newly admitted patient diagnosed with schizophrenia says, "The voices are bothering me. They yell and tell me I am bad. I have got to get away from them." Select the nurse's most helpful reply.

a. "Do you hear the voices often?" b. "Do you have a plan for getting away from the voices?" c. "I'll stay with you. Focus on what we are talking about, not the voices. " d. "Forget the voices and ask some other patients to play cards with you." ANSWER: C

A nurse at the mental health clinic plans a series of psychoeducational groups for persons newly diagnosed with schizophrenia. Which two topics take priority? (Select all that apply.)

a. "The importance of taking your medication correctly" b. "How to complete an application for employment" c. "How to dress when attending community events" d. "How to give and receive compliments" e. "Ways to quit smoking" ANSWERS: A and E

A patient diagnosed with schizophrenia has received fluphenazine decanoate twice a month for 3 years. The clinic nurse notes that the patient grimaces and constantly smacks both lips. The patient's neck and shoulders twist in a slow, snakelike motion. Which problem would the nurse suspect?

a. Agranulocytosis b. Tardive dyskinesia c. Tourette's syndrome d. Anticholinergic effects ANSWER: B

A patient receiving risperidone (Risperdal) reports severe muscle stiffness at 1030. By 1200, the patient has difficulty swallowing and is drooling. By 1600, vital signs are 102.8° F; pulse 110; respirations 26; 150/90. The patient is diaphoretic. Select the nurse's best analysis and action.

a. Agranulocytosis; institute reverse isolation. b. Tardive dyskinesia; withhold the next dose of medication. c. Cholestatic jaundice; begin a high-protein, high-cholesterol diet. d. Neuroleptic malignant syndrome; notify health care provider stat. ANSWER: D

When a patient diagnosed with schizophrenia was discharged 6 months ago, haloperidol was prescribed. The patient now says, "I stopped taking those pills. They made me feel like a robot." What are common side effects the nurse should validate with the patient?

a. Sedation and muscle stiffness b. Sweating, nausea, and diarrhea c. Mild fever, sore throat, and skin rash d. Headache, watery eyes, and runny nose ANSWER:A

A patient diagnosed with schizophrenia begins a new prescription for ziprasidone. The patient is 5'6'' and currently weighs 204 lbs. The patient has dry flaky skin, headaches about twice a month, and a family history of colon cancer. Which intervention has the highest priority for the nurse to include in the patient's plan of care?

a. Skin care techniques b. Scheduling a colonoscopy c. Weight management strategies d. Teaching to limit caffeine intake ANSWER: C

A patient insistently states, "I can decipher codes of DNA just by looking at someone." Which problem is evident

a. Visual hallucinations b. Magical thinking c. Idea of reference d. Thought insertion ANSWER: B

What is an appropriate initial outcome for a patient diagnosed with a personality disorder who frequently manipulates others? The patient will a.identify when feeling angry. b.use manipulation only to get legitimate needs met. c.acknowledge manipulative behavior when it is called to his or her attention. d.accept fulfillment of his or her requests within an hour rather than immediately.

acknowledge manipulative behavior when it is called to her or her attention

In psychiatric nursing, assessment of a "client" refers exclusively to an individual, family, group, or community.

an individual, family, group, or community. Correct Standards of practice for psychiatric nursing indicate that the client can be an individual, a family, a group, or a community.

A nurse plans care for an individual diagnosed with antisocial personality disorder. Which characteristic behaviors will the nurse expect? (Select all that apply.) a.Reclusive behavior b.Callous attitude c.Perfectionism d.Aggression e.Clinginess f.Anxiety

callous attitude, aggression

A patient experiencing acute mania is dancing atop a pool table in the recreation room. The patient waves a cue in one hand and says, "I'll throw the pool balls if anyone comes near me." To best assure safety, the nurse's first intervention is to:

clear the room of all other patients.

When interviewing an adolescent client, the nurse can expect the client to be most concerned about the issue of confidentiality.

confidentiality. Correct Adolescents are often concerned that what they reveal to the nurse or health care team will be shared with parents. Confidentiality should be explained at the outset of the interview.REF: Page 118

The quantitative study of the distribution of mental disorders in human populations is called

epidemiology

the principle that is the basis of nursing outcome planning is individuals have the right to autonomy to make decisions that affect them.

individuals have the right to autonomy to make decisions that affect them. Correct This is the only true statement. The nurse and the client should work collaboratively because each has knowledge to contribute to planning for the attainment of mutually derived outcomes. REF: Page 124-125

A health care provider recently convicted of Medicare fraud says to a nurse, "Sure I overbilled. Everyone takes advantage of the government. There are too many rules to follow and I deserve the money." These statements show a. shame. b. suspiciousness. c. superficial remorse. d. lack of guilt feelings.

lack of guilt feelings

Consider these three anticonvulsant medications: divalproex (Depakote), carbamazepine (Tegretol), and gabapentin (Neurontin). Which medication also belongs to this classification?

lamotrigine (Lamictal)

A health teaching plan for a patient taking lithium should include instructions to:

maintain normal salt and fluids in the diet.

Consider this comment to three different nurses by a patient diagnosed with an antisocial personality disorder, "Another nurse said you don't do your job right." Collectively, these interactions can be assessed as a.seductive. b.detached. c.manipulative. d.guilt-producing.

manipulative

When group therapy is prescribed as a treatment modality, the nurse would suggest placement of a 9-year-old in a group that uses: a. guided imagery. b. talk focused on a specific issue. c. play and talk about a play activity. d. group discussion about selected topics.

play and talk about a play activity.

A patient diagnosed with borderline personality disorder has self-inflicted wrist lacerations. The health care provider prescribes daily dressing changes. The nurse performing this care should a.maintain a stern and authoritarian affect. b.provide care in a matter-of-fact manner. c.encourage the patient to express anger. d.be very rigid and challenging.

provide care in a matter of fact manner

14. A therapy group adds new members as others leave. What type of group is evident? a. Open b. Closed c. Homogeneous d. Heterogeneous

ANS: A An open group is a group that adds members throughout the life of the group as other members leave and as more persons who would benefit from the group become available. A closed group does not add new members; the membership is established at the beginning and, except for the occasional losses as some members leave, does not change thereafter. A homogeneous group includes members who are similar, and a heterogeneous group includes dissimilar members; not enough data are provided here to determine which applies in this case.

16. Which statement provides the best rationale for closely monitoring a severely depressed patient during antidepressant medication therapy? a. As depression lifts, physical energy becomes available to carry out suicide. b. Patients who previously had suicidal thoughts need to discuss their feelings. c. For most patients, antidepressant medication results in increased suicidal thinking. d. Suicide is an impulsive act. Antidepressant medication does not alter impulsivity.

ANS: A Antidepressant medication has the objective of relieving depression. Risk for suicide is greater as the depression lifts, primarily because the patient has more physical energy at a time when he or she may still have suicidal ideation. The other options have little to do with nursing interventions relating to antidepressant medication therapy.

2. A leader plans to start a new self-esteem building group. Which intervention would be most helpful for assuring mutual respect within the group? a. Describe the importance of mutual respect in the first session and establish it as a group norm. b. Exclude potential members whose behavior suggests they are likely to be disrespectful of others. c. Give members a brochure describing the purpose, norms, and expectations of the group. d. Explain that mutual respect is expected and confront those who are not respectful.

ANS: A It is helpful to motivate members to behave respectfully by describing how mutual respect benefits all members and is necessary for the group to be fully therapeutic. Setting a tone and expectation of mutual respect from the outset is the most helpful intervention listed. Excluding members because of how they might behave could exclude members who would have been appropriate, depriving them of the potential benefits of the group. Conveying expectations by brochure is less effective than doing so orally, because it lacks the connection to each member a skilled leader can create to motivate members and impart the expectation of respect. Confronting inappropriate behavior is therapeutic but only addresses existing behavior rather than preventing all such undesired behavior.

13. Three members of a therapy group share covert glances as other members of the group describe problems. When one makes a statement that subtly criticizes another speaker, the others nod in agreement. Which group dynamic should the leader suspect? a. Some members are acting as a subgroup instead of as members of the main group. b. Some of the members have become bored and are disregarding others. c. Three members are showing their frustration with slower members. d. The leadership of the group has been ineffective.

ANS: A Subgroups, small groups isolated within a larger group and functioning separately from it, sometimes form within therapy groups. When this occurs, subgroup members are cohesive with other subgroup members but not with the members of the larger group. Members of the subgroup may be bored or frustrated or expressing passive aggression, but the primary dynamic is the splitting off from the main group.

9. A patient has talked constantly throughout the group therapy session, often repeating the same comments. Other members were initially attentive then became bored, inattentive, and finally sullen. Which comment by the nurse leader would be most effective? a. Say to everyone, "Most of you have become quiet. I wonder if it might be related to concerns you may have about how the group is progressing today." b. Say to everyone, "One person has done most of the talking. I think it would be helpful for everyone to say how that has affected your experience of the group." c. Say to everyone, "I noticed that as our group progressed, most members became quiet, then disinterested, and now seem almost angry. What is going on?" d. Say to the talkative patient, "You have been doing most of the talking, and others have not had a chance to speak as a result. Could you please yield to others now?"

ANS: A The most effective action the nurse leader can take will be the one that encourages the group to solve its own problem. Pointing out changes in the group and asking members to respond to them lays the foundation for a discussion of group dynamics. Asking members to respond to the talkative patient puts that patient in an awkward position, likely increasing her anxiety. As anxiety increases, monopolizing behavior tends to increase as well, so this response would be self-defeating. Asking members what is going on is a broader opening and might lead to responses unrelated to the issue that bears addressing; narrowing the focus to the group process more directly addresses what is occurring in the group. Focusing on the talkative patient would be less effective and involves the leader addressing the issue instead of members first attempting to do so themselves (giving them a chance to practice skills such as assertive communication).

17. Parents of a mentally ill teenager say, "We have never known anyone who was mentally ill. We have no one to talk to because none of our friends understand the problems we are facing." Select the nurse's most helpful intervention. a. Refer the parents to a support group. b. Build the parents' self-concept as coping parents. c. Teach the parents techniques of therapeutic communication. d. Facilitate achievement of normal developmental tasks of the family.

ANS: A The need for support is evident. Referrals are made when working with families whose needs are unmet. A support group will provide the parents with support of others with similar experiences and with whom they can share feelings and experiences. The distracters are less relevant to providing a network of support.

18. A depressed patient says, "Nothing matters anymore." What is the most appropriate response by the nurse? a. "Are you having thoughts of suicide?" b. "I am not sure I understand what you are trying to say." c. "Try to stay hopeful. Things have a way of working out." d. "Tell me more about what interested you before you became depressed."

ANS: A The nurse must make overt what is covert; that is, the possibility of suicide must be openly addressed. The patient often feels relieved to be able to talk about suicidal ideation.

24. After one of their identical twin daughters commits suicide, the parents express concern that the other twin may also have suicidal tendencies. Which reply should the nurse provide? a. "Genetics are associated with suicide risk. Monitoring and support are important." b. "Apathy underlies suicide. Instilling motivation is the key to health maintenance." c. "Your child is unlikely to act out suicide when identifying with a suicide victim." d. "Fraternal twins are at higher risk for suicide than identical twins."

ANS: A Twin studies suggest the presence of genetic factors in suicide; however, separating genetic predisposition to suicide from predisposition to depression or alcoholism is difficult. Primary interventions can be helpful in promoting and maintaining health and possibly counteracting genetic load. The incorrect options are untrue statements or an oversimplification.

10. A patient asks, "What is the major difference between conventional health care and complementary and alternative medicine (CAM)?" The nurse's best reply is that conventional health care a. focuses on what is done to the patient, whereas CAM focuses on body-mind interaction with an actively involved patient. b. has been tested by research so less regulation is needed, but CAM is religiously based and highly regulated. c. is controlled by the health care industry, but CAM is the people's medicine and not motivated by profit. d. is holistic and focused on health promotion, whereas CAM treats illnesses and is symptom-specific.

ANS: A Conventional health care focuses primarily on curative actions implemented on a mostly passive patient, whereas CAM focuses more on the mind-body aspects of health, along with the active involvement of the patient. Conventional health care is largely grounded in scientific research, and its various components are heavily regulated; the opposite tends to be true of CAM. Some forms of CAM have their roots in religious or cultural practices, but this is not characteristic of CAM as a whole. Both conventional health care and CAM can focus on health promotion and treatment of illness. Although critics express concern about the role of profit in conventional health care, the profit motive can also apply in CAM.

13. For which patient would it be most important for the nurse to urge immediate discontinuation of kava? A patient with a comorbid diagnosis of a. cirrhosis. b. osteoarthritis. c. multiple sclerosis. d. chronic back pain.

ANS: A Kava should be used with caution in patients with liver disease because of its potentially hepatotoxic effects. The other health problems do not pose immediate dangers.

22. During an assessment interview, a patient diagnosed with inflammatory bowel disease accompanied by frequent episodes of diarrhea says, "I've been using probiotics in small doses for about a week." When the nurse assesses mental status, expected findings would be a. intact cognitive function. b. slow verbal responses. c. paranoid thinking. d. slurred speech.

ANS: A Probiotics may reduce inflammation and heal the gut. No effect on cognitive function would be associated with use of microbiomes, including probiotics. The patient has taken small doses, so response times would be normal. It does not usually produce the effects cited in the distracters.

8. A patient diagnosed with major depressive disorder tells the nurse, "I want to try supplementing my selective serotonin reuptake inhibitor (SSRI) with St. John's wort." Which action should the nurse take first? a. Advise the patient of the danger of serotonin syndrome. b. Suggest that aromatherapy may produce better results. c. Assess the patient for depression and risk for suicide. d. Suggest the patient decrease the antidepressant dose.

ANS: A Research has suggested that St. John's wort is a mild inhibitor of serotonin reuptake and could lead to serotonin syndrome; this risk is increased if the patient is taking other medications that increase serotonin activity. Assessing the depression would be a secondary intervention. Aromatherapy has not been shown to be an effective adjunct or treatment for depression. Although a dosage reduction in her SSRI medication might reduce the risk of serotonin syndrome, this intervention is not in the nurse's scope of practice.

2. Which nursing interventions will be implemented for a patient who is actively suicidal? (Select all that apply.) a. Maintain arm's length, one-on-one direct observation at all times. b. Check all items brought by visitors and remove risk items. c. Use plastic eating utensils; count utensils upon collection. d. Remove the patient's eyeglasses to prevent self-injury. e. Interact with the patient every 15 minutes.

ANS: A, B, C One-on-one observation is necessary for anyone who has limited or unreliable control over suicidal impulses. Finger foods allow the patient to eat without silverware; "no silver or glassware" orders restrict access to a potential means of self-harm. Every-15-minute checks are inadequate to assure the safety of an actively suicidal person. Placement in a public area is not a substitute for arm's-length direct observation; some patients will attempt suicide even when others are nearby. Vision impairment requires eyeglasses (or contacts); although they could be used dangerously, watching the patient from arm's length at all times would allow enough time to interrupt such an attempt and would prevent the disorientation and isolation that uncorrected visual impairment could create.

3. Which important points should the nurse teach a patient about using herbal preparations? (Select all that apply.) a. Check active and inactive ingredients. b. Discontinue use if side or adverse effects occur. c. Avoid herbals during pregnancy and breast-feeding. d. Buying from online sources is preferable and cheaper. e. Inform your health care provider about the use of herbals.

ANS: A, B, C, E All of the instruction is correct except regarding purchase of herbals. Herbals should be purchased from a reputable firm. Internet purchasing might not be the best plan, unless the reputation of the firm can be confirmed.

1. A nurse assesses five newly hospitalized patients. Which patients have the highest suicide risk? (Select all that apply.) a. 82-year-old white male b. 17-year-old white female c. 22-year-old Hispanic male d. 19-year-old Native American male e. 39-year-old African American male

ANS: A, B, D Whites have suicide rates almost twice those of nonwhites, and the rate is particularly high for older adult males, adolescents, and young adults. Other high-risk groups include young African American males, Native American males, and older Asian Americans. Rates are not high for Hispanic males.

4. A patient reports frequent sleep disturbances. Which interventions could be considered to help improve the patient's sleep pattern? (Select all that apply.) a. Melatonin b. Chamomile c. Vitamin C d. Valerian e. SAM-e

ANS: A, B, D Melatonin, chamomile, and valerian have relaxant effects that help sleep. SAM-e may help with mild depression. Vitamin C has no effect on sleep.

2. A patient who emigrated from India is hospitalized. The patient and family use ayurvedic medicine. The nurse wants to adjust this patient's care so that it is more comfortable and familiar. What changes from usual Western practice should be considered? (Select all that apply.) a. In preparation for discharge, include a significant focus on preventive practices. b. Spend time exploring the patient's life overall, focusing on broader issues than health. c. Involve the patient's entire family and treatment team in decisions about treatment options. d. Anticipate that the patient will prefer and value interventions with high technology features. e. Provide relevant health-related information and then encourage the patient to determine which course of action to pursue.

ANS: A, B, E Ayurvedic medicine, an ancient practice that originated in India, stresses individual responsibility for health, is holistic, promotes prevention, recognizes the uniqueness of the individual, and offers natural methods of treatment. Ayurvedic medicine does not require spiritual cleansing or the involvement of family and the treatment team in all decisions.

3. Which situations are most likely to place severe, disabling stress on a family? (Select all that apply.) a. A parent needs long-term care after sustaining a severe brain injury. b. The youngest child in a family leaves for college in another state. c. A spouse is diagnosed with liver failure and needs a transplant. d. Parents of three children, aged 9, 7, and 2 years, get a divorce. e. A parent retires after working at the same job for 28 years.

ANS: A, C, D Major illnesses and divorce place severe, potentially disabling stress on families. The distracters identify normal milestones in a family's development.

1. A wife believes her husband is having an affair. Lately, he has been disinterested in romance and working late. The husband has an important, demanding project at work. The mother asks her teen, "What have you noticed about your father?" The teen later mentions this to the father, who says, "Tell your mother that I can't deal with her insecurities right now." Family therapy should focus on (Select all that apply.) a. identifying and reducing the cognitive distortion in each parent's perceptions. b. confronting the family with the need for honest, direct, assertive communication. c. helping the parents find ways to cope more effectively with their stress and fears. d. supporting the teen to redirect the parents when they try to communicate through her. e. convincing the mother that her fear of an affair is due to her own insecurities and unfounded. f. helping the husband understand how others might misinterpret the changes in his behavior.

ANS: A, C, D, F Each parent is seeing the other's behavior in a possibly distorted manner, which the nurse would explore and help the parents correct. The nurse would guide the parents to communicate more effectively, but confrontation would likely be non-therapeutic because it would increase the tension and triangulation. Since fear and anxiety contribute to triangulation, increasing the parent's coping abilities as well as reducing anxiety and fear would be areas for intervention. Teaching the adolescent how to protect herself from triangulation, when done in conjunction with interventions to help the parents reduce this behavior, would be protective of the adolescent and would assist the parents in their efforts to change this pattern of communication. The nurse has no facts about whether or not the husband is having an affair; therefore, the nurse should not convince the wife that her fear is only due to her insecurities. Her fears may be well-founded. Helping the husband understand how his wife might see the changes in behavior differently can help him to respond helpfully instead of accusing her of being insecure.

22. Which statement by a depressed patient will alert the nurse to the patient's need for immediate, active intervention? a. "I am mixed up, but I know I need help." b. "I have no one to turn to for help or support." c. "It is worse when you are a person of color." d. "I tried to get attention before I cut myself last time."

ANS: B Hopelessness is evident. Lack of social support and social isolation increases the suicide risk. Willingness to seek help lowers risk. Being a person of color does not suggest higher risk because more whites commit suicide than do individuals of other racial groups. Attention seeking is not correlated with higher suicide risk.

10. A parent is admitted to a unit for treatment of addictions. The patient's spouse and adolescent children participate in a family session. What is the most important aspect of this family's assessment? a. Spouse's codependent behaviors b. Interactions among family members c. Patient's reaction to the family's anger d. Children's responses to the family sessions

ANS: B Interactions among all family members are the raw material for family problem solving. By observing interactions, the nurse can help the family make its own assessments of strengths and deficits. The other options are narrower in scope when compared with the correct option.

13. Two divorced people plan to marry. The man has a teenager, and the woman has a toddler. This family will benefit most from a. role-playing opportunities for conflict resolution regarding discipline. b. guidance about parenting children at two developmental levels. c. formal teaching about problem-solving skills. d. referral to a family therapist.

ANS: B The newly formed family will be coping with tasks associated with the stage of rearing preschool children and teenagers. These stages require different knowledge and skills. There is no evidence of a problem, so the distracters are not indicated.

4. Which change in the brain's biochemical function is most associated with suicidal behavior? a. Dopamine excess b. Serotonin deficiency c. Acetylcholine excess d. aminobutyric acid deficiency

ANS: B Research suggests that low levels of serotonin may play a role in the decision to commit suicide. The other neurotransmitter alterations have not been implicated in suicidality.

23. A patient previously hospitalized for 2 weeks committed suicide the day after discharge. Which initial nursing measure will be most important regarding this event? a. Request the information technology manager to verify the patient's medical record is secure in the hospital information system. b. Hold a meeting for staff to provide support, express feelings, and identify overlooked clues or faulty judgments. c. Consult the hospital's legal department regarding potential consequences of the event. d. Document a report of a sentinel event in the patient's medical record.

ANS: B Support and an opportunity for staff to safely express feelings about the event should occur first. Interventions should help the staff come to terms with the loss and grow because of the incident. Identifying overlooked clues or faulty judgments will provide the groundwork for identifying changes needed in policies and procedures for future patients. Consulting the legal department is not an initial measure. A sentinel event report is not part of the medical record and can be prepared later. The other incorrect options will not control information or would result in unsafe care.

11. The nurse is planning a new sexuality group for patients. Which location would best enhance the effectiveness of this group? a. The hospital auditorium b. A small conference room c. A common area, such as a day room d. The corner of the music therapy room

ANS: B The conference room would provide a quiet, private area with few distractions, separate from other patient areas and effective for teaching and learning about a private topic. The auditorium is too large, and members' anxiety or lack of trust might lead them to spread out too far from each other, interfering with group process. The day room and the music therapy room are too busy and exposed, reducing privacy and increasing distractions.

12. A nurse interviews a homeless parent with two teenage children. To best assess the family's use of resources, the nurse should ask a. "Can you describe a problem your family has successfully resolved?" b. "What community agencies have you found helpful in the past?" c. "What aspect of being homeless is most frightening for you?" d. "Do you feel you have adequate resources to survive?"

ANS: B The correct option asks about use of resources in an open, direct fashion. It will give information about choices the family has made regarding use of resources in the community. The other questions do not address prior use of resources or focus on other aspects of coping.

5. A 15-year-old is hospitalized after a suicide attempt. This adolescent lives with the mother, stepfather, and several siblings. When performing a family assessment, the nurse must first determine a. how the family expresses and manages emotion. b. names and relationships of the family's members. c. the communication patterns between the patient and parents. d. the meaning that the patient's suicide attempt has for family members.

ANS: B The identity of the members of the family is the most fundamental information and should be obtained first. Without this, the nurse cannot fully process the other responses.

15. During a therapy group that uses existential/Gestalt theory, patients shared feelings that occurred at the time of their admission. After a brief silence, one member says, "Several people have described feeling angry. I would like to hear from members who had other feelings." Which group role is evident by this comment? a. Energizer b. Encourager c. Compromiser d. Self-confessor

ANS: B The member is filling the role of encourager by acknowledging those who have contributed and encouraging input from others. An energizer encourages the group to make decisions or take an action. The compromiser focuses on reducing or resolving conflict to preserve harmony. A self-confessor verbalizes feelings or observations unrelated to the group.

10. Select the most critical question for the nurse to ask an adolescent who has threatened to take an overdose of pills. a. "Why do you want to kill yourself?" b. "Do you have access to medications?" c. "Have you been taking drugs and alcohol?" d. "Did something happen with your parents?"

ANS: B The nurse must assess the patient's access to means to carry out the plan and, if there is access, alert the parents to remove from the home and take additional actions to assure the patient's safety. The information in the other questions may be important to ask but are not the most critical.

14. Parents of a teenager recently diagnosed with serious mental illness express dismay. One parent says, "Our hopes for our child's future are ruined. We probably won't ever have grandchildren." The nurse will use interventions to assist with a. denial. b. acceptance. c. acting out. d. manipulation.

ANS: B The parents comment suggests they are experiencing grief, a common reaction to having a family member diagnosed with mental illness. The grief stems from actual or potential losses, such as the family's ability to function, financial well-being, and altered future. Supporting the parents' acceptance of the patient's illness will enhance coping. Data do not support choosing any of the other options.

11. A patient has tried a variety of CAM approaches to manage health concerns. The nurse asks, "How is going to CAM practitioners different from seeing your medical doctors?" The patient is most likely to respond, "The CAM practitioners a. usually prescribe a course of invasive and sometimes painful treatments." b. spend more time talking with me and not just about my symptoms." c. say I need to become much more spiritual to be well." d. order many tests to determine my diagnoses."

ANS: B CAM practitioners often spend considerable time assessing the person in a holistic way. Visits typically involve lengthy discussions, in contrast to traditional physician visits, where contact is often brief. CAM remedies can sometimes be invasive or slightly painful, but usually they are noninvasive and well-tolerated. Some CAM practices are very spiritually focused, but most do not have overt religious elements. Conventional health care involves more diagnostic testing than CAM.

9. A patient tells the nurse, "I get sick so much, so I started taking ginseng to boost my immune system." The patient's only other medication is warfarin daily. Which potential complication should be included in the nursing assessment? a. Gastrointestinal distress b. Spontaneous bleeding c. Thromboembolism d. Drowsiness

ANS: B Ginseng may interact with anticoagulants and cause spontaneous bleeding. Warfarin is such an agent and can predispose the patient to spontaneous bleeding. It would not increase the risk of thromboembolism. Drowsiness and gastrointestinal complaints are common side effects.

19. A patient had a venous thrombosis 3 weeks ago and is now taking warfarin. When visiting the laboratory to have a prothrombin time drawn, the patient reports drinking ginseng tea to stimulate the immune system. Which nursing diagnosis applies? a. Impaired memory related to neurological changes b. Deficient knowledge related to potentially harmful drug interactions c. Ineffective denial related to consequences of mismanagement of therapeutic regime d. Effective management of the therapeutic regime related to augmentation of anti-coagulant therapy

ANS: B Ginseng tea is amongst the top 10 herbal products used in the United States and believed to have multiple beneficial properties. Because it antagonizes platelet-activating factor, it should not be taken by patients who are receiving anticoagulants or who have other potential bleeding problems. Thus, deficient knowledge is an appropriate nursing diagnosis.

16. Which CAM method is associated with using allergy injections of small amounts of an allergen in solution? a. Naturopathy b. Homeopathy c. Chiropractic d. Shiatsu

ANS: B Homeopathy uses small doses of a substance to stimulate the body's defenses and healing mechanisms to treat illness. Naturopathy emphasizes health restoration rather than disease. Chiropractic uses manipulation of the body to restore health. Shiatsu is a type of massage.

18. A patient report, "Last night I had several mixed drinks at a party. When I got home, I had difficulty falling sleep. I made two cups of herbal tea with lavender. This morning, I feel very groggy and have a headache." The nurse should explain that a. lavender should be delayed at least 1 hour after using alcohol to avoid side effects. b. lavender may increase sedation from other central nervous system depressants. c. herbal teas often cause nervous system side effects such as headaches. d. these feelings are actually a hangover from excessive alcohol intake.

ANS: B Lavender has sedative properties that are potentiated when used in combination with other central nervous system depressants. Headaches are another possible side effect of this herbal medicine. The nurse should advise caution in ingesting alcohol and lavender for these reasons. Taking lavender an hour after alcohol will not prevent these interactions, and it is likely that the lavender played a role in her feeling perhaps worse than usual after this episode of drinking. Herbal teas cause headaches in some cases, but it is not characteristic of this group of herbal remedies.

5. A patient says, "I have taken mega doses of vitamins for 3 months to improve my circulation, but I think I feel worse." Which action should the nurse take first? a. Explain to the patient that vitamin mega doses may be harmful and advise caution. b. Assess the patient for symptoms and signs of toxicity from excess vitamin exposure. c. Assess for signs of circulatory integrity to determine whether improvement has occurred. d. Educate the patient that research has not shown that megadoses of vitamins produce benefits.

ANS: B Mega doses of many vitamins, especially when taken over long periods, may produce dangerous side effects or toxicity. The priority for the nurse is to assess for signs of any dangerous consequences of the patient's use of such a regimen. Secondary interventions would include patient education about research findings related to the practice, along with any benefits and undesired effects associated with the practice. A health care provider should also assess the patient for cardiovascular concerns.

1. A patient tells the nurse, "I've been having problems getting a good night's sleep. I read some information on the Internet and started taking kava kava." Select the nurse's priority response. a. "The Internet does not have reliable health information for consumers." b. "The Food and Drug Administration warned against using it due to the link to severe liver damage." c. "Melatonin has been shown to have better effects for treating sleep disturbances." d. "Your sleep disturbances are related to your problems with anxiety. Herbs will not help."

ANS: B The Food and Drug Administration (FDA) warned against using kava kava due to the link to severe liver damage. The nurse has responsibilities to educate patients regarding safe use of complementary therapies. Melatonin may be useful for sleep disturbances, but the patient's safety is a higher priority. The other distracters are misleading.

12. An older male patient has suffered with episodic pruritus and skin eruptions for over 2 years. This patient tells the nurse, "When my skin gets better for a few days, I start worrying that it's going to start itching again soon. I think my worry may actually trigger the problems to start all over again." Which self-help technique should the nurse consider suggesting for this patient? a. Melatonin b. Meditation c. Purification d. Acupuncture

ANS: B The patient's comment suggests an element of anxiety accompanies the skin problem. Meditation is a popular self-help method recommended to reduce physical and emotional stress and to promote wellness. Purification, associated with ayurvedic practices, may or may not appeal to this patient. Acupuncture is performed by a professional practitioner, so it is not a self-help technique. The scenario does not indicate the patient is experiencing insomnia, so melatonin is not indicated.

20. When assessing a patient's plan for suicide, what aspect has priority? a. Patient's financial and educational status b. Patient's insight into suicidal motivation c. Availability of means and lethality of method d. Quality and availability of patient's social support

ANS: C If a person has plans that include choosing a method of suicide readily available and if the method is one that is lethal (i.e., will cause the person to die with little probability for intervention), the suicide risk is high. These areas provide a better indication of risk than the areas mentioned in the other options. See relationship to audience response question.

6. A patient in a group therapy session listens to others and then remarks, "I used to think I was the only one who felt afraid. I guess I'm not as alone as I thought." This comment is an example of a. altruism. b. ventilation. c. universality. d. group cohesiveness.

ANS: C Realizing that one is not alone and that others share the same problems and feelings is called universality. Ventilation refers to expressing emotions. Altruism refers to benefitting by being of help to others. Group cohesiveness refers to the degree of bonding among members of the group.

15. Which intervention will the nurse recommend for the distressed family and friends of someone who has committed suicide? a. Participating in reminiscence therapy b. Psychological postmortem assessment c. Attending a self-help group for survivors d. Contracting for at least two sessions of group therapy

ANS: C Survivors need outlets for their feelings about the loss and the deceased person. Self-help groups provide peer support while survivors work through feelings of loss, anger, and guilt. Psychological postmortem assessment would not provide the support necessary to work through feelings of loss associated with the suicide. Reminiscence therapy is not geared to loss resolution. Contracting for two sessions of group therapy would not provide sufficient time to work through the issues associated with a death by suicide.

7. A nurse at the well child clinic realizes that many parents have misconceptions about effective ways of disciplining their children. The nurse decides to form a group to address this problem. What should be the focus of the group? a. Support b. Socialization c. Health education d. Symptom management

ANS: C The nurse has diagnosed a knowledge deficit. The focus of the group should be education. Support and socialization are beneficial but should not be the primary focus of the group, and symptoms are not identified for intervention here.

13. A tearful, anxious patient at the outpatient clinic reports, "I should be dead." The initial task of the nurse conducting the assessment interview is to a. assess lethality of suicide plan. b. encourage expression of anger. c. establish trust with the patient. d. determine risk factors for suicide.

ANS: C This scenario presents a potential crisis. Establishing trust facilitates a therapeutic alliance that will allow the nurse to obtain relevant assessment data such as the presence of a suicide plan, lethality of plan, and presence of risk factors for suicide.

6. Acupuncture is a traditional Chinese medical treatment based on the belief that a. insertion of needles in key locations will drain toxic energies. b. pressure on meridian points will correct problems in energy flow. c. insertion of needles modulates the flow of energy along body meridians. d. taking small doses of noxious substances will alleviate specific symptoms.

ANS: C Acupuncture involves the insertion of needles to modulate the flow of body energy (qi) along specific body pathways called meridians. Acupressure uses pressure to affect energy flow. Homeopathy involves the use of microdosages of specific substances to effect health improvement. Traditional Chinese medicine (TCM) is more concerned with energy and life force balance, and acupuncture is not predicated on the removal of toxic energies.

14. A patient tells the nurse, "I prefer to treat my physical problems with herbs and vitamins. They are natural substances, and natural products are safe." Which response by the nurse would be most appropriate? a. "Natural substances tend to be safer than conventional medical remedies." b. "Natural remedies give you the idea that you are controlling your treatment." c. "The word natural can be a marketing term used to imply a product is healthy, but that's not always true." d. "You should not treat your own physical problems. You should see your health care provider for these problems."

ANS: C CAM remedies are usually natural substances, but it is a fallacy that products labeled natural are safer than conventional medicines. Some natural products contain powerful ingredients that can cause illness and damage to the body if taken inappropriately and, for some persons, can be dangerous even when used as directed. This is the most important message for the nurse to convey to the patient. So-called natural substances can have a number of significant side effects. Natural substances may give one the belief that he is controlling his own treatment, but that is not the message that most needs to be communicated here. Many patients can safely self-manage minor physical problems.

2. A patient shows a nurse this advertisement: "Our product is a scientific breakthrough helpful for depression, anxiety, and sleeplessness. Made from an ancient formula, it stimulates circulation and excretes toxins. Satisfaction guaranteed or your money back." Select the nurse's best response. a. "Over-the-counter products for sleep problems are ineffective." b. "Do not take anything unless it's prescribed by your doctor." c. "Let's do some additional investigation of that product." d. "It sounds like you are trying to self-medicate."

ANS: C Helping consumers actively evaluate the quality of information available to them is important. It is important for the nurse to work with the patient and include the patient's preferences regarding management of health. Advertisements indicating scientific breakthroughs or promising miracles for multiple ailments are usually for products that are useless and being fraudulently marketed. Some may even be harmful. Some over-the-counter products can be useful, and patients do not need a prescription for these products. The broader issue is safety and efficacy, rather than whether the patient is trying to self-medicate.

24. A patient diagnosed with depression confidently tells the nurse, "I've been supplementing my paroxetine with St. John's wort. It has helped a great deal." What is the nurse's priority action? a. Assess changes in the patient's level of depression. b. Remind the patient to use a secondary form of birth control. c. Educate the patient about the risks of selective serotonin syndrome. d. Suggest adding valerian to the treatment regimen to further improve results.

ANS: C St. John's wort inhibits serotonin reuptake by elevating extracellular sodium; thus, it may interact with medication, particularly selective serotonin reuptake inhibitors, to produce serotonin syndrome. Discussing the patient's birth control method is a secondary priority.

24. Which example of behavior in a family system demonstrates double bind communication? a. A mother tells her daughter, "You make me so mad that sometimes I wish I had never had you." b. A teenager tells her father, "You are treating me like a baby when you tell me I must be home by 10 PM on a school night." c. A son tells his mother, "You worry too much about what might happen. Nothing has happened yet, so why worry?" d. A wife tells her husband, "You go ahead with your bowling trip. Try not to worry about me falling on my crutches while I'm alone at home."

ANS: D A double bind communication is one that is inherently contradictory, that is, a comment that gives conflicting directions. In this case, the wife on crutches suggests that her husband should go bowling but then indicates that she will be at greater risk if he does, which in effect tells him "go ahead" and "don't do it" at the same time. This remark places the husband in a double bind, a situation in which no acceptable response exists. The distracters are clear, direct communications.

5. During a group therapy session, a newly admitted patient suddenly says to the nurse, "How old are you? You seem too young to be leading a group." Select the nurse's most appropriate response. a. "I am wondering what leads you to ask. Please tell me more." b. "I am old enough to be a nurse, which qualifies me to lead this group." c. "My age is not pertinent to why we are here and should not concern you." d. "You are wondering whether I have enough experience to lead this group?"

ANS: D A question such as this is common in the initial phase of group development when members are getting to know one another, dealing with trust issues, and testing the leader. Making explicit the implied serves to role model more effective communication and prompts further discussion of the patient's concern. Asking the patient to tell the leader more about the question focuses on the reason for the member's concern rather than on the issue raised (the experience and ability of the leader) and is a less helpful response. "I am old enough to be a nurse" and "age is not pertinent" are defensive responses and fail to address the patient's valid concern.

8. Which outcome would be most appropriate for a symptom-management group for persons diagnosed with schizophrenia? Group members will a. state the names of their medications. b. resolve conflicts within their families. c. rate anxiety at least two points lower. d. describe ways to cope with their illness.

ANS: D An appropriate psychoeducational focus for patients with schizophrenia is managing their symptoms; coping with symptoms such as impaired memory or impaired reality testing can improve functioning and enhance their quality of life. Names of medications might be appropriate for a medication education group but would be a low priority for symptom management. Addressing intra-family issues would be more appropriate within a family therapy group or possibly a support group. Rating anxiety lower would be an expected outcome for a stress-management group.

20. A parent became unemployed 6 months ago. The parent has subsequently been verbally abusive toward the spouse and oldest child. The child ran away twice, and the spouse has become depressed. What is the most appropriate nursing diagnosis for this family? a. Impaired parenting related to verbal abuse of oldest child b. Impaired social interaction related to disruption of family bonds c. Ineffective community coping related to fears about economic stability d. Disabled family coping related to insecurity secondary to loss of family income

ANS: D Disabled family coping refers to the behavior of a significant family member that disables his or her own capacity as well as another's capacity to perform tasks essential to adaptation. The distracters are inaccurate because the stressors influence more than one individual.

1. A patient tells members of a therapy group, "I hear voices saying my doctor is poisoning me." Another patient replies, "I once heard voices too. They sounded real, but I found out later they were not. The voices you hear are not real either." Which therapeutic factor is exemplified in this interchange? a. Catharsis b. Universality c. Imitative behavior d. Interpersonal learning

ANS: D Here a member gains insight into his own experiences from hearing about the experiences of others through interpersonal learning. Catharsis refers to a therapeutic discharge of emotions. Universality refers to members realizing their feelings are common to most people and not abnormal. Imitative behavior involves copying or borrowing the adaptive behavior of others.

25. Which individual in the emergency department should be considered at highest risk for completing suicide? a. An adolescent Asian American girl with superior athletic and academic skills who has asthma b. A 38-year-old single, African American female church member with fibrocystic breast disease c. A 60-year-old married Hispanic man with 12 grandchildren who has type 2 diabetes d. A 79-year-old single, white male diagnosed recently with terminal cancer of the prostate

ANS: D High-risk factors include being an older adult, single, male, and having a co-occurring medical illness. Cancer is one of the somatic conditions associated with increased suicide risk. Protective factors for African American women and Hispanic individuals include strong religious and family ties. Asian Americans have a suicide rate that increases with age.

12. Which remark by a group participant would the nurse expect during the working stage of group therapy? a. "My problems are very personal and private. How do I know people in this group will not tell others what you hear?" b. "I have enjoyed this group. It's hard to believe that a few weeks ago I couldn't even bring myself to talk here." c. "One thing everyone seems to have in common is that sometimes it's hard to be honest with those you love most." d. "I don't think I agree with your action. It might help you, but it seems like it would upset your family."

ANS: D In the working stage, members actively interact to help each other accomplish goals, and because trust has developed, conflict and disagreement can be expressed. Focusing on trust and confidentiality typically occur in the orientation phase as part of establishing group norms. Commonality and universality are also themes typically expressed in the orientation phase, whereas reflecting on progress is a task addressed in the termination phase.

15. Parents of a teenager recently diagnosed with serious mental illness express dismay. One parent says, "Our child acts so strangely that we don't invite friends to our home. We quit taking vacations. Sometimes we don't get any sleep." Which nursing diagnosis best applies? a. Impaired parenting b. Parental role conflict c. Impaired social interaction d. Interrupted family processes

ANS: D Interrupted family processes are evident in the face of disruptions in family functioning as a result of having a mentally ill member. Assessment data best support this diagnosis. Data are insufficient to support the other diagnoses.

8. A person who attempted suicide by overdose was treated in the emergency department and then hospitalized. The initial outcome is that the patient will a. verbalize a will to live by the end of the second hospital day. b. describe two new coping mechanisms by the end of the third hospital day. c. accurately delineate personal strengths by the end of first week of hospitalization. d. exercise suicide self-restraint by refraining from attempts to harm self for 24 hours.

ANS: D Suicide self-restraint relates most directly to the priority problem of risk for self-directed violence. The other outcomes are related to hope, coping, and self-esteem.

7. The parent of an adolescent diagnosed with mental illness asks the nurse, "Why do you want to do a family assessment? My teenager is the patient, not the rest of us." Select the nurse's best response. a. "Family dysfunction might have caused the mental illness." b. "Family members provide more accurate information than the patient." c. "Family assessment is part of the protocol for care of all patients with mental illness." d. "Every family member's perception of events is different and adds to the total picture."

ANS: D The identified patient usually bears most of the family system's anxiety and may have come to the attention of parents, teachers, or law enforcement because of poor coping skills. The correct response helps the family understand that the opinions of each will be valued. It allows the nurse to assess individual coping and prepares the family for the experience of working together to set goals and solve problems. The other responses are either incorrect or evasive.

6. Which information is the nurse most likely to find when assessing the family of a patient with a serious mental illness? a. The family exhibits many characteristics of dysfunctional families. b. Several family members have serious problems with their physical health. c. Power in the family is maintained in the parental dyad and rarely delegated. d. Stress from living with a mentally ill member has challenged the family's function.

ANS: D The information almost universally obtained is that the family is under stress associated with having a mentally ill member. This stress lowers the family's level of functioning in at least one significant way. Stress does not necessarily mean the family has become dysfunctional.

3. A young female member in a therapy group says to an older female member, "You are just like my mother, always trying to control me with your observations and suggestions." Which therapeutic factor of a group is evident by this behavior? a. Instillation of hope b. Existential resolution c. Development of socializing techniques d. Corrective recapitulation of the primary family group

ANS: D The younger patient is demonstrating an emotional attachment to the older patient that mirrors patterns within her own family of origin, a phenomenon called corrective recapitulation of the primary family group. Feedback from the group then helps the member gain insight about this behavior and leads to more effective ways of relating to her family members. Instillation of hope involves conveying optimism and sharing progress. Existential resolution refers to the realization that certain existential experiences such as death are part of life, aiding the adjustment to such realities. Development of socializing techniques involves gaining social skills through the group's feedback and practice within the group.

2. Four individuals have given information about their suicide plans. Which plan evidences the highest suicide risk? a. Turning on the oven and letting gas escape into the apartment during the night b. Cutting the wrists in the bathroom while the spouse reads in the next room c. Overdosing on aspirin with codeine while the spouse is out with friends d. Jumping from a railroad bridge located in a deserted area late at night

ANS: D This is a highly lethal method with little opportunity for rescue. The other options are lower lethality methods with higher rescue potential. See relationship to audience response question.

3. Which measure would be considered a form of primary prevention for suicide? a. Psychiatric hospitalization of a suicidal patient b. Referral of a formerly suicidal patient to a support group c. Suicide precautions for 24 hours for newly admitted patients d. Helping school children learn to manage stress and be resilient

ANS: D This measure promotes effective coping and reduces the likelihood that such children will become suicidal later in life. Admissions and suicide precautions are secondary prevention measures. Support group referral is a tertiary prevention measure.

15. An immigrant from China needs a colonic resection but is anxious and reluctant about surgery. This patient usually follows traditional Chinese health practices. Which comment by the nurse would most likely reduce the patient's anxiety and reluctance? a. "Surgery will help rebalance the yin and yang forces and return you to harmony." b. "The surgery we are recommending will help you achieve final transformation." c. "I know this is new to you, but you can trust us to take very good care of you." d. "If you would like, we could investigate using acupuncture to help control pain."

ANS: D It would be helpful to incorporate elements of TCM as appropriate; such as acupuncture for pain control. TCM has the goal of healing in harmony with one's environment and all of creation in mind, body, and spirit, as well as balance of yin and yang energies and a state of transition. However, it would not be helpful to suggest that surgery will balance the yin and the yang, since this is not how balance is achieved in TCM. Transformation is recognized as a stage of healing occurring when mutual, creative, active participation occurs between healers and the patient toward changes in the mind, body, and spirit; but "final transformation" could imply the end of corporeal life and might be perceived as hastening his demise. Appealing to him to trust persons whose practices are foreign to him conflicts with the patient's values and would not likely be effective.

4. A patient with a history of asthma says, "I've been very nervous lately. I think aromatherapy will help. I am ordering $250 worth of oils from an Internet site that promised swift results." Select the nurse's best action. a. Support the patient's efforts to become informed and to find health solutions. b. Suggest the patient check with friends who have tried aromatherapy for treatment of anxiety. c. Remind the patient, "If you spend that much on oils, you may not be able to buy your prescribed medication." d. Tell the patient, "Aromatherapy can complicateNURSINGTB.COM respiratory problems such as asthma. Let's consider some other options."

ANS: D Safety is paramount, and aromatherapy may cause complications for a patient with asthma. The nurse should view alternative treatments with an open mind and try to recognize the importance of the treatment to the patient while trying to give the patient accurate, reliable information about the treatment. Although efforts to become health literate should be supported, educating the patient about the pitfalls of relying on the Internet is essential. The opinions of others, whether they are positive or negative, lack a scientific basis and are subject to confounding variables such as the placebo effect and individual factors such as age and health history. Admonishing the patient may jeopardize the relationship.

3. A patient wants to learn more about integrative therapies. Which resource should the nurse suggest for the most reliable information? a. Internet b. American Nurses Association (ANA) c. Food and Drug Administration (FDA) d. National Center for Complementary and Integrative Health (NCCIH)

ANS: D The NCCIH provides reliable, objective, and scientific information to help in making decisions about use of these practices. NCCIH supports not only research, but also the development and sharing of this kind of information. The FDA has information, but it is not as extensive as NCCIH. The Internet has many resources but some are unreliable. The ANA does not provide extensive information about this topic.


Related study sets

The Great Divorce (chapters 1-3)

View Set

peds prep u chapter 28 neoplastic disorder

View Set

Chapter 8: Distractions Inside and Outside the Vehicle

View Set

AP Psychology: UNIT 10 Test Review

View Set

Business Law - Chapter 5 (Exam #2)

View Set